SBA Pathology
SBA Pathology
To my mother for her struggle in life and for leading her children into intellectual pursuits.
To my father for never compromising on principles and for providing his children with the best
of everything possible within his limited resources.
© 2007 PASTEST LTD
Egerton Court
Parkgate Estate
Knutsford
Cheshire
WA16 8DX
All rights reserved. No part of this publication may be reproduced, stored in a retrieval system, or transmitted, in any form or by any
means, electronic, mechanical, photocopying, recording or otherwise without the prior permission of the copyright owner.
A catalogue record for this book is available from the British Library.
The information contained within this book was obtained by the author from reliable sources. However, while every effort has been
made to ensure its accuracy, no responsibility for loss, damage or injury occasioned to any person acting or refraining from action as a
result of information contained herein can be accepted by the publishers or author.
PasTest has been established in the field of postgraduate medical education since 1972, providing revision books and intensive study
courses for doctors preparing for their professional examinations.
M RCGP, M RCP Parts 1 and 2, M RCPCH Parts 1 and 2, M RCPsych, M RCS, M RCOG Parts 1 and 2, DRCOG, D
FRCA, PLAB Parts 1 and 2.
www.pastest.co.uk [email protected]
Preface
Abbreviations
Questions
Neoplasia
Microbiology
Cardiovascular Pathology
Pulmonary Pathology
Renal Pathology
Haematopathology
Endocrine Pathology
Pharmacology
Answers
Neoplasia
Microbiology
Cardiovascular Pathology
Pulmonary Pathology
Renal Pathology
Haematopathology
Endocrine Pathology
Pharmacology
Index
PREFACE
Recent reforms in medical education in the United Kingdom have prompted the replacement of
multiple choice questions (MCQs) with single best answer questions (SBAs). Access to Surgery: 500
Single Best Answer Questions in General and Systemic Pathologyis the first book of its kind and
provides 500 practice SBAs in general and systemic pathology for candidates taking the surgical
examinations. As the title indicates, it is primarily written for surgical trainees, with an emphasis
mainly on the surgical aspects of human pathology. However, it can be used as a practice tool by
undergraduate medical students and by trainees in other medical disciplines (for whom pathology can
often be a major examination hurdle).
Each question has been carefully formulated to cover a given topic in pathology. All the major
aspects of pathology are dealt with, and are organised in sequence in the following sections: Cell
Injury and Wound Healing, Inflammation and Immunology, Neoplasia, Microbiology, Disorders of
Fluids and Electrolytes, Bleeding and Haemostasis, Cardiovascular Pathology, Pulmonary Pathology,
Renal Pathology, Gastrointestinal and Hepatobiliary Pathology, Haematopathology, Endocrine
Pathology, Breast and Female Reproductive Pathology, Male Reproductive Pathology, Bone and Joint
Pathology and Central Nervous System Pathology; a section on Pharmacology has also been included.
The questions also cover the complete MRCS applied pathology syllabus and are also expected to be
useful for specialty FRCS candidates.
This book contains a substantial number of patient-based questions or clinical scenarios that will
enable prospective candidates to test their ability to integrate key basic pathological concepts with
relevant clinical problems. In addition, factual recall questions have also been included that probe for
basic recall of facts. Detailed and comprehensive explanations, rather than just brief answers, have
been provided so that candidates do not have to consult textbooks for clarification, as is the case with
most MCQ books.
The questions in this book can be used in a number of ways: (i) as a diagnostic tool (ie a pre-test),
(ii) as a guide and focus for further study, and (iii) for self-assessment. The least effective use of
these questions is to ‘study’ them by reading them one at a time, and then looking at the correct
answer. These 500 practice questions are intended to be an integral part of a well-planned review as
well as an isolated resource. If used appropriately, these questions can provide self-assessment
information beyond a numerical score. Furthermore, the questions have been planned in such a way
that this book can be used as companion to any textbook of pathology.
I am hopeful that this book will prove a useful revision and self-assessment tool for all those
involved in learning pathology.
Shahzad G. Raja
2007
ABBREVIATIONS
For each question given below choose the ONE BEST option.
A 45-year-old woman with a chronic infective lesion on her leg underwent a full-thickness
1.2 biopsy of the lesion. During histological examination of this lesion a rim of multinuclear
giant cells is seen. The central region is most likely to show:
A Caseous necrosis
B Eosinophilic necrosis
C Fibrinous necrosis
D Foam cells
E Pyogenic necrosis
1.3 A skin biopsy from an anorexic 16-year-old girl showed cellular atrophy. During atrophy:
1.4 A 35-year-old man is a habitual smoker. If a biopsy is taken from the respiratory tract in
this man, the epithelium of respiratory tract is most likely to show:
A Mucous hyperplasia
B Smooth-muscle hyperplasia
C Squamous cell anaplasia
D Squamous cell hypertrophy
E Stratified squamous metaplasia
1.6 Apoptotic cells usually exhibit distinctive morphological features. Which one of the
following morphological features is usually seen in pure apoptosis?
A Cellular swelling
B Chromatin condensation
C Early disruption of the plasma membrane
D Nuclear stabilisation
E Phagocytosis of apoptotic bodies by neutrophils
1.7 A histopathologist reports fat necrosis after examining a slide. Fat necrosis might be found
in which one of the following situations?
A Brain injury
B Muscle injury
C Trauma to the abdomen
D Trauma to the breast
E Trauma to the bowel
A healthy 26-year-old man fractured his right tibia in a road traffic accident. His right leg
1.8 was immobilised in a plaster cast. The cast was removed from his leg after 8 weeks of
immobilisation. Which of the following changes is most likely to have taken place in his
gastrocnemius muscle after this time?
A Decrease in the number of muscle fibres
B Decrease in the number of nerve fibres
C Increase in the number of fast fibres
D Increase in the mitochondrial content
E Increase in the number of satellite cells
1.9 A 21-year-old man sustained a severe soft-tissue injury following a road traffic accident.
Which of the following metabolic effects is most likely to follow this injury?
A Decreased aldosterone secretion
B Inhibition of gluconeogenesis
C Mobilisation of fat stores
D Protein anabolism
E Respiratory alkalosis
1.10 A 32-year-old man, working in a power plant, was exposed to radioactive material. He is
most likely to suffer radiation injury due to:
A Decreased intracellular Na+
B Decreased intracellular Ca2+
C Free radical formation
D Increased adenosine triphosphate (ATP) production
E Inhibition of protein synthesis
A 15-year-old girl with haemophilia A has had episodes of pain in her knees for the past 6
1.11 years. Over time, there has been an increase in size of her knee joints, with deformity.
Laboratory studies show decreased levels of coagulation factor VIII activity. Which of the
following is most likely to be seen within the joint space following episodes of pain?
A Anthracotic pigment
B Cholesterol crystals
C Lipofuscin
D Neutrophils
E Russell bodies
A 62-year-old diabetic and hypertensive man suffered a stroke which affected his speech
and movement in the right arm and leg. A cerebral angiogram revealed an occlusion of his
1.13 left middle cerebral artery. Months later, a computed tomographic (CT) scan shows a
large, 5-cm cystic area in his left parietal lobe cortex. This CT finding most likely
demonstrates a lesion that is the consequence of resolution of which of the following
events?
A Apoptosis
B Atrophy
C Caseous necrosis
D Coagulative necrosis
E Liquefactive necrosis
A 25-year-old woman breastfed her first baby for almost 1 year with no difficulties and no
1.14 complications. Which of the following cellular processes that occurred in the breast during
pregnancy allowed her to nurse the infant for this period of time?
A Ductal epithelial metaplasia
B Epithelial dysplasia
C Lobular hyperplasia
D Stromal hypertrophy
E Steatocyte atrophy
An 80-year-old woman was found dead in her room in a nursing home one morning. An
autopsy was carried out and her death was reported as being secondary to old age. At
1.15 autopsy, her heart was small (250 g) and dark-brown in colour on section. Microscopically,
there was light-brown perinuclear pigment seen after haematoxylin and eosin (H&E)
staining of the cardiac muscle fibres. Which of the following substances is most likely to be
increased in the myocardial fibres to produce this cardiac appearance?
A Calcium, following necrosis
B Cholesterol, as a consequence of atherosclerosis
C Glycogen, resulting from a storage disease
D Haemosiderin, resulting from iron overload
E Lipochrome, from ‘wear and tear’
A 28-year-old woman developed a darker skin complexion after returning from a holiday
1.18 trip to Goa. Her skin did not show warmth, erythema or tenderness. Her skin tone faded
to its original appearance within 1 month. Which of the following substances contributes
most to the biochemical process leading to these skin changes?
A Glycogen
B Haem
C Homogentisic acid
D Lipofuscin
E Tyrosine
Metaplasia is a reversible change in which one adult cell type (epithelial or mesenchymal)
1.19 is replaced by another adult cell type. In which of the following situations is the process of
epithelial metaplasia most likely to have occurred?
A Acute myocardial infarction
B Lactation following pregnancy
C Tanning of the skin following sunlight exposure
D Urinary obstruction due to an enlarged prostate
E Vitamin A deficiency
A histopathologist reviewing a slide noticed a disease process which has led to scattered
1.20 loss of individual cells, with the microscopic appearance of karyorrhexis and cell
fragmentation. The overall tissue structure, however, has remained intact. This process is
most typical of which of the following diseases?
A Barbiturate overdose
B Brown atrophy of the heart
C Chronic alcoholic liver disease
D Renal transplant rejection
E Viral hepatitis
A 55-year-old woman with chronic atrial fibrillation suddenly developed an acute abdomen
1.21 and was rushed to the emergency department. At emergency laparoscopy most of the
small-bowel loops were dusky to purple-red in colour. Her mesenteric veins were patent.
The most probable underlying pathological process is:
A Coagulative necrosis
B Dry gangrene
C Gas gangrene
D Liquefactive gangrene
E Wet gangrene
A 13-year-old girl complained of redness and pain in her cheeks after spending a sunny day
1.22 on the beach. Her GP told her that she was suffering from sunburn on the cheeks. What is
the most probable underlying cause of her complaint?
A Antigen–antibody reaction
B Damage to DNA
C Free radical injury
D Ischaemic injury
E Vasoconstriction
Adenosine triphosphate (ATP) depletion and decreased ATP synthesis are frequently
1.24 associated with both hypoxic and chemical (toxic) injury. Which of the following is most
likely to result from depletion of ATP?
A Decreased rate of anaerobic glycolysis
B Decreased influx of calcium
C Increased activity of ouabain-sensitive Na+, K+-ATPase
D Increased glycogen synthesis
E Increased unfolded protein response
1.25 Accumulation of which of the following substances indicates ageing at a cellular level in a
biopsy taken from an elderly person?
A Beta carotene
B Bilirubin
C Haemosiderin
D Lipofuscin
E Melanin
A 36-year-old man sustained a 5-cm-long incised wound on his forearm during a bar fight.
1.26 Which of the following is NOT likely to be seen as a complication of healing in this
patient?
A Cicatrisation and disfigurement
B Keloid
C Malignancy
D Proud flesh
E Wound dehiscence
1.27 A 26-year-old woman suffered serious burns to her hands and chest. Which of the
following factors is NOT likely to influence wound healing in this woman?
A Blood supply
B Infection
C Steroids
D Vitamin A deficiency
E Zinc
A rugby player sustained a laceration to his right forearm as a result of a rough tackle.
1.28 This wound was closed with sutures. Wound healing proceeded over the next week. Which
of the following factors will be most likely to aid wound healing in this patient?
A Corticosteroid therapy
B Hypoalbuminaemia
C Presence of sutures
D Poor tissue perfusion
E Secondary wound infection
1.32 Which of the following findings will be most striking in a microscopic slide showing atrophy
of an organ?
A A greater number of autophagic vacuoles
B A greater number of myofilaments
C A greater number of mitochondria
D A greater number of rough endoplasmic reticulum
E A greater number of smooth endoplasmic reticulum
1.33 Which of the following cells is an example of a permanent cell, on the basis of a
classification according to regenerative ability?
A Acinar cell of the pancreas
B Colonic mucosal cell
C Erythrocyte
D Hepatocyte
E Osteocyte
1.34
Which of the following provides an example of concomitant hyperplasia and hypertrophy?
A Breast enlargement at puberty
B Cystic hyperplasia of the endometrium
C Enlargement of skeletal muscle in athletes
D Left ventricular cardiac hypertrophy
E Uterine growth during pregnancy
You are looking at a histopathology slide that shows changes associated with cell injury.
1.35 Which of the following cell changes associated with injury is most likely to be accompanied
by disruption of the cell membrane?
A Apoptosis
B Cloudy swelling
C Coagulative necrosis
D Hydropic change
E Pyknosis
1.36 The histopathologist reported the presence of ‘epithelioid cells’ in a biopsied cervical
lymph node from a patient with tuberculosis. Epithelioid cells are transformed from:
A Epithelial cells
B Eosinophils
C Lymphocytes
D Macrophages
E Neutrophils
A lump was excised and sent for histopathological examination. The histopathologist
1.37 reported seeing what appeared to be an abnormal amount and arrangement of normal
tissue than is appropriate or normal for the area in which the tissue arises. This is best
described as:
A Carcinosarcoma
B Embryonal tumour
C Hamartoma
D Mixed tumour
E Teratoma
1.38 A histopathology report states that the periphery of a haematoma is infiltrated by new
capillaries, fibroblasts, and collagen. What is this process called?
A Embolisation
B Lysis of the clot
C Organisation of the haematoma
D Recanalisation
E Thrombosis
1.39 Which of the following terms best describes the passage of leukocytes through the blood
vessel wall?
A Diapedesis
B Euperiporesis
C Migration
D Phagocytosis
E Pavement
A 62-year-old man suffered a small myocardial infarction involving the inferior wall of the
1.40 heart. The length of time required for a scar of a small myocardial infarct to reach full
strength is approximately:
A 3 days
B 1 week
C 2 weeks
D 6 weeks
E Several months
SECTION 2:
INFLAMMATION AND IMMUNOLOGY — QUESTIONS
For each question given below choose the ONE BEST option.
A 45-year-old woman with carcinoma of right breast and enlarged axillary lymph nodes on
2.1 the same side underwent mastectomy with axillary clearance. Histological examination of
the axillary lymph nodes showed no evidence of metastasis. This enlargement of lymph
nodes is most probably due to:
A Lymphoma
B Paracortical lymphoid hyperplasia
C Secondary deposits
D Sinus histiocytosis
E Tuberculosis
A 6-year-old boy has a history of repeated pyogenic infections. He had normal antibody
2.2 responses following childhood immunisations and showed normal recovery from
chickenpox and measles. Decreased numbers or functional defects in which of the
following cells best explains the cause of his repeated pyogenic infections?
A B lymphocytes
B Eosinophils
C Macrophages
D Neutrophils
E T lymphocytes
A woman is stung by a bee. Within 5 minutes, she develops a raised, red, swollen lesion at
2.4 the site of injury, 2 cm in size. Which of the following findings is most likely to be seen in
this lesion?
A Foreign body reaction
B Haemorrhage
C Lymphocytic infiltration
D Neutrophilic migration
E Vasodilation
A Chemotaxis
B Leukocyte adhesion
C Leukocyte margination
D Leukocyte transmigration
E Phagocytosis
A 25-year-old woman develops nasal discharge and itching of the eyes each year in the
2.6 Spring. She was referred by her GP for allergy testing. Fifteen minutes after skin testing
with a mixture of grass pollens, she developed erythema and a 15-mm wheal at the site.
The skin test response is most likely to be a result of which of the following mechanisms?
A Antigen–antibody complexes being formed in blood vessels in the skin
B Influx of phagocytic cells in response to injection of foreign proteins
C Release of histamine from mast cells
D Release of lymphokines by sensitised lymphocytes reacting with antigens
E Release of lymphokines from mast cells
Platelet-derived growth factor binds to its receptor, activating cell growth. The receptor–
2.7 growth factor complex uses which of the following mechanisms to signal to the cell to
divide?
A Activation of tyrosine kinase
B Binding of GTP to a G protein
C Binding to DNA
D Increase in intracellular calcium concentration
E Opening of an ion channel
2.11 Which of the following cytokines is produced by T lymphocytes that express class II MHC
antigen?
A Alpha interferon
B Beta interferon
C Gamma interferon
D Interleukin-1
E Tumour necrosis factor
A 12-year-old boy was brought to the emergency department by his parents because he
had developed marked right lower quadrant abdominal pain over the past 24 hours. On
physical examination there was rebound tenderness on palpation over the right lower
2.12 quadrant. Laparoscopic surgery was performed, and the appendix was swollen,
erythematous, and partly covered by a yellowish exudate. It was removed, and a
microscopic section showed infiltration with numerous neutrophils. The pain experienced
by this patient was predominantly the result of the formation of which of the following
pairs of chemical mediators?
A Complement C3b and IgG
B Histamine and serotonin
C Interleukin-1 and tumour necrosis factor
D Leukotriene and HPETE
E Prostaglandin and bradykinin
A 35-year-old man was seen in a surgical outpatient clinic complaining of a tender lump
which had appeared on his right forearm about 3 weeks previously. On enquiry, he
revealed that he had been involved in a road traffic accident about 6 months before, when
2.13 several pieces of windscreen entered his forearm which were removed at the time of initial
presentation in the emergency department. At elective surgery a further fragment of glass
was recovered from his right forearm. Which of the following cell types is would be most
characteristic of the inflammatory response in this situation?
A Eosinophil
B Giant cell
C Mast cell
D Neutrophil
E Plasma cell
Cultured sputum from a 55-year-old man with a 4-day history of cough and fever grew
2.14 Streptococcus pneumoniae. Clearance of these organisms from the lung parenchyma
would be most effectively accomplished through generation of which of the following
substances by the major inflammatory cell type responding to this infection?
A Hydrogen peroxide
B Kallikrein
C Leukotriene
D Platelet-activating factor
E Prostaglandins
A 64-year-old man with a history of inhaling silica dust for many years in his job has
become increasingly dyspnoeic over the last 3 years. A chest X-ray now shows increased
2.15 intersitital markings and nodules ranging in size from 1 cm to 3 cm in the parenchyma. His
pulmonary problems are most likely to be caused by which of the following inflammatory
processes?
A Foreign-body giant-cell formation
B Histamine release by mast cells
C Neutrophilic infiltration with release of leukotrienes
D Production of immunoglobulin by plasma cells
E Release of growth factors by macrophages
A 16-year-old boy with a 1-day history of sore throat was seen by his GP. On physical
2.16 examination, the most prominent finding was a pharyngeal purulent exudate. Which of the
following types of inflammation does this boy have?
A Acute inflammation
B Abscess formation
C Chronic inflammation
D Granulomatous inflammation
E Resolution of inflammation
A sexually active 21-year-old man has experienced pain when passing urine for 3 days.
Urethritis is suspected, and Neisseria gonorrhoeae is cultured. Numerous neutrophils are
2.18
present in a smear of the exudate from the penile urethra. These neutrophils are most
likely to have undergone diapedesis to reach the organisms as a consequence of release of
which of the following chemical mediators?
A Bradykinin
B Complement C5a
C Hageman factor
D Histamine
E Prostaglandin
A 58-year-old man with osteoarthritis of 4 years’ duration feels better after taking
2.19 acetylsalicylic acid (aspirin) as his pain is markedly reduced. This pain reduction is most
likely to be the result of a reduction in which of the following inflammatory responses?
A Anticoagulation by Hageman factor inhibition
B Fever resulting from interleukin-1
C Neutrophil chemotaxis by leukotriene B4
D Pain resulting from bradykinin generation
E Prostaglandin-mediated vasodilation
C Immunoglobulin M
D P-selectin
E NADPH oxidase
A healthy 25-year-old man with no major medical problems says that he breaks out with
blotchy areas of erythema that are pruritic over the skin of his arms, legs and trunk within
2.21 an hour every time he eats strawberries, this is followed by diarrhoea. These problems
abate within a few hours, and physical examination reveals no abnormal findings. Which of
the following immunological abnormalities is he most likely to have?
A Cell-mediated hypersensitivity
B Hypergammaglobulinaemia
C Immune complex deposition
D Localised anaphylaxis
E Release of complement C3b
A 15-month-old boy with failure to thrive has recently been falling ill with one bacterial
2.23 pneumonia after another, with both Haemophilus influenzae and Streptococcus
pneumoniae cultured from his sputum. He was born at term with no congenital anomalies.
Which of the following diseases is he most likely to have?
A Acute leukaemia
B DiGeorge syndrome
C Epstein–Barr virus (EBV) infection
D Selective IgA deficiency
E X-linked agammaglobulinaemia
While presenting an audit of outcomes involving patients who received renal allografts in a
renal transplant unit, the SHO highlighted the fact that patients who received renal
2.24 allografts with matching to the donor by tissue typing for HLA-DR (class II) antigens had
a low rate of complications. Which of the following immunological abnormalities is most
likely to be diminished by tissue typing?
A Amyloidosis
B Cell lysis by CD8 lymphocytes
C CD4 lymphocyte activation
D Graft-versus-host disease
E Serum sickness
2.25 Which of the following is most likely to be found in a 10-year-old girl showing type I
hypersensitivity response of the immune system accompanied by eosinophilia?
A Amyloidosis
B Dust inhalation
C Liver flukes
D Neoplasm
E Spirochaetes
A 28-year-old nurse has had a chronic cough with fever for 2 months. On physical
examination, her temperature is 37.9 °C. A chest X-ray reveals a diffuse bilateral
2.26 reticulonodular pattern. A transbronchial biopsy is performed. On microscopic examination
of the biopsy, focal areas of inflammation containing epithelioid macrophages, Langhans
giant cells and lymphocytes are found. These findings are most typical for which of the
following immunological responses?
A Graft-versus-host disease
B Polyclonal B-cell activation
C Type I hypersensitivity
D Type II hypersensitivity
E Type IV hypersensitivity
2.27 Tumour necrosis factor (TNF) is a cytokine involved in systemic inflammation. Which of
the following statements about TNF is true?
A It inhibits apoptotic cell death
B It inhibits the acute phase reaction
C It is a soluble intracytoplasmic peptide
D It is an appetite suppressant
E It is mainly produced by basophils
Interleukin-1 (IL-1) is one of the first cytokines ever described. Its initial discovery was as
2.28 a factor that could induce fever. Which of the following statements about interleukin-1 is
correct?
A It increases the expression of adhesion factors on endothelial cells
B It is a lipopolysaccharide
C It is an anti-inflammatory cytokine
D It is produced by endothelial cells
E It suppresses bone marrow cells
2.31 The histopathology report for a granulomatous lesion suggests chronic inflammation.
Which cell types are most commonly seen in tissue undergoing chronic inflammation?
A Eosinophils
B Lymphocytes
C Mast cells
D Neutrophils
E Platelets
2.32 A histopathology report describes the presence of granulation tissue in a lesion. Which of
the following features is characteristic of granulation tissue?
A Giant cells and fibroblasts
B Giant cells and lymphocytes
C Giant cells, plasma cells and lymphocytes
D Neutrophils and necrotic tissue
E Proliferation of new capillaries, with fibroblasts and new collagen formation
2.33 Which of the following terms best describes the unidirectional migration of leukocytes
towards a target?
A Chemotaxis
B Diapedesis
C Endocytosis
D Margination
E Meiosis
2.34 What is the origin of the cells of the mononuclear phagocyte system?
A Bone marrow
B Liver
C Lymph nodes
D Thymus
E Spleen
A 42-year-old woman with end-stage renal disease is prepared to receive a kidney from
her husband. Assays for HLA antigens indicate that the donor and recipient are not 100%
compatible. The patient is given an immunosuppressive drug regimen. Three months after
2.36 transplantation, laboratory tests indicate that the patient’s kidney function is declining.
There is a rapid decrease in urine output and the urine contains blood cells and a high level
of protein. The transplanted kidney is enlarged and tender. After treatment with
antilymphocyte globulin, the patient’s critical condition is reversed. What type of graft did
this patient receive?
A Allograft
B Autograft
C Isograft
D Syngraft
E Xenograft
A 45-year-old man with end-stage renal disease is prepared to receive a kidney from his
best friend. Assays for HLA antigens indicate that the donor and recipient are not 100%
2.37 compatible. The patient is given an immunosuppressive drug regimen. Thirteen weeks
after transplantation, laboratory tests indicate that the patient’s kidney function is
declining. There is a rapid decrease in urine output and the urine contains blood cells and a
high level of protein. The transplanted kidney is enlarged and tender. After treatment with
antilymphocyte globulin, the patient’s critical condition is reversed. During the crisis
period, the patient was most likely experiencing:
A Acute rejection
B Chronic rejection
C Graft-versus-host disease
D Hyperacute rejection
E Toxicity to the drugs
Following a major traffic accident, a wife (blood group B) and husband (blood group A) are
2.38 rushed to the emergency department. Donor blood for the wife is erroneously transfused
into the husband. Within minutes, he develops a fever, chills, dyspnoea and a dramatic
decrease in blood pressure. This reaction is most likely to have been caused by:
A A cell-mediated response against the A antigen of the recipient
B Anti-A isohaemagglutinin of the immunoglobulin M (IgM) class present in the recipient
C Anti-B isohaemagglutinin of the IgM class preformed in the recipient
Immunoglobulin G (IgG) production by the recipient in response to the infused red blood
D
cells
E IgM production by the recipient in response to the B antigen
A 26-year-old woman with blood group type A, Rh-negative, is pregnant with her second
2.39 child. Her first child is Rh-positive, and the father is also Rh-positive. The second child is
most likely to be at risk of developing:
A An autoimmune disease
B ABO incompatibility
C Drug-induced haemolytic anaemia
D Neutropenia
E Haemolytic disease of the newborn
A 25-year-old man with persistent low-grade fever and cough had a blood smear which
2.41 showed a marked increase in the number of cells with a large bi-lobed nucleus. Which of
the following cell types was seen in the smear?
A Basophils
B Eosinophils
C Lymphocytes
D Monocytes
E Neutrophils
2.42 Which of the following white blood cells will be the predominant cell type seen under the
microscope if a blood smear is made from a normal healthy individual?
A Basophils
B Eosinophils
C Lymphocytes
D Monocytes
E Neutrophils
A 58-year-old farmer with a hydatid cyst in the liver was admitted for elective surgery to
2.43 remove the cyst. Which of the following white blood cell types will be raised in this
patient’s preoperative full blood count?
A Basophils
B Eosinophils
C Lymphocytes
D Monocytes
E Neutrophils
D Interleukin-10
E Tumour necrosis factor (TNF)
2.47 Arachidonic acid is one of the essential fatty acids required by most mammals. It is
essential for the synthesis of which of the following mediators of inflammation?
A Bradykinin
B Interferon-gamma
C Interleukin-1
D Prostaglandins
E Tumour necrosis factor
Leukotrienes are involved in asthmatic and allergic reactions and act to sustain
2.48 inflammatory reactions. Blockage of which one of the following enzymes will inhibit the
synthesis of leukotrienes?
A 5-alpha-reductase
B 5-lipoxygenase
C Cyclooxygenase-1
D Cyclooxygenase-2
E Peroxidase
2.49 Which one of the following is a primary function of the Kupffer cells in the liver?
A Protein synthesis
B Recycling of old red blood cells
C Secretion of mucus
D Storage of fat-soluble vitamins
E Synthesis of intrinsic factor
2.50 Which of the following immune cells has perforin in its granules?
A B lymphocyte
B Eosinophil
C Kupffer cell
D Mast cell
E Natural killer cell
SECTION 3:
NEOPLASIA — QUESTIONS
For each question given below choose the ONE BEST option.
A 42-year-old woman noted a lump in her right breast while taking a shower. Her GP
confirmed the presence of a 3-cm, firm, irregular, non-movable mass located in the upper
outer quadrant of her right breast on physical examination. A fine-needle aspiration of this
3.1 mass was performed. Cells obtained from the mass were examined cytologically and were
consistent with infiltrating ductal carcinoma. The mass was excised in a lumpectomy
procedure, and an axillary lymph node dissection was performed. Which of the following
findings will best predict a better prognosis for this patient?
A Flow cytometric analysis demonstrates aneuploidy and a high S-phase
B No metastases are found in the sampled lymph nodes
C She has one relative who had a similar type of breast cancer
D The tumour cells are strongly oestrogen receptor-positive
E The tumour has a high grade
A change in bowel habits prompted a 58-year-old man to see his GP. On physical
examination, there were no lesions noted on digital rectal examination, but his stool was
3.2 positive for occult blood. A colonoscopy was performed and revealed a 6-cm friable mass
located in the caecum. A biopsy of this mass was performed and microscopic examination
showed a moderately differentiated adenocarcinoma. Which of the following findings is
most likely to be present in this patient?
A A high titre of DNA topoisomerase I autoantibody
B A K-ras mutation in the neoplastic cells
C An immunoperoxidase stain positive for vimentin in the neoplastic cells
D A plasma HIV-1 RNA level of 40,000 copies/ml
E A stool culture positive for Shigella flexneri
A healthy 45-year-old woman has a routine health check. She has no chest pain, cough, or
3.3 fever. A chest X-ray is taken, however, which shows a peripheral ‘coin lesion’, 2.5 cm in
diameter, in the right mid-lung field. Which of the following biological characteristics
would best distinguish this lesion as a neoplasm, rather than a granuloma?
A Necrosis
B Rapid increase in size
C Recurrence following excision
D Sensitivity to radiation or chemotherapy
E Uncontrolled (autonomous) growth
A 45-year-old woman was found to have a 4-cm-diameter, non-tender mass in her right
breast. The mass appeared to be fixed to the chest wall. Another 2-cm, non-tender mass
3.5 was palpable in the left axilla. A chest X-ray reveals multiple nodules in both lungs, ranging
in size from 0.5 cm to 2 cm. Which of the following classifications best describes the stage
of her disease?
A T1 N1 M0
B T1 N0 M1
C T2 N1 M0
D T3 N0 M0
E T4 N1 M1
Review of a series of surgical pathology reports indicates that a certain type of neoplasm
3.7 is graded as grade I on a scale of I to IV. Clinically, some of the patients with this
neoplasm are found to be stage I. Which of the following is the best interpretation of a
neoplasm with this stage I designation?
A It has an in-situ component
B It has probably arisen from epithelium
C It is unlikely to be malignant
D It is well differentiated and localised
E It could spread via lymphatics
A 48-year-old woman has had a painless mass in the right parotid gland for the past 8
months. A 2-cm-diameter, discrete, solid mass is enucleated from the parotid gland.
3.8 Histological examination shows a neoplastic lesion with uniform epithelial and
myoepithelial cells forming acini, tubules and ducts and supported by a myxoid and
chondroid stroma. Which of the following is the most likely complication of this type of
parotid lesion?
A Contralateral immune-mediated parotitis
B Haematogenous metastases to lungs and bone
C Ipsilateral submaxillary salivary gland neoplasm
D Local recurrence
E Regional lymph node metastases
A 76-year-old man with back pain was diagnosed with metastatic prostatic carcinoma
3.9 involving his lumbar spine. High serum levels of which of the following tumour markers will
aid the diagnosis of prostatic carcinoma?
A Acid phosphatase
B Alkaline phosphatase
C Alpha-fetoprotein
D CEA
E PSA
3.12 A farmer is diagnosed with hepatic angiosarcoma. Exposure to which of the following
agents is responsible for the occurrence of this neoplasm?
A Aflatoxin
B Arsenic
C Dust particles
D Mouldy hay
E Pollen
A 35-year-old woman is very anxious about her tendency to develop breast cancer as both
3.13 her mother and grandmother died of it. She has no lump palpable on examination. The best
way to assess her risk of developing breast cancer is to perform:
A Mammography
B Magnetic resonance imaging
C TP53 gene analysis
D BRCA1 and BRCA2 gene analysis
E Fine-needle aspiration cytology
3.14 Hypercalcaemia is probably the most common paraneoplastic syndrome. The most likely
mechanism by which cancer causes this abnormality is:
A Bone metastasis
B Increased level of vitamin D
C Increased level of parathyroid hormone (PTH)
D Increased level of PTH-related protein
E Renal failure
3.15 You are reading a histopathology report which describes the excised tumour as benign.
Which one of the following is a benign tumour?
A Adenocarcinoma
B Fibrosarcoma
C Haematoma
D Osteogenic sarcoma
E Warthin’s tumour
3.16 Which of the following features is taken into account when staging a cancer?
A Basophilia
B Local invasion
C Nuclear/cytoplasmic ratio
D Number of mitotic figures
E Pleomorphism
3.17 A 50-year-old woman has got a 7-cm-diameter mass in her right breast, with one enlarged
right axillary lymph node. The most likely stage of disease is:
A IA
B IB
C IIB
D IIIA
E IIIB
3.18 You are reading a histopathology report which describes an excised lesion as a
premalignant condition. Which of the following is a premalignant condition?
A Acne
B Familial polyposis
C Keloid
D Solar keratosis
E Villous adenoma of colon
A 52-year-old woman with carcinoma of left breast and left axillary lymph node
involvement underwent successful mastectomy with axillary clearance. The histopathology
3.20 report described the tumour to be 3.5 cm in its maximum diameter, with three axillary
lymph nodes showing microscopic evidence of tumour. The most likely stage of disease in
this patient is:
AI
B IIA
C IIB
D IIIA
E IIIB
3.21 A 60-year-old woman with suspected carcinoma of cervix is seen by the gynaecologist. The
most reliable and easy method to confirm the diagnosis is:
A Blood culture
B Colposcopy
C Laparotomy
D Pap smear
E Ultrasound
3.22 A 5-year-old child has been diagnosed with a tumour. Which of the following tumours is
most likely to be found in this child?
A Lymphangiosarcoma
B Melanoma
C Meningioma
D Nephroblastoma
E Pleomorphic adenoma
3.23 BRCA1 and BRCA2 genes are used as tumour markers for the diagnosis of a tumour of
which of the following organs?
A Bladder
B Breast
C Kidney
D Lung
E Rectum
A 56-year-old man presents in the surgical outpatient clinic complaining of increasing pain
3.24 in the mid-epigastrium, with occasional radiation to the back. He also mentions that he has
lost nearly 5 kg in weight over the past 3 months. The consultant suspects pancreatic
carcinoma. Which of the following tumour markers would aid in confirming this diagnosis?
A Alpha-fetoprotein (AFP)
B Carcinoembryonic antigen (CEA)
C CA-125
D Human chorionic gonadotrophin (hCG)
E Prostate-specific antigen (PSA)
A 38-year-old woman presented with an enlarging pelvic mass that extended to the
3.25 umbilicus. On clinical examination, no pathological abnormalities of the gastrointestinal
tract were revealed. Which tumour marker is likely to be raised?
A CA-125
B CEA
C hCG
D HER2
E p53s
3.26 Industrial exposure to which of the following substances has been correlated with the
occurrence of malignant mesothelioma?
A Asbestos
B Beryllium
C Coal dust
D Nitrogen dioxide
E Silica
3.27 Which one of the following tumours is associated with Paget’s disease of bone?
A Ewing’s sarcoma
B Giant-cell tumour
C Metastatic duct carcinoma of the breast
D Osteosarcoma
E Multiple enchondromas
3.28 Which of the following is the most common primary malignant tumour of the thyroid?
A Anaplastic carcinoma
B Follicular carcinoma
C Large-cell carcinoma
D Medullary carcinoma
E Papillary carcinoma
3.29 A patient presents in the surgical outpatient clinic with carcinoma of the oral cavity.
Carcinoma of the oral cavity:
A Is more common in women
B Is predisposed to by erythroplasia
C Is predisposed to by hairy leukoplakia
D Is predominantly adenocarcinoma in type
E Occurs most frequently on the hard palate
3.30 Which of the following is the most common benign bone tumour affecting individuals under
the age of 21 years?
A Chondromyxoid fibroma
B Osteochondroma
C Giant-cell tumour
D Aneurysmal bone cyst
E Osteogenic sarcoma
3.32 Which of the following is the most common benign germ-cell tumour of the ovaries in
premenopausal woman?
A Brenner tumour
B Dysgerminoma
C Dermoid cyst
D Hilus-cell tumour
E Mucinous cystadenoma
A 12-year-old boy complains of leg pain and swelling, and an X-ray of the affected limb
3.33 shows the classic sign of Codman’s triangle. Which of the following is the most likely
diagnosis?
A Aneurysmal bone cyst
B Chondrosarcoma
C Multiple myeloma
D Osteomyelitis
E Osteosarcoma
A Neuroblastoma – chromosome 1
B Neurofibromas – chromosome 13
C Osteogenic sarcoma – chromosome 8
D Retinoblastoma – chromosome 11
E Wilms’ tumour – chromosome 7
3.37 In which of the following sites is a tumour more likely to occur in adults than in children?
A Bone
B Central nervous system
C Kidney
D Lung
E Soft tissue
3.38 Which of the following types of cancers is most common in organ transplant recipients?
A Breast cancer
B Lung cancer
C Pancreatic cancer
D Prostate cancer
E Skin cancer
3.39 For which of the following tumours is the malignant potential most often associated with
tumour size?
A Breast adenocarcinoma
B Colon adenocarcinoma
C
D Prostate adenocarcinoma
Renal adenocarcinoma
E Squamous cell carcinoma of the lung
3.40 Which of the following sites in newborns most commonly gives rise to tumours derived
from all three germ-cell layers?
A Central nervous system
B Mediastinum
C Ovaries
D Sacrococcygeal area
E Testis
A 53-year-old-old patient with non-small-cell lung cancer involving the left upper lobe had
3.42 a staging computed tomographic scan which revealed enlarged ipsilateral mediastinal
lymph nodes. What is the nodal staging according to TNM staging system for this patient?
A N0
B N1
C N2
D N3
E N4
3.43 Which of the following is the most common predisposing factor for adenocarcinoma of the
oesophagus?
A Exposure to nitrosamines
B Gastro-oesophageal reflux disease
C Human papilloma virus infection
D Smoking
E Tylosis palmaris
3.44 A 52-year-old man had a stage III colon cancer resected successfully, followed by adjuvant
chemotherapy. What is the most likely 5-year survival rate for this patient?
A 90%
B 70%–85%
C 30%–60%
D 5%
E <1%
3.45 Which of the following types of lung cancer is most likely to present as disseminated
disease at the time of initial presentation?
A Adenocarcinoma
B Bronchoalveolar carcinoma
C Large-cell carcinoma
D Small-cell carcinoma
E Squamous cell carcinoma
A 68-year-old man with haematuria was diagnosed with bladder cancer. The primary
3.47 tumour involved the perivesical fat. What is the primary tumour (T) stage of this patient’s
cancer?
A Ta
B T1
C T2
D T3
E T4
3.48 Which of the following is the most commonly associated risk factor for bladder cancer?
A Alcohol
B Coffee
C Human papilloma virus infection
D Radon
E Smoking
A 52-year-old man was diagnosed with renal cell carcinoma. The tumour was seen to be
3.49 extending into the inferior vena cava on computed tomography. What is the clinical stage
of this tumour?
A Stage 0
B Stage I
C Stage II
D Stage III
E Stage IV
A 53-year-old woman was diagnosed with a squamous cell carcinoma on the forehead
3.50 measuring more than 2 cm but less than 4 cm in its greatest dimension. What is the tumour
stage of this primary tumour according to the TNM staging system for head and neck
cancers?
A T1
B T2
C T3
D T4
E TX
SECTION 4:
MICROBIOLOGY — QUESTIONS
For each question given below choose the ONE BEST option.
A 4-year-old child was brought to the emergency department with fever, hypotension,
4.1 erythema and neck stiffness. Which of the following toxins is most likely to be responsible
for this child’s condition?
A Botulinum toxin
B Endotoxin
C Erythrotoxin
D Exotoxin
E Neurotoxin
A 55-year-old man was brought to the emergency department 2 hours after the onset of
delirium. On arrival, he was confused and uncooperative. His vital signs were:
temperature 39.4 °C, blood pressure 95/50 mmHg, pulse 140/minute and regular,
4.2 respiratory rate 24/minute. Clinical examination was unremarkable. Laboratory
investigations showed: serum glucose 3.9 mmol/l, serum creatinine 1.4 mg/dl; urinary
glucose 0, urinary protein 2+, urinary white blood cell count >200 cells/high-power field,
and no urinary casts. Which of the following organisms is most likely to be grown from
blood cultures in this patient?
A Bacteroides fragilis
B Candida albicans
C Escherichia coli
D Pseudomonas aeruginosa
E Staphylococcus aureus
A sexually active 19-year-old male university student has had fatigue and malaise for the
past 10 days. Early in the course he had a mild pharyngitis. On physical examination, he
has enlarged and tender posterior cervical, axillary and inguinal lymph nodes. His liver
edge is slightly tender and the spleen tip is palpable. He has mild scleral icterus. A full
4.3 blood count shows: haemoglobin 13.5 g/dl, haematocrit 40%, mean corpuscular volume
(MCV) 93 fl, platelet count 263 × 109/l, and white blood cell count (WBC) 13 × 109/l, with
a WBC differential count of 45 neutrophils, 3 bands, 25 lymphocytes, 15 atypical
lymphocytes, 10 monocytes, and 2 eosinophils. Which of the following infectious agents is
most likely responsible for these findings?
A Epstein–Barr virus
B Hepatitis A virus
C Human immunodeficiency virus
D Rubella virus
E Rubeola virus
The vaginal culture of a 28-year-old woman with excessive vaginal discharge showed non-
4.4 pathogenic bacteria. A vaginal smear showed numerous bacilli under the microscope. The
most likely organism is:
A Escherichia coli
B Gardnerella vaginalis
C Lactobacillus species
D Proteus species
E Pseudomonas species
4.5 Which of the following structures is found in Gram-negative bacteria but not in Gram-
positive bacteria?
A Capsule
B Cell wall
C Cytoplasmic membrane
D Endospore
E Outer membrane
A 26-year-old woman presents with a 2-month history of cough that has recently become
productive, fatigue, night sweats and a recent weight loss of 5 lb. She has a history of
4.6 intravenous drug abuse and has recently been diagnosed as HIV-positive. Sputum samples
contain many acid-fast bacilli and her Mantoux test is positive. Which of the following is
the most appropriate initial empirical therapy for this patient?
A Cefoxitin
B Erythromycin
C Clarithromycin + isoniazid
D Rifampicin + isoniazid
E Rifampicin + isoniazid + pyrazinamide + ethambutol
A week after undergoing sigmoid resection with diverting colostomy for ruptured sigmoid
diverticulum, a 64-year-old man has a temperature of 39.7 °C and shaking chills. He
4.7 received gentamicin and ampicillin therapy after the operation and his postoperative
course had been uncomplicated until now. A blood culture grew Gram-negative bacilli.
Which of the following is the most likely causative organism for this patient’s present
condition?
A Bacteroides fragilis
B Brucella abortus
C Escherichia coli
D Proteus mirabilis
E Pseudomonas aeruginosa
4.9 A 32-year-old man developed fatal food poisoning. Which of the following organisms is
most likely to be associated with fatal food poisoning?
A Bacillus cereus
B Clostridium botulinum
C Escherichia coli
D Staphylococcus aureus
E Vibrio cholerae
4.10 A 26-year-old man has developed folliculitis. Which of the following organisms is most
likely to be responsible for this condition?
A Escherichia coli
B Klebsiella species
C Proteus species
D Pseudomonas species
E Staphylococcus aureus
A 32-year-old man developed a high-grade fever with rusty-coloured sputum. His chest X-
4.11 ray showed right-sided consolidation. The most accurate test for diagnosis in this case
would be:
A Blood culture
B Cold antibody titre
C Lung biopsy
D Sputum culture
E Sputum Gram stain
4.12 The drug of choice for treating pulmonary anthrax is:
A Ceftriaxone
B Clindamycin
C Ciprofloxacin
D Metronidazole
E Tetracycline
A 21-year-old woman who had been feeling unwell since returning from a holiday trip to
4.13 India presented to the emergency department on day 16 after the onset of fever. She had
a positive Widal test. The most likely organism resposnsible for the fever is:
A Bacteroides fragilis
B Escherichia coli
C Klebsiella species
D Mycobacterium tuberculosis
E Salmonella typhi
4.14 Which of the following is the most common portal of entry in Blastomyces dermatitidis
infection?
A Central nervous system
B Circulatory system
C Mouth
D Respiratory tract
E Skin
4.15 A 45-year-old hospital patient acquires a Pseudomonas infection. The most likely
mechanism for pathogenesis in Pseudomonas infection is:
A Activation of cAMP
B Activation of EF-2
C Endotoxin
D Exotoxin
E Inhibition of cGMP
A previously healthy, 24-year-old, male intravenous drug addict was admitted to hospital
with complaints of fever and malaise over the past 5 days. Clinical examination suggested
4.16 infective endocarditis, which was confirmed on echocardiography. From two sets of blood
culture there was growth of Gram-positive cocci arranged in clusters. Which of the
following organisms is most likely to be responsible for endocarditis in this patient?
A Enterococcus
C Escherichia
B Pseudomonascolio
aeruginosao
D Staphylococcus aureuso
E Streptococcus pneumoniae
A Cats
B Ticks
C Flies
D Mosquito bite
E Unpasteurised milk
A Actinomycosis
B Amoebiasis
C Brucellosis
D Histoplasmosis
E Staphylococcus aureus infection
4.24 A 12-year-old boy is suffering from a parasitic infestation. Which type of anaemia is most
likely to be seen in this boy?
A Aplastic anaemia
B Anaemia of chronic disease
C Folic acid deficiency anaemia
D Haemolytic anaemia
E Iron deficiency anaemia
A 22-year-old medical student presented in the emergency department with sore throat
4.25 and lymphadenopathy. Her peripheral smear shows atypical lymphocytes. What is the most
probable infectious cause for this patient’s clinical condition?
A Cytomegalovirus
B Epstein–Barr virus
C Herpes simplex virus
D Human immunodeficiency virus
E Mycobacterium tuberculosis
A 73-year-old man with chronic prostatitis who declined surgery for benign prostatic
4.27 hypertrophy was commenced on doxycycline after an acute exacerbation of prostatitis.
Which of the mechanisms given below describes the mechanism of action of doxycycline?
A Acts as a competitive inhibitor of the enzyme dihydropteroate synthetase
B Blockage of bacterial DNA replication
C Inhibition of bacterial wall synthesis
D Inhibition of translocation of peptides by binding to the 50S subunit of the bacterial ribosome
E Prevents the amino-acyl t-RNA from binding to the A site of the ribosome
A 26-year-old man with cystic fibrosis was commenced on amikacin to treat pneumonia
4.30 caused by a multidrug-resistant strain of Pseudomonas aeruginosa. Which of the
mechanisms given below describes the mechanism of action of amikacin?
A Acts as a competitive inhibitor of the enzyme dihydropteroate synthetase
B Blockage of bacterial DNA replication
C Inhibition of bacterial wall synthesis
D Inhibition of protein synthesis by binding to the 30S subunit of the bacterial ribosome
E Prevents the amino-acyl t-RNA from binding to the A site of the ribosome
A 68-year-old man had a catheter placed in his urethra at the time of abdominal aortic
aneurysm repair. The catheter had to be left in for longer than anticipated as this patient
developed postoperative acute renal failure. One week later he experienced suprapubic
4.31 and flank pain, with urinary urgency and frequency. He also had chills and fever. After
examining him and evaluating the sediment of his centrifuged urine, he was informed by
the intensive care consultant that he had acute ascending pyelonephritis. If the diagnosis is
accurate and this man’s urine is cultured, which of the following organisms is most likely to
be isolated?
A Clostridium difficile
B Escherichia coli with pili
C Pseudomonas aeruginosa
D Staphylococcus aureus
E Streptococcus pneumoniae
A 28-year-old man presented to his GP complaining of fatigue, night sweats and a dry
unproductive cough 6 months after a holiday trip to South Africa. Until the past few
4.32 months, he had apparently been in good health. A full blood count and differential blood
count reveal that he is lymphopenic. X-ray examination reveals an interstitial pneumonia.
Skin test reactions to a battery of materials are normal. Which of the following is the most
appropriate next step in evaluating this patient’s illness?
A CH50 assay
B Chemotaxis assay
C Identification of the organism that is causing the pneumonia
D Intracellular killing assay
E Nitroblue tetrazolium reduction assay
A 12-year-old boy with recurrent pulmonary infections has been brought to the hospital
with meningitis by his mother. Gram staining of the spinal fluid reveals numerous
4.33
polymorphonuclear neutrophils and Gram-positive cocci in grape-like clusters. Which of
the following will be the most appropriate empirical drug of choice to be employed until the
antibiotic sensitivity report is received from the laboratory?
A Ampicillin
B Chloramphenicol
C Methicillin
D Streptomycin
E Penicillin
A toddler is unprovokedly attacked and bitten on the shoulder by a stray dog and suffers a
4.34 slight wound. In addition to flushing the wound, cleaning it surgically, and giving
antitetanus prophylaxis and antibiotics as indicated, the physician should immediately:
A Observe the boy very carefully
B Order a search for the attacking dog for autopsy
C Report the incident to the local police
D Start rabies vaccine
E Start rabies vaccine and give antirabies serum
An FY2 doctor who had previously worked on the infectious diseases ward has been
diagnosed as having tuberculosis. She has been ill for 10 months with symptoms that
4.35 include a productive cough, intermittent fever, night sweats, and has lost nearly 6 kg in
weight. Numerous acid-fast bacilli are seen in a sputum examination, and more than 50
colonies of organisms grow out in culture. In a situation such as this, those contacts who
have a positive skin test but no other signs of disease should:
A Be checked periodically by chest X-ray
B Be immunised with bacillus Calmette–Guérin (BCG) vaccine
C Be vaccinated with purified protein derivative (PPD)
D Receive a full course of isoniazid and ethambutol
E Receive prophylactic isoniazid
4.36 Certain surgical instruments are sterilized with ultraviolet light. Ultraviolet light is used as
an antimicrobial physical agent because it:
A Acts as an alkylating agent
B Causes the formation of pyrimidine dimers
C Disrupts the bacterial cell membrane
D Is a common protein denaturant
E Removes free sulphhydryl groups
4.37 A 45-year-old patient who was receiving chemotherapy for cancer developed a
disseminated varicella-zoster infection. The most likely reason for this viral infection is:
A Deficiency in the third component of complement
B Hypogammaglobulinaemia
C Outgrowth of virus from varicella-zoster immunisation
D Synergism between varicella-zoster and chemotherapy
E T-cell deficiency
A 2-year-old boy was brought to the emergency department with a suspected diagnosis of
4.38 diphtheria. Which of the following modes of pathogenesis is most compatible with
diphtheria?
A Immune complex formation
B Lysis of natural killer T lymphocytes by bacterial envelope enzymes
C Possession of a capsule by the causative agent
D Production of an exotoxin by the causative agent
E Tumour necrosis factor production
4.39 A laboratory test used to identify Staphylococcus aureus is based on the clotting of
plasma. The microbial product that is responsible for this activity is:
A Coagulase
B Coagulase reactive factor
C Plasmin
D Prothrombin
E Thrombin
A 56-year-old diabetic patient in the intensive care unit was commenced on an antifungal
4.40 agent that inhibits the biosynthesis of fungal ergosterol. Which of the following agents was
most likely to have been given to the patient?
A Amphotericin B
B Flucytosine
C Griseofulvin
D Ketoconazole
E Nystatin
SECTION 5:
DISORDERS OF FLUIDS AND ELECTROLYTES —
QUESTIONS
For each question given below choose the ONE BEST option.
A 22-year-old woman was brought to the emergency department with profound nausea
5.1 and vomiting for 24 hours. An arterial blood gas analysis was performed. Which of the
following sets of values is most likely to be obtained?
A pH 6.25, p(CO2) 28 mmHg, HCO3− 15 mmol/l
B pH 7.20, p(CO2) 55 mmHg, HCO3− 27 mmol/l
C pH 7.30, p(CO2) 40 mmHg, HCO3− 24 mmol/l
D pH 7.35, p(CO2) 35 mmHg, HCO3− 28 mmol/l
E pH 7.50, p(CO2) 47 mmHg, HCO3− 35 mmol/l
5.2 A 26-year-old woman with nephrotic syndrome has developed generalised oedema. The
most likely mechanism for oedema in this patient is:
A Decreased colloid osmotic pressure
B Decreased protein synthesis
C Decreased protein intake
D Increased capillary hydrostatic pressure
E Lymphatic obstruction
A 26-year-old man with a 1-week history of severe diarrhoea feels dizzy when he stands
5.3 up. His blood pressure (supine) is 112/76 mmHg with a pulse of 88/minute; blood pressure
(on standing) is 80/60 mmHg with a pulse of 120/minute. In addition to controlling his
diarrhoea, the most appropriate initial therapy is intravenous administration of:
A 5% dextrose in water
B Desmopressin
C Fresh frozen plasma
D Isotonic saline
E Verapamil
A 28-year-old woman has presented in the emergency department with a 3-day history of
5.4 vomiting and diarrhoea. She has postural hypotension and poor tissue turgor. Her serum
sodium concentration is 130 mmol/l. Which of the following findings is most likely to be
seen in this patient?
A Decreased serum aldosterone concentration
B Increased serum atrial natriuretic peptide concentration
C Increased effective circulating volume
D Increased serum ADH (vasopressin) concentration
E Urine osmolality less than the serum osmolality
A 45-year-old diabetic woman was admitted to the emergency ward. Her electrolyte
5.5 report showed: sodium 135 mmol/l, potassium 5 mmol/l, bicarbonate − 10 mmol/l, chloride −
102 mmol/l, pH 7.3, and p(CO2) 40 mmHg. The patient is most likely to have:
A Compensated respiratory acidosis
B Compensated metabolic alkalosis
C Compensated respiratory acidosis
D Respiratory alkalosis
E Wide anion gap metabolic acidosis
One day after an emergency repair of a ruptured abdominal aortic aneurysm, a 64-year-
old man has urine output of 40 ml over a 4-hour period. A Foley catheter is still in place.
He received 14 units of blood during the operation. His temperature is 37.8 °C, blood
5.6 pressure 100/50 mmHg, pulse 126/minute, and central venous pressure (CVP) 3 mmHg.
Examination shows diffuse peripheral oedema. Heart sounds are normal. The lungs are
clear to auscultation. The abdomen is soft. Laboratory studies show: haematocrit 34%,
serum sodium 145 mmol/l, serum potassium 5.0 mmol/l, and urine sodium 6 mmol/l. Which
of the following is the most likely cause of the oliguria?
A Heart failure
B Hypovolaemia
C Occluded Foley catheter
D Renal artery thrombosis
E Transfusion reaction
A 48-year-old woman underwent surgery for an abdominal mass. While still in hospital, on
5.7 the 6th postoperative day, she developed a cardiac problem. Her serum potassium was
markedly elevated. What is the most likely cause for hyperkalaemia in this patient?
A Prolonged dependency
B Prolonged use of antibiotics
C Multiple blood transfusions
D Pulmonary embolism
E Volume overload
5.8 A blood gas analysis report of a 36-year-old patient admitted to hospital shows: pH 7.6,
p(O2) 75 mmHg, p(CO2) 46 mmHg and bicarbonate 44 mmol/l. The most likely
interpretation of this set of values is:
A Metabolic acidosis
B Metabolic alkalosis
C Respiratory acidosis
D Respiratory alkalosis
E Respiratory failure
5.9 A patient with renal failure has a serum potassium of 7 mmol/l. The most likely
electrocardiographic manifestation of this abnormality will be:
A Flattened T waves
B Prolonged QT interval
C ST depression
D Tented T waves
E U waves
5.12 The serum electrolyte analysis of a patient shows hypokalaemia, with a serum potassium
of 3.1 mmol/l. What is the most likely cause of this abnormality?
A Hyperglycaemia
B
C Liddle syndrome
Metabolic acidosis
D Renal failure
E Rhabdomyolysis
5.13 The serum electrolyte analysis of a patient shows hyperkalaemia, with serum potassium of
6.5 mmol/l. Which of the following drugs could be responsible for this abnormality?
A Amphotericin B
B Carbenicillin
C Theophylline
D Thiazide diuretic
E Spironolactone
5.14 A 46-year-old-woman with chronic renal failure has hypermagnesaemia. Which of the
following abnormalities is most likely to be seen in this patient?
A Anorexia
B Generalised seizures
C Respiratory depression
D Tetany
E Vomiting
The serum electrolyte analysis of a patient in the intensive care unit showed
5.15 hyperphosphataemia, with a serum phosphate of more than 1.46 mmol/l. What is the most
likely cause for this abnormality?
A Acute alcoholism
B Hyperparathyroidism
C Refeeding after prolonged malnutrition
D Renal insufficiency
E Severe chronic respiratory alkalosis
5.16 A 26-year-old woman has developed euvolaemic hyponatraemia. Which of the following
conditions is most likely to be associated with this abnormality?
A Diuretic therapy
B Pancreatitis
C Protracted vomiting
D Psychosis
E Salt-losing nephropathy
5.17 A 56-year-old man has developed hypervolaemic hyponatraemia. Which of the following
conditions is most likely to be associated with this abnormality?
A Cirrhosis
B Diuretic therapy
C Pancreatitis
D Protracted vomiting
E Salt-losing nephropathy
A Burns
B Diarrhoea
C Nephrotic syndrome
D Sweating
E Vomiting
5.19 A 26-year-old woman has hypocalcaemia. Which of the following conditions might be
responsible for this electrolyte abnormality?
A Hypoparathyroidism
B Primary hyperparathyroidism
C Prolonged immobilisation
D Secondaries in bone
E Vitamin D toxicity
5.20 What percentage of calcium is available for buffering changes in Ca2+ balance in the
body?
A 1%
B 10%
C 20%
D 50%
E 99%
A 28-year-old patient was seen in the emergency department with metabolic acidosis and a
5.21 decreased anion gap. What is the most likely cause for the decreased anion gap in this
patient?
A Hypoalbuminaemia
B Hypocalcaemia
C Hypomagnesaemia
D Lactic acidosis
E Uraemia
A 36-year-old man was admitted to the emergency ward with metabolic acidosis and
5.22 increased anion gap. What is the most likely cause for the increased anion gap in this
patient?
5.23 Which of the following conditions is associated with the generation of a metabolic alkalosis
due to intracellular shifting of hydrogen ions?
A Chloride diarrhoea
B Hypokalaemia
C Loop diuretic therapy
D Nasogastric suction
E Vomiting
5.24 A 28-year-old woman was admitted through the emergency department with profuse
vomiting. Arterial blood gas analysis suggested severe metabolic alkalosis. In this patient:
A Ammonia production is decreased
B Cerebral perfusion is decreased
C Coronary blood flow is increased
D Diffuse arteriolar vasodilatation increases tissue perfusion
E Serum concentration of ionised calcium is increased
5.25 A 63-year-old woman on long-term total parenteral nutrition was noticed to have
hypophosphataemia. What is the normal plasma level of phosphate in adults?
A 0.5–0.75 mmol/l
B 0.8–1.45 mmol/l
C 1.50–3.0 mmol/l
D 10.0–12.5 mmol/l
E 15.0–17.5 mmol/l
5.27 A 46-year-old woman with chronic renal insufficiency has hyperphosphataemia. Which of
the following is the most likely complication of hyperphosphataemia?
A Depressed cardiac output
B Impaired diaphragmatic contractility
C Metastatic calcification
D Osteomalacia
E Respiratory insufficiency
Ammonia is an important urine-buffering system for secreted hydrogen ions. Which of the
5.28 following electrolyte abnormalities is associated with increased synthesis of ammonia in
the kidneys?
A Hypercalcaemia
B Hyperkalaemia
C Hypernatraemia
D Hypokalaemia
E Hyponatraemia
A 52-year-old woman was admitted into the surgical intensive care unit after a prolonged
5.29 and complex cardiac surgical operation. Her tissue perfusion was poor in the immediate
postoperative period and arterial blood gas analysis showed lactic acidosis. What type of
lactic acidosis did this woman have in the immediate postoperative period?
A Type A
B Type B1
C Type B2
D Type B3
E Type C
5.30 Which of the following conditions is most likely to cause hypervolaemic hypernatraemia?
A Burns
B Central diabetes insipidus
C Diarrhoea
D Hyperalimentation
E Nephrogenic diabetes insipidus
SECTION 6:
BLEEDING and HAEMOSTASIS — QUESTIONS
For each question given below choose the ONE BEST option.
A 26-year-old woman was informed by the haematologist that she had a quantitative as
6.2 well as qualitative platelet defect. Which of the following is typically associated with a
qualitative or quantitative platelet defect?
A Epistaxis
B Haemarthroses
C Normal bleeding time
D Prolonged partial thromboplastin time
E Soft tissue haemorrhages
A 16-year-old girl was seen by the haematologist because she had developed skin and
6.3 mucosal petechiae. Her prothrombin time (PT), partial thromboplastin time (PTT), platelet
count and bleeding time were all normal. What would explain skin and mucosal petechiae
in the presence of a normal PT, PTT, platelet count and bleeding time?
A Aspirin ingestion
B Chronic alcoholism
C Scurvy
D Uraemia
E vonWillebrand’s disease
A 10-year-old boy was referred to the haematologist because he had a bleeding disorder
6.4 characterised by haemarthrosis. On investigation, his prothrombin time was normal, but
the partial thromboplastin time was quite elevated. His platelet count was normal. Which
of the following statements regarding his clinical condition is correct?
A He has an underlying liver disease
B He has had episodes of mucocutaneous bleeding
C His sisters are affected as well
D There is typically no family history in this condition
E Transfusion of factor VIII concentrate is helpful
A 55-year-old man had a preoperative full blood count (FBC) performed prior to elective
right hemicolectomy for a malignant growth in the ascending colon. The FBC revealed:
6.5 haemoglobin 12.1 g/dl, haematocrit 35.8%, mean corpuscular volume (MCV) 90 fl, white
blood cell count 13.5 × 109/l, platelet count of 1000 × 109/l. These findings are most
indicative of:
A A myeloproliferative disorder
B Chronic liver disease
C Hereditary spherocytosis
D Postoperative haemorrhage
E von Willebrand’s disease
A 26-year-old man who has had multiple episodes of deep venous thrombosis presented in
6.6 the emergency department with a suspected pulmonary embolism. Which of the following
blood proteins is most likely to be deficient in this patient?
A Alpha-2-antiplasmin
B Alpha-2-macroglobulin
C Antithrombin III
D Factor V
E Fibrinogen
A 25-year-old man admitted for elective hydrocoele repair had a full blood count that
showed: haemoglobin 14.8 g/dl, haematocrit 45%, mean corpuscular volume 96 fl, white
6.9
blood cell count 8.5 × 109/l and platelet count 275 × 109/l. His prothrombin time was
normal, but the partial thromboplastin time was prolonged to 210 seconds. A deficiency of
which of the following is most likely to be associated with these findings?
A Antithrombin III
B Factor XII
C Protein C
D Protein S
E Vitamin C
A 46-year-old alcoholic with sepsis was admitted into the intensive care unit for
6.10 management of coagulopathy. Which of the following laboratory tests best distinguishes
the coagulopathy of liver disease from disseminated intravascular coagulopathy (DIC)?
A D-Dimer test
B Factor XIII assay
C Partial thromboplastin time
D Platelet count
E Prothrombin time
A 36-year-old woman was admitted through the emergency department with fever and
neurological abnormalities. She had a full blood count which showed: white blood cell count
9.6 × 109/l, haemoglobin 8.4 g/dl, haematocrit 25.9%, mean corpuscular volume 100 fl,
6.11 platelet count 10.0 × 109/l. Her serum haptoglobin was markedly decreased. Her serum
total bilirubin was 5.5 mg/dl, with a direct bilirubin of 5.1 mg/dl. The on-call consultant
decided to perform a plasmapheresis on this woman but she died despite plasmapheresis.
At autopsy, hyaline thrombi were observed in the small arteries of the kidney, heart and
brain. Which of the following diseases did she probably have?
A Alcoholic liver disease
B Malaria
C Systemic lupus erythematosus (SLE)
D Thrombotic thrombocytopenic purpura (TTP)
E von Willebrand’s disease (vWD)
A 12-year-old boy was seen in the orthopaedics outpatient clinic with recurrent
6.12 spontaneous joint haemorrhages. Which of the following laboratory findings are most
likely to be seen in this patient?
A Factor V deficiency
B Factor VIII deficiency
C Platelet count of 100,000
D Positive protamine sulphate test
E Prothrombin time twice normal control
6.13 In a patient who is to undergo elective surgery, the presence of which of the following
findings on preoperative investigations is most often associated with clinical bleeding?
A Factor IX deficiency
B Factor XII deficiency
C High-molecular-weight kininogen deficiency
D Lupus anticoagulant
E Prekallikrein deficiency
A 55-year-old man undergoes an elective direct inguinal hernia repair. During this
procedure there is significant loss of blood, with oozing from small vessels. Preoperative
6.15 coagulation tests would have shown a normal prothrombin time, normal partial
thromboplastin time and normal platelet count, but a prolonged bleeding time. What over-
the-counter drug was the probable cause for these findings?
A Acetaminophen
B Acetylsalicylic acid
C Codeine
D
E Ephedrine
Phenylpropanolamine
A Factor VII
B Factor VIII
C Factor XIII
D Fibrinogen
E Tissue factor
A 65-year-old woman was admitted into the intensive care unit with sepsis originating
from an Escherichi coli urinary tract infection. A full blood count performed on admission
showed: white blood cell count 17.7 × 109/l, haemoglobin 10.5 g/dl, haematocrit 29.7%,
6.17 mean corpuscular volume (MCV) 96 fl, platelet count 45 × 109/l. On the peripheral blood
smear, it was noted that schistocytes were present. The haematologist phoned the on-call
intensive care unit SHO and asked her to send some more blood for a further
investigation. Which of the following tests is the haematologist most likely to perform for
this patient?
A D-Dimer
B Factor VIII
C Reticulocyte count
D Serum ceruloplasmin
E Serum ferritin
A 56-year-old man complaining of vague abdominal pain in the right hypochondrium had an
6.18 abdominal ultrasound scan that revealed a small, nodular liver. His routine blood
investigations revealed only a prolonged prothrombin time. This isolated coagulation
abnormality is most likely to be due to:
A Abnormal fibrinolysis
B A myeloproliferative disorder
C Factor VII deficiency
D von Willebrand’s disease
E Thrombocytopenia
Warfarin is prescribed for a 40-year-old man who underwent aortic valve replacement
6.20 with a mechanical prosthesis as treatment for infective endocarditis of his native aortic
valve. Which of the following statements regarding the use of this medication is correct?
A His anticoagulation should be monitored with the partial thromboplastin time
B If he also takes aspirin, the dosage of warfarin must be decreased
C Renal disease will affect the response to warfarin
D The action of warfarin is immediate after ingestion
E Warfarin acts by inhibiting factor XII
Which of the following laboratory profiles best describes a patient with haemophilia A (PT,
6.21 prothrombin time; PTT, partial thromboplastin time; AHF, antihaemophilic factor;
vWFAg, von Willebrand factor antigen)?
A Increased PT, normal PTT, low factor VIII AHF, decreased vWFAg
B Increased PT, increased PTT, low factor VIII AHF, normal vWFAg
C Normal PT, increased PTT, low factor VIII AHF, normal vWFAg
D Normal PT, increased PTT, low factor IX, normal vWFAg
E Normal PT, increased PTT, low factor VIII AHF, decreased vWFAg
A 72-year-old man with an enlarged prostate and type 2 diabetes has a history of urinary
tract infections. Recently, one of these episodes was complicated by acute pyelonephritis.
He became septic, and a blood culture grew Escherichia coli. He developed severe
6.22 hypotension, with purpuric areas on his skin. A stool for occult blood was positive. He had
a prothrombin time of 50 seconds (control 12 seconds), a partial thromboplastin time of
100 seconds (control 25 seconds), a platelet count of 20 × 109/l and a D-dimer level of 4
µg/ml. These findings are most characteristic of which of the following conditions?
A Acute fulminant hepatitis
B Antiphospholipid syndrome
C Disseminated intravascular coagulation
D Haemophilia A
E von Willebrand’s disease
A 42-year-old woman who was admitted for elective subtotal thyroidectomy gave a history
6.27 of a bleeding disorder. Preoperative evaluation showed a prolonged thrombin clotting time.
What is the most likely cause for this abnormality?
A Factor VIII deficiency
B Factor XII deficiency
C Hypofibrinogenaemia
D von Willebrand factor deficiency
E Warfarin therapy
A 27-year-old man has a deficiency of a coagulation factor which forms a complex with
6.28 tissue factor to activate factors IX and X. Which of the following coagulation factors is
most likely to be deficient in this patient?
A Prothrombin
B Fibrinogen
C Factor VII
D Factor XI
E Factor XII
A 36-year-old woman who was commenced on warfarin following mechanical aortic valve
6.29 replacement for a congenital bicuspid valve developed skin necrosis. Deficiency of which
of the following proteins could be responsible for this disorder?
A Heparin cofactor II
B Plasmin
C Protein C
D Protein S
E Protein Z
6.30 A 22-year-old woman with repeated episodes of deep venous thrombosis was diagnosed
with protein S deficiency. What is the normal role of protein S in the body?
A It activates plasmin
B It activates thrombin
C It functions as a cofactor to protein C
D It inhibits heparin cofactor II
E It inhibits protein C
A 26-year-old woman who bled excessively after a therapeutic abortion was diagnosed as
6.31 having a rare coagulation defect characterised by deficiency of an enzyme of the
coagulation cascade that cross-links fibrin. Which of the following factors was deficient in
this patient?
A Factor II
B Factor VII
C Factor VIII
D Factor XII
E Factor XIII
A 26-year-old patient who bled excessively after excision of a thyroglossal cyst was
6.32 investigated and found to have deficiency of a glycoprotein coagulation factor. Which of
the following factors was deficient in this patient?
A Factor II
B Factor V
C Factor IX
D Factor XII
E Factor XIII
A 35-year-old woman has been diagnosed with deficiency of a protein that functions as a
6.33 cofactor in the thrombin-induced activation of protein C in the anticoagulant pathway.
Which of the following proteins was deficient in this patient?
A Alpha-2-antiplasmin
B Fibronectin
C High-molecular-weight kininogen
D Prekallikrein
E Thrombomodulin
6.34 Heparin is a naturally occurring anticoagulant. Which of the following cell types produces
heparin?
A Endothelial cells
B Eosinophils
C Hepatocytes
D Mast cells
E Platelets
A 48-year-old woman with a mechanical mitral valve who was on long-term warfarin
6.35 therapy was started on intravenous heparin prior to undergoing an elective
cholecystectomy. What is the mechanism of action of heparin?
A Activation of antithrombin III
B Activation of protein C
C Activation of protein S
D Inhibition of fibrinogen degradation
E Inhibition of synthesis of vitamin K-dependent clotting factors
A 36-year-old pregnant woman with deep venous thrombosis was prescribed a low-
6.36 molecular-weight heparin (LMWH) as an outpatient. What is the mechanism of action of
LMWHs?
A Inhibition of antithrombin III
B Inhibition of factor Xa
C Inhibition of fibrinogen degradation
D Inhibition of protein C
E Inhibition of synthesis of vitamin K-dependent clotting factors
A 45-year-old Ashkenazi Jewish man was admitted for elective cholecystectomy and told
6.38 the SHO that he suffers from haemophilia C. Which of the following coagulation factors is
deficient in haemophilia C?
A Factor VIII
B Factor IX
C Factor X
D Factor XI
E Factor XII
A 35-year-old woman admitted for elective excision of a preauricular dermoid cyst told the
6.39 SHO clerking her that she has a coagulation defect. This is known to be due to deficiency
of a coagulation factor which is activated by factor XIIa and is produced in the liver.
Which coagulation factor is deficient in this woman?
A Factor VIII
B Factor IX
C Factor X
D Factor XI
E Factor XIII
A 22-year-old patient with a bleeding tendency was investigated and diagnosed with grey
6.40 platelet syndrome. Which of the following components is deficient in this patients’s
platelets?
A Alpha granules
B Canalicular system
C Dense bodies
D Lysosomes
E Mitochondria
SECTION 7:
CARDIOVASCULAR PATHOLOGY — QUESTIONS
For each question given below choose the ONE BEST option.
A 32-year-old man was seen in the emergency department after a road traffic accident in
7.1 which he sustained fracture to the mid-shaft of the right femur and an unstable fracture of
the pelvis. On arrival in the department, his blood pressure is 80/45 mmHg and his pulse is
184/minute. What type of shock is this patient likely to be in?
A Anaphylactic shock
B Cardiogenic shock
C Hypovolaemic shock
D Neurogenic shock
E Septic shock
A 15-year-old girl with a mitral valve that was repaired in infancy because she had a
congenital mitral cleft was seen in the emergency department 1 month after an attack of
7.2 acute pharyngitis. She had a severe febrile illness, prominent and changing heart murmurs
and petechiae. She was diagnosed with infective endocarditis. In this patient the most
likely pathogenic organism will be:
A Chlamydia
B Haemophilus influenzae
C Staphylococcus aureus
D Streptococcus faecalis
E Streptococcus viridans
7.3 Histological examination of a biopsy specimen from a 55-year-old man with vasculitis
revealed giant-cell arteritis. The patient most likely has:
A Hypersensitivity vasculitis
B Polyarteritis nodosa
C Takayasu’s arteritis
D Temporal arteritis
E Wegener’s granulomatosis
A 65-year-old bedridden man with paraplegia developed acute shortness of breath and a
heart rate of 120/minute. He died suddenly and an autopsy is performed to determine the
7.6 cause of death. His right lung showed an embolus completely obstructing the right
pulmonary artery. Which of the following is the underlying factor most likely to be
responsible for this patient’s death?
A Hypercoagulable state
B Injury to the vascular endothelium
C Release of endotoxin
D Stasis of blood
E Turbulent blood flow
A 32-year-old pregnant woman in her third trimester was severely injured in a road traffic
7.7 accident, with a left-sided femoral shaft fracture and a right-sided tibial shaft fracture. On
arrival in the emergency department she deteriorated and died suddenly. The most likely
cause of her sudden death was:
A Amniotic fluid embolism
B Anaphylaxis
C Antepartum haemorrhage
D Fat embolism
E Spinal cord compression
A 55-year-old man dies from metastatic stomach carcinoma. At autopsy, small vegetations
7.8 are found along the line of closure of the mitral valve. Which of the following is the most
likely diagnosis?
A Acute infectious endocarditis
B Calcific valvular disease
C Carcinoid heart disease
D Marantic endocarditis
E Small mural thrombi
A 36-year-old man who has been a smoker for the last 20 years develops gangrenous toes
on his left foot. His blood pressure is 118/76 mmHg. His serum cholesterol level is 3.5
7.9 mmol/l and his serum glucose is 4.9 mmol/l. The left anterior tibial artery is biopsied, which
shows luminal thrombus and vasculitis. The most likely cause for this patient’s clinical
presentation is:
A Giant-cell arteritis
B Kawasaki’s disease
C Monckeberg’s arteriosclerosis
D Severe atherosclerosis
E Thromboangiitis obliterans
A 48-year-old man in the surgical intensive care unit develops sudden onset of chest pain,
cough, dyspnoea, tachypnoea and marked anxiety. Two days ago, he underwent a right
hemicolectomy for cancer of the ascending colon. An accentuated pulmonary S2 is heard
on auscultation. An electrocardiogram shows non-specific ST-segment and T-wave
7.11
changes. The leukocyte count is 12 × 109/l. An X-ray film of the chest shows no pulmonary
infiltrates and no pleural effusions. Arterial blood gas analysis on room air shows an
arterial p(CO2) of 30 mmHg and an arterial p(O2) of 55 mmHg. Which of the following
conditions is most likely to be responsible for causing the symptoms experienced by this
man?
A Acute pericarditis
B Pericardial tamponade
C Pleuritis
D Pulmonary embolism
E Spontaneous pneumothorax
A 52-year-old man with long-standing hypertension was brought to the emergency
department 30 minutes after the sudden onset of severe chest pain that radiated to his
7.12 back and arms. On arrival, his blood pressure was 180/80 mmHg in his right arm, with no
pressure reading obtainable from the left arm. Cardiac examination elicited a murmur of
aortic insufficiency. Which of the following is the most likely diagnosis?
A Acute aortic dissection
B Acute myocardial infarction
C Embolus to the right subclavian artery
D Pulmonary embolism
E Spontaneous pneumothorax
A 57-year-old man was brought to the emergency department because he had had blood-
tinged sputum for 2 weeks. He had a 6-month history of exertional dyspnoea, especially
when walking uphill or climbing stairs. He takes no medications and does not smoke
cigarettes. He is allergic to penicillin. On examination, his blood pressure is 120/80 mmHg,
7.13 pulse 88/minute and respiratory rate 16/minute. There are crackles at both lung bases and
a diastolic murmur can be heard at the cardiac apex. Electrocardiography shows a broad,
notched P wave in the limb leads. An X-ray film of the chest shows pulmonary vascular
redistribution to the upper lobes of the lungs. Which of the following is the most
appropriate next step in diagnosis?
A Bronchoscopy
B Coronary angiography
C Echocardiography
D Magnetic resonance imaging
E Pulmonary artery catheterisation
A previously healthy, 21-year-old man comes to the emergency department 12 hours after
the onset of chest pain in the precordial area. He has had an upper respiratory tract
7.15 infection for 9 days. A friction rub is heard over the precordium. An electrocardiogram
shows an increase in the J point of all leads except aVR and Vl. After administration of
aspirin, the pain subsides. Which of the following conditions is most likely to be responsible
for this patient’s symptoms?
A Acute pericarditis
B Cardiac tamponade
C Cardiogenic shock
D Mitral valve disease
E Myocarditis
A 65-year-old diabetic man presents to the emergency department with a history of chest
7.16 pain of 12 hours’ duration. This pain has been getting worse for several hours. It is a
substernal, ‘crushing’ type of pain that radiates down the left arm. Which of the following
laboratory tests would it be most important to perform at the time of presentation?
A Antinuclear antibody test
B Creatine kinase-MB
C Erythrocyte sedimentation rate
D Glucose
E White blood cell count
Five years after a cardiac transplant for dilated cardiomyopathy, a 42-year-old woman
7.17 develops worsening congestive heart failure. She has had multiple episodes of rejection,
but a recent endomyocardial biopsy shows no evidence of rejection this time. The most
likely cause of her worsening cardiac function is:
A Amyloidosis
B Constrictive pericarditis
C Coronary atherosclerosis
D Ciclosporin toxicity
E Cytomegalovirus myocarditis
7.18 A baby was born with complete failure of development of the spiral septum in the heart.
He is most likely to have:
A Aortic arch interruption
B Atrioventricular septal defect
C Overriding aorta
D Persistent truncus arteriosus
E Transposition of the great vessels
7.19 A patient was admitted with a mycotic aneurysm. Which of the following conditions is most
likely to be associated with a mycotic aneurysm?
A Disseminated aspergillosis
B Endocarditis with Staphylococcus aureus
C Marfan syndrome
D Metastatic adenocarcinoma
E Polyarteritis nodosa
A 10-year-old boy was born with a large ventricular septal defect (VSD) that was never
7.20 surgically corrected. Now he has increasing dyspnoea with hypoxia and cyanosis. The
reason for these symptoms is:
A Acute myocardial infarction
B Endocardial fibroelastosis
C Left atrial thrombosis
D Natural closure of the VSD
E Reversal of the shunt
SECTION 8:
PULMONARY PATHOLOGY — QUESTIONS
For each question given below choose the ONE BEST option.
8.1 A 15-year-old boy was was found in infancy to have an elevated sweat chloride, indicative
of cystic fibrosis. This puts him at a greater risk for development of:
A Adenocarcinoma of the lung
B Bronchiectasis
C Lymphangiectasis
D Pleural plaques
E Pneumocystis carinii pneumonia
8.2 A patient was admitted through the emergency department with lobar pneumonia. Which
of the following sets of characteristics is most often associated with lobar pneumonia?
A Community-acquired, Mycoplasma infection, Hodgkin’s disease
B Community-acquired, Streptococcus pneumoniae infection, alcoholism
C Congenital, Escherichia coli infection, premature rupture of membranes
D Hospital-acquired, Staphylococcus aureus infection, AIDS
E Hospital-acquired, Klebsiella infection, postoperative
You are asked to review the chest X-ray of an asymptomatic 9-year-old child. The X-ray
8.3 shows a subpleural nodule in the right mid-lung field and enlarged mediastinal lymph nodes.
After looking at the X-ray you feel that the child most probably has:
A Aspergillosis
B Coccidiodomycosis
C Primary tuberculosis
D Secondary tuberculosis
E Miliary tuberculosis
You are informed by the nurse in charge of the surgical high dependency unit that a 68-
8.4 year-old woman on postoperative day 10 walked to the bathroom, but on returning to bed
became extremely dyspnoeic and diaphoretic. You should strongly suspect:
A Pleural effusion
B Pneumonia
C Postoperative atelectasis
D Pulmonary embolus
E Pulmonary oedema
While doing a ward round in the intensive care unit you are asked to review a chest X-ray
8.6 of a 78-year-old man who was admitted unconscious 3 days ago after a stroke. The X-ray
shows a 4-cm-diameter mass lesion with an air–fluid level in the right lung. The left lung is
normal. The chest X-ray appearance is most strongly suggestive of:
A Lung abscess
B Bronchiectasis
C Bronchopulmonary sequestration
D Septicaemia
E Squamous cell carcinoma
While doing the surgical outpatient clinic you come across a 68-year-old man who is
complaining of increasing dyspnoea over the past year. He has never smoked. On further
8.7 enquiry, he divulges that he worked for 5 years in a shipyard in the 1950s. His chest X-ray
reveals diaphragmatic pleural plaques and interstitial lung disease. The clinical picture is
suggestive of:
A Asbestosis
B Berylliosis
C Byssinosis
D Silicosis
E Siderosis
A 56-year-old male smoker complaining of persistent dry cough had a chest X-ray which
showed a 1.5-cm mass lesion in the left upper lung. Bronchoscopy revealed this mass lesion
8.8 to be involving the left superior segmental bronchus. Biopsies of the mass were sent for
histopathological examination. The histopathology report described polygonal pink cells
with dark, angular nuclei. The most likely diagnosis is:
A Adenocarcinoma
B Bronchial carcinoid
C Mesothelioma
D Small cell anaplastic carcinoma
A 35- year-old man who had been involved in a serious road traffic accident had a large
flail segment involving his right chest. He was intubated and placed on a ventilator in the
intensive care unit. He required multiple blood transfusions and external fixation of his
8.9 right femur and pelvis. On day 5 he became increasingly difficult to oxygenate despite
ventilation with positive end-expiratory pressure (PEEP) and an Fi(O2) of 100%. He
remained afebrile. His chest X-ray revealed bilateral fluffy deposits. He died several days
later. At autopsy, the lung showed hyaline membranes, thickened alveolar walls, and type
II pneumocyte proliferation. This man had:
A Adult respiratory distress syndrome
B Bronchopneumonia
C Bronchiectasis
D Chronic bronchitis
E Viral pneumonia
A 35-year-old woman with fever, weight loss and progressively increasing shortness of
breath of 6 weeks’ duration had a chest X-ray which showed prominent hilar
8.11 lymphadenopathy and diffuse pulmonary interstitial disease. She had a video-assisted
thoracoscopic lung biopsy. Microscopic examination of the lung biopsy showed
noncaseating granulomas. She is most likely to have:
A Asbestosis
B Interstitial pneumonitis
C Sarcoidosis
D Silicosis
E Tuberculosis
A 65-year-old man, a heavy smoker, presented with chronic cough, weight loss and two
episodes of haemoptysis over the last 2 months. He has developed truncal obesity, easy
8.12
bruising and osteoporosis. Which of the following pulmonary diseases is most likely to be
the cause of these findings?
A Bronchial carcinoid
B Bronchioloalveolar carcinoma
C Large-cell anaplastic carcinoma
D Small-cell anaplastic carcinoma
E Squamous cell carcinoma
8.13 Of the neoplasms that involve the lung, the most common are:
A Bronchial adenomas
B Pulmonary metastases
C Peripheral adenocarcinomas in non-smokers
D Small-cell anaplastic carcinomas in smokers
E Squamous cell carcinomas in smokers
8.14 A 55-year-old man has a serous pleural effusion. Which of the following conditions is most
likely to be associated with a serous pleural effusion?
A Bronchogenic carcinoma
B Congestive heart failure
C Metastatic carcinoma
D Pulmonary infarction
E Tuberculosis
A 55-year-old man has smoked two packs of cigarettes a day for many years. He has had a
8.16 worsening cough for the past few months. In the past couple of weeks he has occasionally
noted blood-tinged sputum. Otherwise, he has no major health problems. Which of the
following procedures should be done first to begin the investigation of his disease?
A Sputum cytology
B Bronchoalveolar lavage
C Fine-needle aspiration
D Pleural fluid cytology
E Arterial blood gases
A 50-year-old heart transplant recipient has recently been treated with increasing dosages
of imunosuppressive medications because an endomyocardial biopsy demonstrated
8.17 moderate acute rejection. About 1 month later, he is diagnosed with a chronic abscessing
pneumonia that is involving the right middle lobe. Which of the following organisms is most
likely to be causing this?
A Candida albicans
B Cytomegalovirus
C Nocardia asteroides
D Pneumocystis carinii
E Streptococcus pneumoniae
While obtaining informed consent for a fine-needle aspiration of the right lung to obtain a
8.18 tissue diagnosis of a lower-lobe mass in a 58-year-old man, the radiologist tells you that
the most common problem associated with this procedure is:
A Chronic pain after the procedure
B Empyema
C Haemothorax
D Inability to determine the cell type of a malignancy
E Pneumothorax
A chest X-ray reveals a 3-cm, right upper lobe peripheral coin lesion in a healthy,
8.20 asymptomatic 44-year-old male non-smoker. Which of the following conditions is most
likely to be responsible for this radiographic appearance?
A Bronchial carcinoid
B Mesothelioma
C Metastatic adenocarcinoma
D Large-cell undifferentiated carcinoma
E Pulmonary hamartoma
SECTION 9:
RENAL PATHOLOGY — QUESTIONS
For each question given below choose the ONE BEST option.
A 52-year-old man with a long history of smoking has atypical epithelial cells in a urinalysis
9.1 specimen, but cystoscopy is negative. Which of the following conditions is most likely to be
associated with these findings?
A Acute interstitial nephritis
B Adenocarcinoma of prostate
C Nodular glomerulosclerosis
D Squamous cell carcinoma of penis
E Transitional cell carcinoma of renal pelvis
A 60 year-old-man is diagnosed with chronic renal failure. The findings from urine dipstick
9.2 testing show no protein, blood, glucose, nitrite or ketones. However, the semi-quantitative
sulphosalicylic acid test for urine protein is positive. Which of the following conditions is
most likely to have caused the chronic renal failure in this patient?
A Diabetes mellitus
B Membranous glomerulonephritis
C Minimal-change disease
D Multiple myeloma
E Systemic lupus erythematosus
9.3 Which of the following is lacking in the renal cortex and medulla if you are looking at a
slide of kidney under the microscope?
A Capillaries
B Fenestrated endothelium
C Henle’s loop
D Squamous epithelium
E Type IV collagen in glomerular basement membrane
9.4 Which of the following findings will enable you to make a definite diagnosis of nephrotic
syndrome?
A Haematuria, with >10 red blood cells/high-power field
B Lipiduria in association with hypercholesterolaemia
C No evidence of inflammation in a urinalysis specimen
D Proteinuria of >3 g/24 hours
E Renal tubular epithelial cells and casts
9.5 Which of the following renal functions will be assessed if you are measuring the urine
specific gravity?
A Blood flow
B Concentration
C Filtration
D Reabsorbtion
E Secretion
Which of the following patients is most likely to have a hyperplastic arteriolosclerosis with
9.6 fibrinoid necrosis, petechial haemorrhages, and microinfarcts in the kidneys, in conjunction
with a markedly elevated plasma renin?
A A 56-year-old man with an acute myocardial infarction
B A 6-year-old boy with albuminuria
C A 62-year-old woman with end-stage renal disease
D A 15-year-old man with recent streptococcal infection
E A 45-year-old woman with scleroderma
A patient had nephrectomy for a neoplasm involving his right kidney. Microscopically, the
9.7 tumour resembled an embryonic nephrogenic zone and the pathologist suggested that it
might have resulted from a lack of a tumour suppressor gene on chromosome 11. Which of
the following neoplasms was the pathologist most likely to be suggesting?
A Angiomyolipoma
B Medullary fibroma
C Renal cell carcinoma
D Transitional cell carcinoma
E Wilms’ tumour
A 9-year-old girl is noted by her mother to have been lethargic for several weeks, and she
appears to have some puffiness around her eyes. Dipstick urinalysis reveals no glucose,
9.8 ketones or blood, but she has 4+ of protein. Microscopic urinalysis reveals no casts, but
oval fat bodies are seen. Which of the following conditions is most likely to be responsible
for this child’s clinical features?
A Berger’s disease
B Goodpasture syndrome
C Focal segmental glomerulosclerosis
D Membranoproliferative glomerulonephritis
E Minimal-change disease
A 46-year-old man with a history of chronic alcoholism was admitted in the emergency
9.9 department after drinking a litre of antifreeze solution containing ethylene glycol. Which
of the following conditions is he most likely to develop?
A Acute interstitial nephritis
B Acute pyelonephritis
C Acute tubular necrosis
D Chronic interstitial nephritis
E Rapidly progressive glomerulonephritis
If you are an SHO on the renal unit and are asked to do a percutaneous needle biopsy of
9.10 the kidney, which of the following situations do you think will be most appropriate for
performing this investigation?
A Fever with suspected acute pyelonephritis
B Prostatic hyperplasia with suspected hydronephrosis
C Suspected polycystic kidney disease
D Suspected renal cyst
E Systemic lupus erythematosus and acute renal failure
A Adenocarcinoma of prostate
B Renal cell carcinoma
C Squamous cell carcinoma of penis
D Transitional cell carcinoma
E Wilms’ tumour of the kidney
9.12 A 36-year-old man has been diagnosed with rapidly progressive glomerulonephritis. Which
of the following features is most characteristic of this condition?
A Glomerular crescents
B IgA deposited in glomerular capillaries
C Lipiduria
D Polymorphonuclear infiltrates
E Widened proximal tubules
A 45-year-old man visits his GP after feeling generally unwell and lethargic for a couple of
weeks. His clinical examination was normal, except for a blood pressure of 150/95 mmHg.
9.13 Dipstick urinalysis showed no glucose, blood, ketones, nitrite or urobilinogen, but he had
4+ of proteinuria, and a 24-hour urine protein was 3.8 g. Microscopic urinalysis showed no
casts. Which of the following conditions is most likely to be responsible for these findings?
A Amyloidosis
B Membranous glomerulonephritis
C Minimal-change disease
D Post-streptococcal glomerulonephitis
E Systemic lupus erythematosus
A 62-year-old man was admitted to the intensive care unit after a prolonged operation to
repair an abdominal aortic aneurysm. He had been hypotensive throughout the operation
9.14 and for several hours afterwards. His serum urea and creatinine were noted to be
increasing. Granular and hyaline casts were present on microscopic urinalysis. Which of
the following renal lesions is most likely to be present in this patient?
A Acute tubular necrosis
B Chronic pyelonephritis
C Minimal-change disease
D Nodular glomerulosclerosis
E Renal vein thrombosis
A 68-year-old man who had been feeling unwell and pyrexial for 10 days was seen in the
emergency department. He described dull pain on palpation of his left lower back. He had
9.15 burning dysuria. A full blood count revealed an elevated white blood cell count with a left
shift. Which of the following urinalysis findings would be most diagnostic for his renal
condition?
A Broad renal casts
B Oval fat bodies
C Proteinuria
D Renal tubular epithelial cells
E White blood cell casts
A 38-year-old man had sudden-onset, severe, right flank pain that came in waves all night
long. When he was seen in the emergency department (after waiting for 2 hours) he was
9.16 exhausted. His urine specimen was examined by dipstick and this revealed no ketones,
glucose, protein, nitrite or urobilinogen; the urine contained blood, but few white blood
cells. The specific gravity of the urine was 1.015 and the pH was 5.5. The most likely
diagnosis is:
A Benign prostatic hyperplasia
B Membranous glomerulonephritis
C Renal angiomyolipoma
D Transitional cell carcinoma of bladder
E Ureteric calculus
9.17 Which of the following congenital anomalies of the urinary tract carries the greatest
significance in terms of morbidity?
A Bilateral ureteral duplication
B Bladder exstrophy
C Horseshoe kidney
D Medullary sponge kidney
E Unilateral renal agenesis
9.18 Which of the following organisms is most frequently responsible for non-gonoccocal
urethritis in a young, sexually active male?
A Chlamydia
B Haemophilus
C Herpes simplex virus
D Human papilloma virus
E Treponema
A patient with renal problems had a urinalysis which showed oval fat bodies. Which of the
9.19 following conditions is most likely to be associated with presence of oval fat bodies on
urinalysis?
A Ascending pyelonephritis
B Nephritic syndrome
C Nephrotic syndrome
D Obstructive uropathy
E Renal infarction
A young man was admitted through the emergency department after passing bloodstained
9.20 urine. The urine dipstick test for blood was positive but no red blood cells were seen on
urine sediment microscopy. Which of the following conditions is most likely to be
associated with these findings?
A Myoglobinuria
B Post-streptococcal glomerulonephritis
C Renal infarction
D Renal papillary necrosis
E Ureteric lithiasis
SECTION 10:
GASTROINTESTINAL AND HEPATOBILIARY PATHOLOGY
— QUESTIONS
For each question given below choose the ONE BEST option.
An SHO working in a surgical ward developed hepatitis, which resolved in 2 weeks. His
10.1 blood tests showed that he had hepatitis A. Six months after this episode a liver biopsy is
performed for this SHO. What is it most likely to show?
A Central necrosis
B Chronic septal fibrosis
C Lobular fibrosis
D Normal architecture
E Periportal fibrosis
10.2 A histopathology report of a liver biopsy taken from a patient mentions the presence of
Mallory bodies. Mallory bodies are characteristically present in:
A Alcoholic hepatitis
B Alcoholic fatty liver
C Hepatitis B
D Hepatocellular carcinoma
E Primary biliary cirrhosis
A 16-year-old boy was brought to the emergency department complaining of pain in the
10.5 abdomen that started from the umbilical region and later shifted to the right lower
abdomen. He also had anorexia, nausea and fever. Which of the following findings is most
likely to be present on investigation?
A Anaemia
B Gas shadow in the right iliac fossa on erect abdominal X-ray
C Increased haematocrit
D Leucocytosis
E Microscopic haematuria
A rectal polyp was excised from a patient and sent for biopsy. At the follow-up outpatient
10.6 clinic appointment the surgeon told the patient: ‘It has no tendency to turn malignant,
and there is no need to do any further surgery.’ What is the most likely type of polyp that
was excised?
A Adenomatous polyp
B Hyperplastic polyp
C Tubular adenoma
D Tubulovillous adenoma
E Villous adenoma
A 2-week-old baby was brought to the emergency department by his mother. She
reported that the infant has not passed stools for almost 4 days. Clinical examination
10.7 revealed a crying baby with a distended and diffusely tender abdomen. Bowel sounds
were absent. Abdominal X-ray showed a distended colon. A sigmoid colon biopsy was
unremarkable except for a lack of mural ganglion cells. Which of the following is the most
likely diagnosis?
A Chagas’ disease
B Congenital pyloric stenosis
C Cystic fibrosis
D Hirschsprung’s disease
E Rectal atresia
A 25-year-old man with jaundice had laboratory investigations performed which showed:
10.8 total bilirubin 7.0 mg/dl, direct bilirubin 1.2 mg/dl and anaemia. What is the most likely
cause of his jaundice?
A Fibrosis of the common bile duct
B Haemolysis
C Hepatitis
D Sclerosing cholangitis
E Schistosomiasis
A 43-year-old man has developed sclerosing cholangitis and iron deficiency anaemia
10.10 secondary to chronic bloody diarrhoea. A colonic biopsy would be most likely to show
which of the following?
A Colonic adenocarcinoma
B Diverticulitis
C Granulomatous inflammation
D Pseudopolyps
E Villous adenoma
A 58-year-old man had an adenocarcinoma of the colon that was surgically resected.
10.11 Which of the following features of the tumour’s growth most strongly suggest a poor
prognosis?
A Circumferential growth
B Extension to the muscularis mucosa
C Extension to the serosa
D Polypoid growth
E Surface growth
A farm worker was seen in the surgical outpatient clinic complaining of right
10.12 hypochondrial pain. Clinical examination revealed an enlarged liver with ascites. Analysis
of the ascitic fluid revealed malignant cells. Hepatocellular carcinoma was diagnosed.
Which of the following is the most likely cause of this tumour?
A Aflatoxin
B Aromatic amines
C Hydrocarbons
D Oestrogens
E Saccharin
A 60-year-old man is noted to have mild jaundice and some weight loss. His alkaline
10.13 phosphatase is very high. He has been passing very pale stools. The most likely diagnosis
is:
A Acute viral hepatitis
B Cirrhosis of liver
C Gilbert’s syndrome
D Haemolysis secondary to G6PD deficiency
E Pancreatic carcinoma
A 38-year-old man with ulcerative colitis develops pruritus and fatigue. His alkaline
10.15 phosphatase is elevated. The biliary tree appears to have a ‘beaded’ appearance on
barium radiography. Which of the following is the most likely diagnosis?
A Acute cholecystitis
B Chronic cholelithiasis
C Cholesterosis
D Gallstone ileus
E Sclerosing cholangitis
You are asked to examine a 42-year-old man in the surgical outpatient clinic who gives a
10.16 history of pain in the upper central abdomen that occurs 2–3 hours after meals. He also
tells you that he often wakes up during the night with a similar pain. Based on this
history, you feel that the most likely cause of the pain is:
A Duodenal ulcer
B Gastritis
C Gastric ulcer
D Pyloric stenosis
E Zollinger–Ellison syndrome
A 44-year-old woman notes that her fingers turn blue on exposure to cold, and that this
has been happening increasingly over the past 2 years. She also has difficulty with fine
movement of her fingers because the skin has become increasingly taut. Her face has
10.17 become mask-like, and she has been gradually losing weight. Laboratory findings include
a positive anticentromere antibody, a negative antinuclear antibody and a negative
rheumatoid factor. Which of the following pathological conditions is she most likely to
have?
A Acute pancreatitis
B Chronic cholecystitis
C Ischaemic enteritis
D Micronodular cirrhosis
E Oesophageal stricture
A 48-year-old woman has noted increasing abdominal enlargement for several months.
Physical examination reveals no abdominal tenderness, but a fluid thrill is present. A
10.18 paracentesis is performed and 2 litres of clear, yellow ascitic fluid is removed. This fluid
has a protein of 2.1 g/dl. Cytologically, the fluid contains a few mesothelial cells and a few
mononuclear cells. Which of the following underlying conditions is she is most likely to
have?
A Acute pancreatitis
B Colonic adenocarcinoma
C Crohn’s disease
D Micronodular cirrhosis
E Perforated gastric ulcer
A 21-year-old man was seen in the surgical outpatient clinic complaining of lethargy and
generalised weakness over a period of several weeks. Investigations showed:
haemoglobin 9 g/dl, haematocrit 26.3%, mean corpuscular volume (MCV) 72 fl, platelet
10.19 count 189 × 109/l, and white blood cell count 7.5 × 109/l. On physical examination, there
was no tenderness and no masses; bowel sounds were present. His stool was positive for
occult blood. A small-bowel series with barium enema revealed no masses or
perforations, only a solitary, 2-cm outpouching in the ileum. The presence of which of the
following is most likely to have led to these findings?
A Antiphospholipid antibody
B Elaboration of enterotoxin by Escherichia coli
C Inheritance of a faulty APC gene
You are examining a 55-year-old man in the emergency department who has presented
with a 1-day history of increasing abdominal pain. He also has abdominal distension.
While examining him you notice that he has diffuse abdominal pain. An old, right lower
10.20 quadrant, 8-cm transverse scar is noted. Bowel sounds are high-pitched, faint and
sporadic. A stool sample is negative for occult blood. A plain abdominal X-ray reveals
dilated loops of bowel with air-fluid levels but no free air. Which of the following is the
most likely predisposing factor with regard to his current condition?
A Adhesions from previous surgery
B Chronic persistent hepatitis
C Entamoeba histolytica infection
D Ileal adenocarcinoma
E Meckel’s diverticulum
SECTION 11:
HAEMATOPATHOLOGY — QUESTIONS
For each question given below choose the ONE BEST option.
A 68-year-old man admitted for elective inguinal hernia repair was found to have the
following haematology results: haemoglobin 12.8 g/dl, haematocrit 36.9%, mean
11.3 corpuscular volume (MCV) 88 fl, platelet count 179 × 109/l, and white blood cell count
30.5 × 109/l. The peripheral blood smear showed numerous small, mature lymphocytes.
He is most likely to have:
A Acute lymphocytic leukaemia
B Chronic lymphocytic leukaemia
C Cytomegalovirus infection
D Infectious mononucleosis
E Leukaemoid reaction
A 45-year-old woman who was admitted for elective cholecystectomy had a full blood
count, with the following results: haemoglobin 9.5 g/dl, haematocrit 28.1%, mean
11.4 corpuscular volume (MCV) 134 fl. The reticulocyte index was low. Hypersegmented
polymorphonuclear neutrophils were seen on the peripheral blood smear. On further
enquiry, she mentioned that she had been feeling tired for months. Which of the following
tests should be ordered next?
A Bone marrow biopsy
B Haemoglobin electrophoresis
C HIV antibody test
D Serum vitamin B12 and folate
E Serum ferritin
An 18-year-old university student was seen in the emergency department with fever,
11.5 lymphadenopathy and mild scleral icterus. He had a positive Monospot test. Which of the
following will be the most characteristic peripheral blood finding in this patient?
A Atypical lymphocytosis
B Eosinophilia
C Increased band neutrophils
D Mild thrombocytopenia
E Normocytic anaemia
A 72-year-old man has had chronic back pain for several months. He has lost some
weight. He is not febrile. A full blood count is performed and shows a white blood cell
count of 9.8 × 109/l, with a differential count of 63 polymorphonuclear leukocytes, 7
11.6 bands, 2 metamyelocytes, 2 myelocytes, 18 lymphocytes, 8 monocytes, and 4 nucleated
red blood cells. The haemoglobin is 12.2 g/dl with a haematocrit of 37.1%, a mean
corpuscular volume (MCV) of 84 fl, and a platelet count of 124 × 109/l. Which of the
following conditions is most likely to be associated with these findings?
A Chronic lymphocytic leukaemia
B Haemolytic anaemia
C Metastatic carcinoma
D Previous splenectomy
E Staphylococcus aureus osteomyelitis
11.7 In which of the following patients would you NOT be able to palpate the spleen tip?
A 42-year-old woman is seen in the surgical outpatient clinic with a mass in the left
hypochondrium. On clinical examination, it is discovered that the mass is actually a
palpable spleen tip. Her haematological investigations show: haemoglobin 22.3 g/dl,
11.8 haematocrit 65.7%, mean corpuscular volume (MCV) 89 fl, mean cell haemoglobin
concentration (MCHC) 33.9 g/dl, mean corpuscular haemoglobin (MCH) 34.2 pg, platelet
count 445 × 109/l, and white blood cell count 13.5 × 109/l, with 80% polymorphonuclear
leukocytes, 5% bands, 3% monocytes, and 7% lymphocytes. Which of the following is the
most likely diagnosis?
A Chronic myelogenous leukaemia
B Dehydration
C Epstein–Barr virus infection
D Miliary tuberculosis
E Polycythaemia vera
A lymph node biopsy was performed on a 24-year-old FY1 doctor who presented with
low-grade fevers, night sweats and generalised malaise for a couple of months. He was
11.9 found to have cervical and supraclavicular lymphadenopathy. The nodes were non-
tender. The microscopic appearance at high magnification demonstrated the presence of
Reed–Sternberg cells. Which of the following conditions is the most probable diagnosis in
this doctor?
A Burkitt’s lymphoma
B Cat scratch disease
C Hodgkin’s lymphoma
D Multiple myeloma
E Mycosis fungoides
A 40-year-old patient admitted for elective thyroidectomy has a white blood cell count of
11.10
55 × 109/l and 4% blasts. The leukocyte alkaline phosphatase (LAP) is elevated. This is
most consistent with:
A Acute myelogenous leukaemia
B Chronic lymphocytic leukaemia
C Chronic myelogenous leukaemia
D Infectious mononucleosis
E Leukaemoid reaction
A 52-year-old woman with chronic abdominal pain was noticed to be pale at the time of
11.11 assessment in the surgical outpatient clinic. Investigations showed that she had a
decreased serum iron and total iron-binding capacity (TIBC) in association with an
increased serum ferritin. These findings are most indicative of:
A Anaemia of chronic disease
B Autoimmune haemolytic anaemia
C Chronic blood loss
D Malabsorption
E Vitamin B12 deficiency
A 65-year-old woman has several non-tender, movable lymph nodes palpable in both the
11.12 neck and axillae. Biopsy shows numerous crowded follicles of small, monomorphic
lymphocytes without any Reed–Sternberg cells. This is most typical of:
A Chronic lymphocytic leukaemia
B Hodgkin’s disease, lymphocyte predominance type
C Infectious mononucleosis
D Poorly differentiated lymphocytic lymphoma
E Reactive hyperplasia
A 36-year-old man is seen in the surgical outpatient clinic with massive splenomegaly
11.13 (3000 g). Which of the following conditions is most likely to have caused the massive
splenomegaly in this patient?
A Haemochromatosis
B Infectious mononucleosis
C Myelofibrosis
D Portal hypertension
E Sickle cell anaemia
A 42-year-old woman presented with a 1-week history of fever and mental confusion.
Physical examination revealed widespread petechiae of the skin and mucosal surfaces.
11.14 The urea and creatinine were elevated. She had marked thrombocytopenia, but
deteriorated rapidly following platelet transfusion and died. At autopsy, pink hyaline
thrombi were found in small myocardial arteries. Which of the following disorders is most
likely to be responsible for this woman’s condition?
A Disseminated intravascular coagulopathy
B Idiopathic thrombocytopenic purpura
C Thrombotic thrombocytopenic purpura
D Trousseau syndrome
E Warm autoimmune haemolytic anaemia
An 18-year-old man who was admitted for elective thyroglossal cyst excision was noted
11.15 to have hypochromic anaemia in association with splenomegaly and haemochromatosis on
preoperative evaluation. Which of the following conditions is he most likely to have?
A β-Thalassaemia
B G6PD deficiency
C Hereditary spherocytosis
D Malaria
E Sickle cell anaemia
11.16 A 38-year-old woman has been diagnosed with atrophic gastritis. Which of the following
findings is most likely to be found in this patient?
A Decreased mean corpuscular volume in the red blood cells
B Decreased serum ferritin
C Decreased serum folate
D Increased neutrophil segmentation
E Increased reticulocyte count
11.17 A 65-year-old woman has bone pain, renal failure and pneumococcal pneumonia. In this
setting, a bone marrow biopsy is most likely to show numerous:
A Blasts
B Granulomas
C Plasma cells
D Reed–Sternberg cells
E Small mature lymphocytes
A 55-year-old man who is admitted for elective resection of a colonic carcinoma has a
11.19 haemoglobin level of 9.2 g/dl, a white blood cell count of 1.5 × 109/l and a mean
corpuscular volume (MCV) of 132 fl. Which of the following will be most prominent on
this patient’s peripheral blood smear?
A Blasts
B Hypersegmented neutrophils
C Hypochromic microcytic red blood cells
D Nucleated red blood cells
E Schistocytes
11.20 A 52-year-old man has skin infiltration by neoplastic T lymphocytes. His condition is
known as:
A Acute lymphocytic leukaemia
B Burkitt’s lymphoma
C Hodgkin’s disease
D Mycosis fungoides
E Hairy cell leukaemia
SECTION 12:
ENDOCRINE PATHOLOGY — QUESTIONS
For each question given below choose the ONE BEST option.
A 52-year-old woman who presented with a lump in her neck was seen in the surgical
12.1 outpatient clinic. Examination of the lump revealed it to be a solitary nodule in the
thyroid gland. A fine-needle aspiration of this lump was reported as medullary carcinoma.
Which tumour-associated marker is most likely to be elevated in her serum?
A Alpha-fetoprotein
B CA15-3
C CA-125
D Calcitonin
E Human chorionic gonadotrophin
A 32-year-old woman with a solitary lump in the thyroid gland undergoes fine-needle
12.2 aspiration of this lump, and the results were reported as papillary carcinoma. Which of
the following conditions is most likely to be associated with papillary carcinoma of the
thyroid gland?
A Endemic goitrous area
B Exposure to ionising radiation in childhood
C Follicular adenoma
D Hashimoto’s thyroiditis
E Riedel’s thyroiditis
A previously fit and healthy 32-year-old female lawyer consulted her GP because of
nervousness, tremors, emotional lability and excessive sweating for 3 weeks. She told him
12.5 that she had lost nearly 3 kg during this period in spite of having an increased appetite.
Her blood pressure was 120/80 mmHg, pulse 95/minute and respiratory rate 12/minute.
She had warm and moist skin, a fine tremor of the fingers and tongue and hyperreflexia.
What is the most likely diagnosis?
A Alcohol withdrawl
B Hyperthyroidism
C Hypoglycaemia
D Major depressive disorder
E Phaeochromocytoma
A 45-year-old man was brought to the emergency department after a road traffic
accident. He complained of tenderness in the right lower quadrant of the abdomen. On
clinical examination, no injuries were noted externally and he had faint bowel sounds.
12.7 Thinking of blunt abdominal trauma, the on-call consultant requested a computed
tomographic scan of the chest, abdomen and pelvis, which was essentially normal except
for a 1-cm left adrenal cortical mass. No other abnormality was detected on laboratory
tests. A decision was made to leave the mass alone on this occasion because it was most
likely to be a:
A Haematoma
B Histoplasma granuloma
C Metastasis from a lung carcinoma
D Non-functioning adrenal adenoma
E Simple cyst
A 52-year-old man with newly diagnosed hypertension was investigated by his GP. His
routine laboratory investigations showed: sodium 145 mmol/l, potassium 2.9 mmol/l,
12.8 chloride 107 mmol/l and bicarbonate 26 mmol/l. His blood glucose was 5.7 mmol/l. He felt
fine otherwise and a physical examination revealed no abnormal findings. Which of the
following conditions is most likely to be responsible for this patient’s hypertension?
A Congenital adrenal hyperplasia
B Conn syndrome
C Cushing syndrome
D Multiple endocrine neoplasia, type IIA
E Nelson syndrome
12.9 A 45-year-old woman has been diagnosed with subacute granulomatous thyroiditis. Which
of the following statements would NOT be true of this condition?
A An influenza virus infection of the lung preceded this lesion
B Psammoma bodies are a common histological feature
C The course of this disease can run for only 3 months
D The patient presented with an enlarged, painful thyroid
E The patient was hyperthyroid at the time of initial presentation
A 13-year-old girl with a 5-month history of headaches had a computed tomographic scan
12.10 of the head that revealed a 4-cm suprasellar mass with calcifications that is eroding the
bone of the surrounding sella turcica. No abnormal findings were present on physical
examination. Which of the following is this lesion most likely to be:
A ACTH-secreting pituitary adenoma
B Astrocytoma
C Craniopharyngioma
D Null cell adenoma
E Prolactinoma
A 15-year-old boy was brought into the emergency department with features suggestive
12.11 of acute adrenocortical insufficiency. According to his parents, he had a high fever for
only 24 hours prior to becoming unconscious. On clinical examination, his skin showed
extensive purpura. The most likely cause of these clinical features is:
A Amyloidosis
B Idiopathic adrenalitis
C Meningococcaemia
D Tuberculosis
E Adrenal cortex destruction by tumour
A 49-year-old man presented in the orthopaedic outpatient clinic with an X-ray that
showed a compressed fracture at T10. He complained of increasing weakness over the
12.13 past few months. Clinical examination revealed a blood pressure of 165/110 mmHg.
Laboratory findings included a serum glucose of 8.6 mmol/l. He was mildly obese and had
purplish striae on his thighs and abdomen. The pathological lesion most likely to explain
these findings is:
A Adrenal cortical carcinoma
B Anaplastic thyroid carcinoma
C Multinodular goitre
D Parathyroid adenoma
E Phaeochromocytoma
You were asked to do the endocrine surgery outpatient clinic by your consultant and on
12.14 that particular day you only saw patients with endocrine malignancies. Based on your
knowledge about the natural history of various endocrine malignancies, which of the
following neoplasms has the best prognosis?
A Adrenal cortical carcinoma
B Anaplastic carcinoma of the thyroid
C Follicular carcinoma of the thyroid
D Papillary carcinoma of the thyroid
E Parathyroid carcinoma
12.16 Which of the following conditions is most likely to develop over time in a 66-year-old man
with type 2 diabetes and chronic renal failure?
A Addison’s disease
B Cushing syndrome
C Hypopituitarism
D Primary hyperparathyroidism
E Secondary hyperparathyroidism
A 45-year-old, full-time typist who is suffering from a constant headache consulted his
12.17 GP. Clinical examination revealed a bitemporal hemianopia. Which of the following
conditions is the most likely cause of his visual field defect?
A Craniopharyngioma
B Empty sella syndrome
C Metastatic carcinoma
D Prolactinoma
E Sheehan syndrome
A 48-year-old male smoker presented to his GP complaining of chronic dry cough and
backache. The GP noticed truncal obesity, muscular weakness, hypertension, purplish
12.18 abdominal striae and tenderness in the region of the lower thoracic spine. The patient
was taking no medications. Which of the following diseases is most likely to be the cause
for these findings?
A 21-hydroxylase enzyme deficiency
B Extra-adrenal paraganglioma
C Multiple endocrine neoplasia, type I
D Small-cell anaplastic (oat cell) carcinoma
E Tuberculosis
A 42-year-old woman has just been transferred from the operating theatre to the ward
12.19 after a total thyroidectomy. If you were asked to write the postoperative orders for this
patient, which of the following laboratory tests will you request in the early postoperative
period (the first day) to help you to manage her?
A Antithyroglobulin antibody
B Serum calcium
C Free catecholamines
D Serum iodine
E Thyroid-stimulating hormone
A 16-year-old girl presented in the surgical outpatient clinic with a mobile, 2-cm mass in
12.20 the midline of her neck. A fine-needle aspiration of the mass yielded only clear, mucoid
fluid. What is the most likely diagnosis?
A Follicular adenoma
B Lymph node metastasis of follicular carcinoma
C Nodule of a multinodular goitre
D Parathyroid cyst
E Thyroglossal duct cyst
SECTION 13:
BREAST AND FEMALE REPRODUCIVE PATHOLOGY —
QUESTIONS
For each question given below choose the ONE BEST option.
While doing the breast clinic you see a 19-year-old female university student whose left
13.2 breast has developed to be double the size of her right breast since puberty. Which of the
following is the most likely cause for this?
A An ovarian tumour
B Cystosarcoma phyllodes
C Fibrocystic disease
D Infiltrating ductal carcinoma
E Virginal breast hypertrophy
A 25-year-old woman presented in the surgical outpatient clinic complaining that she had
felt a lump in her right breast while showering. You examine her and discover that she
13.3 does have a firm, rubbery, mobile, 1-cm mass in the upper outer quadrant of her right
breast. No axillary lymph nodes are palpable. If mammography confirms that the lesion
has no microcalcifications, and reveals no lesions of the opposite breast, then the most
likely diagnosis is:
A Cystosarcoma phyllodes
B Fibroadenoma
C Focus of fat necrosis
D Infiltrating ductal carcinoma
E Intraductal papilloma
A 25-year-old lactating mother presented in the surgical outpatient clinic with a tender
13.4
and swollen right breast 4 weeks after the delivery of a healthy baby boy. The most likely
diagnosis is:
A Acute mastitis
B Fat necrosis
C Fibrocystic disease
D Galactocele
E Intraductal papilloma
A 32-year-old woman presented in the surgical outpatient clinic with a palpable lump in
13.5 her right breast that has appeared recently. On enquiry she mentioned that she has been
taking oral contraceptives for many years. Which of the following conditions is most
likely to be associated with oral contraceptive use?
A Acute mastitis
B Cyst formation
C Fat necrosis
D Galactocele
E Hypertrophy
13.7 A 56-year-old woman with a leaking silicone breast implant is at an increased risk of:
A Breast abscess
B Cystosarcoma phyllodes
C Infiltrating ductal carcinoma
D Pain and contracture
E Scleroderma
13.8 A 55-year-old woman with a palpable 1.5-cm lump in the upper outer quadrant of her
right breast had a biopsy of the lump which suggested that the lump was a carcinoma.
Which of the following features of her carcinoma would suggest a poor prognosis?
A Axillary lymph node metastases
B Family history of breast carcinoma
C Lack of aneuploidy
D Oestrogen receptor positivity
E Presence of an in-situ component
A 22-year-old woman who is breastfeeding develops a tender 2-cm mass beneath the
13.9 nipple in the right breast that shows several painful fissures. Which of the following
pathological findings is most likely to be present in this breast?
A Fat necrosis
B Infiltrating ductal carcinoma
C Numerous plasma cells
D Sclerosing adenosis
E Staphylococcus aureus infection
A 43-year-old woman presented in the surgical outpatient clinic with a 6-cm, soft, fleshy
13.12 left breast lump. A biopsy of the lump revealed lymphoid stroma with little fibrosis,
surrounding sheets of large vesicular cells with frequent mitoses. The most likely
diagnosis is:
A Cystosarcoma phyllodes
B Infiltrating ductal carcinoma
C Lobular carcinoma
D Medullary carcinoma of breast
E Paget’s disease of the breast
13.13 A 52-year-old man presented in the surgical outpatient clinic with bilateral
gynaecomastia. Which of the following conditions is he most likely to have?
A Hepatic failure
B History of antidepressant drug therapy
C Increased risk of breast carcinoma
D Increased testosterone levels
E Seminoma of the testis
A 35-year-old woman who had been on oral contraceptives for nearly 3 years presented
in the gynaecology outpatient clinic with abnormal vaginal bleeding. On speculum
13.14 examination, an endocervical polypoid mass was discovered, which was biopsied and sent
for histopathology reporting. Which of the following will be reported by the
histopasthologist?
A Clear cell adenocarcinoma
B Ectopic pregnancy
C Endocervical adenocarcinoma
D Microglandular hyperplasia
E Sarcoma botryoides
13.15 Endometrial biopsy of a woman revealed endometrial hyperplasia. Which of the following
neoplasms is most likely to be associated with endometrial hyperplasia?
A Choriocarcinoma
B Fibrothecoma
C Krukenberg tumour
D Mature cystic teratoma
E Sertoli–Leydig cell tumour
13.16 A 28-year-old woman has been diagnosed with endometriosis. Which of the following
statements regarding endometriosis is correct?
A It causes amenorrhoea
B It causes dyspareunia
C It develops through metaplasia of the peritoneal mesothelium
D It transforms into well-differentiated adenocarcinoma
E Obesity is a risk factor
A 48-year-old woman with a large abdominal mass underwent laparotomy. The mass was
found to be an ovarian tumour involving the right ovary. There was also some free ascitic
13.17 fluid in the peritoneal cavity. The tumour was excised and cytological examination of fluid
from this unilocular cystic tumour revealed clusters of malignant epithelial cells
surrounding psammoma bodies. What is the most likely diagnosis?
A Adenocarcinoma of Fallopian tube
B Endometrial adenocarcinoma
C Mesothelioma
D Ovarian mature cystic teratoma
E Ovarian serous cystadenocarcinoma
13.18 A 35-year-old woman had a Pap test which showed cervical intraepithelial neoplasia
(CIN). The presence of CIN on biopsy is most strongly associated with:
A Epstein–Barr virus infection
B Herpes simplex virus infection
C HLA DR5
D Human papillomavirus infection
E Previous pregnancy
A 23-year-old woman passed grape-like masses of tissue per vagina in week 16 of her
13.19 pregnancy. A dilation and curettage was then performed, and the microscopic appearance
of the tissue obtained showed large avascular villi with trophoblastic proliferation. Which
of the following is the best investigation to use for her follow-up?
A Chest X-ray
B Endometrial biopsy
C Pap smear
D Pelvic ultrasound
E Serum β-hCG
13.20 A 13-year-old girl is seen in the gynaecology outpatient clinic with haematocolpos. Which
of the following conditions is most likely to be associated with haematocolpos?
A Cervical condyloma
B Cervical gonorrhoea
E Endometriosis
D Imperforate hymen
E Ruptured Bartholin’s cyst
SECTION 14:
MALE REPRODUCTIVE PATHOLOGY — QUESTIONS
For each question given below choose the ONE BEST option.
14.2 A 32-year-old man with an uncircumcised penis presented in the emergency department
complaining that he could not fully retract the foreskin. This condition is known as:
A Cryptorchidism
B Epispadias
C Exstrophy
D Hypospadias
E Phimosis
While performing the initial clinical examination of a newborn baby boy, the paediatrician
14.3 noted that the baby had an abnormal opening of the urethra onto the ventral surface of
the penis. This is known as:
A Cryptorchidism
B Epispadias
C Exstrophy
D Hypospadias
E Phimosis
A 25-year-old man with an enlarged right testis was diagnosed with a testicular tumour.
14.5 He underwent inguinal orchidectomy and the histopathology report described the tumour
as a germ cell tumour. Which of the following tumours was most likely to have been
excised?
A Embryonal carcinoma
B Gonadoblastoma
C Lymphoma
D Leydig cell tumour
E Sertoli cell tumour
A 35-year-old man presented in the surgical outpatient clinic complaining of a left scrotal
14.6 swelling. On examination, he had a painless, transilluminant swelling at the upper pole of
the left testis. No cough impulse was visible or palpable. What is the most likely
diagnosis?
A Haematocoele
B Inguinal hernia
C Pyocoele
D Spermatocoele
E Varicocoele
A 25-year-old man presented in the surgical outpatient clinic complaining of a mass in his
14.7 left scrotum. On examination, he had a non-tender, twisted mass along the spermatic
cord. The mass was more prominent when the patient was standing and felt like a bag of
worms. The left testis was smaller than the right testis. What is the most likely diagnosis?
A Haematocoele
B Inguinal hernia
C Pyocoele
D Spermatocoele
E Varicocoele
A 72-year-old man with complaint of urinary retention had a digital rectal examination
14.8 which suggested that he had an enlarged prostate with a nodular feel to it. Biopsy of the
prostatic nodule suggested a malignant lesion. Which of the following is the most common
malignant lesion involving the prostate gland?
A Adenocarcinoma
B Ductal transitional carcinoma
C Prostatic intraepithelial neoplasia
D Squamous cell carcinoma
E Undifferentiated carcinoma
A 46-year-old man with painful erections presented in the surgical outpatient clinic. On
14.9 further enquiry, he told the consultant that his penis deviates to the right side when it is
erect. What is the most likely diagnosis?
A Balanitis
B Orchitis
C Peyronie’s disease
D Posthitis
E Priapism
A 25-year-old man is diagnosed with a germ cell tumour of the testis. Which of the
14.10 following histological subtypes is most likely to be found on histopathological examination
of the involved testis after orchidectomy?
A Choriocarcinoma
B Embryonal carcinoma
C Seminoma
D Teratocarcinoma
E Teratoma
A 40-year-old man with a seminoma of the left testis had a staging computed tomographic
14.11 scan which demonstrated retroperitoneal lymph nodes that were larger than 2 cm but
smaller than 5 cm in size. What is the nodal stage of this tumour?
A N0
B N1
C N2
D N3
E N4
A 42-year-old man has been diagnosed with a seminoma of the right testis that is
14.12 involving the tunica vaginalis. What is the tumour stage of this primary testicular
tumour?
A Tis
B T1
C T2
D T3
E T4
A 40-year-old man was diagnosed with a testicular seminoma. His serum tumour marker
14.13 levels were: lactate dehydrogenase (LDH) less than 1.5 times the reference range, beta-
human chorionic gonadotrophin (β-hCG) <5000 mIU/ml and alpha-fetoprotein (AFP)
<1000 ng/ml. What is the serum tumour marker stage of this tumour?
A S0
B S1
C S2
D S3
E S4
A 40-year-old man was diagnosed with a testicular seminoma that was extending through
the tunica albuginea, with involvement of the tunica vaginalis, retroperitoneal lymph
14.14 nodes greater than 5 cm in greatest dimension, no distant metastasis, a serum lactate
dehdrogenase (LDH) less than 1.5 times the reference range, beta-human chorionic
gonadotrophin (β-hCG) <5000 mIU/ml and alpha-fetoprotein (AFP) <1000 ng/ml. What is
the clinical stage of this tumour?
A Stage IA
B Stage IB
C Stage IIA
D Stage IIB
E Stage IIC
14.15 A patient with prostatic adenocarcinoma has very high serum levels of prostate-specific
antigen (PSA). What is the normal function of PSA?
A It converts testosterone to dihydrotestosterone in the prostate
B It increases FSH secretion from the anterior pituitary gland
C It inhibits the action of 5-alpha reductase
D It inhibits testosterone secretion
E It liquifies gelatinous semen after ejaculation
A 72-year-old man with an elevated serum prostate-specific antigen (PSA) level was
14.16 diagnosed with prostatic cancer on needle biopsy. What is the stage of this primary
prostatic tumour?
A T1a
B T1b
C T1c
D T2a
E T2b
A 78-year-old man with an elevated prostate-specific antigen level had a tumour palpable
14.17 in both lobes of the prostate gland, with no nodal involvement or distant metastases.
After a radical prostatectomy the tumour was reported as histological grade G2. What is
the overall stage of the prostatic cancer?
A Stage 0
B Stage I
C Stage II
D StageIII
E Stage IV
14.18 A 72-year-old man was prescribed finasteride as treatment for benign prostatic
hyperplasia. What is the mechanism of action of finasteride?
A It blocks the action of FSH on prostate
B It enhances the activity of dihydrotestosterone
C It increases serum levels of testosterone-binding protein
D It is a 5-alpha-reductase inhibitor
E It is an anti-oestrogen
A 68-year-old man presented in the urology outpatient clinic with a tumour that was
14.19 invading the subepithelial connective tissue of the penis. What is the stage of this primary
penile cancer?
A Ta
B T1
C T2
D T3
E T4
A 5-year-old boy was brought to the urology outpatient clinic after his mother noted that
his left testis was larger than the right. Investigations confirmed the presence of a
14.20 tumour, which was later excised. The histopathology report described the presence of
elements similar to skin and its appendages in the excised tumour. What is the most likely
diagnosis?
A Choriocarcinoma
B Embryonal carcinoma
C Juvenile granulosa cell tumour
D Seminoma
E Teratoma
SECTION 15:
BONE AND JOINT PATHOLOGY — QUESTIONS
For each question given below choose the ONE BEST option.
A 46-year-old patient was seen in the orthopaedic outpatient clinic complaining of pain in
15.1 the first metatarsophalangeal joint. On examination, the joint was swollen, red and
tender. Synovial fluid analysis revealed needle-shaped, strongly negatively birefringent
crystals. What is the most likely diagnosis?
A Gout
B Infectious arthritis
C Osteoarthritis
D Pseudogout
E Rheumatoid arthritis
A 56-year-old man complaining of pain in his right tibia presented in the orthopaedic
outpatient clinic. On clinical examination, bowing of the affected tibia was noticed. An X-
15.2 ray revealed increased bone density, abnormal architecture with coarse cortical
trabeculations, bowing and bony enlargement. Which of the following is the most likely
diagnosis?
A Osteitis fibrosa cystica
B Osteopetrosis
C Osteoporosis
D Paget’s disease of bone
E Tuberculous osteomyelitis
A 62-year-old woman was seen in the orthopaedic outpatient clinic complaining of severe
pain and stiffness in her neck, shoulders and hips for 2 months. She said that her
symptoms were most pronounced in the morning, shortly after awakening. She has had
15.4 chronic fatigue and low-grade fevers during this period. On examination, the range of
movement in the neck, shoulders and hips was normal. The muscles were minimally
tender to palpation. Muscle strength, sensations and deep tendon reflexes were all
normal. The serum creatine kinase activity was 40 U/l and the erythrocyte sedimentation
rate (ESR) was 80 mm/hour. The serum rheumatoid factor and antinuclear antibody
assays were negative. Which of the following is the most likely diagnosis?
A Fibromyositis
B Osteoarthritis
C Polymyalgia rheumatica
D Polymyositis
E Seronegative rheumatoid arthritis
A 52-year-old man presented in the orthopaedic outpatient clinic complaining of left hip
pain of several months’ duration. Clinical examination elicited deep tenderness. An X-ray
15.7 revealed a 9 cm × 12 cm mass involving the ischium of the pelvis. The mass had irregular
borders and there were extensive areas of bony destruction, as well as some scattered
calcifications. The lesion was resected, and grossly the mass had a bluish-white cut
surface. Which of the following statements regarding this lesion is correct?
A It is associated with Paget’s disease of bone
B It is more common in women
C It sometimes arises in benign cartilagenous tumours
D It is the most common primary tumour of bone
E It is usually seen in distal skeletal bones
A 16-year-old girl presented in the orthopaedic outpatient clinic complaining of pain and
swelling of her left distal thigh that was associated with activity. She noticed tenderness
15.11 after performing physical exercise. She had no history of trauma. Radiography of the
affected thigh showed an expansile, eccentric, lytic lesion located in the metaphysis of the
distal femur that was surrounded by a rim of reactive new bone. What is the most likely
diagnosis?
A Aneurysmal bone cyst
B Chondrosarcoma
C Ewing’s sarcoma
D Fibrous dysplasia
E Osteosarcoma
Which of the following bone diseases is most likely to be seen in a 48-year-old woman
15.12 who has been on long-term corticosteroid therapy for the treatment of severe
rheumatoid arthritis?
A Osteochondritis
B Osteomalacia
C Osteoporosis
D Paget’s disease of bone
E Rickets
An 18-year-old professional footballer noted some minor discomfort over the lateral
aspect of his right knee after a fall during a game. There was a palpable ‘bump’ in this
region. He consulted an orthopaedic surgeon, who ordered an X-ray of the affected knee.
15.13 The X-ray revealed a lateral projection from the metaphyseal region of his lower femur.
There was no soft-tissue swelling. The lesion was excised and was found to be composed
of a 3-cm stalk of bony cortex, capped by cartilage. Which of the following is the most
probable diagnosis in his case:
A Aneurysmal bone cyst
B Enchondroma
C Giant-cell tumour
D Osteochondroma
E Osteoid osteoma
A man who had been experiencing persistent lower back pain and stiffness that
diminished with activity since his twenties was diagnosed with ankylosing spondylitis. In
15.14 his thirties he also developed hip and shoulder arthritis and in his forties he was bothered
by decreased lumbar spine mobility. Which of the following HLA alleles is he most likley
to have?
A HLA-B3
B HLA-B27
C HLA-D4
D HLA-W6
E HLA-DR2
A 26-year-old cricketer complained to his bowling coach that for the previous 5 days he
had had backache, with muscle spasms, weakness and pain in his right hip which was
15.16 radiating all the way down to his toes. He was examined by an orthopaedic specialist, who
noticed that this bowler also had paraesthesiae in an L5 distribution and pain radiating
down the leg on straight-leg raising. These findings are most likely to be due to:
A Herniated nucleus pulposus
B Osteoporosis
C Spina bifida
D Spondylolisthesis
E Paget’s disease of bone
A 48-year-old man has noticed that he cannot completely extend the middle and little
15.17 fingers of his left hand. In the palm of his hand, at the base of these fingers, ill-defined,
tender, firm nodules are palpable. This man is a chronic alcoholic. What histological
appearance will this mass be most likely to show?
A Dystrophic calcification
B Fibromatosis
C Giant-cell tumour of the tendon sheath
D Granulomatous inflammation
E Rhabdomyosarcoma
A 35-year-old female typist in a busy legal firm notices that she develops tingling and
15.18 numbness over the palmar surface of her thumb, index and middle fingers after several
hours at her computer workstation. Pain in the same area often occurs at night as well.
Which of the following conditions is most likely to be responsible for her symptoms?
A Carpal tunnnel syndrome
B Dupuytren’s contracture
C Gout
D Hypertrophic osteoarthropathy
E Rheumatoid arthritis
A 76-year-old woman presented in the surgical outpatient clinic with a swollen and
painful right knee joint. On enquiry, she reported that she has had this swelling for
15.20 several months and that it occasionally bothers her as the joint becomes very painful. The
orthopaedic consultant aspirated synovial fluid from the joint and subjected it to
polarised light microscopy. Rhomboid or rod-shaped, weakly positively birefringent
crystals were seen on polarised light microscopy. What is the most likely diagnosis?
A Calcium oxalate crystal deposition disease
B Gout
C Osteoarthritis
D Pseudogout
E Rheumatoid arthritis
SECTION 16:
CENTRAL NERVOUS SYSTEM PATHOLOGY — QUESTIONS
For each question given below choose the ONE BEST option.
A 65-year-old man with known mitral valve disease and atrial fibrillation suffered a
16.1 massive stroke and died. At autopsy, a 5-cm-diameter area of softening in the region of
the left middle cerebral artery distribution was noticed. This is most consistent with:
A Hypertension
B Mycotic aneurysm
C Thromboembolism
D Vasculitis
E Venous thrombosis
A 65-year-old woman presented in the emergency department after having a grand mal
seizure. She had been healthy prior to this event. A neurological examination revealed no
16.3 focal abnormalities. Magnetic resonance imaging of the brain revealed a large, poorly
demarcated mass with central necrosis in the right frontal lobe. The most likely diagnosis
is:
A Choroid plexus tumour
B Glioblastoma multiforme
C Low-grade astrocytoma
D Medulloblastoma
E Meningioma
16.4 Which of the following histopathological findings are most likely to be found in the
cerebrum of a 75-year-old woman who suffered a cerebral infarction 1 week ago?
A Caseous necrosis
B Coagulative necrosis
C Fat necrosis
D Gangrenous necrosis
E Liquefactive necrosis
16.5 Which of the following laboratory tests that are done in the second trimester will most
strongly suggest the presence of a neural tube defect in a fetus?
A Decreased human chorionic gonadotrophin
B Decreased serum folate
C Hypochromic microcytic anaemia
D Increased alpha-fetoprotein
E Positive serological test for syphilis
A 55-year-old man with a 1-week history of headaches and fever was brought to the
emergency department in a confused state. Computed tomography revealed a 4-cm, ring-
16.6 enhancing mass in the right parietal region. Biopsy of the mass revealed gliosis and
fibrosis with necrosis, neutrophils and lymphocytes. These findings strongly suggest a
diagnosis of:
A Cerebral abscess
B Glioblastoma multiforme
C Herpes simplex type 2 encephalitis
D Subacute infarction
E Vascular malformation
A patient with an intracranial neoplasm was informed by the neurosurgeon that he had a
16.8 tumour which has the best prognosis following surgery. Which of the following
intracranial neoplasms is this patient most likely to have?
A
B Astrocytoma
Glioblastoma multiforme
C Medulloblastoma
D Schwannoma
E Solitary metastasis
A 22-year-old cricketer was hit on the left side of his head by a fast and rising cricket
ball. He fell to the ground and was immediately seen by the team physiotherapist. Only a
minor scalp abrasion was present at the site of the impact, with minimal bleeding that
16.9 stopped in a few minutes. He was initially alert after this accident, but then became
unconcious about 30 minutes later. A head computed tomographic (CT) scan revealed a
convex, lens-shaped area of haemorrhage centered over the left parietal region. This
history and the CT features are most likely to be associated with:
A Brain concussion
B Epidural haematoma
C Intracerebral haematoma
D Subdural haematoma
E Subarachnoid haemorrhage
A previously fit and healthy 48-year-old man with sudden onset of a generalised seizure
was referred for computed tomography (CT) of the brain. The CT scan revealed a 4-cm,
16.14 solid mass lesion in the right cerebral hemisphere which was distorting the lateral
ventricle. The patient had an urgent operation to excise the tumour. The histopathology
report of the tumour noted the presence of glial fibrillary acidic protein (GFAP). Which
of the following tumours was excised from this patient’s right cerebral hemisphere?
A Acoustic neuroma
B Astrocytoma
C Ependymoma
D Meningioma
E Neuroblastoma
A previously healthy, 26-year-old male office assistant was noticed by his colleagues to
be a bit confused. He then had a seizure and was brought to the hospital. His vital signs
showed: blood pressure 100/60 mmHg, temperature 37 °C, pulse 88/minute and
16.15 respiratory rate 22/minute. A lumbar puncture revealed a normal opening pressure and
clear, colourless cerebrospinal fluid was obtained with 1 red blood cell and 20 white blood
cells (all lymphocytes), with normal glucose and protein. Magnetic resonance imaging
revealed swelling of the right temporal lobe with haemorrhagic areas. Which of the
following infectious agents is the most likely cause of these findings?
A Haemophilus influenzae
B Herpes simplex virus
C Influenza virus
D Neisseria meningitidis
E Mycobacterium tuberculosis
A 55-year-old man was examined by an SHO in the neurology outpatient clinic because
he had developed a distal, symmetric, primarily sensory polyneuropathy over a period of
16.18 several months. He also had a non-healing ulcer on the ball of his right foot. He had had a
myocardial infarction 1 year previously, but had recovered and was doing well on anti-
anginal medication. Which of the following abnormal laboratory test findings would the
SHO expect to find in this patient?
A Decreased glucose level in the cerebrospinal fluid (CSF) (20 mg/dl)
B Elevated serum glucose of 10.8 mmol/l
C Elevated protein in the CSF
D Multiple osteoblastic lesions on X-ray of the vertebral column
E Markedly increased blood lead level of 2.4 µmol/l
For each question given below choose the ONE BEST option.
A 38-year-old patient with central abdominal pain was seen by an SHO in the surgical
17.1 oupatient clinic. Enquiries about his current medication therapy revealed that the patient
was taking cimetidine for duodenal ulcer. What is the mechanism of action of cimetidine?
A Adsorbs bile salts
B Competitively inhibits H2 receptors
C Dopamine antagonist
D Holds water in the stool
E Lowers the surface tension of the stool, facilitating penetration of water and fats
A 42-year-old woman who was receiving chemotherapy for breast cancer was prescribed
17.2 ondansetron for chemotherapy-induced vomiting. What is the mechanism of action of
ondansetron?
A Competitively inhibits H2 receptors
B Holds water in the stool
C Inactivates pepsin
D Opiate agonist
E Serotonin antagonist
A 74-year old man with chronic constipation and diverticulosis uses methylcellulose on a
17.4 regular basis as it helps him to move his bowels. What is the mechanism of action of
methylcellulose?
A Adsorbs bile salts
B Holds water in the stool
C Irreversibly inhibits H+/K+-ATPase
D Lowers the surface tension of the stool, facilitating penetration of water and fats
E Neutralises gastric acid
17.5 A 68-year-old woman with diverticulosis uses psyllium on a daily basis. What is the
mechanism of action of psyllium?
A Competitively inhibits H2 receptors
B Holds water in the stool
C Inactivates pepsin
D Lowers the surface tension of the stool, facilitating penetration of water and fats
E Opiate agonist
An SHO prescribes metoclopramide for the treatment of nausea and vomiting in a patient
17.7 on day 3 after closure of an ileostomy. What is the mechanism of action of
metoclopramide?
A Irreversibly inhibits H+/K+-ATPase
B Dopamine antagonist
C Neutralises gastric acid
D Serotonin antagonist
E Competitively inhibits H2 receptors
17.8 A 42-year-old patient with constipation and haemorrhoids is prescribed docusate sodium
(dioctyl sodium sulphosuccinate). What is the mechanism of action of docusate sodium?
A Competitively inhibits H2 receptors
B Holds water in the stool
C Inactivates pepsin
D Lowers the surface tension of the stool, facilitating penetration of water and fats
E Promotes release of acetylcholine
17.9 A 32-year-old patient with diarrhoea is prescribed loperamide. What is the mechanism of
action of loperamide?
A Competitively inhibits H2 receptors
B Irreversibly inhibits H+/K+-ATPase
C Inactivates pepsin
D Lowers the surface tension of the stool, facilitating penetration of water and fats
E Opiate agonist
17.10 An anaesthetist gave a trauma patient a shot of sufentanil prior to changing his wound
dressings in the intensive care unit. What is the mechanism of action of sufentanil?
A Modulates the GABAergic system
B Modulates the noradrenergic system
C Modulates the serotonergic system
D Opiate agonist
E Promotes the release of acetylcholine
17.11 A 48-year old man complaining of pain in his thoracotomy wound was prescribed regular
tramadol. What is the mechanism of action of tramadol?
A Competitively inhibits H2 receptors
B Inhibits synthesis of prostaglandins
C Modulation of GABAergic, noradrenergic and serotonergic systems
D Opiate antagonist
E Promotes release of acetylcholine
17.12 A 58-year old man was prescribed atorvastatin after coronary artery bypass surgery.
What is the mechanism of action of atorvastatin?
A Inhibits apoprotein synthesis
B Inhibits cholesterol synthesis
C Inhibits lipoprotein lipase
D Inhibits reabsorption of bile acids
E Inhibits VLDL synthesis
A Angina
B Cataract surgery
C Congestive heart failure
D Hypertriglyceridaemia
E Paroxysmal atrial tachycardia provoked by emotion or exercise
17.15 A 36-year-old multiple trauma patient who had been in the intensive care unit for over 3
weeks was prescribed griseofulvin. What class of drugs does griseofulvin belong to?
A Antifungal
B Antimycobacterial
C Autonomic nervous system drugs
D Beta-blockers
E Vasodilators
17.16 A 38-year-old patient in the intensive care unit is on mexiletine. Which of the following is
a recognised indication for mexiletine?
A Paroxysmal atrial tachycardia provoked by emotion or exercise
B Prophylaxis of Mycobacterium avium infection in patients with AIDS
C Streptococcus pneumoniae infection
D Thyrotoxicosis (preparation for surgery)
E Ventricular arrhythmias
C It induces the release of stored factor VIII and von Willebrand factor
D It inhibits hepatic gluconeogenesis
E It inhibits angiotensin-converting enzyme
17.19 A 63-year-old woman admitted for elective resection of a rectal carcinoma was noted to
be on buspirone (prescribed by her GP). Buspirone acts as:
A Anticonvulsant
B Antimycobacterial
C Anxiolytic
D Benzodiazepine
E Beta-blocker
A Macrolide
B Penicillin
C Quinolone
D Sulphonamide
E Tetracycline
17.22 A 56-year-old woman who was admitted for elective right hemicolectomy was noted to be
on spironolactone. What class of drugs does spironolactone belong to?
A Beta-lactamase inhibitors
B Hydantoins
C Lipid-lowering drugs
D Oral hypoglycaemics
E Potassium-sparing diuretics
17.23 Which of the following is a recognised indication for abciximab treatment?
A Bleeding due to deficiency of factor VIII
B Generalised motor seizures
C Muscle relaxation
D Neisseria gonorrhoeae infection
E Prevention of re-stenosis after angioplasty
A 45-year-old patient who had been in the intensive care unit for nearly 1 month was
17.24 commenced on fluconazole. Which of the following is the correct mechanism of action of
fluconazole?
A Inhibits cytochrome P450
B Inhibits phospholipase C
C Inhibits transpeptidase
D Inhibits viral protease
E Inhibits viral DNA polymerase
17.25 A 55-year-old man underwent emergency surgery for a perforated duodenal ulcer and
was commenced on ceftriaxone. The subclass of antibiotics that ceftriaxone belongs to is:
A First-generation cephalosporins
B Second-generation cephalosporins
C Third-generation cephalosporins
D Fluoroquinolones
E Macrolides
17.26 Etoposide is a chemotherapeutic agent. Which of the following is the correct indication
for this drug?
A Breast cancer
B Colon cancer
C Hairy cell leukaemia
D Lung cancer
E Myxoma
A Beta-antagonist
B Inhibits cholesterol synthesis
C Inhibits dihydrofolate reductase
D Mitotic spindle poison
E Potentiates glucose-mediated insulin secretion
17.29 The prescription of a patient in the intensive care unit includes tazobactam. What is the
mechanism of action of tazobactam?
A Inhibits beta-lactamase
B Inhibits CD3 receptor
C Inhibits calcineurin
D Inhibits translocase
E Opiate agonist
17.32 A 42-year-old man with lymphoma who was treated with cyclophosphamide 8 years ago is
potentially at risk of developing cancer of which of the following organs?
A Bone
B Colon
C Liver
D Oesophagus
E Urinary bladder
A 63-year-old man who is on broad-spectrum antibiotics and who has been in an intensive
17.35 care unit for nearly 3 weeks developed Clostridium difficile infection. He was treated
with oral metronidazole but the diarrhoea persisted. Which of the following antibiotics
could be used to treat this infection?
A Amoxicillin
B Cetriaxone
C Clindamycin
D Ciprofloxacin
E Oral vancomycin
17.36 A 35-year-old patient in the intensive care unit was commenced on linezolid. Which of the
following organisms is most likely to be effectively treated by linezolid?
A Entamoeba histolytica
B Methicillin-resistant Staphylococcus aureus
C Pseudomonas aeruginosa
D Salmonella typhi
E Vibrio cholerae
17.37 A 39-year-old multiple trauma patient with a methicillin-resistant Staphylococcus aureus
chest infection was commenced on linezolid because he was allergic to vancomycin. What
is the mechanism of action of linezolid?
A Inhibits bacterial cell wall synthesis
B Inhibits bacterial cholesterol synthesis
C Inhibits dihydrofolate reductase
D Inhibits initiation of bacterial protein synthesis
E Mitotic spindle poison
17.41 A 26-year-old woman with tuberculous meningitis was treated with rifampicin. Which of
the following is the mechanism of action of rifampicin?
A Disrupts the bacterial DNA helical structure
B Inhibits bacterial protein synthesis by binding to the 30S ribosomal subunit
C Inhibits bacterial protein synthesis by binding to the 50S ribosomal subunit
17.42 A 35-year-old man with a Staphylococcus aureus wound infection was commenced on
fusidic acid. Which of the following is the mechanism of action of fusidic acid?
A Disrupts the bacterial DNA helical structure
B Inhibits bacterial protein synthesis by binding to the 30S ribosomal subunit
C Inhibits bacterial protein synthesis by binding to the 50S ribosomal subunit
D Inhibits DNA-dependent RNA polymerase
E Prevents the translocation of elongation factor G from the ribosome
Chemotherapy regimens are often known by their acronyms, identifying the combination
17.47 of agents used. Which of the following chemotherapy regimens is used for treating
Hodgkin’s lymphoma?
A ABVD
B BEP
C CHOP
D COP
E FOLFOX
Chemotherapy regimens are often known by their acronyms, identifying the combination
17.48 of agents used. Which of the following chemotherapy regimens is used for treating non-
Hodgkin’s lymphoma?
A ABVD
B BEP
C CHOP
D ECF
E FOLFOX
17.49 A 28-year-old woman with severe Crohn’s disease was prescribed methotrexate. What is
the mechanism of action of methotrexate in this condition?
A Disruption of the DNA helical structure
B Inhibition of dihydrofolate reductase
C Inhibition of mast cells
D Inhibition of T-cell activation
E Mitotic spindle poison
17.50 A 48-year-old man with small-cell lung cancer was given cisplatin chemotherapy. What is
the mechanism of action of cisplatin?
A Cross-linking of DNA
B Disruption of the DNA helical structure
C Inhibition of dihydrofolate reductase
D Inhibition of T-cell activation
E Mitotic spindle poison
SECTION 1:
CELL INJURY AND WOUND HEALING — ANSWERS
1.1
Liquefactive necrosis is characteristic of focal bacterial or, occasionally, fungal infections, because
microbes stimulate the accumulation of inflammatory cells. Hypoxic death of cells within the central
nervous system often evokes liquefactive necrosis, though the reasons for this are obscure. Whatever
the pathogenesis, liquefaction completely digests the dead cells. The end result is transformation of
the tissue into a viscous liquid mass. If the process was initiated by acute inflammation the material is
frequently creamy yellow in colour because of the presence of dead white cells and this is called
‘pus’.
1.2
Caseous necrosis, a distinctive form of coagulative necrosis, is encountered most often in foci of
tuberculous infection. In this disease the characteristic lesion is called a ‘granuloma’ or a ‘tubercle’
and is classically characterised by the presence of central caseous necrosis. The term ‘caseous’ is
derived from the cheesy white gross appearance of the area of necrosis. On microscopic examination,
the necrotic focus appears as amorphous granular debris, seemingly composed of fragmented,
coagulated cells and amorphous granular debris enclosed within a distinctive inflammatory border
known as a ‘granulomatous reaction’. Unlike coagulative necrosis, the tissue architecture is
completely obliterated. In a granuloma, macrophages are transformed into epithelium-like cells
surrounded by a collar of mononuclear leukocytes, principally lymphocytes and occasionally plasma
cells. In the usual haematoxylin and eosin-stained tissue sections, the epithelioid cells have a pale
pink granular cytoplasm with indistinct cell boundaries, and often appear to merge into one another.
The nucleus is less dense than that of a lymphocyte, is oval or elongated in shape, and can show
folding of the nuclear membrane. Older granulomas develop an enclosing rim of fibroblasts and
connective tissue. Epithelioid cells often fuse to form giant cells in the periphery or sometimes in the
centre of granulomas. These giant cells can attain diameters of 40–50 µm. They have a large mass of
cytoplasm containing 20 or more small nuclei arranged either peripherally (Langhans-type giant cell)
or haphazardly (foreign body-type giant cell). There is no known functional difference between these
two types of giant cells. The chronic infective lesion in this scenario is most probably a tuberculous
granuloma.
1.3
Answer: C Cell size decreases
Atrophy is the shrinkage in the size of the cell due to loss of cell substance. It represents a form of
adaptive response and can result in cell death. Involvement of a sufficient number of cells causes the
entire tissue or organ to diminish in size or become atrophic. Atrophy can be physiological or
pathological. Physiological atrophy is commonly seen during early development and is exemplified
by atrophy of the notochord and the thyroglossal duct during foetal development. Another common
example of physiological atrophy is the decrease in size of the uterus shortly after childbirth.
Pathological atrophy can be local or generalised, depending on the underlying cause. The common
causes of atrophy are the following:
• Ageing (senile atrophy). Ageing is associated with cell loss, typically seen in tissues containing
permanent cells, particularly the brain and heart.
• Decreased workload (atrophy of disuse). This is best seen when a broken limb is immobilised in a
plaster cast or when a patient is restricted to complete bed rest resulting in rapid skeletal muscle.
The initial rapid decrease in cell size is reversible once activity is resumed. With more prolonged
disuse, however, skeletal muscle fibres decrease in number as well as in size, and this atrophy can
be accompanied by increased bone resorption, leading to osteoporosis of disuse.
• Diminished blood supply. A decrease in blood supply (ischaemia) to a tissue occurring as a result
of occlusive arterial disease results in atrophy of tissue because of progressive cell loss. An
example of this is seen in late adult life as the brain undergoes progressive atrophy due to reduced
blood supply secondary to atherosclerosis.
• Loss of endocrine stimulation. Many endocrine glands, the breast, and the reproductive organs are
dependent on endocrine stimulation for normal metabolism and function. The loss of oestrogen
stimulation after the menopause results in physiological atrophy of the endometrium, vaginal
epithelium, and breast.
• Loss of innervation (denervation atrophy). Normal function of skeletal muscle depends on its nerve
supply. Damage to the nerves leads to rapid atrophy of the muscle fibres supplied by those nerves.
• Pressure. Compression of tissues for any length of time can lead to atrophy. An enlarging benign
tumour can cause atrophy in the surrounding compressed tissues. Atrophy in this setting is probably
the result of ischaemic changes caused by compromise of the blood supply to the surrounding
tissues by the expanding mass.
Atrophy is brought about by a reduction in the structural components of the cell with concomitant
diminished function. For instance in atrophic muscle the cells contain fewer mitochondria and
myofilaments and a reduced amount of endoplasmic reticulum. The fundamental cellular changes
associated with atrophy are identical in all of the above settings and represent an adaptive response
to ensure survival.
1.4
Metaplasia is a reversible change in which one adult cell type (epithelial or mesenchymal) is
replaced by another adult cell type. It can represent an adaptive substitution of cells that are sensitive
to stress by cell types better able to withstand the adverse environment. The most common type of
epithelial metaplasia is columnar to squamous, as occurs in the respiratory tract in response to
chronic irritation. In the habitual cigarette smoker, the normal ciliated columnar epithelial cells of the
trachea and bronchi are often replaced focally or widely by stratified squamous epithelial cells.
The more rugged stratified squamous epithelium is able to survive under circumstances in which the
more fragile, specialised columnar epithelium would have succumbed. Although the metaplastic
squamous cells in the respiratory tract are capable of surviving this environment, an important
protective mechanism — mucus secretion — is lost. Epithelial metaplasia is therefore a two-edged
sword and, in most circumstances, represents an undesirable change. Moreover, the influences that
predispose to metaplasia, if persistent, can induce malignant transformation in metaplastic epithelium.
A common form of cancer in the respiratory tract is therefore composed of squamous cells, which
arise in areas of metaplasia of normal columnar epithelium into squamous epithelium.
1.5
Hypertrophy is caused by an increase in the size of cells, resulting in an increase in the size of the
organ. The hypertrophied organ has no new cells, just larger cells. The increased size of the cells is
due not to cellular swelling but to the synthesis of more structural components. Cells capable of
division can respond to stress by undergoing both hyperplasia and hypertrophy, whereas in non-
dividing cells (eg myocardial fibres), hypertrophy occurs. The nuclei in hypertrophied cells can have
a higher DNA content than normal cells, probably because the cells arrest in the cell cycle without
undergoing mitosis.
The massive physiological growth of the uterus that occurs during pregnancy is a good example of a
hormone-induced increase in the size of an organ that results from both hypertrophy and hyperplasia.
The cellular hypertrophy is stimulated by oestrogenic hormones acting on smooth-muscle oestrogen
receptors, eventually resulting in increased synthesis of smooth-muscle proteins and an increase in
cell size. Similarly, prolactin and oestrogen cause hypertrophy of the breasts during lactation. These
are examples of physiological hypertrophy induced by hormonal stimulation.
1.6
Apoptosis is a pathway of cell death that is induced by a tightly regulated intracellular programme in
which cells destined to die activate enzymes that degrade the cells’ own nuclear DNA and nuclear
and cytoplasmic proteins. The cell’s plasma membrane remains intact, but its structure is altered in
such a way that the apoptotic cell becomes an avid target for phagocytosis. The dead cell is rapidly
cleared, before its contents have leaked out, and therefore cell death by this pathway does not elicit
an inflammatory reaction in the host. Apoptosis is therefore fundamentally different from necrosis,
which is characterised by loss of membrane integrity, enzymatic digestion of cells, and frequently a
host reaction. However, apoptosis and necrosis sometimes coexist, and they can share some common
features and mechanisms.
The following morphological features, some best seen with the electron microscope, characterise
cells undergoing apoptosis:
• Cell shrinkage. The cell is smaller in size; the cytoplasm is dense; and the organelles, although
relatively normal, are more tightly packed.
• Chromatin condensation. This is the most characteristic feature of apoptosis. The chromatin
aggregates peripherally, under the nuclear membrane, into dense masses of various shapes and
sizes. The nucleus itself can break up, producing two or more fragments.
• Formation of cytoplasmic blebs and apoptotic bodies. The apoptotic cell first shows extensive
surface blebbing, then undergoes fragmentation into membrane-bound apoptotic bodies composed
of cytoplasm and tightly packed organelles, with or without nuclear fragments.
• Phagocytosis of apoptotic cells or cell bodies, usually by macrophages. The apoptotic bodies are
rapidly degraded within lysosomes, and the adjacent healthy cells migrate or proliferate to replace
the space occupied by the now deleted apoptotic cell.
Plasma membranes are thought to remain intact during apoptosis, until the last stages, when they
become permeable to solutes that are normally retained. This classic description is accurate with
respect to apoptosis during physiological conditions such as embryogenesis and deletion of immune
cells. However, forms of cell death with features of necrosis as well as of apoptosis are not
uncommon in reaction to injurious stimuli. Under such conditions, the severity, rather than the
specificity, of the stimulus determines the form in which death is expressed. If necrotic features are
predominant, early plasma membrane damage occurs, and cell swelling, rather than shrinkage, is
seen.
1.7
Fat necrosis can be seen after trauma to the breast. It can present as a painless palpable mass, skin
thickening or retraction, a mammographic density, or mammographic calcifications. The majority of
women will give a history of trauma or prior surgery. The major clinical significance of the condition
is its possible confusion with breast carcinoma when there is a palpable mass or mammographic
calcifications.
Grossly, the lesion can consist of haemorrhage in the early stages and, later, central liquefactive
necrosis of fat. Later still, it can appear as an ill-defined nodule of grey-white, firm tissue containing
small foci of chalky-white or haemorrhagic debris. The central focus of necrotic fat cells is initially
surrounded by macrophages and an intense neutrophilic infiltration. Then, during the next few days,
progressive fibroblastic proliferation, increased vascularisation, and lymphocytic and histiocytic
infiltration wall off the focus. Subsequently, foreign-body giant cells, calcifications, and
haemosiderin make their appearance. Eventually, the focus is replaced by scar tissue or is encysted
and walled off by collagenous tissue.
1.8
Shrinkage in the size of the cell by loss of cell substance is known as ‘atrophy’. It represents a form
of adaptive response and can culminate in cell death. When a sufficient number of cells are involved,
the entire tissue or organ diminishes in size, or becomes atrophic. When a broken limb is immobilised
in a plaster cast or when a patient is restricted to complete bed rest, skeletal muscle atrophy rapidly
ensues. The initial rapid decrease in cell size is reversible once activity is resumed. With more
prolonged disuse, however, skeletal muscle fibres decrease in number as well as in size; this atrophy
can be accompanied by increased bone resorption, leading to osteoporosis of disuse (see also
Answer to 1.3).
1.9
The metabolic response to injury is an important part of the stress reaction in that it improves the
individual organism’s chances of surviving under adverse circumstances or when injured. The
metabolic response to trauma includes:
• Immunosuppression
1.10
Cells generate energy by reducing molecular oxygen to water. During this process small amounts of
partially reduced reactive oxygen forms are produced as an unavoidable byproduct of mitochondrial
respiration. Some of these forms are free radicals that can damage lipids, proteins and nucleic acids.
They are referred to as ‘reactive oxygen species’. Cells have defence systems for preventing injury
caused by these products. An imbalance between free radical-generating and free radical-scavenging
systems results in oxidative stress, a condition that has been associated with the cell injury seen in
many pathological conditions. Free radical-mediated damage contributes to such varied processes as
chemical and radiation injury, ischaemia-reperfusion injury (induced by restoration of blood flow in
ischaemic tissue), cellular ageing and microbial killing by phagocytes.
1.11
In haemophiliac joints the lipid from the red cell membranes is broken down and cholesterol crystals
form. Accumulation of lipofuscin is not a feature of haemorrhage. Russell bodies are intracellular
accumulations of immunoglobulins in plasma cells. Neutrophils suggest acute inflammation.
Anthracotic pigment is an exogenous carbon pigment from dusts in the air that accumulate in lung.
1.12
Answer: E Metaplasia
Metaplasia is the substitution of one tissue type normally found at a site for another. The epithelium
undergoes metaplasia in response to ongoing inflammation from reflux of gastric contents. It is
common in the lower oesophagus with gastro-oesophageal reflux disease. The growth of the
epithelial cells must become disordered in order to be dysplastic. Hyperplasia can occur with
inflammation, as the number of cells increases, but hyperplasia does not explain the presence of the
columnar cells. Carcinoma is characterised by cellular atypia with hyperchromatism and
pleomorphism. Goblet cells would not be seen. Ischaemia would be unusual at this site and would be
marked by coagulative necrosis.
1.13
The brain undergoes liquefactive necrosis with infarction. As it resolves, a cystic area forms in the
region of infarction. Atrophy would be a more generalised process, whereas a single cystic area in
the brain suggests a remote infarction. Coagulative necrosis is more typical of parenchymal organs
such as the kidney or spleen, which do not have as high a lipid content as the brain. Caseous necrosis
is more typical of granulomatous inflammation with Mycobacterium tuberculosis. Apoptosis is
single-cell necrosis that does not result in a grossly visible cystic area (see also Answer to 1.1).
1.14
There is an increase in lobules under hormonal influence to provide for lactation. Lobular
hyperplasia would allow this woman to nurse the infant. The stroma of the breast consists of
connective tissue, which provides structural support but does not have cells that produce milk.
Dysplasia in epithelia is a premalignant change and not a normal physiological event. With poor
nutrition and weight loss, steatocytes can decrease in size. Metaplasia is the exchange of normal
epithelium for another type in response to chronic irritation. Metaplasia is not a normal physiological
process.
1.15
Lipochrome is a very common finding in older people, but it has little effect on cardiac function.
Lipochrome is also known as ‘lipofuscin’. Haemosiderosis is not a complication of ageing. Glycogen
storage diseases are inherited conditions that appear early in life. Glycogen does not appear
pigmented in haematoxylin and eosin-stained tissue sections. Cholesterol accumulates in
atheromatous plaques in the arteries, not in the myocardium. Calcium deposits appear as irregular,
dark-blue areas and are not associated with ageing of the myocardium.
1.16
Malignant tumours of diverse origin often resemble one another because they are poorly
differentiated. These tumours are often quite difficult to distinguish on the basis of routine
haematoxylin and eosin- (H&E-) stained tissue sections. For example, certain anaplastic carcinomas,
malignant lymphomas, melanomas and sarcomas can look quite similar, but they must be accurately
identified because their treatment and prognosis are different. Antibodies against intermediate
filaments have proved to be of value in such cases because tumour cells often contain intermediate
filaments characteristic of their cell of origin. For example, the presence of cytokeratins, detected
immunohistochemically, points to an epithelial origin (carcinoma), whereas desmin is specific for
neoplasms of muscle cell origin. Cytoskeletal alterations occur with ischaemia, but are not a useful
marker for such an event. Many cell types contain intermediate filaments. Mallory’s alcoholic hyaline
can be observed by H&E staining. However, it is not entirely specific for alcoholism. Metaplasia and
dysplasia are assessed using their light-microscopic appearances after H&E staining.
1.17
Answer: D Hypertrophy
In this patient the pressure load of hypertension led to myocardial fibre hypertrophy and a heart twice
normal size. Fat in the heart does not increase in response to the increase in workload from
hypertension. Myocardial fibres do not undergo hyperplasia. Fatty degeneration of the myocardium is
typically the result of a toxic or hypoxic injury. Myocardial oedema is not a characteristic feature of
myocardial injury or increased workload. However, heart failure could lead to peripheral oedema
(see also Answer to 1.5).
1.18
Answer: E Tyrosine
The tanning process in skin is stimulated by ultraviolet light exposure. Melanocytes have the enzyme
tyrosinase to oxidize tyrosine to dihydroxyphenylalanine in the pathway for melanin production.
Haem as part of haemosiderin from breakdown of red blood cells can impart a brownish colour, but
this is typically local from trauma or more global as part of an iron storage disease such as
haemochromatosis. Lipochrome (lipofuscin) is a ‘wear and tear’ pigment that imparts a golden-brown
appearance to granules in cells (such as myocytes or hepatocytes), but this is not a feature of skin.
Homogentisic acid can be part of the process of the rare disease alkaptonuria, in which a black
pigment is deposited in connective tissues. Glycogen in sufficient quantity is starch-like and imparts a
paler colour to organs in which it is stored in excess. It does not involve the skin.
1.19
Vitamin A is necessary to maintain epithelia, and squamous metaplasia of the respiratory tract can
occur if there is a deficiency. Tanning of the skin is a physiological event resulting from the
accumulation of melanin pigment — there is no change of one cell type to another involved. Lactation
following pregnancy is a form of physiological hyperplasia of breast lobules that occurs as a result of
hormonal influences. An acute myocardial infarction will lead to cardiac muscle fibre necrosis,
which will heal with the production of fibrous scar tissue, but this is not a reversible metaplasia. The
enlarged prostate represents primarily glandular hyperplasia.
1.20
Viral infection leads to individual hepatocyte necrosis, which is characterised by the microscopic
appearances of karyorrhexis and cell fragmentation. Brown atrophy of the heart results when there is
marked lipofuscin deposition in the myocardium. Tissue destruction with transplant rejection is more
widespread. Single cell necrosis is not evident in chronic alcoholic liver disease. Barbiturate
overdose causes hypertrophy of smooth endoplasmic reticulum, not individual cell necrosis.
1.21
1.22
Free radicals are chemical species that have a single unpaired electron in an outer orbit. Energy
created by this unstable configuration is released through reactions with adjacent molecules, such as
inorganic or organic chemicals (proteins, lipids, carbohydrates), particularly with key molecules in
membranes and nucleic acids. Moreover, free radicals initiate autocatalytic reactions, whereby
molecules with which they react are themselves converted into free radicals to propagate the chain of
damage. Absorption of radiant energy such as ultraviolet rays in sunlight can hydrolyse water into
hydroxyl (OH) and hydrogen (H) free radicals, which cause sunburn. The free radicals induce cell
injury by the following mechanisms:
• Lipid peroxidation of membranes: free radicals in the presence of oxygen can cause peroxidation
of lipids within plasma and organellar membranes. Oxidative damage is initiated when the double
bonds in unsaturated fatty acids of membrane lipids are attacked by oxygen-derived free radicals,
particularly by OH. The lipid-free radical interactions yield peroxides, which are themselves
unstable and reactive, and an autocatalytic chain reaction ensues (called ‘propagation’), which can
result in extensive membrane, organellar and cellular damage. Other more favorable termination
options take place when the free radical is captured by a scavenger, such as vitamin E, embedded
in the cell membrane.
• Oxidative modification of proteins: free radicals promote oxidation of amino acid residue side
chains, formation of protein–protein cross-linkages (eg disulfide bonds), and oxidation of the
protein backbone, resulting in protein fragmentation. Oxidative modification enhances degradation
of critical proteins by the multicatalytic proteasome complex, causing havoc throughout the cell.
• Lesions in DNA: reactions with thymine in nuclear and mitochondrial DNA produce single-
stranded breaks in DNA. This DNA damage has been implicated in cell ageing and in malignant
transformation of cells.
1.23
Answer: D Liver
Carbon tetrachloride (CCl4) was once used widely in the dry-cleaning industry. The toxic effect of
CCl4 results from its conversion by P450 to the highly reactive toxic free radical, CCl3 (CCl4 + e =
CCl3 + Cl–). The free radicals produced locally cause auto-oxidation of the polyenoic fatty acids
present within the membrane phospholipids. There, oxidative decomposition of the lipid is initiated,
and organic peroxides are formed after reacting with oxygen (lipid peroxidation). This reaction is
autocatalytic in that new radicals are formed from the peroxide radicals themselves. Rapid
breakdown of the structure and function of the endoplasmic reticulum is therefore due to
decomposition of the lipid. It is no surprise, therefore, that CCl4-induced liver cell injury is both
severe and extremely rapid in onset. Within less than 30 minutes there is a decline in hepatic protein
synthesis; within 2 hours there is swelling of smooth endoplasmic reticulum and dissociation of
ribosomes from the rough endoplasmic reticulum. Lipid export from the hepatocytes is reduced
because of their inability to synthesise apoprotein to complex with triglycerides and thereby facilitate
lipoprotein secretion. The result is the fatty liver of CCl4 poisoning. Mitochondrial injury then occurs,
and this is followed by progressive swelling of the cells due to increased permeability of the plasma
membrane. Plasma membrane damage is thought to be caused by relatively stable fatty aldehydes,
which are produced by lipid peroxidation in the smooth endoplasmic reticulum but which are able to
act at distant sites. This is followed by massive influx of calcium and cell death.
1.24
ATP depletion and decreased ATP synthesis are frequently associated with both hypoxic and
chemical (toxic) injury. High-energy phosphate in the form of ATP is required for many synthetic and
degradative processes within the cell. These include membrane transport, protein synthesis,
lipogenesis, and the deacylation-reacylation reactions necessary for phospholipid turnover. ATP is
produced in two ways. The major pathway in mammalian cells is oxidative phosphorylation of
adenosine diphosphate, in a reaction that results in reduction of oxygen by the electron transfer system
of mitochondria. The second is the glycolytic pathway, which can generate ATP in the absence of
oxygen using glucose derived either from body fluids or from the hydrolysis of glycogen. Tissues with
greater glycolytic capacity (eg liver) therefore have an advantage when ATP levels are falling
because of inhibition of oxidative metabolism by injury.
Depletion of ATP to < 5%–10% of normal levels has widespread effects on many critical cellular
systems:
• The activity of the plasma membrane energy-dependent sodium pump (ouabain-sensitive Na+, K+-
ATPase) is reduced. Failure of this active transport system, due to diminished ATP concentration
and enhanced ATPase activity, causes sodium to accumulate intracellularly and potassium to
diffuse out of the cell. The net gain of solute is accompanied by isosmotic gain of water, causing
cell swelling, and dilatation of the endoplasmic reticulum.
• Cellular energy metabolism is altered. If the supply of oxygen to cells is reduced, as in ischaemia,
oxidative phosphorylation ceases and cells rely on glycolysis for energy production. This switch to
anaerobic metabolism is controlled by energy pathway metabolites acting on glycolytic enzymes.
The decrease in cellular ATP and associated increase in adenosine monophosphate stimulate
phosphofructokinase and phosphorylase activities. These result in an increased rate of anaerobic
glycolysis that is designed to maintain the cell’s energy sources by generating ATP through
metabolism of glucose derived from glycogen. As a consequence, glycogen stores are rapidly
depleted. Glycolysis results in the accumulation of lactic acid and inorganic phosphates from the
hydrolysis of phosphate esters. This reduces the intracellular pH, resulting in decreased activity of
many cellular enzymes.
• Failure of the calcium pump leads to influx of calcium ions, with damaging effects on numerous
cellular components.
• With prolonged or worsening depletion of ATP, structural disruption of the protein synthetic
apparatus occurs, manifest as detachment of ribosomes from the rough endoplasmic reticulum and
dissociation of polysomes into monosomes, with a consequent reduction in protein synthesis.
Ultimately, there is irreversible damage to mitochondrial and lysosomal membranes, and the cell
undergoes necrosis.
• In cells that are deprived of oxygen or glucose, proteins can become misfolded, and misfolded
proteins trigger a cellular reaction called the ‘unfolded protein response’ that can lead to cell
injury and even death. Protein misfolding is also seen in cells exposed to stress, such as heat, and
when proteins are damaged by enzymes such as calcium-responsive enzymes and free radicals.
1.25
Answer: D Lipofuscin
1.26
Answer: C Malignancy
Complications of wound healing can arise from abnormalities in any of the basic components of the
repair process. These aberrations can be grouped into three general categories: (1) deficient scar
formation, (2) excessive formation of the repair components, and (3) formation of contractures.
1. Inadequate formation of granulation tissue or assembly of a scar can lead to two types of
complications: wound dehiscence and ulceration. Dehiscence or rupture of a wound is most
common after abdominal surgery and is due to increased abdominal pressure. This mechanical
stress on the abdominal wound can be generated by vomiting, coughing or ileus. Wounds can
ulcerate because of inadequate vascularisation during healing. For example, lower-extremity
wounds in individuals with atherosclerotic peripheral vascular disease typically ulcerate. Non-
healing wounds also form in areas devoid of sensation. These neuropathic ulcers are occasionally
seen in patients with diabetic peripheral neuropathy.
2. Excessive formation of the components of the repair process can also complicate wound healing.
Aberrations of growth can occur even in what may begin initially as normal wound healing. The
accumulation of excessive amounts of collagen can give rise to a raised scar known as a
‘hypertrophic scar’; if the scar tissue grows beyond the boundaries of the original wound and does
not regress, it is called a ‘keloid’. Keloid formation appears to be an individual predisposition,
and for unknown reasons this aberration is somewhat more common in African Americans. The
mechanisms of keloid formation are still unknown. Another deviation in wound healing is the
formation of excessive amounts of granulation tissue, which protrudes above the level of the
surrounding skin and blocks re-epithelialisation. This has been called ‘exuberant granulation’ (or,
with more literary fervour, ‘proud flesh’). Excessive granulation must be removed by cautery or
surgical excision to permit restoration of the continuity of the epithelium. Finally (fortunately
rarely), incisional scars or traumatic injuries can be followed by exuberant proliferation of
fibroblasts and other connective tissue elements that can even recur after excision. Called
‘desmoids’, or ‘aggressive fibromatoses’, these lie in the interface between benign proliferations
and malignant (though low-grade) tumours. The line between the benign hyperplasias characteristic
of repair and neoplasia is frequently finely drawn.
3. Contraction in the size of a wound is an important part of the normal healing process. An
exaggeration of this process is called a ‘contracture’ and results in deformities of the wound and
the surrounding tissues. Contractures are particularly prone to develop on the palms, the soles, and
the anterior aspect of the thorax. Contractures are commonly seen after serious burns and can
compromise the movement of joints.
1.27
Answer: D Vitamin A deficiency
Healing is modified by a number of known influences and some unknown ones, frequently impairing
the quality and adequacy of both inflammation and repair. These influences include both systemic and
local host factors.
• Blood supply
• Denervation
• Local infection
• Foreign body
• Haematoma
• Mechanical stress
• Necrotic tissue
• Surgical techniques
• Protective dressings
• Type of tissue.
• Nutrition has profound effects on wound healing. Protein deficiency, for example, and particularly
vitamin C deficiency, inhibit collagen synthesis and retard healing. Similarly, deficiency of trace
elements such as zinc affects wound healing. Vitamin A deficiency is not a recognised factor that
retards wound healing.
• Metabolic status can affect wound healing. Diabetes mellitus, for example, is associated with
delayed healing, as a consequence of the microangiopathy that is a frequent feature of this disease.
• Circulatory status can modulate wound healing. Inadequate blood supply, usually caused by
arteriosclerosis or venous abnormalities (eg varicose veins) that retard venous drainage, also
impairs healing.
Although any foreign body will delay wound healing, the net effect of having sutures is to aid the
process of healing more than to hinder it. Secondary wound infection is a serious postoperative
complication. Diabetics and people with severe atherosclerosis with poor tissue perfusion have
notoriously poor wound healing. Corticosteroids dampen the inflammatory response that contributes
to healing. Poor nutrition, leading to hypoalbuminaemia, is a detrimental factor in wound healing (see
also Answer to 1.27).
1.29
Cell-surface growth-factor receptors recruit intracellular protein kinases such as tyrosine kinase that
begin a sequence of events leading to cell division and growth. A tyrosine kinase is an enzyme that
can transfer a phosphate group from adenosine triphosphate (ATP) to a tyrosine residue in a protein.
Tyrosine kinases are a subgroup of the larger class of protein kinases. Phosphorylation of proteins by
kinases is an important mechanism in signal transduction for regulation of enzyme activity.
Fibronectin acts in the extracellular matrix to bind macromolecules (such as proteoglycans) via
integrin receptors to aid attachment and migration of cells. Laminin is an extracellular matrix
component that is abundant in basement membranes. Hyaluronic acid is one of the proteoglycans in
the extracellular matrix. Collagen fibres are part of the extracellular matrix that gives strength and
stability to connective tissues.
1.30
Hageman factor (factor XII in the intrinsic coagulation pathway) activates factor XI and prekallikrein.
It is an enzyme of the serine protease (or serine endopeptidase) class. Hageman factor deficiency is a
rare hereditary disorder with a prevalance of about one in a million, although it is a little more
common among Asians. Deficiency does not cause excessive haemorrhage as the other coagulation
factors make up for the deficiency of factor XII. It can increase the risk of thrombosis due to
inadequate activation of the fibrinolytic pathway. The deficiency leads to activated partial
thromboplastin times (PTT) greater than 200 seconds.
1.32
Autophagic vacuoles appear to form when breakdown of intracellular components and organelles
occurs. Separation of damaged cell substances in the form of autophagic vacuoles appears to be an
adaptive response in atrophy.
1.33
Answer: C Erythrocyte
The erythrocyte is an end-stage cell that is not capable of futher division and regeneration.
Hepatocytes, osteocytes, and acinar cells are stable cells that are capable of some regeneration under
certain conditions. Colonic mucosal cells are labile cells.
1.34
In uterine growth during pregnancy, both cell proliferation involving the endometrial glands and
muscle enlargement of the uterine wall occur. These processes offer models of both hyperplasia and
hypertrophy. When both are present, DNA synthesis is markedly accelerated. Hyperplasia is an
increase in the number of cells, whereas hypertrophy is an increase in cell size, as is seen in cardiac
muscle in response to volume overload or systemic hypertension. Breast tissue enlargement resulting
from hormonal influences is due solely to an increase in cell numbers.
1.35
1.36
Answer: D Macrophages
Tuberculous granulomas are often called ‘tubercles’ and consist of round, plump, mononuclear
phagocytes, Langhans cells, and epithelioid cells. The enlarged macrophages are called ‘epithelioid
cells’ because of their abundant cytoplasm and their tendency to arrange themselves very closely
together, which makes them resemble epithelial cells.
1.37
Answer: C Hamartoma
A hamartoma is a conglomeration of tissues that are normal to the area but haphazardly arranged in an
abnormal fashion. All of the neoplastic lesions listed contain a mixture of different cell types, but
only in the hamartoma are the cells normal to that particular area. These are best regarded as
developmental anomalies rather than true neoplasms, and they are never malignant so do not
metastasise.
1.38
New capillaries, fibroblasts, and collagen describe granulation tissue occurring at the periphery of a
haematoma or collection of blood. Although lysis of the blood clot can occur as a result, the actual
formation of this response is known as ‘organisation’ and is an attempt to heal the area and fill the
defect with collagen or scar tissue. If this occurs within a blood vessel, recanalisation of the
occluded lumen can take place subsequently and embolisation can be an eventual complication.
Thrombosis describes clot formation in the vascular lumen but this has nothing to do with the process
described.
1.39
Answer: A Diapedesis
Diapedesis is the movement of leukocytes across the endothelial lining of blood vessels into the
interstitial fluid. The process is driven by chemotactic factors that serve to up-regulate expression of
adhesion molecules on the endothelial cells of postcapillary venules adjacent to the site of infection.
Neutrophils, monocytes and natural killer (NK) cells use ‘roll-to-stop’ kinetics in order to slow
down and ‘ooze through’ (diapedesis) interendothelial spaces between endothelial cells and infiltrate
the infected tissue.
On recognition of and activation by pathogens, resident macrophages in the infected tissue release
cytokines such as interleukin 1 (IL-1), tumour necrosis factor (TNF) and chemokines. IL-1 and TNF
cause the endothelial cells of blood vessels close to the site of infection to express adhesion
molecules such as integrin receptors and selectin. Like Velcro, selectin ligands on the leukocyte bind
selectin molecules on the inner wall of the vessel with marginal affinity in order to slow the leukocyte
down near the infected tissue. This causes the leukocyte to slow down and begin rolling on the inner
surface of the vessel wall. At the same time, chemokines released by macrophages activate the rolling
leukocytes and cause their integrin molecules to switch from the default low-affinity state to a high-
affinity state in which they can bind tightly to complementary integrin receptors that have been
expressed on the endothelial cells. At this point, the integrin (VLA-4, LFA-1 or Mac-1) on the surface
of the leukocytes is now able to produce the ‘stopping’ effect by binding the integrin receptors
(VCAM-1 or ICAM-1) on the inner wall of the vessel with high affinity. This leads to immobilisation
of leukocytes and the reorganisation of their cytoskeletons in such a way that they are ‘spread out’
over the endothelial cells. In this form, they are able to pass between gaps in the vessel’s endothelial
cells.
Diapedesis usually occurs when an area is injured or damaged and an inflammatory response is
needed. This trafficking system is regulated by the background cytokine environment produced by the
inflammatory response. In leukocyte adhesion deficiency (LAD) there is defective integrin and this
impairs the ability of the leukocytes to stop and undergo diapedesis. Neutrophilia is a hallmark of
LAD.
Euperiporesis describes the movement of lymphocytes through the cytoplasm of endothelial cells
rather than between the cells. After the injury, neutrophils are attracted to the injured site
(chemotaxis) from the blood vessels. The neutrophils marginate or ‘pavement’ on the inner walls of
the vessel and then pass through the widened pores between the endothelial cells in a movement
known as ‘migration’. Foreign bodies (bacteria, antigen–antibody complexes, tissue debris) are
engulfed by the neutrophils (phagocytosis) aided by serum opsonin. The lysosomal granules of the
neutrophils empty their hydrolytic enzymes (proteases, esterases) into the phagocytic vacuole for
digestion. The undigested residue might be deposited as lipofuscin residual body.
1.40
A week after myocardial infarction, little collagen has formed. At the end of 2 weeks, a cellular
vascular scar is present, some collagen has been laid down in a disorderly fashion, and the scar has
attained some strength. During the next 6 months, collage is laid down and rearranged to make the
scar smaller. Most of the blood vessels are pinched off, leading to decreased cellularity in the scar.
The end result of these processes is a mature scar. The slow healing of a myocardial infarct is the
result of the decreased blood supply.
SECTION 2:
INFLAMATIOIN AND IMMUNOLOGY — ANSWERS
2.1
Sinus histiocytosis, also known as ‘reticular hyperplasia’ refers to distension and prominence of the
lymphatic sinusoids. Although non-specific, this form of hyperplasia can be particularly prominent in
lymph nodes draining cancers, such as carcinoma of the breast. The lining lymphatic endothelial cells
are markedly hypertrophied, and macrophages are greatly increased in numbers, resulting in
expansion and distension of the sinuses. In the setting of cancer, this pattern of reaction has been
thought to represent a host immune response against the tumour or its products.
In this vignette there is no evidence that the patient has infection or another malignancy that could
account for the enlargement of the lymph nodes. Paracortical lymphoid hyperplasia is caused by
stimuli that trigger cellular immune responses. It is characterised by reactive changes within the T-
cell regions of the lymph node that encroach on, and sometimes appear to efface, the B-cell follicles.
Within interfollicular regions, activated T cells (immunoblasts) are observed. These cells are three to
four times the size of resting lymphocytes and have round nuclei, open chromatin, several prominent
nucleoli, and moderate amounts of pale cytoplasm. In addition, there is hypertrophy of sinusoidal and
vascular endothelial cells and a mixed cellular infiltrate, principally of macrophages and sometimes
of eosinophils. Such changes are encountered in immunological reactions induced by drugs
(especially phenytoin), in acute viral infections, particularly infectious mononucleosis, and following
vaccination against certain viral diseases. In florid reactions, immunoblasts can be so numerous that
special studies are needed to exclude a lymphoid neoplasm.
2.2
Answer: D Neutrophils
A reduction in the number of granulocytes in the peripheral blood (neutropenia) can be seen in a wide
variety of circumstances. A marked reduction in neutrophil count, referred to as ‘agranulocytosis’,
has serious consequences by making individuals susceptible to infections. A reduction in circulating
granulocytes will occur if there is (1) reduced or ineffective production of neutrophils, or (2)
accelerated removal of neutrophils from the circulating blood.
• Suppression of myeloid stem cells, as occurs in aplastic anaemia and a variety of infiltrative
marrow disorders (tumours, granulomatous diseases etc.). In these conditions, granulocytopenia is
accompanied by anaemia and thrombocytopenia.
• Suppression of committed granulocytic precursors due to exposure to certain drugs (see below).
• Disease states associated with ineffective granulopoiesis, such as megaloblastic anaemias due to
vitamin B12 or folate deficiency and myelodysplastic syndromes, where defective precursors are
susceptible to death in the marrow.
• Rare inherited conditions (such as Kostmann syndrome) in which genetic defects in specific genes
result in impaired granulocytic differentiation.
• Immunologically mediated injury to the neutrophils, which may be idiopathic, associated with a
well-defined immunological disorder (such as systemic lupus erythematosus), or produced by
exposure to drugs.
2.3
Type II hypersensitivity is mediated by antibodies (mainly IgG or IgM) directed towards antigens
present on cell surfaces or in the extracellular matrix. The antigenic determinants can be intrinsic to
the cell membrane or matrix, or they can take the form of an exogenous antigen, such as a drug
metabolite, that is adsorbed on a cell surface or matrix. In either case, the hypersensitivity reaction
results from the binding of antibodies to normal or altered cell-surface antigens. Clinically, antibody-
mediated cell destruction and phagocytosis occur in the following situations: (1) transfusion
reactions, in which cells from an incompatible donor react with and are opsonised by preformed
antibody in the host; (2) erythroblastosis fetalis, in which there is an antigenic difference between the
mother and the fetus, and antibodies (of the IgG class) from the mother cross the placenta and cause
destruction of fetal red cells; (3) autoimmune haemolytic anaemia, agranulocytosis, and
thrombocytopenia, in which individuals produce antibodies to their own blood cells, which are then
destroyed; and (4) certain drug reactions, in which antibodies are produced that react with the drug,
which can be attached to the surface of erythrocytes or to other cells.
2.4
Answer: E Vasodilation
Inflammation is the first response of the immune system to infection or irritation. Inflammation is
characterised by the following quintet: redness (rubor), heat (calor), swelling (tumur), pain (dolor)
and dysfunction of the organs involved (functio laesa). The first four characteristics have been known
since ancient times and are attributed to Celsus; functio laesa was added to the definition of
inflammation by Rudolf Virchow in 1858.
Inflammation is not a synonym for infection. Even in cases when it is caused by infection it is
incorrect to use the terms as synonyms — infection is caused by an outside agent, while inflammation
is the body’s response.
Inflammation has two main components, vascular (exudative) and cellular. The exudative component
involves the movement of fluid, usually containing many important proteins such as fibrin and
immunoglobulins (antibodies). Blood vessels are dilated upstream of an inflamed area (causing
redness and heat) and constricted downstream, while capillary permeability to the affected tissue is
increased, resulting in a net loss of blood plasma into the tissue, giving rise to oedema or swelling.
The swelling distends the tissues, compresses nerve endings, and so causes pain.
The cellular component involves the movement of white blood cells from blood vessels into the
inflamed tissue. The white blood cells, or leukocytes, take on an important role in inflammation: they
extravasate (filter out) from the capillaries into tissue, and act as phagocytes, picking up bacteria and
cellular debris. They may also aid by walling off an infection and preventing its spread.
If inflammation of the affected site persists, released cytokines, interleukin-1 (IL-1) and tumour
necrosis factor (TNF) will activate endothelial cells to up-regulate receptors VCAM-1, ICAM-1,
Eselectin, and L-selectin for various immune cells. Receptor up-regulation increases extravasation of
neutrophils, monocytes, activated T-helper and T-cytotoxic cells, and memory T and B cells to the
inflamed site.
Neutrophils are characteristic of inflammation in the early stages. They are the first cells to appear in
an infected area, and any section of recently inflamed tissue (within a couple of days or so) viewed
under a microscope will appear packed with them. They are easily identified by their multilobed
nuclei and granular cytoplasm and perform many important functions, including phagocytosis and the
release of extracellular chemical messengers. Neutrophils only live for a couple of days in these
interstitial areas, so if the inflammation persists for any longer they are gradually replaced by longer-
lived monocytes.
2.5
VCAM-1 (vascular cell adhesion molecule-1), also known as ‘CD106’, is a molecule with a
considerable role in the human immune system. It contains six or seven immunoglobulin domains, and
is expressed on both large and small vessels only after the endothelial cells are stimulated by
cytokines. Up-regulation of VCAM-1 in endothelial cells by cytokines occurs as a result of increased
gene transcription (eg in response to TNF-α and IL-1) and through stabilisation of mRNA (eg in
response to IL-4). The promoter region of the VCAM-1 gene contains functional tandem NF-κB sites.
The sustained expression of VCAM-1 lasts over 24 hours. Primarily, VCAM-1 is an endothelial
ligand for VLA-4 (very late antigen-1 or α4β1) of the β1 subfamily of integrins, and for integrin α4β7.
VCAM-1 expression has also been observed in other cell types (eg smooth muscle cells). VCAM-1
promotes the adhesion of lymphocytes, monocytes, eosinophils and basophils. Interestingly, certain
melanoma cells can use VCAM-1 to adhere to the endothelium, and VCAM-1 might participate in
monocyte recruitment to atherosclerotic sites. As a result, VCAM-1 is a potential drug target.
2.6
Skin allergy testing is a method used for medical diagnosis of allergies. A microscopic amount of an
allergen is introduced to a patient’s skin by:
• pricking the skin with a needle or pin containing a small amount of the allergen (’prick testing’ or
‘scratch testing’); or by
• applying a patch to the skin, where the patch contains the allergen.
If an immune response is seen in the form of a rash, hives, urticaria or, much worse, anaphylaxis, it
can be concluded that the patient has a hypersensitivity (or allergy) to that allergen. Further testing
can be performed in order to identify the particular allergen. Release of histamine from mast cells
produces the characteristic response in a skin test.
2.7
Platelet-derived growth factor (PDGF) is one of the numerous growth factors, or proteins that
regulate cell growth and division. In particular, it plays a significant role in angiogenesis, the growth
of blood vessels from already existing blood vessel tissue. Uncontrolled angiogenesis is a
characteristic of cancer. PDGF also plays a role in embryonic development, cell proliferation and
cell migration. PDGF has also been linked to several diseases, including atherosclerosis, fibrosis and
malignant diseases.
In addition, PDGF has demonstrated that not only is it a cell proliferator, but a required element in
cellular division for fibroblasts. Specifically, fibroblasts have PDGF receptors on their plasma
membranes that allow for their signal transductions from PDGFs. These plasma membrane receptors
are called ‘tyrosine kinases’ and allow the signal transduction and inevitably the cell division
stimulation to occur. In essence, the PDGFs allow a cell to skip the G1 checkpoints in order to
divide.
2.8
In type III hypersensitivity, soluble immune complexes (aggregations of antigens and IgG and IgM
antibodies) form in the blood and are deposited in various tissues (typically the skin, kidney and
joints), where they can trigger an immune response according to the classical pathway of complement
activation. The reaction takes hours to days to develop. Some clinical examples include:
• Immune-complex glomerulonephritis
• Rheumatoid arthritis
• Serum sickness
• Arthus reaction.
2.9
Many local type I hypersensitivity reactions have two well-defined phases. The immediate, or initial
response is characterised by vasodilation, vascular leakage and, depending on the location, smooth
muscle spasm or glandular secretions. These changes usually become evident within 5–30 minutes
after exposure to an allergen and tend to subside in 60 minutes. In many instances (eg allergic rhinitis,
bronchial asthma), a second, late-phase reaction sets in, 2–24 hours later without additional exposure
to antigen and can last for several days. This late-phase reaction is characterised by infiltration of
tissues with eosinophils, neutrophils, basophils, monocytes and CD4+ Tcells, as well as tissue
destruction, typically in the form of mucosal epithelial cell damage.
2.10
In type II hypersensitivity the antibodies produced by the immune response bind to antigens on the
patient’s own cell surfaces. The antigens recognised in this way can either be intrinsic (‘self’ antigen,
innately part of the patient’s cells) or extrinsic (absorbed onto the cells during exposure to some
foreign antigen, possibly as part of an infection with a pathogen, or a drug metabolite). Type II
hypersensitivity diseases are mediated by:
• Inappropriate hindering or stimulating of normal cellular processes. This mechanism does not
involve tissue damage but occurs when IgG antibody acts as an analogue by binding to cell-surface
receptors for hormones or neurotransmitters. This binding can cause non-specific stimulation of a
cellular process or can inhibit a process by preventing normal ligand binding to the receptor. An
example of this type of interference is seen in a form of insulin-resistant diabetes, where
autoantibody bound to the insulin receptor initially mimics insulin activity but after prolonged
exposure induces insulin resistance.
• Erythroblastosis fetalis
• Pemphigus
• Pernicious anaemia
• Immune thrombocytopenia
• Transfusion reactions
• Hashimoto’s thyroiditis
• Graves’ disease
• Myasthenia gravis
• Farmer’s lung
• Rheumatic fever
2.11
Interferon-gamma (IFN-γ) is the only member of the type II class of interferons. It is a dimerised
soluble cytokine that was originally called ‘macrophage-activating factor’. In human beings, the gene
which codes for IFN-γ is located on chromosome 12. It is produced by a variety of immune cells.
These include: T-helper cells, 0 and 1 types (CD4); cells with immunological memory (CD45PA); T-
killer cells (CD8); natural killer (NK) cells (CD16, CD56); dendrite cells (CD23, CD35); and B
lymphocytes (CD22, CD23). During secretion, IFN-γ influences the secreting cells (as well as the
surrounding cells) through IFN-γ receptors. The first necessary step in the functioning of IFN-γ is the
interaction of IFN-γ with receptors located on the surface of the cells. IFN-γ can stimulate defending
or pathological effects. It can induce the differentiation process of myeloid cells in bone marrow,
forming cells with high-affinity Fcγ receptors, which combine with IgG monomers. In contrast, in
mature granulocytes, IFN-γ induces the expression of medium-affinity Fcγ receptors, which combine
with only the aggregated IgG. IFN-γ also activates the antibody-dependent cytotoxins, implemented
by the matured granulocytes.
IFN-γ activates macrophages and thus increases their anti-tumour activities. If the macrophages are
infected by intracellular parasites, it activates macrophages which in turn destroy the parasites.
Suppression of intracellular parasites under the influence of IFN-γ takes place in non-macrophage
cells as well.
IFN-γ reinforces the anti-tumour activities of the cytotoxic lymphocytes. Together with lymphotoxins
CD4 or CD8, produced by lymphocytes, it suppresses the growth of the tumour cells. IFN-γ induces
expression of the receptors of lymphotoxins by acting in the nucleus of the target cells. It increases the
non-specific activities of NK cells. It is one of the factors that controls the differentiation of B cells.
It can either increase or decrease B-cell immune responses. In the late stages, for example, IFN-γ
increases the secretion of immunoglobins.
IFN-γ plays an very important role in increasing the expression of HLA class I and II molecules on
the cell membranes. Moreover, IFN-γ induces the expresion of HLA-DR and HLA-DP molecules
more quickly than the expression of HLA-DQ molecules. If the expression of HLA class I and II
molecules on the pathological cells takes place more vigorously, then they become more readily
recognised for the destructive process.
In the case of viral infections, IFN-γ can cause considerable changes on the surface of cell membrane
which inhibit the adhesion and penetration of virus into the cells. IFN-γ promotes the synthesis of
oligoadenylate synthetase in cells. The polymers of oligoadenylate activate the endogenous
endonucleases, which promote the destruction of mRNA and rRNA, disturbing the intracellular
synthesis in viral cells. IFN-γ also promotes the formation of ferment-proteinkinases, resulting in a
decrease in protein synthesis. IFN-γ also activates osteoclasts, increasing the resorption of bone-
tissue.
2.12
Prostaglandin and bradykinin production is associated with pain. Bradykinin, a product of the kinin-
kallikrein system, is a physiologically and pharmacologically active peptide of the kinin group of
proteins. It is composed of nine amino acids. It is produced by proteolytic cleavage of its kininogen
precursor, high-molecular weight kininogen (HMWK), by the enzyme kallikrein. Bradykinin is a
potent endothelium-dependent vasodilator, causes contraction of non-vascular smooth muscle,
increases vascular permeability, and also is involved in the mechanism of pain. In some aspects, it
has similar actions to those of histamine and, like histamine, is released from venules rather than from
arterioles.
Bradykinin raises internal calcium levels in neocortical astrocytes, causing them to release glutamate.
Bradykinin is also thought to be the cause of the dry cough experienced by some patients who are on
angiotensin-converting enzyme (ACE) inhibitor drugs. This refractory cough is a common cause for
stopping ACE-inhibitor therapy.
2.13
The glass fragment is a foreign material and produces a foreign body reaction characterised by giant
cells. Mast cells are not numerous in tissues and do not proliferate significantly as part of a foreign
body inflammatory response. Eosinophils are typical of parasitic and allergic inflammatory
responses, not of inflammatory responses to foreign bodies. A few neutrophils might be present, but
giant cells are more specific for a foreign body inflammatory response. Plasma cells are more typical
of chronic inflammation.
2.14
Hydrogen peroxide is reduced by myeloperoxidase to form a powerful oxidant that kills bacteria. In
chronic granulomatous disease, this system is not operative against many bacterial species. Platelet-
activating factor leads to leukocyte aggregation, adhesion, and chemotaxis, but does not directly
facilitate bacterial destruction. Prostaglandins potentiate vasodilation, but not neutrophil function.
Kallikrein is released from neutrophil lysosomes and promotes bradykinin formation, leading to
vasodilation. Leukotrienes promote vascular permeability, but not leukocyte function.
2.15
This patient has developed interstitial lung disease over the course of several years. Growth factors
are released from macrophages that have ingested silica crystals and these factors stimulate collagen
production by fibroblasts. Leukotrienes are more important mediators in acute inflammation, leading
to chemotaxis and increased vascular permeability, among other functions. Foreign body giant cells
are not a significant feature of silicosis. Plasma cell proliferation is not a key feature of silicosis or of
other pneumoconioses. Histamine release by mast cells is an acute inflammatory response.
2.16
The short course of 1 day and the purulent exudate are typical features of acute inflammation. The
acute inflammatory response has three main functions:
• The affected area is occupied by a transient material called the ‘acute inflammatory exudate’. The
exudate carries proteins, fluid and cells from local blood vessels into the damaged area to mediate
local defences.
• If an infective causative agent (eg bacteria) is present in the damaged area, it can be destroyed and
eliminated by components of the exudate.
• The damaged tissue can be broken down and partially liquefied, and the debris removed from the
site of damage.
Acute inflammation can result from physical damage, chemical substances, micro-organisms, or other
agents. The inflammatory response consists of changes in blood flow, increased permeability of
blood vessels, and escape of cells from the blood into the tissues. The changes are essentially the
same whatever the cause and wherever the site.
Acute inflammation is short-lived, lasting only a few days. If inflammation lasts longer than this, it is
referred to as ‘chronic inflammation’. Examples of acute inflammation include sore throat, reactions
in the skin to a scratch or a burn or insect bite, and acute hepatitis.
Granulomatous inflammation typically pursues a course lasting months to years. An abscess usually
takes a bit longer to form than 1 day, and the purulent inflammation is localised to an abscess cavity.
With resolution, the purulent exudate should be gone, and the course would probably be longer.
Chronic inflammation pursues a course of weeks, and purulent exudate is not typical.
2.17
Answer: C Macrophage
Epithelioid cells and giant cells are derived from macrophages and are important in the development
of granulomatous inflammation, as seen in this patient with tuberculosis. The tuberculous granuloma
(caseating tubercle) has central caseous necrosis bordered by giant multinucleated cells (Langhans
cells), and surrounded by epithelioid cell aggregates, lymphocytes and fibroblasts. Granulomatous
tubercles tend to become confluent. Multinucleated giant cells (mature, Langerhans-type) are 50–100
µm in diameter, with numerous small nuclei (over 20) lying peripherally in the cell (in a crown or
horseshoe distribution). They have abundant eosinophilic cytoplasm. They are formed when activated
macrophages merge. Epithelioid cells are activated macrophages that resemble epithelial cells. They
are elongated, with finely granular, pale eosinophilic (pink) cytoplasm and a central, ovoid nucleus.
They have indistinct shape and contour and form aggregates. At the periphery of the tubercle are the
lymphocytes (T-cells) and rare plasma cells and fibroblasts. Caseous necrosis is a central area,
amorphous, finely granular, and eosinophilic. If recent, it can contain nuclear fragments. The caseous
material is the result of the accumulated destruction of giant cells and epithelioid cells.
Fibroblasts lay down collagen. Although collagen is laid down around a granuloma as part of the
response, it is not the major component involved in granuloma formation. Although some neutrophils
might be present in a granuloma, they are not the major contributors to granuloma formation. Mast
cells are few in number. They are involved in the release of mediators that are primarily involved in
acute inflammatory responses. Platelets are mainly involved in coagulation.
2.18
C5a is a protein fragment released from complement component C5. It is chemotactic for neutrophils
similar to tumour necrosis factor (TNF), leukotrienes, and bacterial products. In humans, the
polypeptide contains 74 amino acids but is rapidly metabolised by a serum enzyme, carboxypeptidase
B to a 73-amino-acid form, C5a des-Arg. C5a is an anaphylatoxin and causes the release of histamine
from mast cells. C5a des-Arg is a much less potent anaphylatoxin. However, both forms are effective
leukocyte chemoattractants, causing the accumulation of white blood cells, especially neutrophil
granulocytes, at sites of complement activation.
C5a binds to a receptor protein on the surface of target cells, C5aR or CD88. This is a member of the
G-protein-coupled receptor superfamily of proteins, predicted to have seven transmembrane helical
domains of largely hydrophobic amino acid residues, forming three intracellular and three
extracellular loops, with an extracellular N-terminus and an intracellular C-terminus. C5a binding to
the receptor is a two-stage process: an interaction between basic residues in the helical core of C5a
and acidic residues in the extracellular N-terminal domain allows the C-terminus of C5a to bind to
residues in the receptor transmembrane domains. The latter interaction leads to receptor activation,
and the transduction of the ligand binding signal across the cell plasma membrane to a cytoplasmic G
protein.
2.19
Prostaglandins are pain mediators produced via the cyclo-oxygenase pathway of arachidonic acid
metabolism. Aspirin and non-steroidal anti-inflammatory drugs block the synthesis of prostaglandins,
which can reduce pain. Aspirin suppresses the production of prostaglandins and thromboxanes
through non-competitive and irreversible inhibition of the cyclo-oxygenase (COX) enzyme. Cyclo-
oxygenase is required for prostaglandin and thromboxane synthesis. Aspirin acts as an acetylating
agent, where an acetyl group is covalently attached to a serine residue in the active site of the COX
enzyme. This makes aspirin different from other nonsteroidal anti-inflammatory drugs (NSAIDs) such
as diclofenac and ibuprofen, which are reversible inhibitors.
Prostaglandins are local hormones (paracrine) produced in the body and have diverse effects in the
body, including but not limited to transmission of pain information to the brain, modulation of the
hypothalamic thermostat, and inflammation. Thromboxanes are responsible for the aggregation of
platelets and aspirin prevents platelet aggregation by blocking the synthesis of thromboxanes.
Aspirin has two additional modes of action, contributing to its strong analgesic, antipyretic and anti-
inflammatory properties:
• It induces the formation of NO-radicals in the body that enable the white blood cells (leukocytes) to
fight infections more effectively.
More recent data suggest that salicylic acid and its derivatives will modulate NF-κB signalling. NF-
κB is a transcription factor complex that plays a central role in many biological processes, including
inflammation.
2.20
The C3b is a protein fragment released from complement component C3. It serves as an effective
opsonin. Soluble C3-convertase, also known as ‘iC3Bb’, catalyses the proteolytic cleavage of C3
complement component into C3a and C3b as part of the alternative complement system. C3b can then
bind to microbial cell surfaces within an organism’s body. This can lead to the production of surface-
bound C3 convertase and so more C3b components. Also known as ‘C3bBb’, this convertase is
similar to soluble C3-convertase, except that it is membrane bound. Alternatively, bound C3b can
help in the opsonisation of the microbe by macrophages. Complement receptor-1 (CR1) on
macrophages allows the engaging of C3b-covered microbes.
Glutathione peroxidase does not act as an opsonin, but is involved in scavenging free radicals. IgM
does not act as an opsonin, although IgG does. Selectins aid in the initial binding of leukocytes to
endothelial surfaces. NADPH oxidase in leukocytes aids in the killing of phagocytosed microbes.
2.21
A food allergy is an allergic (type I hypersensitivity) reaction to a particular food. Food allergies are
typically seen in 6%–8% of children and 2% of adults. Many different foods can cause allergic
reactions. However, food allergies are most commonly triggered by certain nuts, peanuts, shellfish,
fish, milk, eggs, wheat, and soybeans. Allergic reactions to food may be severe and sometimes
include an anaphylactic reaction. There is currently no cure for food allergies. Treatment consists of
avoidance diets, where the allergic person avoids any and all forms of the food to which they are
allergic. For people who are extremely sensitive, this can involve the total avoidance of any exposure
with the allergen, including touching or inhaling the problematic food as well as any surfaces that may
have come into contact with it. Food allergy is distinct from food intolerance, which is not caused by
an immune reaction.
2.22
These non-specific findings, put together, suggest the possibility of autoimmune disease. An
antinuclear antibody (ANA) test is a good way to begin the investigations, then more specific tests
can be ordered. The ANA test measures the pattern and amount of autoantibody, which can attack the
body’s tissues as if they were foreign material. Everyone has a small amount of autoantibody but in
about 5% of the population this is raised, and about half of this 5% have an autoimmune disease. The
test is used to aid in the diagnosis of systemic lupus erythematosus and drug-induced lupus, but may
also be positive in cases of scleroderma, Sjögren’s syndrome, Raynaud’s disease, juvenile chronic
arthritis, rheumatoid arthritis, antiphospholipid antibody syndrome, autoimmune hepatitis, and many
other autoimmune and non-autoimmune diseases. A thorough medical history, physical examination
and other tests are needed to confirm the presence of a suspected autoimmune disease.
The findings given in the scenario are non-specific and do not point to a septicaemia. A decreased
CD4 lymphocyte count is more typical of AIDS (an immune deficiency) than of an immunologically
mediated disease process such as systemic lupus erythematosus. Creatine phosphokinase (CPK) is
more useful for diagnosis of acute disease (it is raised after myocardial infarction). CPK can be
increased with myositis, but with the multiple manifestations in this patient, it is not the best screening
test. The sedimentation rate is a very non-specific test that is elevated in many inflammatory
conditions. The history here points more to an immunological disease.
2.23
Recurrent bacterial infections suggest a lack of B-cell immune function, with lack of gamma globulin
production. Once maternal antibody is gone, the disease manifests more severely. X-linked
agammaglobulinaemia (also known as ‘X-linked hypogammaglobulinaemia’, ‘XLA’ or ‘Bruton-type
agammaglobulinaemia’) is a rare X-linked genetic disorder that affects the body’s ability to fight
infection. XLA is classified with other inherited defects of the immune system in a group of disorders
known as primary immunodeficiency disorders.
It results from mutations in a gene on the X chromosome that encodes the Bruton tyrosine kinase (Btk).
Btk is essential for B-cell development and maturation; without it, there are no B-cells and no
antibodies. People with XLA do not generate mature B-cells that are capable of manufacturing
immunoglobulins.
Patients with untreated XLA are prone to develop serious and even fatal infections. Patients typically
present in early childhood with recurrent infections, particularly with extracellular, encapsulated
bacteria. XLA is an X-linked disorder, and therefore is almost always limited to males, with a
frequency of about 1 in 100,000 male newborns, and shows no ethnic predisposition. XLA is treated
by infusion of human gamma globulins. Treatment with pooled gamma globulins cannot restore a
functional population of B-cells, but it is sufficient to reduce the severity and number of infections
due to the passive immunity afforded by the exogenous antibodies.
Acute leukaemias can result in immune deficiency, but not so selective and not typically in infancy.
DiGeorge syndrome is predominantly the result of a loss of T-cell function. This is pronounced in
complete DiGeorge syndrome, but less severe in partial DiGeorge syndrome. There are often
congenital anomalies. Epstein–Barr virus infection does not lead to immunodeficiency states. A
selective IgA deficiency is seen in about 1 in 600 people and leads to mild diarrhoea and/or
occasional respiratory tract infections, but is not life-threatening.
2.24
The human leukocyte antigen (HLA) system, the major histocompatibility complex (MHC) in humans,
is located on chromosome 6. It encodes cell surface molecules specialized to present antigenic
peptides to the T-cell receptor (TCR) on T-cells. MHC molecules which present antigen are divided
into 2 major classes: Class I and Class II. HLA were originally defined as cell-surface antigens that
mediate graft-versus-host disease, which results in the rejection of tissue transplants in HLA
mismatched donors. Identification of these antigens has led to greater success and longevity in organ
transplant patients.
Class II HLA antigens are part of the process of CD4 lymphocyte activation. HLA-DR is a major
histocompatibility complex, class II, cell surface receptor encoded by the human leukocyte antigen
complex on chromosome 6, region 6p21.31. Receptor is frequently found with ligand, a peptide that
is nine amino acids in length or longer, within the binding groove. The receptor/peptide complex is a
ligand for the TCR. HLA-DR is also involved in several autoimmune conditions, disease
susceptibility and disease resistance. It is also closely linked to HLA-DQ and this linkage often
makes it difficult to elucidate the principle causative factor in diseases.
Amyloidosis is a complication of chronic inflammation that has lasted for many years or of multiple
myeloma. Cell lysis by CD8 lymphocytes is a function of class I antigen recognition. Graft-versus-
host disease is not a consequence of renal transplantation, and few if any donor lymphocytes are
present in the allograft. Serum sickness is caused by antigen–antibody complexes.
2.25
2.26
Graft-versus-host disease does not produce a granulomatous reaction. Granulomatous reactions are
based mainly on cell-mediated immunity. Type I reactions are associated with allergy and
anaphylaxis. Type II reactions are associated with complement-mediated immune reactions.
2.27
Answer: D It is an appetite suppressant
Tumour necrosis factor (TNF), which used to be known as ‘tumour necrosis factor alpha’, is a
cytokine involved in systemic inflammation and is a member of a group of cytokines that all stimulate
the acute phase reaction. TNF causes apoptotic cell death, cellular proliferation, differentiation,
inflammation, tumorigenesis, and viral replication. TNF’s primary role is in the regulation of immune
cells. Dysregulation and, in particular, overproduction of TNF have been implicated in a variety of
human diseases, including cancer.
The 17-kDa TNF protomers (185 amino acids long) are composed of two antiparallel β-pleated
sheets with antiparallel β-strands, forming a Swiss roll β-structure, typical of the TNF family, but
also found in viral capsid proteins.
TNF is mainly produced by macrophages, but also by a wide variety of other cell types, including
lymphoid cells, mast cells, endothelial cells, cardiac myocytes, fibroblasts and neuronal tissue. Large
amounts of sTNF are released in response to lipopolysaccharides, other bacterial products, and
interleukin-1 (IL-1).
TNF has a number of actions on various organ systems, generally together with IL-1 and IL-6:
suppresses appetite
causes fever.
stimulates the acute phase response, leading to an increase in C-reactive protein and a number of
other mediators
• Effects on neutrophils:
attracts neutrophils very potently, helping them to stick to the endothelial cells for migration.
• Effects on macrophages:
stimulates phagocytosis and production of IL-1 oxidants and the inflammatory lipid, prostaglandin
E2 (PGE2).
A locally increasing concentration of TNF will cause the cardinal signs of inflammation to occur:
heat, swelling, redness and pain. High concentrations of TNF induce shock-like symptoms, whereas
prolonged exposure to low concentrations of TNF can result in cachexia, a wasting syndrome. This
can be found for example in patients with cancer.
2.28
Interleukin-1 (IL-1) actually composed of two distinct proteins, IL-1α and IL-1β, is one of the first
cytokines ever described. Its initial discovery was as a factor that could induce fever, control
lymphocytes, increase the number of bone marrow cells and cause degeneration of bone joints. It is
also known as ‘endogenous pyrogen’, ‘lymphocyte-activating factor’, ‘haemopoetin-1’ and
‘mononuclear cell factor’. Both, IL-1α and IL-1β, belong to a family of cytokines known as the
‘interleukin-1 superfamily’.
The original members of the IL-1 superfamily are IL-1α, IL-1β and the IL-1 receptor antagonist (IL-
1RA). IL-1α and IL-β are pro-inflammatory cytokines involved in immune defence against infection.
IL-1RA is a molecule that competes for receptor binding with IL-1α and IL-1β, blocking their role in
immune activation. Both IL-1α and IL-1β are produced by macrophages, monocytes and dendritic
cells. They form an important part of the inflammatory response of the body against infection. These
cytokines increase the expression of adhesion factors on endothelial cells, enabling transmigration of
leukocytes to sites of infection and re-set the hypothalamic thermoregulatory centre, leading to an
increased body temperature which expresses itself as fever. IL-1 is therefore called an ‘endogenous
pyrogen’. The increased body temperature helps the body’s immune system to fight infection. IL-1 is
also important in the regulation of haematopoiesis.
IL-1α and IL-1β are produced as precursor peptides which are then processed to release a shorter,
active molecule, which is the ‘mature’ protein. Mature IL-1β, for instance, is released from Pro-IL-1β
following cleavage by a member of the caspase family of proteins (caspase-1) or by the interleukin-
1-converting enzyme (ICE). The three-dimensional structure of the mature forms of each member of
the human IL-1 superfamily is composed of 12 to 14 β strands, producing a barrel-shaped protein.
2.29
Answer: A It binds to a cytosolic protein (cyclophilin) of immunocompetent lymphocytes
Ciclosporin is an immunosuppressant drug that is widely used after allogeneic organ transplantation
to reduce the activity of the patient’s immune system and so the risk of organ rejection. It has been
studied in transplants of skin, heart, kidney, lung, pancreas, bone marrow and small intestine.
Ciclosporin is a cyclic nonribosomal peptide of 11 amino acids (an undecapeptide) produced by the
fungus Tolypocladium inflatum Gams and was initially isolated from a Norwegian soil sample.
Apart from its use in transplant medicine, ciclosporin is also used in psoriasis and infrequently in
rheumatoid arthritis and related diseases, although it is only used in severe cases. It has been
investigated for use in many other autoimmune disorders. Ciclosporin has also been used to help treat
patients with ulcerative colitis who do not respond to treatment with steroids, and is also used to treat
non-infective posterior or intermediate uveitis. Ciclosporin has been investigated as a possible
neuroprotective agent in conditions such as traumatic brain injury, and has been shown in animal
experiments to reduce brain damage associated with injury. As mentioned above, ciclosporin blocks
the formation of the mitochondrial permeability transition pore, which has been found to cause much
of the damage associated with head injury and neurodegenerative diseases.
Treatment with ciclosporin can be associated with a number of potentially serious adverse drug
reactions and drug interactions. Ciclosporin interacts with a wide variety of other drugs and other
substances, including grapefruit juice, although this interaction with grapefruit juice has been used
beneficially to increase the blood level of ciclosporin in expermental studies.
Adverse drug reactions include gum hyperplasia, convulsions, peptic ulcers, pancreatitis, fever,
vomiting, diarrhoea, confusion, breathing difficulties, numbness and tingling, pruritis, high blood
pressure, potassium retention and possibly hyperkalaemia, nephrotoxicity and hepatotoxicity, and also
an increased vulnerability to opportunistic fungal and viral infections. Cytokine release syndrome is
seen after anti-thymocyte globulin administration.
2.30
Answer: C Is complement-mediated
2.31
Answer: B Lymphocytes
The lymphocyte and monocyte series are the characteristic cells seen in more prolonged forms of
inflammation. The eosinophils and the neutrophils are usually associated with early or acute
inflammatory responses. The mast cell is involved in liberating histamine in early inflammation. The
platelets, although they may aggregate, are not characteristic of either type of inflammation.
2.32
Answer: E Proliferation of new capillaries, with fibroblasts and new collagen formation
Granulation tissue is the first phase of the healing process at the end of acute inflammation. New
capillaries proliferate in the tissue, with fibroblasts and the first laying-down of new collagen, which
eventually will become largely avascular scar tissue. It is not to be confused with granulomatous
inflammation or granuloma, which is the hallmark of a form of chronic inflammation in which healing
and the stimulation for damage occur concurrently. Granulation tissue is an abnormal response in the
normal healing process.
• Light red or dark-pink in colour, being perfused with new capillary loops or ‘buds’
• Moist
• Granular in appearance.
2.33
Answer: A Chemotaxis
During the vascular stasis stage of hyperaemia, neutrophils and monocytes adhere to the vascular
endothelium prior to migration into the extravascular space in a process known as ‘margination’.
Leukocytes emigrate (diapedesis) through gaps between the endothelial cells. Chemotaxis is the
process by which leukocytes undergo unidirectional migration towards a specific target. Various
chemotactic substances or factors influence the rate of movement of the cells. Several chemotactic
factors have an apparently specific action on selected cell types.
2.34
There is evidence that most, if not all macrophages originate from a committed bone marrow stem
cell, which differentiates into a monoblast and then a promonocyte, which in turn matures into a
monocyte in the circulating peripheral blood. When called upon, the circulating monocyte can enter
into an organ or tissue bed as a tissue macrophage (previously called a ‘histiocyte’). Examples of
tissue macrophages are Kupffer cells (liver), alveolar macrophages (lung), osteoclasts (bone),
Langerhans cells (skin), microglial cells (central nevous system), and possibly the dendritic
immunocytes of the dermis, spleen and lymph nodes. The entire system, including the peripheral
blood monocytes, constitutes the mononuclear phagocyte system.
2.35
Langhans giant cells are the hallmarks of granulomatous inflammation. They are formed by the fusion
of epithelioid cells (macrophages) and contain nuclei arranged in a horseshoe-shaped pattern in the
cell periphery.
2.36
Answer: A Allograft
This type of graft is an allograft (ie the graft is transferred between two members of the same
species). Because the donor and recipient are histo-incompatible (genetic differences exist in the
human leukocyte antigen or HLA system; genetic differences are also likely in minor
histocompatibility loci), rejection of the graft is likely but can be controlled with immunosuppressive
therapy (eg with ciclosporin). ‘Isograft’ and ‘syngraft’ are synonymous terms. They refer to a graft
transferred between genetically identical individuals (ie identical twins) or highly inbred laboratory
animals. The donor and recipient are histocompatible and rejection will not occur. An autograft is
performed by transferring cells or tissues from one part of the body to another part of the same
individual. Autografting of skin and bone is performed quite commonly. Rejection will not occur after
this kind of graft. A xenograft is tissue transferred from a member of one species to a member of
another species (eg baboon to human). The donor and recipient are histo-incompatible. Rejection is
highly likely without immunosuppressive therapy.
2.37
The patient in this clinical scenario was most likely to be experiencing acute rejection. This type of
rejection is primarily a cell-mediated attack against the transplanted organ. Antigen presentation can
occur in at least two different ways. In the direct path, blood-borne cells within the graft (ie
‘passenger’ dendritic cells and monocytes expressing class I and class II major histocompatibility
complex [MHC] molecules) act as the antigen-presenting cells. In the indirect path, the recipient’s
antigen-presenting cells could be presenting allogeneic class I and class II molecules that have been
shed by the graft. Peptides presented in the context of MHC class I molecules will activate CD8-
positive T-cytotoxic lymphocytes, which can directly attack the graft. Peptides presented in the
context of MHC class II molecules can activate CD4-positive T-helper cells, resulting in secretion of
helper factors. This intensifies the rejection by expansion of antigen-specific cytotoxic T-cell clones
and accumulation and activation of macrophages. Antilymphocyte globulin (anti-T-cell antibodies
made in an animal) is the agent used to reverse acute rejection episodes. Without immunosuppression
of the recipient, allograft rejection usually takes place in approximately 11–14 days.
Chronic rejection is a slow, smouldering phenomenon that is poorly understood. The mechanisms that
are responsible can include monocyte secretion of interleukin-1 and release of platelet-derived
growth factor (PDGF) from platelets and endothelial cells. If it is going to occur, it generally begins
several months to years after the transplantation is performed. It is characterised by proliferation of
endothelial cells of blood vessels feeding the graft and fibrotic changes within the vessel wall.
Blockage of the vessels results in necrosis and demise of the graft. It is not reversed by
antilymphocyte globulin.
In a graft-versus-host reaction, the cells of the graft attack the recipient, either directly or indirectly,
by recruiting the recipient’s cells to attack the host. This type of reaction is seen primarily in
recipients of bone marrow grafts.
The case history does not mention reduction in drug dosage. Administration of antilymphocyte
globulin will not reverse immunosuppressive drug toxicity.
2.38
In this scenario the husband, who has blood group A, has preformed immunoglobulin M (IgM) against
the B antigen on the transfused erythrocytes. The IgM binds to the cells, activates complement, and
lysis occurs (type II hypersensitivity). The antibody–antigen complexes on the erythrocyte surface
also activate Hageman factor, which leads to production of kinins (vasoactive peptides that cause
hypotension and shock by increasing capillary permeability). If the infusion of ABO-incompatible
blood is not stopped promptly, kidney failure caused by renal vasoconstriction and intravascular
thrombi can occur.
IgM isohaemagglutinins begin to appear shortly after birth, presumably because of colonisation of the
gastrointestinal and respiratory tracts by normal flora, which have oligosaccharide antigenic
determinants that are similar to those on the A and B antigens. However, an individual will produce
antibodies only against those antigens that are different from self.
The term ‘isohaemagglutinin’ refers to antibodies in one species that are directed against major
antigens on erythrocytes of other individuals of the same species.
2.39
The infant is at risk of developing haemolytic disease of the newborn. The mechanism involves
passage of Rh-positive erythrocytes from the fetus into the blood circulation of a Rh-negative mother,
which usually occurs near the time of delivery. The mother becomes sensitised and can begin
producing anti-Rh antibodies. The firstborn child is generally not at great risk, because it is born
before an appreciable level of immunoglobulin G (IgG) antibody is formed. However, in a
subsequent pregnancy with an Rh-positive fetus, the mother can have a strong secondary immune
response. Maternal anti-Rh antibody of the IgG isotype crosses the placenta and binds to the fetal red
blood cells, which are removed by macrophages (type II hypersensitivity). IgG antibodies against the
Rh antigen are the principal cause of haemolytic disease of the newborn.
2.40
Infusion of Rh-positive red blood cells could further increase the risk of the mother producing anti-Rh
antibodies. Plasma from randomly selected donors is useful in immunodeficiency diseases. Some
preparations could conceivably contain anti-Rh antibodies, but this is not the best choice. Albumin
from Rh-positive donors or platelet transfusion would not protect the infant against anti-Rh
antibodies.
2.41
Answer: D Monocytes
A monocyte is a leukocyte that protects against blood-borne pathogens and moves quickly (in
approximately 8–12 hours) to sites of infection in the tissues. Monocytes are usually identified in
stained smears by their large bilobed nucleus.
Monocytes are produced by the bone marrow from haemopoietic stem cell precursors called
‘monoblasts’. Monocytes circulate in the bloodstream for about 1–3 days and then typically move into
tissues throughout the body. They comprise 3%–8% of the leukocytes in the blood. In the tissues,
monocytes mature into different types of macrophages at different anatomical locations.
Monocytes are responsible for phagocytosis (ingestion) of foreign substances in the body. Monocytes
can perform phagocytosis using intermediary (opsonising) proteins such as antibodies or complement
that coat the pathogen, as well as by binding to the microbe directly via pattern-recognition receptors
that recognise pathogens. Monocytes are also capable of killing infected host cells via antibody
(antibody-mediated cellular cytotoxicity). A cell that has recently phagocytosed foreign matter can
have a vacuolated appearance.
2.42
Answer: E Neutrophils
Neutrophils are the most abundant type of white blood cell and form an integral part of the immune
system. Their name is derived from the staining characteristics on haematoxylin and eosin
histological preparations: whereas basophilic cellular components stain dark-blue and eosinophilic
components stain bright-red, neutrophilic components stain a neutral pink. These phagocytes are
normally found in the bloodstream. However, during the acute phase of inflammation, particularly as
a result of bacterial infection, neutrophils leave the vasculature and migrate towards the site of
inflammation in a process called ‘chemotaxis’. They are the predominant cells in pus, accounting for
its whitish appearance. Neutrophils react within an hour of tissue injury and are the hallmark of acute
inflammation.
Neutrophil granulocytes have an average volume of 330 fl and a diameter of 12–15 µm in peripheral
blood smears. Together with the eosinophil and the basophil, they form the class of
polymorphonuclear cells, so-named because of the characteristic multilobulated shape of their nuclei
(quite different from that of lymphocytes and monocytes, the other types of white cells). Neutrophils
are the most abundant white blood cells, accounting for 70% of all white blood cells.
The stated normal range for blood counts varies between laboratories, but a neutrophil count of 2.5–
7.5 × 109/l is a standard normal range. People of African and Middle Eastern descent may have
lower counts which are still normal.
2.43
Answer: B Eosinophils
Eosinophils are white blood cells that are responsible for combating infection by parasites. They are
also involved in the pathogenesis of allergy and asthma. They are granulocytes that develop in the
bone marrow before migrating into blood. These cells are naturally transparent but appear brick-red
when stained with eosin using Romanowsky’s method; the red colour stains small granules within the
cellular cytoplasm which contain chemical mediators such as histamine and proteins such as
eosinophil peroxidase, RNase, DNases, lipase, plasminogen and major basic protein. These
mediators are released by degranulation following activation of the eosinophil, and are toxic to both
parasite and host tissues.
Eosinophils make up about 1%–5% of all white blood cells, and are about 10–12 µm in size. They
are found in the medulla and the junction between the cortex and medulla of the thymus, and in the
lower gastrointestinal tract, ovary, uterus, spleen and lymph nodes, but not in the lung, skin,
oesophagus or some of the other internal organs under normal conditions. The presence of eosinophils
in these latter organs is associated with disease. Eosinophils persist in the circulation for 6–12 hours,
and can survive in tissue for an additional 2–3 days in the absence of stimulation.
2.44
Answer: D Interleukin-6
IL-6 is one of the most important mediators of fever and of the acute phase response. In muscle and
fatty tissue, IL-6 stimulates energy mobilisation, which leads to increased body temperature. IL-6 can
be secreted by macrophages in response to specific microbial molecules, known as ‘pathogen-
associated molecular patterns’ (PAMPs). These PAMPs bind to highly important detection molecules
of the innate immune system called ‘Toll-like receptors’ (TLRs) that are present on the cell surface
(or in intracellular compartments), which induces intracellular signalling cascades that give rise to
inflammatory cytokine production. Inhibitors of IL-6 (including oestrogen) are used to treat
postmenopausal osteoporosis.
2.45
Answer: D Interleukin-10
Interleukin-10 (IL-10), also known as ‘human cytokine synthesis inhibitory factor’ (CSIF), is an anti-
inflammatory cytokine. It is capable of inhibiting the synthesis of pro-inflammatory cytokines such as
interferon-gamma, IL-2, IL-3, TNF-α and granulocyte macrophage colony-stimulating factor made by
cells such as macrophages and the type 1 T-helper cells. IL-10 also displays a potent ability to
suppress the antigen presentation capacity of antigen-presenting cells. However, it is also stimulatory
towards certain T cells, mast cells and B cells. It is mainly expressed in monocytes and type 2 T-
helper cells and mast cells and also in a particular subset of activated T cells and B cells. It is
released by cytotoxic T cells to inhibit the actions of natural killer cells during the immune response
to viral infection.
2.46
Answer: C Interleukin-5
IL-5 has long been associated with a number of allergic diseases, including allergic rhinitis and
asthma, where a large increase in the number of circulating, airway-tissue, and induced sputum
eosinophil numbers has been observed. Given that eosinophils are the primary IL-5Rα-expressing
cells, it is not surprising that this cell type responds to IL-5. In fact, IL-5 was originally discovered as
an eosinophil colony-stimulating factor, is a major regulator of eosinophil accumulation in tissues,
and can modulate eosinophil behaviour at every stage from maturation to survival.
2.47
Answer: D Prostaglandins
Arachidonic acid is a polyunsaturated fatty acid that is present in the phospholipids of cell
membranes (especially phosphatidylethanolamine, phosphatidylcholine and phosphatidylinositides),
and is abundant in the brain. Chemically, arachidonic acid is a carboxylic acid with a 20-carbon
chain and four cis double bonds; the first double bond is located at the sixth carbon from the omega
end. It is involved in cellular signalling as a second messenger. Arachidonic acid is freed from
phospholipid molecules by the enzyme phospholipase A2, which cleaves off the fatty acid precursor
(usually linoleic acid). Arachidonic acid is a precursor in the production of eicosanoids:
• The enzymes cyclo-oxygenase and peroxidase lead to prostaglandin H2 synthesis, which in turn is
used to produce the prostaglandins, prostacyclin, and thromboxanes.
The production of these derivatives and their action in the body are collectively known as the
‘arachidonic acid cascade’.
2.48
Answer: B 5-lipoxygenase
Leukotrienes are naturally produced eicosanoid lipid mediators, and they are thought to be
responsible for a number of the effects of asthma and allergies. Leukotrienes use both autocrine
signalling and paracrine signalling to regulate the body’s response. Examples of leukotrienes are
LTA4, LTB4, LTC4, LTD4, LTE4 and LTF4. LTC4, LTD4 and LTE4 are often called ‘cysteinyl
leukotrienes’ due to the presence of the amino acid cysteine in their structure. Collectively, the
cysteinyl leukotrienes make up the slow-reacting substance of anaphylaxis (SRS-A). Another
substance, LTG4, has been postulated to exist, a metabolite of LTE4, in which the cysteinyl moiety has
been oxidised to an alpha-keto-acid (ie the cysteine has been replaced by a pyruvate). Very little is
known about this putative leukotriene.
Leukotrienes are synthesised in the cell from arachidonic acid by 5-lipoxygenase. The catalytic
mechanism involves the insertion of an oxygen moiety at a specific position in the arachidonic acid
backbone. The lipoxygenase pathway is active in leukocytes, including mast cells, eosinophils,
neutrophils, monocytes and basophils. When such cells are activated, arachidonic acid is liberated
from cell membrane phospholipids by phospholipase A2 and donated by the 5-lipoxygenase-
activating protein to 5-lipoxygenase, which converts it in two steps to leukotriene A4, an unstable
epoxide.
In cells equipped with LTA4 hydrolase, such as neutrophils and monocytes, LTA 4 is converted to the
dihydroxy acid leukotriene LTB4, which is a powerful chemoattractant for neutrophils, acting at BLT1
and BLT2 receptors on the plasma membrane of these cells. In cells that express LTC4 synthase, such
as mast cells and eosinophils, LTA 4 is conjugated with the tripeptide glutathione to form the first of
the cysteinyl-leukotrienes, LTC4. Outside the cell, LTC4 can be converted by ubiquitous enzymes to
form, successively, LTD4 and LTE4, which retain biological activity.
The cysteinyl-leukotrienes act at their cell-surface receptors, CysLT1 and CystLT2, on target cells to
contract bronchial and vascular smooth muscle to increase permeability of small blood vessels, to
enhance the secretion of mucus in the airway and gut, and to recruit leukocytes to sites of
inflammation. Both LTB4 and the cysteinylleukotrienes (LTC4, LTD4, LTE4) are partly degraded in
local tissues, and ultimately become inactive metabolites in the liver.
2.49
Kupffer cells (or Browicz–Kupffer cells) are specialised macrophages located in the liver that form
part of the reticuloendothelial system (mononuclear phagocyte system). The primary function of
Kupffer cells is to recycle old red blood cells that are no longer functional. The red blood cell is
broken down by phagocytic action and the haemoglobin molecule is split. The globin chains are
reutilised, while the iron-containing portion or haem is further broken down into iron (which is
reutilised) and bilirubin, which is conjugated with glucuronic acid within hepatocytes and secreted
into the bile.
The cells were first observed by Karl Wilhelm von Kupffer in 1876. The scientist called them
‘sternzellen’ (star cells or stellate cells) but thought falsely that they were an integral part of the
endothelium of the liver blood vessels and that they originated from it. In 1898, after several years of
research, Tadeusz Browicz identified them correctly as macrophages. Their development begins in
the bone marrow with the genesis of promonocytes and monoblasts into monocytes and then on to
peripheral blood monocytes, completing their differentiation into Kupffer cells.
2.50
Perforin is a cytolytic protein found in the granules of CD8 T cells and natural killer (NK) cells. On
degranulation, perforin inserts itself into the target cell’s plasma membrane, forming a pore. Although
purified perforin is sufficient to lyse cells at high doses, the biology of perforin itself does not explain
the ability of CD8 T cells and NK cells to induce apoptosis in target cells. This induction of
apoptosis might require at least one other granule protein, granzyme B.
SECTION 3:
NEOPLASIA–ANSWERS
3.1
The lack of metastases suggests a lower stage and a better prognosis. Many breast cancers are
oestrogen receptor-positive, which suggests that hormonal therapy will be helpful. Both aneuploidy
and a high S-phase are characteristic of malignancy and suggest a worse prognosis. A history of
breast cancer suggests a greater risk for breast cancer, but does not predict prognosis. A higher grade
suggests a worse prognosis.
3.2
Many human carcinomas are associated with K-ras mutations that contribute to oncogenesis. Vimentin
is a more typical marker for soft-tissue malignancies (sarcomas). Carcinomas are positive for
cytokeratin by immunoperoxidase. Bacterial infections such as Shigella flexneri do not tend to
increase the risk of adenocarcinoma. Adenocarcinomas are not generally seen significantly more
frequently in people with immunodeficiency diseases. Adenocarcinomas are also generally not seen
with significantly increased frequency in people with collagen vascular diseases and therefore a
finding of a high titre of DNA topoisomerase I is less likely in this patient.
3.3
A neoplasm is new, uncontrolled growth of cells. Neoplasms and granulomas can both recur. Some
neoplasms can increase in size quickly, but so can granulomas associated with certain infections.
Response to therapy is not always a reliable indicator that the presumed disease was actually present.
Any mass lesion can undergo necrosis, particularly if it is large and outgrows the blood supply.
3.4
Answer: D A 15-year-old boy with a mass in the left femur and lung metastases
3.5
Answer: E T4 N1 M1
This woman has a large invasive primary tumour mass with axillary node and lung metastases,
making this stage T4 N1 M1. Looking at the other stems, T1 N1 M0 signifies a small primary cancer
with nodal metastases but no distant metastases; T1 N0 M1 signifies a small primary cancer with no
lymph node metastases but with distant metastases; T2 N1 M0 signifies a larger primary cancer with
nodal metastases but no distant metastases; and T3 N0 M0 indicates a larger primary cancer with no
metastases to either lymph nodes or to distant sites.
3.6
Answer: C Invasion
Metastasis would be an even better indicator, but invasion suggests malignancy more than the other
items listed here. While pleomorphism is more prominent in malignant neoplasms, it can be present in
benign neoplasms too. Atypia can be part of benign or dysplastic processes that are not malignant. A
high nuclear/cytoplasmic ratio is one feature of malignancy, but is not the best indicator. Necrosis can
occur for many reasons, and even a benign tumour can cause pressure necrosis or outgrow its blood
supply.
3.7
A well-differentiated and localised neoplasm usually has both a low grade and a low stage. Criteria
for malignancy must be satisfied first, then grading and staging follow. Grading and staging are most
useful for epithelial malignancies, but are not reserved specifically for them. It can indeed spread to
nodes, particularly if it is a carcinoma, but is less likely to do so if it is a low-grade tumour and it has
not already done so. It may have an in-situ component, but the behaviour of most neoplasms is judged
by the worst part of it.
3.8
The description in this scenario is of a pleomorphic adenoma of the parotid gland. It is a benign
tumour of the salivary glands. It is the most common type of salivary gland tumour and the most
common tumour of the parotid gland. It derives its name from the architectural pleomorphism (ie
variable appearance) seen by light microscopy. It is also known as ‘mixed tumour’, due to its
pleomorphic appearance as opposed to its dual origin from epithelial and myoepithelial elements.
The tumour is usually solitary and presents as a slow-growing, painless, firm, single nodular mass.
Isolated nodules are generally outgrowths of the main nodule rather than a multinodular presentation.
It is usually mobile unless found in the palate and can cause atrophy of the mandibular ramus when
located in the parotid gland. When found in the parotid tail, it can present as an eversion of the ear
lobe. Though it is a benign tumour, pleomorphic adenomas have the capacity to grow quite large.
3.9
Prostate-specific antigen (PSA) is a protein produced by the cells of the prostate gland. PSA is
present in small quantities in the serum of normal men and is often elevated in the presence of
prostate cancer and in other prostate disorders. A blood test to measure PSA is the most effective test
currently available for the early detection of prostate cancer. Higher than normal levels of PSA are
associated with both localised and metastatic prostate cancer.
PSA (also known as ‘kallikrein III’, ‘seminin’, ‘semenogelase’, γ-seminoprotein’ or ‘P-30 antigen’)
is a glycoprotein produced almost exclusively by the prostate gland. PSA liquifies the semen and
allows sperms to swim freely. It is also believed to be instrumental in dissolving the cervical mucus
cap, allowing the entry of sperm. Biochemically, it is a serine protease enzyme, the gene of which is
located on chromosome 19 (19q13).
Normal PSA levels lie in the range 0–4.0 ng/ml. Increased levels of PSA can suggest the presence of
prostate cancer. However, prostate cancer can also be present in the complete absence of an elevated
PSA level, in which case the test result would be a false negative. PSA levels can also be elevated
due to prostate infection, irritation, benign prostatic hypertrophy or hyperplasia, or recent ejaculation,
in which case it may give a false-positive result. It is a myth, however, that digital rectal examination
raises the PSA level.
3.10
Answer: E Seminoma
Seminoma is a very radiosensitive tumour. Standard treatment for seminoma after unilateral
orchidectomy is radiation therapy, usually 20–40 Gy to the para-aortic regions up to the diaphragm (a
higher dose is used for patients with a nodal mass); the ipsilateral ilioinguinal region is not treated
routinely. Occasionally, the mediastinum and left supraclavicular regions are also irradiated,
depending on the clinical stage.
3.11
Answer: A Arrhenoblastoma
Arrhenoblastoma, also known as ‘Sertoli–Leydig tumour’, is a rare ovarian stromal neoplasm that
secretes testosterone. Although it can occur at any age, it is mostly seen in women in the reproductive
years. It is typically unilateral. The key clinical features of this tumour are related to its endocrine
activity. The excessive production of testosterone leads to progressive masculinisation in a woman
who typically had been normal beforehand: she might experience not only anovulation and
amenorrhoea, but usually also acne and hirsutism, voice deepening, clitoromegaly, temporal hair
recession, and an increase in musculature. On ultrasonography, a unilateral solid lesion can be seen in
the ovary. The serum testosterone level is high. The treatment consists of surgical removal and the
prognosis is generally good as the lesion tends to grow slowly and rarely metastasises.
3.12
Answer: B Arsenic
The rare hepatic angiosarcomas are associated with distinct carcinogens, including arsenic (exposure
to arsenical pesticides), Thorotrast (a radioactive contrast medium at one time widely used in
radiology), and polyvinyl chloride (PVC, a widely used plastic). The increased frequency of
angiosarcomas among workers in the PVC industry is one of the well-documented instances of
chemical carcinogenesis in humans. With all three agents, there is a very long latent period of many
years between exposure and the development of tumours.
3.13
BRCA1 is a human gene that belongs to a class of genes known as ‘tumour suppressor genes’. Like
many other tumour suppressor genes, BRCA1 regulates the cycle of cell division by keeping cells
from growing and dividing too rapidly or in an uncontrolled way. In particular, it inhibits the growth
of cells that line the milk ducts in the breast. The protein made by BRCA1 is directly involved in the
repair of damaged DNA. In the nucleus of many types of normal cells, the BRCA1 protein interacts
with the protein produced by the RAD51 gene to mend breaks in DNA. These breaks can be caused by
natural radiation or other exposures, but also occur when chromosomes exchange genetic material in
preparation for cell division. The BRCA2 protein, which has a function similar to that of the BRCA1
protein, also interacts with the RAD51 protein. By repairing DNA, these three proteins play a role in
maintaining the stability of the human genome.
Research suggests that both the BRCA1 and BRCA2 proteins regulate the activity of other genes and
play a critical role in embryo development. The BRCA1 protein probably interacts with many other
proteins, including tumour suppressors and regulators of the cell division cycle. BRCA1 is located on
the long (q) arm of chromosome 17 at position 21.
Certain variations of BRCA1 lead to an increased risk for breast cancer. Researchers have identified
more than 600 mutations in BRCA1, many of which are associated with an increased risk of cancer.
These mutations can be changes in one or a small number of DNA base pairs. In some cases, large
segments of DNA are rearranged. A mutated BRCA1 gene usually makes a protein that does not
function properly because it is abnormally short. Researchers believe that the defective BRCA1
protein is unable to help fix mutations that occur in other genes. These defects accumulate and may
allow cells to grow and divide uncontrollably to form a tumour.
In addition to breast cancer, mutations in the BRCA1 gene also increase the risk of ovarian, Fallopian
tube, prostate and colon cancers. Moreover, precancerous lesions (dysplasia) within the Fallopian
tube have been linked to BRCA1 gene mutations.
BRCA2 is another tumour suppressor, and although the structures of the BRCA1 and BRCA2 genes are
very different, their functions appear to be similar. Like BRCA1, BRCA2 probably regulates the
activity of other genes and plays a critical role in embryo development. The BRCA2 gene is located
on the long (q) arm of chromosome 13 at position 12.3.
3.14
Several humoral factors have been associated with paraneoplastic hypercalcaemia of malignancy.
Perhaps the most important is a molecule related to, but distinct from, parathyroid hormone (PTH).
Parathyroid hormone-related protein (PTHRP) resembles the native hormone only in its amino
terminus. It has some biological actions similar to those of PTH, and both hormones share a G-
protein-coupled receptor known as PTH/PTHRP receptor (often referred to as PTH-R or PTHrP-R)
In contrast to PTH, PTHrP is produced by many normal tissues, including keratinocytes, muscles,
bone and ovary. The amounts produced by normal cells, however, are small. It is thought that PTHrP
regulates calcium transport in the lactating breast and across the placenta. Tumours most often
associated with paraneoplastic hypercalcaemia are carcinomas of the breast, lung, kidney and ovary.
In breast cancers, PTHrP production is associated with osteolytic bone disease, bone metastasis and
humoral hypercalcaemia. The most common lung neoplasm associated with hypercalcaemia is the
squamous cell bronchogenic carcinoma (in contrast to the small-cell cancer of the lung, which is more
often associated with endocrinopathies). In addition to PTHrP, several other factors, including
interleukin-1, tumour growth factor-alpha, tumour necrosis factor and dihydroxyvitamin D, have also
been implicated in causing the hypercalcaemia of malignancy.
3.15
Warthin’s tumour is a benign tumour of the salivary glands. It is also known as ‘benign papillary
cystadenoma lymphomatosum’. Its aetiology is unknown, but there is a strong association with
cigarette smoking. Smokers are at an eight-fold greater risk of developing Warthin’s tumour than non-
smokers. The gland most likely to be affected is the parotid gland. Though much less common than
pleomorphic adenoma, Warthin’s tumour is the second most common benign parotid tumour.
Warthin’s tumour is more likely to occur in older adults aged between 60 years and 70 years. There
is only a slight male predilection according to recent studies, but historically it has been associated
with a strong male predilection. This change is possibly due to the tumour’s association with cigarette
smoking and the growing use of cigarettes in women. The tumour is slow-growing, painless, and
usually appears in the tail of the parotid gland, near the angle of the mandible. In 5%–14% of cases,
Warthin’s tumour is bilateral, but the two masses usually arise at different times.
The appearance of this tumour under the microscope is unique. There are cystic spaces surrounded by
two uniform rows of cells with centrally placed, pyknotic nuclei. The cystic spaces have epithelium
referred to as ‘papillary infoldings’ that protude into them. In addition, the epithelium has a lymphoid
stroma with germinal centre formation.
3.16
The stage of a cancer is a descriptor (usually numbered I to IV) of how much the cancer has spread.
The stage often takes into account the size of a tumour, how deep it has penetrated, whether it has
invaded adjacent organs, if and how many lymph nodes it has metastasised to, and whether it has
spread to distant organs. Staging of cancer is important because the stage at diagnosis is the most
powerful predictor of survival, and treatments are often changed based on the stage.
Cancer staging can be divided into a clinical stage and a pathological stage. In the TNM (tumour-
node-metastasis) system, clinical stage and pathological stage are denoted by a small ‘c’ or ‘p’
before the stage, for example cT3 N1 M0 or pT2 N0 M0:
• Clinical stage is based on all of the available information obtained before surgery to remove the
tumour. This might include information about the tumour obtained by physical examination,
radiological examination, and endoscopy.
Because they use different information, the clinical stage and pathological stage are often different.
Pathological staging is usually considered the ‘better’ or ‘truer’ stage because it is based on direct
examination of the tumour and its spread, in contrast to clinical staging, which is limited by the fact
that the information is obtained by making indirect observations of a tumour which is still in the body.
However, clinical staging and pathological staging should complement each other. Not every tumour
is treated surgically, so sometimes pathological staging is not available. Also, sometimes surgery is
preceded by other treatments such as chemotherapy and radiotherapy which shrink the tumour, so the
pathological stage might be an underestimate of the true stage.
Correct staging is critical because treatment is directly related to disease stage. Incorrect staging
could lead to improper treatment, and reduced patient survival. Correct staging, however, can be
difficult to achieve. Pathological staging, where a pathologist examines sections of tissue, can be
particularly problematic for two reasons: visual discretion and random sampling of tissue. ‘Visual
discretion’ means being able to identify single cancerous cells intermixed with healthy cells on a
slide. Oversight of just one cell can lead to mis-staging and serious, unexpected spread of cancer.
‘Random sampling’ refers to the fact that lymph nodes are cherry-picked from patients and random
samples of these are examined. If cancerous cells present in the lymph node happen not to be present
in the slices of tissue viewed, incorrect staging and improper treatment can result.
Staging systems are specific for each type of cancer (eg systems for breast cancer and lung cancer).
Some cancers, however, do not have a staging system. There are often competing staging systems for
the same type of cancer. However, the universally accepted staging system is that of the Union
Internationale Contre le Cancer (UICC), which has the same definitions of individual categories as
the American Joint Committee on Cancer (AJCC).
3.17
Answer: D IIIA
Metastasis (M):
MX: presence of distant spread (metastasis) cannot be assessed
M0: no distant spread
M1: spread to distant organs is present
Stage 0 breast cancer is Tis N0 M0 disease. Ductal carcinoma in situ (DCIS) is the earliest form of
breast cancer. In DCIS, cancer cells are located within a duct and have not invaded the surrounding
fatty breast tissue. Lobular carcinoma in situ (LCIS) is sometimes classified as stage 0 breast cancer,
but most oncologists believe it is not a true breast cancer. In LCIS, abnormal cells grow within the
lobules or milk-producing glands, but they do not penetrate through the wall of these lobules. Paget’s
disease of the nipple is stage 0. In all cases the cancer has not spread to lymph nodes or distant sites.
Stage I breast cancer is T1 N0 M0 disease. The tumour is 2 cm or less in diameter and has not
spread to lymph nodes or distant sites.
Stage IIA breast cancer is T0 N1 M0, T1 N1 M0, or T2 N0 M0 disease.No tumour is found in the
breast but it is found in one to three axillary lymph nodes; or the tumour is less than 2 cm in size and
has spread to one to three axillary lymph nodes; or the cancer is found by sentinel node biopsy as
microscopic disease in internal mammary nodes, but not on imaging studies or by clinical
examination; or the tumour is larger than 2 cm but less than 5 cm in diameter, but has not spread to
axillary nodes. In all cases the cancer has not spread to distant sites.
Stage IIB breast cancer is T2 N1 M0 or T3 N0 M0 disease.The tumour is larger than 2 cm but less
than 5 cm in diameter and has spread to one to three axillary lymph nodes; or cancer is found by
sentinel node biopsy as microscopic disease in internal mammary nodes; or the tumour is larger than
5 cm but does not grow into the chest wall and has not spread to lymph nodes. In all cases, the cancer
has not spread to distant sites.
Stage IIIA breast cancer is T0–2 N2 M0 or T3 N1-2 M0 disease.The tumour is smaller than 5 cm
in diameter and has spread to 4 to 9 axillary lymph nodes; or it is found through imaging studies or
clinical exam to have spread to internal mammary nodes; or the tumour is larger than 5 cm and has
spread to 1 to 9 axillary nodes, or to internal mammary nodes. In all cases, the cancer hasn’t spread to
distant sites.
Stage IIIB breast cancer is T4 N0-2 M0 disease.The tumour has grown into the chest wall or skin
and may have spread to no lymph nodes or to as many as nine axillary nodes. It might or might not
have spread to internal mammary nodes. The cancer has not spread to distant sites.
Stage IIIC breast cancer is T0-4 N3 M0 disease.The tumour can be of any size, has spread to ten
or more nodes in the axilla; or to one or more lymph nodes under the clavicle (infraclavicular) or
above the clavicle (supraclavicular); or to internal mammary lymph nodes, which are enlarged
because of the cancer. All of these are on the same side as the breast cancer. The cancer has not
spread to distant sites.
Inflammatory breast cancer is classified as stage III, unless it has spread to distant organs or lymph
nodes that are not near the breast, in which case it would be stage IV.
Stage IV breast cancer is T0-4 N0-3 M1 disease. The cancer, regardless of its size, has spread to
distant organs such as bone, liver or lung, or to lymph nodes far from the breast.
According to this staging system, therefore, the woman in this scenario has stage IIIA disease (see
also Answer to 3.5).
3.18
A premalignant condition is a disease, syndrome or finding that, if left untreated, can lead to cancer.
Examples of premalignant conditions include solar (actinic) keratosis, Barrett’s oesophagus and
cervical dysplasia. Solar keratosis (also called ‘actinic keratosis’, ‘senile keratosis’, or ‘AK’) is a
premalignant condition of thick, scaly, or crusty patches of skin. It is most common in fair-skinned
people who are frequently exposed to the sun, because their skin pigment is not very protective. It is
usually accompanied by solar damage. Because some of these pre-cancers progress to squamous cell
carcinoma, they should be treated.
When skin is exposed to the sun constantly, thick, scaly, or crusty bumps appear. The scaly or crusty
part of the bump is dry and rough. The growths start out as flat scaly areas, and later grow into a
tough, wart-like area. A solar keratosis site commonly ranges in size between 2 mm and 6 mm, and
can be dark or light, tan, pink, red, a combination of all these, or the same colour as the rest of the
skin. It can appear on any sun-exposed area, such as the face, ears, neck, scalp, chest, the back of
hands, forearms or lips. Solar keratosis can appear as early as 30 years of age in susceptible people
who spend a lot of time outdoors. People with skin phototypes I and II are more likely to be affected,
as are albinos and immunosuppressed patients. Up to 100% of elderly white people get AK, but it is
rare in darker-skinned people. About 10% of people with AK eventually develop squamous cell
carcinoma of the skin.
3.19
Answer: B Liposarcoma
Liposarcoma is a malignant tumour that arises in fat cells in deep soft tissue, such as that inside the
thigh or in the retroperitoneum. They are typically large bulky tumours which tend to have multiple
smaller satellites extending beyond the main confines of the tumour. Patients usually notice a deep-
seated mass in their soft tissue. Only when the tumour is very large do symptoms of pain or functional
disturbances occur.
Retroperitoneal tumours can present with signs of weight loss and emaciation and abdominal pain.
These tumours can also compress the kidney or ureter, leading to kidney failure. Most frequent in
middle-aged and older adults (over the age of 40 years), liposarcomas are the most common of all
soft-tissue sarcomas. The annual incidence is 2.5 per million. The prognosis varies, depending on the
site of origin, the type of cancer cell, the tumour size, the depth, and the proximity to lymph nodes.
Metastases are common. They are very radioresistant and the 5-year survival rate for a high-grade
liposarcoma is less than 50%.
3.20
Answer: C IIB
In this case the tumour is larger than 2 cm and less than 5 cm in diameter and has spread to three
axillary lymph nodes. According to the TNM staging system, this is stage IIB (see alsoAnswer to 3.5
and 3.17).
3.21
The Papanicolaou test (also called ‘Pap smear’, ‘Pap test’, ‘cervical smear’, or ‘smear test’) is a
medical screening method primarily designed to detect premalignant and malignant processes in the
ectocervix. It may also detect infections and abnormalities in the endocervix and endometrium. The
endocervix can be partially sampled with the device used to obtain the ectocervical sample but
consistent and reliable sampling cannot be guaranteed because of the anatomy of this area, and
abnormal endocervical cells might be sampled. The endometrium is not directly sampled with the
device used to sample the ectocervix. Cells can exfoliate onto the cervix and be collected from there,
so as with endocervical cells, abnormal cells can be recognised if present but the Pap test should not
be used as a screening tool for endometrial malignancy.
It is generally recommended that sexually active females undergo Pap smear testing annually, although
guidelines vary from country to country. If results are abnormal, and depending on the nature of the
abnormality, the test might need to be repeated in 3–12 months. If the abnormality requires closer
scrutiny, the patient might be referred for detailed inspection of the cervix by colposcopy. The patient
might also be referred for HPV DNA testing, which can serve as an adjunct (or even as an
alternative) to Pap testing.
About 5%–7% of Pap smears produce abnormal results, such as dysplasia, possibly indicating a
precancerous condition. Although many low-grade cervical dysplasias spontaneously regress without
ever leading to cervical cancer, dysplasia can serve as an indication that increased vigilance is
needed. Endocervical and endometrial abnormalities can also be detected, as can a number of
infectious processes, including yeast and Trichomonas vaginalis infections. A small proportion of
abnormalities are reported as being of ‘uncertain significance’.
3.22
Answer: D Nephroblastoma
Wilms’ tumour is a neoplasm of the kidneys that typically occurs in children. It is named after Dr Max
Wilms, a German surgeon (1867–1918) who first identified this form of cancer. It is also known as a
‘nephroblastoma’. The majority (75%) occur in otherwise normal children; a minority (25%) are
associated with other developmental abnormalities. It is highly responsive to treatment, with about
90% of patients surviving at least 5 years. Wilms’ tumour can affect any child, regardless of race,
sex, country of origin or parental occupation. The disease is usually noticed around the age of 3 years,
but has been recorded in people at the age of 32 years. Most people initially experience the following
symptoms:
• Abdominal mass
• Haematuria
• Fever
It can be associated with a WAGR complex. This complex includes Wilms’ tumour, aniridia,
genitourinary malformation, and mental/motor retardation. It can also be associated with Beckwith–
Wiedmann syndrome (hemihypertrophy, macroglossia, omphalocele).
3.23
Answer: B Breast
Both BRCA1 and BRCA2 are tumour suppressor genes. Certain variations of the BRCA1 and BRCA2
genes lead to an increased risk for breast cancer. Researchers have found numerous mutations in these
genes, many of which are associated with an increased risk of cancer. These mutations can be
changes in one or a small number of DNA base pairs. In some cases, large segments of DNA are
rearranged. A mutated BRCA1 or BRCA2 gene usually makes a protein that does not function properly
because it is abnormally short. Researchers believe that the defective BRCA1 or BRCA2 protein is
unable to help fix mutations that occur in other genes. These defects accumulate and may allow cells
to grow and divide uncontrollably to form a tumour (see also Answer to 3.13).
3.24
3.25
Answer: A CA-125
CA-125 (or CA125) is an abbreviation for cancer antigen-125. CA-125 is a tumour marker or
biomarker that can be elevated in the blood of some people with specific types of cancers. CA-125 is
a mucinous glycoprotein and is the product of the MUC16 gene. It is best known as a marker for
ovarian cancer, but it can also be elevated in other malignant cancers, including those originating in
the endometrium, Fallopian tubes, lungs, breast and gastrointestinal tract. CA-125 can also be
elevated in a number of relatively benign conditions, such as endometriosis, several diseases of the
ovary, and in pregnancy. It also tends to be elevated in the presence of any inflammatory condition in
the abdominal area, whether cancerous and benign. CA-125 is therefore not completely specific for
cancer nor is it 100% sensitive since not every patient with cancer will have elevated levels of CA-
125 in the blood. For example, 79% of all ovarian cancers are positive for CA-125, whereas the
remainder do not express this antigen at all.
CA-125 is clinically approved for following the response to treatment and predicting prognosis after
treatment. It is especially useful for detecting the recurrence of ovarian cancer. Its potential role for
the early detection of ovarian cancer is controversial and it has not yet been adopted for widespread
screening programmes in asymptomatic women. The key problems in using the CA-125 test as a
screening tool are its lack of specificity and its inability to detect early-stage cancers.
3.26
Answer: A Asbestos
Mesothelioma is the most common malignant tumour of the pleura. It is a highly invasive lesion and
has been linked to inhalation of asbestos fibres, especially by workers in the shipbuilding and
insulation industries. A history of smoking dramatically increases the risk of developing a
mesothelioma. Histologically, the tumour can be either sarcomatous (composed of mesenchymal
stromal cells), carcinomatous (resembling tubular or papillary structures), or a combination of these
two types. These tumours are highly malignant and most patients die within 1 year of diagnosis.
3.27
Answer: D Osteosarcoma
Paget’s disease of bone can lead to osteosarcoma. It should not be confused with Paget’s disease of
the breast, which is closely associated with an underlying duct carcinoma.
3.28
Papillary carcinoma is by far the most common primary malignant tumour of the thyroid.
3.29
Carcinoma of the oral cavity accounts for approximately 54% of all human malignancies. More than
90% are squamous cell type and precursor lesions include dysplastic leukoplakia and erythroplasia,
with transformation rates of approximately 15% and 50% respectively. It is more common in men.
Smoking, tobacco chewing, chronic irritation, heat exposure and irradiation are all thought to
contribute to carcinogenesis. The lower lip is the most common site, followed by the floor of the
mouth, anterior tongue, palate and posterior tongue. The prognosis varies according to site but is best
for lesions of the lip and worst for lesions in the floor of the mouth.
3.30
Answer: B Osteochondroma
Osteochondroma is also referred to as ‘exostosis’. It is benign new bone growth that often protrudes
from the outer contour of bones and is capped by growing cartilage. The multifocal and clearly
hereditary form of this lesion is known as ‘hereditary multiple cartilaginous exostosis’. Whether
multiple or isolated, nearly 80% of these lesions are noted prior to the age of 21 years.
3.31
3.32
Benign cystic teratoma, also known as ‘dermoid cyst’, is the most common ovarian tumour found in
premenopausal women. A dermoid cyst originates from the germ-cell component of the ovary. The
tumour can therefore contain tissues that are characteristic of all three germ cell layers: endoderm,
mesoderm and ectoderm. The dermoid tumour is benign; the malignant counterpart is known as an
‘immature teratoma’.
3.33
Answer: E Osteosarcoma
When an osteogenic sarcoma (osteosarcoma) penetrates the bone cortex, it elevates the periosteum.
This periosteal elevation usually produces an acute angle with the underlying remaining cortical
bone, which is known as ‘Codman’s triangle’. A significant radiographic sign, Codman’s triangle
aids in the diagnosis of osteogenic sarcoma.
3.34
Answer: B Burkitt’s lymphoma – c-myc oncogene
Burkitt’s lymphoma is associated with the c-myc oncogene, which is involved in the synthesis of
nuclear regulatory proteins necessary for the growth process of B lymphocytes. Some breast cancers
(eg comedocarcinomas) are associated with overexpression of the erb-B3 oncogene. The presence of
this oncogene implies a poor prognosis for the patient. Chronic myelogenous leukaemia is associated
with the c-abl oncogene, which is normally involved in non-receptor tyrosine kinase activity for the
generation of second messengers. Colon cancer is associated with the ras oncogene, which is
normally involved in guanosine-5-triphosphate (GTP) binding and the generation of second
messengers. Neuroblastomas have an abnormality in the N-myc oncogene, which is normally
involved in the generation of nuclear regulatory proteins.
3.35
Thymomas are often associated with pure red blood cell aplasia, this sign sometimes leading to the
discovery of the underlying thymoma. Paraneoplastic syndromes often indicate the presence of an
underlying neoplasm before it has progressed to an advanced stage. Unfortunately, they only occur in
10% to 15% of cases. To qualify as a paraneoplastic sydrome, the syndrome must not be caused by
the direct effects of tumour invasion or metastasis. For example, hypercalcaemia caused by lysis of
bone by metastatic tumour does not qualify, but secretion of a parathormone-like peptide producing
the hypercalcemia does qualify.
3.36
3.37
Answer: D Lung
The lungs are a more common primary site for a tumour in adults than in children. Other tumour sites
that are more common in adults than in children are the skin (eg basal-cell carcinoma, malignant
melanoma), the breast (eg infiltrating ductal carcinoma), the prostate and the colon. The most
important risk factor for breast, prostate and colon cancer is increasing age. Exposure of the skin to
ultraviolet light increases the risk of skin cancer.
3.38
Carcinomas of the skin are the most common cancers in organ transplant recipients. There is 4- to 21-
fold higher incidence than that of the general population. The skin cancers differ from the carcinomas
in the general population because they have increased aggressiveness, are of squamous cell origin,
and occur more frequently in relatively young patients. Other common cancers in transplant recipients
are: anogenital cancer (eg carcinoma of the cervix, vulva, perineum, penis and anus, possibly
associated with the human papilloma viruses); non-Hodgkin’s lymphoma, particulary large-cell and
immunoblastic types (most non-Hodgkin’s lymphomas are of B-cell origin and are commonly
associated with central nervous system involvement); and Kaposi’s sarcoma, which has a 400 times
to 500 times higher incidence in these patients compared with the incidence in the general population.
Kaposi’s sarcoma has an endothelial-cell origin and is highly aggressive.
3.39
The distinction between a benign renal adenoma and renal adenocarcinoma is commonly made on the
basis of size, tumours less than 2 cm in size rarely being malignant and those greater than 3 cm in size
behaving in a malignant fashion. This distinction is also made for carcinoid tumours, which most
commonly occur in the appendix. These tumours rarely metastasise because they are less than 2 cm.
Carcinoids in the small intestine, however, commonly metastasise because they are often larger than 2
cm. Although size is an important criterion in staging breast, prostate, colon and lung cancers, the
relationship between tumour size and malignant potential is much more marked for renal
adenocarcinoma and carcinoid tumours.
3.40
Tumours derived from all three cell layers are called ‘teratomas’. In a newborn infant, the most
common location is the sacrococcygeal area. Overall, however, a haemangioma is the most common
tumour in infancy, and the majority of these regress over time.
3.41
Answer: D T3
The staging of all non-small-cell lung cancer follows the TNM system. The current TNM staging
system came into effect in 1997 after revisions for stage groupings for stages I, II and III. According
to TNM staging for primary lung cancer, the tumour (T) stages are as follows:
3.42
Answer: C N2
The most important prognostic indicator in lung cancer is the extent of disease. The American Joint
Committee for Cancer (AJCC) developed the TNM staging system, which takes into account the
degree of spread of primary tumour, the extent of regional lymph-node involvement, and the presence
or absence of distant metastases. The TNM system is used for all lung carcinomas except small-cell
lung carcinomas. According to the TNM staging system, regional lymph-node involvement (N) is as
follows:
3.43
Gastro-oesophageal reflux disease (GORD) is the most common predisposing factor for
adenocarcinoma of the oesophagus. As a consequence of the irritation caused by the reflux of acid
and bile, 10%–15% of patients who undergo endoscopy for evaluation of GORD symptoms are found
to have Barrett’s epithelium. Adenocarcinoma can develop in these patients, representing the last
event of a sequence that starts with the development of GORD and progresses to Barrett’s metaplasia,
low-grade dysplasia, high-grade dysplasia and adenocarcinoma. With the premalignant nature of
Barrett’s oesophagus well established, many investigators have searched for markers of oesophageal
carcinoma that could facilitate earlier diagnosis and follow-up of tumour recurrence. The risk of
adenocarcinoma among patients with Barrett’s metaplasia has been estimated to be 30–60 times that
of the general population. The oncosuppressor gene, TP53 and various oncogenes, particularly erb-
b2, have been studied as potential markers. Casson and colleagues have identified mutations in the
TP53 gene in patients with Barrett’s epithelium that are associated with adenocarcinoma.
3.44
Answer: C 30%–60%
An excellent correlation exists between stage and 5-year survival rate in patients with colon cancer.
For stage I or Dukes’ stage A disease, the 5-year survival rate after surgical resection exceeds 90%.
For stage II or Dukes’ stage B disease, the 5-year survival rate is 70%–85% after resection, with or
without adjuvant therapy. For stage III or Dukes’ stage C disease, the 5-year survival rate is 30%–
60% after resection and adjuvant chemotherapy. For stage IV or Dukes’ stage D disease, the 5-year
survival rate is poor (approximately 5%).
3.45
Small-cell lung cancer (SCLC) is considered to be distinct from the other lung cancers (the so-called
‘non-small-cell lung cancers’ or NSCLCs), because of its clinical and biological characteristics.
SCLC exhibits aggressive behaviour, with rapid growth, early spread to distant sites, exquisite
sensitivity to chemotherapy and radiation and frequent association with distinct paraneoplastic
syndromes. Surgery usually plays no role in its management, except in rare situations where it
presents at a very early stage as a solitary pulmonary nodule (<5% of patients). Even then, adjuvant
chemotherapy after surgical resection is recommended because SCLC should always be considered
to be a systemic disease. Approximately 65%–70% of patients with SCLC have disseminated disease
at presentation. Late-stage SCLCs are incurable, and patients with extensive disease have a median
survival of less than 1 year. Even patients presenting with localised disease have a median survival
of less than 2 years. The overall 5-year survival rate for SCLC is less than 20%.
3.46
The disease is usually observed in middle-aged and older people but children and young people can
also be affected. Because of its rarity, the exact incidence is unknown. Lambert–Eaton myasthenic
syndrome is usually a solitary diagnosis but it can also occur as a paraneoplastic syndrome
associated with lung cancer (small-cell histology). It can also be associated with cancers such as
lymphoma, non-Hodgkin’s lymphoma, T-cell leukaemia, non-small-cell lung cancer, prostate cancer
and thymoma. In both conditions, the disease is of autoimmune origin, caused by antibodies that are
directed against the antigens of the neuromuscular junction. In 1989 the antibodies were demonstrated
to be directed against presynaptic calcium channels, which are located in the neuromuscular junction
and are responsible for the efficient release of acetylcholine. The antibodies prevent normal
functioning of calcium channels and so prevent the release of acetylcholine that is essential for normal
nerve–muscle interactions, which in turn maintain the normal muscle strength.
The major clinical finding is progressive weakness that does not usually involve the respiratory
muscles and the muscles of the face. In patients with affected ocular and respiratory muscles, the
involvement is not as severe as in myasthenia gravis. The proximal parts of the legs and arms are
predominantly affected. Many patients have autonomic symptoms such as dry mouth or impotence.
Reflexes are usually reduced or absent.
3.47
Answer: D T3
The Union Internationale Contre le Cancer (UICC) and the American Joint Committee on Cancer
(AJCC) developed the tumour-node-metastasis (TNM) staging system, which is used to stage bladder
cancer. Ta and T1 tumours and carcinoma in situ (CIS) are considered to be superficial bladder
tumours. T2, T3 and T4 tumours are invasive bladder tumours. Transitional cell carcinoma (TCC) is
histologically graded as low-grade (formerly graded 1–2) or high-grade (formerly graded as 3). CIS
is characterised by full-mucosal-thickness and high-grade dysplasia of the bladder epithelium and is
associated with a poorer prognosis.
3.48
Answer: E Smoking
Exposure to environmental carcinogens of various types is responsible for the development of nearly
80% of bladder cancers. Smoking is the most commonly associated risk factor and accounts for
approximately 50% of all bladder cancers. Nitrosamine, 2-naphthylamine and 4-aminobiphenyl are
possible carcinogenic agents found in cigarette smoke. Bladder cancer is also associated with
industrial exposure to aromatic amines in dyes, paints, solvents, leather dust, inks, combustion
products, rubber and textiles. Higher-risk occupations associated with bladder cancer therefore
include painting, driving trucks and working with metal.
Several medical risk factors are associated with bladder cancer. Patients with prior exposure to
radiation treatment of the pelvis have an increased risk of bladder cancer. Chemotherapy with
cyclophosphamide increases the risk of bladder cancer via exposure to acrolein, a urinary metabolite
of cyclophosphamide. Patients with spinal cord injuries who have long-term indwelling catheters
have a 16- to 20-fold increased risk of developing squamous cell carcinoma of the bladder.
Coffee consumption does not increase the risk of developing bladder cancer and there is no
significant correlation between artificial sweeteners (eg saccharin, cyclamate) and bladder cancer.
Although no convincing evidence exists for a hereditary factor in the development of bladder cancer,
familial clusters of bladder cancer have been reported. Several genetic mutations have been
identified in bladder cancer. Mutations of the tumour suppressor gene for p53, found on chromosome
17, are associated with high-grade bladder cancer and CIS. Mutations of the tumour suppressor gene
for p15 and p16, found on chromosome 9, are associated with low-grade and superficial tumours.
Retinoblastoma (Rb) tumour suppressor gene mutations have also been noted. Bladder cancer is
associated with increased expression of the epidermal growth factor gene and the erb-b2 oncogene,
and mutations of the oncogenes p21 ras, c-myc, and c-jun.
3.49
Renal cell carcinomas can be staged by using the American Joint Committee on Cancer (AJCC) TNM
(tumour-node-metastasis) classification, as follows:
3.50
Answer: B T2
The TNM staging system used for head and neck cancers is a clinical staging system that allows
physicians to compare results across patients, assess prognosis, and design appropriate treatment
regimens: ‘T’ refers to tumour size at the primary site, ‘N’ refers to the status of the cervical chain of
lymph nodes, and ‘M’ refers to the presence or absence of distant metastases. The same system is
employed for laryngeal tumours. The basic premise of these systems is that smaller cancers with no
nodal disease have a better prognosis than a larger lesion with positive neck nodes.
Staging:
Stage I: T1 N0 M0
Stage II: T2 N0 M0
Stage III: T2NOMO, T3N1MO
Stage IV: T4N1M0, Any TN2M0, Any TN3M0, Any T Any NM1
The depth of infiltration is predictive of the prognosis. With increasing depth of invasion of the
primary tumour, the risk of nodal metastasis increases and survival decreases.
4.1
Answer: B Endotoxin
Endotoxins are potentially toxic natural compounds found inside pathogens such as bacteria.
Classically, an ‘endotoxin’ is a toxin which, unlike an ‘exotoxin’, is not secreted in soluble form by
live bacteria, but is a structural component in the bacterium itself and which is released mainly when
bacteria are lysed. The prototypical examples of endotoxin are lipopolysaccharides (LPS) or
lipooligo-saccharides found in the outer membrane of various Gram-negative bacteria. The term
‘LPS’ is often used interchangeably with ‘endotoxin’ because of the history of its discovery: in the
early nineteenth century it became understood that bacteria could secrete toxins into their
environment, which became broadly known as ‘exotoxin’. The term ‘endotoxin’ resulted from the
discovery that portions of Gram-negative bacteria can cause toxicity, hence the name ‘endotoxin’.
Studies of endotoxin over the next 50 years revealed that the effects of ‘endotoxin’ were in fact due to
lipopolysaccharide.
There are, however, endotoxins other than LPS. For example, delta endotoxin ofBacillus
thuringiensis makes crystal-like inclusion bodies next to the endospore inside the bacteria. It is toxic
to larvae of insects feeding on plants, but is harmless to humans (as we do not possess the enzymes
and receptors necessary for its processing followed by toxicity). The only Gram-positive bacteria
that produces endotoxin is Listeria monocytogenes.
LPS consist of a polysaccharide (sugar) chain and a lipid moiety, known as lipid A, which is
responsible for the toxic effects. The polysaccharide chain is highly variable between different
bacteria. Endotoxins are approximately 10 kDa in size but can form large aggregates of up to 1000
kDa. Humans are able to produce antibodies to endotoxins after exposure but these are generally
directed at the polysaccharide chain and do not protect against a wide variety of endotoxins. Injection
of a small amount of endotoxin in human volunteers produced fever, a lowering of the blood pressure,
and activation of inflammation and coagulation. Endotoxins are in large part responsible for the
dramatic clinical manifestations of infections with pathogenic Gram-negative bacteria such as
Neisseria meningitidis (also simply known as ‘meningococcus’), the pathogen that causes fulminant
meningitis. The clinical features in this scenario are also suggestive of meningitis.
4.2
Escherichia coli is one of the main species of bacteria living in the lower intestines of mammals,
known as the ‘gut flora’. When located in the large instestine, it actually assists with waste
processing, vitamin K production and food absorption. Discovered in 1885 by Theodor Escherich, a
German paediatrician and bacteriologist, E. coli are abundant: the number of individual E. coli
bacteria in the faeces that a human defecates in 1 day averages between 100 billion and 10 trillion.
As with all Gram-negative organisms, E. coli are unable to sporulate. Treatments which kill all
active bacteria, such as pasteurisation or simple boiling, are therefore effective for their eradication,
without requiring the more rigorous sterilisation which also deactivates spores. As a result of their
adaptation to mammalian intestines, E. coli grow best in vivo or at the higher temperatures
characteristic of such an environment, rather than the cooler temperatures found in soil and other
environments.
E. coli can generally cause several intestinal and extra-intestinal infections, including urinary tract
infections, meningitis, peritonitis, mastitis, septicaemia and Gram-negative pneumonia. Although it is
more common in females due to the shorter urinary tract, urinary tract infection is seen in both males
and females. It is found in roughly equal proportions in elderly men and women. Because bacteria
invariably enter the urinary tract through the urethra (‘ascending infections’), poor toilet habits can
predispose to infection, but other factors are also important (pregnancy in women, prostate
enlargement in men) and in many cases the initiating event is unclear. While ascending infections are
generally the rule for lower urinary tract infections and cystitis, the same may not necessarily hold for
upper urinary tract infections such as pyelonephritis, which can be haematogenous in origin. Most
cases of lower urinary tract infections in women are benign and do not need exhaustive laboratory
investigations. However, young infants with urinary tract infections must undergo some form of
imaging study, typically a retrograde urethrogram, to investigate for congenital urinary tract
anomalies. Males with urinary tract infections also must be investigated further.
The clinical features and laboratory findings point to E. coli septicaemia in this patient.
4.3
These are the findings of infectious mononucleosis, which is typically acquired through close
personal contact. Infectious mononucleosis, also known as ‘the kissing disease’, ‘Pfeiffer’s disease’,
‘mono’, or ‘glandular fever’, is a disease seen most commonly in adolescents and young adults, and is
characterised by fever, sore throat, muscle soreness and fatigue. White patches on the tonsils or in the
back of the throat can also be seen. Mononucleosis is usually caused by the Epstein–Barr virus
(EBV), which infects B lymphocytes, producing a reactive lymphocytosis and atypical T lymphocytes
known as ‘Downey bodies’.
The virus is typically transmitted from asymptomatic individuals through blood or saliva, or by
sharing a drink or eating utensils, though the disease is far less contagious than is commonly thought.
In rare cases a person may have a high resistance to infection. The mononuclear leukocyte count is
significantly raised. There are two main types of mononuclear leukocytes, monocytes and
lymphocytes. They normally account for about 35% of all white blood cells. In infectious
mononucleosis, this can rise to 50%–70%. In addition, the overall white blood cell count is almost
invariably raised, and can increase to 10–20 × 109/l.
Mononucleosis leads to the production of heterophile antibodies, which cause agglutination of non-
human red blood cells: the Monospot test is a non-specific test that screens for mononucleosis by
looking for these antibodies. Confirmation of the exact cause of the infection can be obtained through
tests to detect specific antibodies to the causative viruses. The Monospot test might be negative in the
first week, so negative tests are often repeated at a later date. Because the Monospot test is usually
negative in children under the age of 6–8 years, EBV serology should be requested if mononucleosis
is suspected. An older test for heterophile antibodies is the Paul–Bunnell test, in which the patient’s
serum is mixed with sheep red blood cells and checked for agglutination of these cells.
4.4
Lactobacillus is a genus of Gram-positive facultative anaerobic bacteria. They are a major part of the
lactic acid bacteria group, named as such because most of its members convert lactose and other
sugars to lactic acid. They are common and usually benign. In humans they are present in the vagina
and the gastrointestinal tract, where they are symbiotic and make up a small portion of the gut flora.
Many species are prominent in decaying plant material. The production of lactic acid makes its
environment acidic, which inhibits the growth of some harmful bacteria. Several members of the
genus have had their genome sequenced.
4.5
Gram-negative bacteria do not retain crystal violet dye in the Gram staining protocol. Gram-positive
bacteria will retain the dark-blue dye after an alcohol wash. In a Gram stain test, a counterstain
(commonly Safranin) is added after the crystal violet, which colours all Gram-negative bacteria a red
or pink colour. The test itself is useful in classifying two distinctly different types of bacteria on the
basis of structural differences in their cell walls.
Many species of Gram-negative bacteria are pathogenic, meaning they can cause disease in a host
organism. This pathogenic capability is usually associated with certain components of Gram-negative
cell walls, in particular the lipopolysaccharide (LPS) or endotoxin layer. The LPS is the trigger
which the body’s innate immune response receptors sense to begin a cytokine reaction. It is toxic to
the host. It is this response which begins the inflammation cycle in tissues and blood vessels.
• The cell walls only contain a few layers of peptidoglycan (which is present in much higher levels
in Gram-positive bacteria).
• The cells are surrounded by an outer membrane containing lipopolysaccharide (which consists of
lipid A, core polysaccharide and O-polysaccharide) outside the peptidoglycan layer.
• Porins exist in the outer membrane, which act like pores for particular molecules.
• There is a space between the layers of peptidoglycan and the secondary cell membrane called the
‘periplasmic space’.
• The S-layer is directly attached to the outer membrane, rather than to the peptidoglycan.
4.6
Tuberculosis is treated in two phases, an initial phase using four drugs and a continuation phase using
two drugs in fully sensitive cases. Treatment requires specialised knowledge, particularly where the
disease involves resistant organisms or non-respiratory organs.
The regimens given below are recommended for the treatment of tuberculosis in the UK; variations
occur in other countries. Either the unsupervised regimen or the supervised regimen described below
should be used; the two regimens should not be used concurrently.
Initial phase. The concurrent use of four drugs during the initial phase is designed to reduce the
bacterial population as rapidly as possible and to prevent the emergence of drug-resistant bacteria.
The drugs are best given as combination preparations unless one of the components cannot be given
because of resistance or intolerance. The treatment of choice for the initial phase is the daily use of
isoniazid, rifampicin, pyrazinamide and ethambutol. Streptomycin is rarely used in the UK but it can
be used in the initial phase of treatment if resistance to isoniazid has been established before therapy
is commenced. Treatment should be started without waiting for culture results if clinical features or
histology results are consistent with tuberculosis; treatment should be continued even if initial culture
results are negative. The initial phase drugs should be continued for 2 months. Where a positive
culture for Mycobacterium tuberculosis has been obtained, but susceptibility results are not
available after 2 months, treatment with rifampicin, isoniazid, pyrazinamide and ethambutol should be
continued until full susceptibility is confirmed, even if this is for longer than 2 months.
Continuation phase. After the initial phase, treatment is continued for a further 4 months with
isoniazid and rifampicin (preferably given as a combination preparation). Longer treatment is
necessary for meningitis, direct spinal cord involvement, and for resistant organisms (which might
also require modification of the regimen).
4.7
Bacteroides fragilis is an obligate anaerobe of the gut. It is involved in 90% of anaerobic peritoneal
infections. B. fragilis is generally susceptible to metronidazole, carbapenems, and beta-lactam/beta-
lactamase inhibitor combinations, but it has inherent high-level resistance to penicillin. Clindamycin
is no longer recommended as the first-line agent for B. fragilis due to emerging high-level resistance
(>30% in some reports).
4.8
Cholera toxin is a protein complex secreted by the bacterium Vibrio cholerae. Cholera toxin is
responsible for the harmful effects of cholera infection. The cholera toxin is an oligomeric complex
made up of six protein subunits: a single copy of the A subunit and five copies of the B subunit. The
five B subunits, each weighing 12 kDa, form a five-membered ring. The A subunit has two important
segments: the A1 portion of the chain (CTA1) is a globular enzyme payload, while the A2 chain
(CTA2) forms an extended alpha helix which sits snugly in the central pore of the B subunit ring. This
structure is similar in shape, mechanism and sequence to the heat-labile enterotoxin secreted by some
strains of the Escherichia coli bacterium.
Once secreted, the B subunit ring of cholera toxin will bind to GM1 gangliosides on the surface of the
host’s cells. After binding takes place, the entire cholera toxin complex is internalised by the cell and
the CTA1 chain is released by the reduction of a disulphide bridge. CTA1 is then free to bind with a
human partner protein called ‘ADP-ribosylation factor 6’ (Arf6); binding to Arf6 drives a change in
the conformation of CTA1, which exposes its active site and enables its catalytic activity. The CTA1
fragment catalyses ADP ribosylation from NAD to the regulatory component of adenylate cyclase,
thereby activating it. Increased adenylate cyclase activity increases cyclic AMP synthesis, causing
massive fluid and electrolyte efflux, resulting in diarrhoea.
4.9
Botulism (from the Latin botulus, sausage) is a rare but serious paralytic illness caused by a nerve
toxin, botulin, that is produced by the bacterium Clostridium botulinum. Botulinum toxin is one of the
most powerful known, with a lethal dose of a microgram. It acts by blocking nerve function and leads
to respiratory and musculoskeletal paralysis.
• Food-borne botulism is a form of food-borne illness and is caused by eating foods that contain the
botulinum toxin.
• Infant botulism is caused by consuming the spores of the botulinum bacteria, which then grow in
the intestines and release toxin.
• Wound botulism is caused by toxin produced from a wound infected with C. botulinum. This is the
rarest type of botulism.
All forms of botulism can be fatal and are considered medical emergencies. Food-borne botulism can
be especially dangerous as a public health problem because many people can be poisoned from a
single contaminated food source.
4.10
Folliculitis is inflammation of one or more hair follicles. The condition can occur anywhere on the
skin. Most carbuncles and furuncles and other cases of folliculitis develop as a result of
Staphylococcus aureus infection. This organism is a bit different from the regular staphylococcus
found on the skin, mostly within the nostrils. S. aureus is an important pathogen as most of the
important skin infections are caused by it. Folliculitis starts when hair follicles are damaged by
friction from clothing, blockage of the follicle or shaving. In most cases of folliculitis, the damaged
follicles are then infected with S. aureus.
4.11
Sputum culture is used to detect and identify bacteria or fungi that are infecting the lungs or breathing
passages. Sputum is a thick fluid produced in the lungs and in the airways leading to the lungs. A
sample of sputum is placed in a container with substances that promote the growth of bacteria or
fungi. If no bacteria or fungi grow, the culture is negative. If pathogenic organisms do grow, the
culture is positive. The type of bacterium or fungus is identified by microscopy or using chemical
tests. If bacteria or fungi that can cause infection grow in the culture, other tests can be performed in
order to determine which antibiotic will be most effective in treating the infection, ie susceptibility or
sensitivity testing. The test is done on a sample of sputum that is usually collected by coughing. For
people who cannot cough deeply enough to produce a sample, a suction tube or needle may be
inserted into the airway to collect the sputum.
4.12
Answer: C Ciprofloxacin
Treatment for anthrax infection includes large doses of intravenous and oral antibiotics, such as
ciprofloxacin, doxycycline, erythromycin and vancomycin. For inhalation infections, antibiotic
treatment is not very effective unless initiated within 1 day of exposure, before any symptoms appear.
Antibiotic treatment is crucial in cases of pulmonary anthrax to prevent death. Antibiotic-resistant
strains of Bacillus anthracis are known.
4.13
The Widal test is a serological test for Salmonella typhi. It is a demonstration of salmonella
antibodies against antigens O-somatic and H-flagellar in the blood. It is used to ascertain the presence
of typhoid fever. However, it is not a very accurate method because patients are often exposed to
other bacteria in this species that induce cross-reactivity (eg S. enteritidis, S. typhimurium). Many
people have antibodies against these enteric pathogens, which also react with the antigens in the
Widal test, leading to a false-positive result. Typhidot is the other test used to diagnose typhoid fever.
As with all serological tests, the rise in antibody levels needed to make the diagnosis takes 7–14
days, which limits their usefulness. Other means of diagnosing S. typhi (and paratyphus infections)
include cultures of blood, urine and faeces. The organism also produces hydrogen sulphide from
thiosulphate.
4.14
4.15
Answer: D Exotoxin
P. aeruginosa uses the virulence factor exotoxin A to ADP-ribosylate eukaryotic elongation factor 2
in the host cell, much as the diphtheria toxin does. Without elongation factor 2, eukaryotic cells cannot
synthesise proteins and necrose. The release of intracellular contents induces an immunological
response in immunocompetent patients.
4.16
Staphylococus aureus is commonly found on the skin and can attack either healthy or deformed
valves. This organism is responsible for 10% to 20% of cases of infective endocarditis. S. aureus is
the main offender in intravenous drug abusers.
4.17
The tuberculous granuloma (caseating tubercle) has central caseous necrosis bordered by giant
multinucleated cells (Langerhans cells), and surrounded by epithelioid cell aggregates, lymphocytes
and fibroblasts. Granulomatous tubercles tend to become confluent. Multinucleated giant cells
(mature, Langerhans-type) are 50–100 µm in diameter, with numerous small nuclei (over 20) lying
peripherally in the cell (in a crown or horseshoe distribution). They have abundant eosinophilic
cytoplasm. They are formed when activated macrophages merge. Epithelioid cells are activated
macrophages that resemble epithelial cells. They are elongated, with finely granular, pale
eosinophilic (pink) cytoplasm and a central, ovoid nucleus. They have indistinct shape and contour
and form aggregates. At the periphery of the tubercle are the lymphocytes (T cells) and rare plasma
cells and fibroblasts. Caseous necrosis is a central area, amorphous, finely granular, and
eosinophilic. If recent, it can contain nuclear fragments. The caseous material is the result of the
accumulated destruction of giant cells and epithelioid cells.
4.18
‘Virulence’ refers to the degree of pathogenicity of a microbe, in other words, the relative ability of a
microbe to cause disease. The virulence factors of bacteria are typically proteins or other molecules
that are synthesised by protein enzymes. These proteins are coded for by genes in chromosomal DNA,
bacteriophage DNA or plasmids. Factors which affect pathogens’ ability to cause disease include
adhesion, colonisation, invasion, immune response inhibition and toxins:
• Adhesion. Many bacteria must first bind to host cell surfaces. Many bacterial and host molecules
that are involved in the adhesion of bacteria to host cells have been identified. Often, the host cell
receptors for bacteria are essential proteins for other functions.
• Colonisation. Some virulent bacteria produce special proteins that allow them to colonise parts of
the host body. Helicobacter pylori is able to survive in the acidic environment of the human
stomach by producing the enzyme urease, for example. Colonisation of the stomach lining by this
bacterium can lead to gastric ulcer and cancer. The virulence of various strains of H. pylori tends
to correlate with the level of production of urease.
• Invasion. Some virulent bacteria produce proteins that either disrupt host cell membranes or
stimulate endocytosis into host cells. These virulence factors allow the bacteria to enter host cells
and facilitate entry into the body across epithelial tissue layers at the body surface.
• Immune response inhibitors. Many bacteria produce virulence factors that inhibit the host’s
immune system defences. For example, a common bacterial strategy is to produce proteins that bind
host antibodies. The polysaccharide capsule of Streptococcus pneumoniae inhibits phagocytosis
of the bacterium by host immune cells, for example.
• Toxins. Many virulence factors are proteins made by bacteria that poison host cells and cause
tissue damage. For example, there are many food-poisoning toxins produced by bacteria that can
contaminate human foods. Some of these can remain in ‘spoiled’ food even after cooking and cause
illness when the contaminated food is consumed. Some bacterial toxins are chemically altered and
inactivated by the heat of cooking.
4.19
The disease is transmitted either through contaminated or untreated milk (and its derivatives) or
through direct contact with infected animals, which can include dogs, pigs, camels and ruminants, but
primarily includes sheep, goats and cattle. This also includes contact with their carcasses. Parturition
rests are extremely rich in highly virulent Brucella organisms. Along with Leptospira, Brucella
organisms have the unique property of being able to penetrate through intact human skin, so infection
by mere hand contact with infectious material is likely to occur.
The disease is now usually associated with the consumption of unpasteurised milk and soft cheeses
made from the milk of infected animals and with occupational exposure of veterinarians and
slaughterhouse workers. Some vaccines used in livestock management, most notably derived from the
B. abortus strain 19, can also cause disease in humans if accidentally injected. The incubation period
of brucellosis is usually 1–3 weeks, but the disease can occasionally take several months to surface.
4.20
Answer: A Actinomycosis
4.21
Answer: B Pseudomonas aeruginosa
The colour of the pus gives a clue to the likely causative pathogen here. Definitive clinical
identification of Pseudomonas aeruginosa often includes identifying the production of both
pyocyanin (blue-green) and fluorescein (yellow-green and fluorescent), which are responsible for the
greenish colour of the pus.
4.22
Cryptosporidium parvum is one of several species that cause cryptosporidiosis. C. parvum infection
is a protozoal infection that causes an acute diarrhoea in immunocompromised patients. In HIV
infection, it can cause a watery diarrhoea which can be associated with anorexia, nausea, vomiting
and abdominal pain. There is no effective treatment (other than supportive treatment) for this infection
apart from an antibiotic called paromomycin. This drug can have a limited effect on the diarrhoea.
4.23
Endocarditis of native but previously damaged or otherwise abnormal valves is caused most
commonly (in 50%–60% of cases) by Streptococcus viridans. It is, however, important to remember
that S. viridans is not the organism responsible for rheumatic disease.
4.24
Worldwide, the most common cause of iron deficiency anemia is parasitic infestation (hookworm,
amoebiasis, schistosomiasis and whipworm).
4.25
Epstein–Barr virus (EBV), also called human herpesvirus 4 (HHV-4), is a virus of the herpes family
(which includes herpes simplex virus and cytomegalovirus). Its host is humans and most people
become infected with EBV. Infection with EBV is often asymptomatic but it commonly causes
infectious mononucleosis. EBV is also statistically associated with and likely has a causal role in
Burkitt’s lymphoma, certain B-cell tumours in immunocompromised patients, and nasopharyngeal
carcinoma. It is named after Michael Epstein and Yvonne Barr, who together with Bert Achong
discovered the virus in 1964 (see also Answer to 4.3).
4.26
4.27
Answer: E Prevents the amino-acyl t-RNA from binding to the A site of the ribosome
Doxycycline is a member of the tetracycline antibiotic group. As well as the general indications for
all members of the tetracycline antibiotic group, doxycycline is frequently used to treat chronic
prostatitis, sinusitis, syphilis, chlamydia infections, pelvic inflammatory disease, acne and rosacea. In
addition, it is used in the treatment and prophylaxis of anthrax and malaria. Its mechanism of action
against malaria is to specifically impair in the progeny the apicoplast genes resulting in their
abnormal cell division.
It is also effective against Yersinia pestis (the infectious agent of bubonic plague) and is prescribed
for the treatment of Lyme disease and Rocky Mountain spotted fever. Doxycycline, like other
antibiotics, will not work for colds, flu or other viral infections.
Elephantiasis is a disease caused by a nematode Wuchereria bancrofti . It causes swollen limbs and
genitals (filariasis) and affects over 120 million people in the world. Previous anti-nematode
treatments have been limited by poor levels of effectiveness, drug side-effects and high costs. In 2003
doxycycyline was shown to kill the symbiotic Wolbachia bacteria which the nematodes depend on.
Field trials in 2005 showed that doxycycline almost completely eliminates blood-borne filaria when
given as an 8-week course.
Doxycycline like other tetracyclines inhibits cell growth by inhibiting translation. It binds to the 16S
part of the 30S ribosomal subunit and prevents the amino-acyl t-RNA from binding to the A site of the
ribosome. The binding is reversible in nature.
4.28
Answer: D Sulphonamides
SJS is characterised by a flu-like prodromal period of fever, sore throat and headache, followed by
the sudden development of circular mucocutaneous lesions that can cover the majority of the skin.
These lesions begin as macules and can develop into papules, vesicles, blisters or urticarial weals.
Stevens–Johnson syndrome is the term usually used to describe those cases where less than 10% of
body surface is involved. When more than 30% of the body surface area is involved the condition is
called toxic epidermal necrolysis syndrome (TENS) or Lyell’s syndrome. Intermediate cases (10%–
30% body surface involved) are classified as ‘SJS/TENS’ overlap.
SJS can be caused by infections (usually viral infections such as herpes simplex, influenza, mumps,
cat-scratch fever, histoplasmosis, Epstein–Barr virus infection), allergic reactions to drugs,
(valdecoxib, penicillins, barbiturates, sulphonamides, phenytoin, lamotrigine, nevirapine, ibuprofen,
ethosuximide, carbamazepine), malignancy (carcinomas and lymphomas), or idiopathic factors (up to
50% of cases). SJS has also been consistently reported as an uncommon side-effect of herbal
supplements containing ginseng. SJS can also be caused by cocaine usage.
4.29
4.30
Answer: D Inhibition of protein synthesis by binding to the 30S subunit of the bacterial ribosome
Side-effects of amikacin are similar to those of other aminoglycosides. Kidney damage and hearing
loss are the most important effects. Because of this potential, blood levels of the drug and markers of
kidney function (creatinine) should be monitored.
4.31
Piliated strains of Escherichia coli ascend the urethra to infect the bladder and kidney. Infections of
the kidney cause pyelonephritis. The vast majority of cases of bacterial pyelonephritis, cystitis, and
other urinary tract infections are caused by E. coli. Introduction of catheters into the urethra has been
associated with the occurrence of urinary tract infections. Clostridium difficile is the cause of
pseudomembranous colitis. Staphylococcus aureus is usually the cause of boils, skin sepsis,
postoperative wound infections, scalded skin syndrome, food-borne infection, septicaemia,
endocarditis, toxic shock syndrome, osteomyelitis and pneumonia. Pseudomonas aeruginosa is
usually the cause of infections of skin and burns. It is also the major pathogen in cystic fibrosis, and
can cause urinary infections, but not as commonly as E. coli. Streptococcus pneumoniae is the most
common micro-organism that causes pneumonia.
4.32
The patient described in the question probably has acquired immunodeficiency syndrome (AIDS).
Such individuals will be lymphopenic and have greatly reduced immunity. People with AIDS
commonly develop Pneumocystis carinii pneumonia. The most important evaluative procedure for
the patient described here is determination of the cause of the pneumonia so the problem can be
corrected. Identifying the infecting organism might assist in the overall diagnosis. About 30% of
patients with HIV infection have P. carinii pneumonia as the initial AIDS-defining diagnosis, and
over 80% of AIDS patients have this infection at some time if prophylaxis is not given. Patients with
HIV infection become vulnerable to P. jiroveci pneumonia when the CD4+ helper cell count is less
than 200/µl.
Choices B to E involve innate immunity, either complement activity or phagocytic cell functions, and
would not be of much diagnostic assistance.
4.33
Answer: C Methicillin
The boy in this scenario appears to be suffering from meningitis due to S. aureus, which should
always be assumed to be a betaβ-lactamase-producing organism until the laboratory reports its
antibiotic sensitivity. Methicillin is a betaβ-lactamase-resistant penicillin that would be the drug of
choice among those listed. It is also bactericidal and is not associated with toxicity, which is a
feature of streptomycin and chloramphenicol. Other antibiotics that might be used include the
cephalosporins, gentamicin, or vancomycin, which can be injected intrathecally.
4.34
If exposure to rabies virus appears definite, as in this case, treatment with human diploid-cell live-
derived vaccine and hyperimmune antirabies gamma globulin should be started immediately. Serum
antibodies provide an immediate barrier to the growth of virus; meanwhile, antibodies are elicited by
the vaccine. If the level of exposure is minimal (ie there is no skin puncture) and the animal probably
is not rabid, vaccine is not recommended. Ordering a search for the attacking dog for autopsy to
determine if it has rabies is like searching for a needle in a haystack, and therefore not the best
approach to address a possible rabies infection, which requires immediate action. Initiation of rabies
vaccine will lead to production of antibodies against the rabies virus, but it will require
approximately two weeks to develop protective antibodies. By that time, severe damage could have
already occurred. Post-exposure immunisation and human rabies immunoglobulin is the best
approach. Observation of the patient or reporting of the incident to the local police do not address the
real needs of an individual running the risk of rabies.
4.35
Exposure to the tuberculosis (TB) bacillus does not assure disease, but a positive skin test makes the
diagnosis more likely. Chemoprophylaxis with isoniazid would be the treatment of choice for
contacts of actively infected people. Ethambutol is very effective against most mycobacteria, but
would be used in the therapy of TB, not in prophylaxis. The skin test is the most sensitive index of
infection, and for individuals who have already shown a positive response, chest X-ray would not
add much information. Sequential X-rays taken months apart might indicate if a lesion was increasing
in size, but that is certainly not a high-priority procedure for the people described in the question.
Immunisation with the live attenuated TB strain bacillus Calmette–Guérin (BCG) would be pointless
because the contacts have already experienced an infection with Mycobacterium tuberculosis.
Immunisation with purified protein derivative does not give protection against tuberculosis. It is used
to determine exposure to M. tuberculosis.
4.36
Answer: B Causes the formation of pyrimidine dimers
The mode of action of ultraviolet light on micro-organisms is related to its absorption by the DNA.
This absorption leads to the formation of covalent bonds between adjacent pyrimidine bases. These
pyrimidine dimers alter the form of the DNA and thus interfere with normal base pairing during the
synthesis of DNA. Disruption of the bacterial cell membrane, removal of free sulphhydryl groups,
protein denaturation, and addition of alkyl groups to cellular components are caused by detergents,
heavy metals, heat or alcoholic compounds, and ethylene oxide or formaldehyde, respectively.
4.37
4.38
4.39
Answer: A Coagulase
In a laboratory test used to identify Staphylococcus aureus, coagulase reacts with a prothrombin-like
compound in plasma to produce an active enzyme (a complex of thrombin and coagulase) that
converts fibrinogen to fibrin. This activity of S. aureus has a very high correlation with the
organism’s virulence, although coagulase-negative organisms can cause less severe disease.
Coagulase reactive factor is a plasma protein with which coagulase reacts. This protein is
presumably a modified derivative of prothrombin. Prothrombin is the substrate from which coagulase
splits a number of amino acids to convert it to thrombin. Thrombin is the proteolytic enzyme that
converts fibrinogen to fibrin, forming the clot. Plasmin is a plasma protein associated with the
destruction of such a clot.
4.40
Answer: D Ketoconazole
Ketoconazole inhibits the biosynthesis of ergosterol by blocking demethylation at the C14 site of the
ergosterol precursor lanosterol. This results in the accumulation of lanosterol-like sterols in the cell,
which alters the properties of the cell membrane, and permits the leakage of potassium ions.
Amphotericin B and nystatin impair the permeability of the cell membrane by directly complexing
with the membrane sterol. The target of griseofulvin is the microtubules. Flucytosine is incorporated
into RNA after being deaminated and then phosphorylated. It also interferes with DNA synthesis
because it is a non-competitive inhibitor of thymidylate synthetase.
SECTION 5:
DISORDERS OF FLUIDS AND ELECTROLYTES —
ANSWERS
5.1
Metabolic alkalosis results in decreased hydrogen ion concentration (increased pH) with
accompanying increased bicarbonate and carbon dioxide concentrations. The organ systems involved
are mainly the kidneys and gastrointestinal tract. The pathogenesis of metabolic alkalosis involves
two processes, the generation of metabolic alkalosis and the maintenance of metabolic alkalosis,
events that usually overlap.
The generation of metabolic alkalosis occurs with the loss of acid, the gain of alkali, or the
contraction of the extracellular fluid compartment, with a consequent change in bicarbonate
concentration. The kidneys usually have an enormous capacity to excrete excess bicarbonate
generated and to restore normal acid–base balance by the following mechanisms: (1) less
reabsorption of bicarbonate because infused sodium bicarbonate leads to volume expansion, which
reduces reabsorption of sodium ions and bicarbonate in the proximal tubule, and (2) bicarbonate
secretion by B-type intercalated cells in the collecting duct that exchange bicarbonate for chloride via
the apical chloride/bicarbonate countertransporter. Therefore, to sustain metabolic alkalosis, the
kidneys must participate to maintain the alkalosis by overriding these mechanisms.
5.2
Nephrotic syndrome is a disorder where the kidneys have been damaged, causing them to leak protein
from the blood into the urine. It is a fairly benign disease when it occurs in childhood, but it can lead
to chronic renal failure, especially in adults, or be a sign of an underlying serious disease such as
systemic lupus erythematosus or a malignancy. Nephrotic syndrome is characterised by proteinuria
and low plasma albumin levels. As a compensation, the liver begins to make more of all its proteins,
and levels of large proteins (such as alpha-2-macroglobulin) increase. Oedema usually occurs due to
salt and water retention by the diseased kidneys as well as due to the reduced colloid osmotic
pressure (because of reduced albumin in the plasma). Oedema might also be caused by congestive
heart failure.
5.3
5.4
Dehydration is a state of negative fluid balance that can be caused by a number of disease entities,
though diarrhoeal illnesses are the most common cause. The negative fluid balance causing
dehydration results from decreased intake, increased output (renal, gastrointestinal or insensible
losses) or fluid shift (ascites, effusions, and capillary leak states such as burns and sepsis). The
decrease in total body water causes reductions in both the intracellular and extracellular fluid
volumes. Clinical manifestations of dehydration are most closely related to intravascular volume
depletion. As dehydration progresses, hypovolaemic shock eventually develops, resulting in end-
organ failure and death.
5.5
As sodium is the main extracellular cation and chloride and bicarbonate are the main anions, the
result should reflect the remaining anions. Normally, this concentration is about 8–16 mmol/l (12 ±
4). An elevated anion gap (ie >16 mmol/l) can indicate particular types of metabolic acidosis,
particularly acidosis due to certain poisons, lactate acidosis and ketoacidosis.
5.6
Answer: B Hypovolaemia
Oliguria and anuria are the decreased or absent production of urine, respectively. The decreased
production of urine may be a sign of dehydration/hypovolaemia, renal failure or urinary
obstruction/urinary retention. Patients usually have a decrease in urine output after a major operation
that may be a normal physiological response to:
• Response of adrenal cortex to stress — increase in aldosterone (sodium and water retention) and
antidiuretic hormone release.
5.7
• Renal insufficiency
trimethoprim
pentamidine
Addison’s disease
• Gordon syndrome (familial hypertension with hyperkalaemia), a rare genetic disorder caused by
defective modulators of salt transporters, including the thiazide-sensitive sodium chloride
cotransporter.
• Rhabdomyolysis, burns or any cause of rapid tissue necrosis, including tumour lysis syndrome
• Shifts/transport out of cells caused by acidosis, low insulin levels, beta-blocker therapy, digoxin
overdose, or the paralysing anaesthetic, succinylcholine.
5.8
Metabolic alkalosis is a primary increase in serum bicarbonate (HCO3-) concentration. This occurs
as a consequence of a loss of H+ from the body or a gain in HCO3-. In its pure form, it manifests as
alkalemia (pH >7.40). As a compensatory mechanism, metabolic alkalosis leads to alveolar
hypoventilation with a rise in arterial carbon dioxide tension p(CO2), which diminishes the change in
pH that would otherwise occur.
Normally, arterial p(CO2) increases by 0.5–0.7 mmHg for every 1 mmol/l increase in plasma
bicarbonate concentration, a compensatory response that occurs very rapidly. If the change in p(CO2)
is not within this range, then a mixed acid–base disturbance occurs. For example, if the increase in
p(CO2) is more than 0.7 times the increase in bicarbonate, then metabolic alkalosis coexists with
primary respiratory acidosis. Likewise, if the increase in p(CO2) is less than the expected change,
then a primary respiratory alkalosis is also present.
The first clue to metabolic alkalosis is often an elevated bicarbonate concentration that is observed
when serum electrolytes are obtained. Remember that an elevated serum bicarbonate concentration
can also be observed as a compensatory response to primary respiratory acidosis. However, a
bicarbonate concentration greater than 35 mmol/l is almost always caused by metabolic alkalosis (as
is the case in this clinical scenario).
Calculation of the serum anion gap can also help to differentiate between primary metabolic alkalosis
and the metabolic compensation for respiratory acidosis. The anion gap is frequently elevated to a
modest degree in metabolic alkalosis because of the increase in the negative charge of albumin and
the enhanced production of lactate. However, the only definitive way to diagnose metabolic alkalosis
is by performing a simultaneous blood gases analysis, which reveals elevation of both pH and arterial
p(CO2) and increased calculated bicarbonate.
There are two ways to determine the serum bicarbonate concentration. The first method is calculating
serum bicarbonate concentration from a blood gas sample using the Henderson–Hasselbalch equation,
as follows:
pH = 6.10 + log[HCo3-] ÷ 0.03 × p(CO2)
Alternatively:
[HCO3-] = 24 × p(CO2) ÷ [H+]
The second method is by measuring the total carbon dioxide content in serum, which is routinely
measured with serum electrolytes obtained from venous blood. In this method, a strong acid is added
to serum, which interacts with bicarbonate in the serum sample, forming carbonic acid. Carbonic acid
dissociates to form carbon dioxide and water and then the carbon dioxide is measured. Note that the
carbon dioxide measured includes bicarbonate and dissolved carbon dioxide. The contribution of
dissolved carbon dioxide is quite small (0.03 × p(CO2)) and is usually ignored, although it accounts
for a difference of 1–3 mmol/l between the measured total carbon dioxide content in venous blood
and the calculated bicarbonate in arterial blood. Thus, at a p(CO2) of 40 mmHg, a total carbon
dioxide content of 25 means a true bicarbonate concentration of 23.8 mmol/l (ie, 25 – 0.03 × 40) (see
also Answer to 5.1).
5.9
Electrocardiography (ECG) is generally done early on in order to identify any influences on the heart,
as hyperkalaemia can cause fatal arrhythmias. With moderate hyperkalaemia, there is reduction of the
size of the P wave and development of tent-shaped T waves. Further hyperkalaemia will lead to
widening of the QRS complex, which ultimately can become sinusoidal in shape. There appears to be
a direct effect of elevated potassium on some of the potassium channels which increases their activity
and speeds membrane repolarisation. Also, hyperkalaemia causes an overall membrane
depolarisation that inactivates many sodium channels. The faster repolarisation of the cardiac action
potential causes the tenting of the T waves, and the inactivation of sodium channels causes a sluggish
conduction of the electrical wave around the heart, which leads to smaller P waves and widening of
the QRS complex.
5.10
Answer: D Na+
The electrolyte disturbance hyponatraemia is defined as a plasma sodium concentration below 135
mmol/l and reflects an excess of total body water content relative to total body sodium content. At
lower levels, water intoxication can result, a very dangerous condition. Hyponatraemia is an
abnormality that can occur in isolation or, most commonly, as a complication of other medical
illnesses. An abnormally low plasma sodium level is best considered in conjunction with the
person’s plasma osmolarity and extracellular fluid volume status.
Most cases of hyponatraemia are associated with reduced plasma osmolarity. In fact, the vast
majority of adult cases are due to increased antidiuretic hormone (ADH). ADH is a hormone that
causes retention of water, but not salt. It is the physician’s task to identify the cause of the increased
ADH activity in each case. In patients who are volume-depleted, that is their blood volume is too
low, ADH secretion is increased, because volume depletion is a potent stimulus for ADH secretion.
As a result, the kidneys of such patients hold on to water and produce a very concentrated urine.
Treatment is simple (if not without risk) — simply restore the patient’s blood volume, thereby turning
off the stimulus for ongoing ADH release and water retention.
Some patients with hyponatraemia have normal blood volume. In these patients the increased ADH
activity and subsequent water retention might be due to ‘physiological’ causes of ADH release, such
as pain or nausea. Alternatively, they might have the syndrome of inappropriate ADH secretion
(SIADH). In SIADH there is sustained, non-physiological release of ADH and this most often occurs
as a side-effect of certain medicines, lung problems such as pneumonia or abscess, brain disease, or
certain cancers (most often small-cell lung carcinoma).
A third group of patients with hyponatraemia are often said to be hypervolaemic. They are identified
by the presence of peripheral oedema. In fact, the term ‘hypervolaemic’ is misleading because their
blood volume is actually low. The oedema underscores the fact that fluid has left the circulation,
meaning that the oedema represents fluid that has exited the circulation and settled in dependent areas.
Because such patients do, in fact, have reduced blood volume, and because reduced blood volume is
a potent stimulus for ADH release, it is easy to see why they have retained water and become
hyponatraemic. Treatment of these patients involves treating the underlying disease that caused the
fluid to leak out of the circulation in the first place. In many cases, this is easier said than done
because the responsible underlying conditions are diseases such as liver cirrhosis or heart failure,
conditions that are notoriously difficult to manage, let alone cure.
5.11
Answer: D Hypokalaemia
Cardiac effects of hypokalaemia are usually minimal until plasma potassium levels are less than 3
mmol/l. Hypokalaemia leads to sagging of the ST segment, depression of the T wave and elevation of
the U wave. With marked hypokalaemia, the T wave becomes progressively smaller and the U wave
becomes increasingly larger. Sometimes, a flat or positive T wave merges with a positive U wave,
which can be confused with QT prolongation. Hypokalemia can also cause premature ventricular and
atrial contractions, ventricular and atrial tachyarrhythmias, and second-or third-degree
atrioventricular block. Such arrhythmias become more severe with increasingly severe hypokalaemia;
eventually, ventricular fibrillation can occur. Patients with significant pre-existing heart disease
and/or those receiving digoxin are at risk of cardiac conduction abnormalities, even with mild
hypokalaemia.
5.12
Hypokalaemia can be caused by decreased intake of potassium but is usually caused by excessive
losses of potassium in the urine or from the gastrointestinal tract. Liddle syndrome is a rare autosomal
dominant disorder characterised by severe hypertension and hypokalaemia. Liddle syndrome is
caused by unrestrained sodium reabsorption in the distal nephron due to one of several mutations
found in genes encoding for epithelial sodium channel subunits. Inappropriately high reabsorption of
sodium results in both hypertension and renal potassium wasting.
5.13
Answer: E Spironolactone
Hyperkalaemia resulting from total body potassium excess is particularly common in oliguric states
(especially acute renal failure) and with rhabdomyolysis, burns, bleeding into soft tissue or the
gastrointestinal tract and in adrenal insufficiency. In chronic renal failure hyperkalaemia is uncommon
until the glomerular filtration rate (GFR) falls to less than 10–15 ml/minute unless dietary potassium
intake is excessive or another source of excess potassium load is present, such as oral or parenteral
potassium therapy, gastrointestinal bleeding, tissue injury or haemolysis. Other potential causes of
hyperkalaemia in chronic renal failure are hyporeninaemic hypoaldosteronism (type 4 renal tubular
acidosis), angiotensin-converting enzyme (ACE) inhibitors, potassium-sparing diuretics such as
spironolactone, fasting (suppression of insulin secretion), beta-blockers, and non-steroidal anti-
inflammatory drugs (NSAIDs). If sufficient potassium chloride is ingested or given parenterally,
severe hyperkalaemia can result, even with normal renal function. Causes are usually iatrogenic, such
as giving potassium supplements to patients taking ACE inhibitors. Other drugs that can limit renal
potassium output, thereby leading to hyperkalaemia, include ciclosporin, lithium, heparin and
trimethoprim.
5.14
5.15
Hyperphosphataemia occasionally results from a transcellular shift of phosphate into the extracellular
space that is so large that the renal excretory capacity is overwhelmed. This occurs most frequently in
diabetic ketoacidosis (despite total body phosphate depletion), crush injuries and non-traumatic
rhabdomyolysis, as well as in overwhelming systemic infections and tumour lysis syndrome.
Hyperphosphataemia also plays a critical role in the development of secondary hyperparathyroidism
and renal osteodystrophy in patients on dialysis. Lastly, hyperphosphataemia can be spurious in cases
of hyperproteinaemia (in multiple myeloma or Waldenström’s macroglobulinaemia, for example),
hyperlipidaemia, haemolysis or hyperbilirubinaemia.
5.16
Answer: D Psychosis
In euvolemic hyponatraemia the total body sodium and so the extracellular fluid volume are normal;
however, total body water is increased. Primary polydipsia can cause hyponatraemia only when
water intake overwhelms the kidneys’ ability to excrete water. Because normal kidneys can excrete
up to 25 litres of urine a day, hyponatraemia due solely to polydipsia results only from the ingestion
of large amounts of water or from defects in renal diluting ability. Patients affected include those with
psychosis or more modest degrees of polydipsia plus renal insufficiency. Dilutional hyponatraemia
can also result from excessive water intake without sodium retention in the presence of Addison’s
disease, myxoedema or non-osmotic antidiuretic hormone (ADH) secretion (due to: stress;
postoperative states; or the use of drugs such as chlorpropamide or tolbutamide, opioids,
barbiturates, vincristine, clofibrate or carbamazepine). Postoperative hyponatraemia occurs because
of a combination of non-osmotic ADH release and excessive administration of hypotonic fluids after
surgery. Certain drugs (eg cyclophosphamide, non-steroidal anti-inflammatory drugs,
chlorpropamide) potentiate the renal effect of endogenous ADH, whereas others (eg oxytocin) have a
direct ADH-like effect on the kidney. A deficiency in water excretion is common in all these
conditions.
5.17
Answer: A Cirrhosis
Hypervolaemic hyponatraemia is characterised by an increase in both total body sodium (and thus
extracellular fluid volume) and total body water, with a relatively greater increase in total body
water. Various oedematous disorders, including heart failure and cirrhosis, cause hypervolaemic
hyponatraemia. Rarely, hyponatraemia occurs in nephrotic syndrome, although pseudohyponatraemia
can be due to interference with sodium measurement by elevated lipids. In each of these disorders, a
decrease in effective circulating volume results in the release of antidiuretic hormone (ADH) and
angiotensin II. Hyponatraemia results from the antidiuretic effect of ADH on the kidney as well as the
direct impairment of renal water excretion by angiotensin II. Decreased glomerular filtration rate and
stimulation of thirst by angiotensin II also potentiate the development of hyponatraemia. Urine sodium
excretion is usually <10 mmo//l and urine osmolality is high relative to plasma osmolality.
5.18
Answer: C Nephrotic syndrome
An increase in total body sodium is the key pathophysiological event in extracellular fluid volume
expansion. It increases osmolality, which triggers compensatory mechanisms that produce water
retention. Movement of fluid between interstitial and intravascular spaces depends on Starling’s
forces at the capillaries: increased capillary hydrostatic pressure (as in heart failure), decreased
plasma oncotic pressure (as in nephrotic syndrome) or a combination of these factors (as in severe
cirrhosis) shift fluid into the interstitial space, producing oedema. In these conditions, subsequent
intravascular volume depletion increases renal sodium retention, which maintains fluid overload.
5.19
Answer: A Hypoparathyroidism
5.20
Answer: A 1%
Despite its important intracellular roles, roughly 99% of body calcium is in bone, mainly as
hydroxyapatite crystals. Roughly 1% of bone calcium is freely exchangeable with the extracellular
fluid and therefore is available for buffering changes in calcium ion balance. The normal total plasma
calcium range is 2.20–2.60 mmol/l. About 40% of the total blood calcium is bound to plasma
proteins, primarily albumin. The remaining 60% includes ionized calcium plus calcium complexed
with phosphate and citrate. Total calcium (ie protein-bound plus complexed plus ionised calcium) is
usually what is determined by clinical laboratory measurement. Ideally, the ionised or free calcium
should be determined, because this is the physiologically active form of calcium in plasma; this
determination, because of its technical difficulty, is usually restricted to patients in whom significant
alteration of protein binding of plasma calcium is suspected, however. Ionised calcium is generally
assumed to be roughly 50% of the total plasma calcium.
5.21
Answer: A Hypoalbuminaemia
5.22
An increased anion gap is most commonly caused by metabolic acidosis in which negatively charged
acids (mostly ketones, lactate, sulphates or metabolites of methanol, ethylene glycol and salicylate)
consume (are buffered by) bicarbonate ion. Other causes of increased anion gap include
hyperalbuminaemia and uraemia (increased anions) and hypocalcaemia or hypomagnesaemia
(decreased cations).
5.23
Answer: B Hypokalaemia
• Loss of hydrogen ions. Whenever a hydrogen ion is excreted, a bicarbonate ion is gained into the
extracellular space. Hydrogen ions can be lost through the kidneys or the gastrointestinal tract.
Vomiting or nasogastric suction generates metabolic alkalosis by the loss of gastric secretions,
which are rich in hydrochloric acid. Renal losses of hydrogen ions occur whenever the distal
delivery of sodium increases in the presence of excess aldosterone, which stimulates the
electrogenic epithelial sodium channel in the collecting duct. As this channel reabsorbs sodium
ions, the tubular lumen becomes more negative, leading to the secretion of hydrogen ions and
potassium ions into the lumen.
• Shift of hydrogen ions into the intracellular space. This mainly develops with hypokaalemia. As
the extracellular potassium concentration decreases, potassium ions move out of the cells. To
maintain neutrality, hydrogen ions move into the intracellular space.
• Alkali administration. Administration of sodium bicarbonate in amounts that exceed the capacity of
the kidneys to excrete this excess bicarbonate can cause metabolic alkalosis. This capacity is
reduced when a reduction in filtered bicarbonate occurs, as observed in renal failure, or when
enhanced tubular reabsorption of bicarbonate occurs, as observed in volume depletion.
5.24
Severe metabolic alkalosis (blood pH >7.55) is a serious medical problem. Mortality rates have
been reported as 45% in patients with an arterial blood pH of 7.55 and 80% when the pH was greater
than 7.65. Severe alkalosis causes diffuse arteriolar constriction, with reduction in tissue perfusion.
By decreasing cerebral blood flow, alkalosis can lead to tetany, seizures and decreased mental status.
Metabolic alkalosis also decreases coronary blood flow and predisposes to refractory arrhythmias.
Metabolic alkalosis causes hypoventilation, which can cause hypoxaemia, especially in patients with
poor respiratory reserve, and it can impair weaning from mechanical ventilation. Alkalosis decreases
the serum concentration of ionised calcium by increasing calcium-ion binding to albumin. In addition,
metabolic alkalosis is almost always associated with hypokalaemia, which can cause neuromuscular
weakness and arrhythmias and, by increasing ammonia production, can precipitate hepatic
encephalopathy in susceptible individuals.
5.25
Phosphorus is one of the most abundant elements in the human body. Most phosphorus in the body is
complexed with oxygen as phosphate (PO43-). About 85% of the roughly 500–700 g of phosphate in
the body is contained in bone, where it is an important constituent of hydroxyapatite. In soft tissues,
phosphate is mainly found in the intracellular compartment as an integral component of several
organic compounds, including nucleic acids and cell membrane phospholipids. Phosphate is also
involved in aerobic and anaerobic energy metabolism. Red blood cell 2,3-diphosphoglycerate (2,3-
DPG) plays a crucial role in the delivery of oxygen to tissues. Adenosine diphosphate (ADP) and
adenosine triphosphate (ATP) contain phosphate and use chemical bonds between PO43- groups to
store energy. Inorganic phosphate is a major intracellular anion but is also present in plasma. The
normal plasma inorganic phosphate concentration in adults ranges from 0.8 mmol/l to 1.45 mmol/l.
Phosphate levels are up to 50% higher in infants and 30% higher in children, possibly because
additional phosphate is required for growth.
5.26
Answer: D Seizures
The primary clinical findings in hypomagnesaemia are neuromuscular irritability, central nervous
system (CNS) hyperexcitability and cardiac arrhythmias. The severity of symptoms is not related
directly to the magnesium level. The reference range for serum magnesium level is 0.75–1.05 mmol/l,
and patients usually become symptomatic at levels of around 0.75 mmol/l, although the typical
physical findings might not be present in all cases. The clinical manifestations are as follows:
• Neuromuscular irritability:
muscle cramps
muscle fibrillation
• CNS hyperexcitability:
disorientation
psychosis
repolarisation alternans.
5.27
Excess free serum phosphorus is taken up into vascular smooth muscle via a sodium–phosphate
cotransporter. The increased cellular phosphate activates a gene (cbfa-1) that promotes calcium
deposition in the vascular cell, making smooth muscle cells engage in osteogenesis. Vascular walls
become calcified and arteriosclerotic, leading to increased systolic blood pressure, widened pulse
pressure and, eventually, left ventricular hypertrophy.
5.28
Answer: D Hypokalaemia
NH3 is produced in the proximal tubule from the amino acid glutamine, and this reaction is enhanced
by an acid load and by hypokalaemia. Ammonia is converted to ammonium ion (NH4+) by
intracellular H+ and is secreted into the proximal tubular lumen by the apical Na+/H+(NH4+)
antiporter. The apical Na+/K+(NH4+)/2Cl- cotransporter in the thick ascending limb of the loop of
Henle then transports NH4+ into the medullary interstitium, where it dissociates back into NH3 and
H+. The NH3 diffuses into the lumen of the collecting duct, where it is available to buffer H+ ions and
becomes NH4+. NH4+ is trapped in the lumen and excreted as the chloride salt, and every H+ ion
buffered is an HCO3- gained to the systemic circulation.
The increased secretion of H+ in the collecting duct shifts the equation to the right and decreases the
NH3 concentration, facilitating continued diffusion of NH3 from the interstitium down its
concentration gradient and allowing more H+ to be buffered. The kidneys can adjust the amount of
NH3 synthesised to meet demand, making this a powerful system to buffer secreted H+ in the urine.
5.29
Answer: A Type A
Lactic acidosis was described and classified by Cohen and Woods into two categories: type A lactic
acidosis occurs with decreased tissue ATP in the setting of poor tissue perfusion or oxygenation; and
type B lactic acidosis occurs when evidence of poor tissue perfusion or oxygenation is absent.
However, in many cases of type B lactic acidosis, occult tissue hypoperfusion is now recognised as
an accompanying factor. Type B lactic acidosis is further divided into three subtypes on the basis of
the underlying aetiology:
• Type B1 lactic acidosis occurs in association with systemic disease such as renal and hepatic
failure, diabetes and/or malignancy.
• Type B2 lactic acidosis is caused by several classes of drugs and toxins, including biguanides,
alcohols, iron, isoniazid and salicylates.
5.30
Answer: D Hyperalimentation
Hypernatraemia in rare cases is associated with volume overload. In this case, hypernatraemia results
from a grossly elevated sodium intake associated with limited access to water. One example is
excessive administration of hypertonic sodium bicarbonate given during cardiopulmonary
rescuscitation or during the treatment of lactic acidosis. Hypernatraemia can also be caused by the
administration of hypertonic saline or hyperalimentation.
SECTION 6:
BLEEDING AND HAEMOSTASIS — ANSWERS
6.1
There are a variety of causes of DIC, all usually causing the release of chemicals into the blood that
instigates the coagulation:
• Obstetric complications (most common cause), with chemicals from the uterus being released into
the blood, or from amniotic fluid embolism, and eclampsia can be causes. Another obstetric
condition which can cause DIC is abruptio placentae.
• Liver disease.
• Incompatible blood transfusion reactions or massive blood transfusion (more than the total
circulatory volume).
• Malignant cancers or hypersensitivity reactions can produce the chemicals leading to DIC.
• Viral haemorrhagic fevers bring about their frank effects, paradoxically, by causing DIC.
• Envenomation by some species of venomous snakes, such as those belonging to the genus Echis
(saw-scaled vipers).
Under homeostatic conditions, the body is maintained in a finely tuned balance of coagulation and
fibrinolysis. The activation of the coagulation cascade yields thrombin that converts fibrinogen to
fibrin; the stable fibrin clot being the final product of haemostasis. The fibrinolytic system then
functions to break down fibrinogen and fibrin. Activation of the fibrinolytic system generates plasmin
(in the presence of thrombin), which is responsible for the lysis of fibrin clots. The breakdown of
fibrinogen and fibrin results in polypeptides called ‘fibrin degradation products’ (FDPs) or ‘fibrin
split products’ (FSPs). In a state of homeostasis, the presence of thrombin is critical because it is the
central proteolytic enzyme of coagulation and is also necessary for the breakdown of clots, or
fibrinolysis.
In DIC, the processes of coagulation and fibrinolysis lose control, and the result is widespread
clotting with resultant bleeding. Regardless of the triggering event of DIC, once initiated, the
pathophysiology of DIC is similar in all conditions. One critical mediator of DIC is the release of a
transmembrane glycoprotein called ‘tissue factor’ (TF). TF is present on the surface of many cell
types (including endothelial cells, macrophages and monocytes) and is not normally in contact with
the general circulation, but is exposed to the circulation after vascular damage. For example, TF is
released in response to exposure to cytokines (particularly interleukin), tumour necrosis factor and
endotoxin. This plays a major role in the development of DIC in septic conditions. TF is also
abundant in tissues of the lungs, brain and placenta. This helps to explain why DIC develops so
readily in patients with extensive trauma. Once activated, TF binds with coagulation factors that then
trigger both the intrinsic and the extrinsic pathways of coagulation.
Excess circulating thrombin results from the excess activation of the coagulation cascade. The excess
thrombin cleaves fibrinogen, which ultimately leaves behind multiple fibrin clots in the circulation.
These excess clots trap platelets to become larger clots, which leads to microvascular and
macrovascular thrombosis. This lodging of clots in the microcirculation, in the large vessels and in
the organs is what leads to the ischaemia, impaired organ perfusion and end-organ damage that occurs
with DIC.
Coagulation inhibitors are also consumed in this process. Decreased inhibitor levels will permit
more clotting so that a feedback system develops in which increased clotting leads to more clotting.
At the same time, thrombocytopenia occurs because of the entrapment of platelets. Clotting factors are
consumed in the development of multiple clots, which contributes to the bleeding seen with DIC.
Although numerous blood tests are often performed on patients prone to develop DIC, the important
measures are: full blood count (especially the platelet count), FDPs or D-dimer tests (markers of
fibrinolysis), bleeding time and fibrinogen levels. Decreased platelets, elevated FDPs or D-dimers,
prolonged bleeding time and decreased fibrinogen are markers of DIC.
6.2
Answer: A Epistaxis
Platelets (thrombocytes) are the cell fragments circulating in the blood that are involved in the
cellular mechanisms of primary haemostasis that lead to the formation of blood clots. Dysfunction or
low levels of platelets predisposes to bleeding. Epistaxis, petechiae and purpura are likely findings
with platelet abnormalities. A normal platelet count in a healthy person is 150–400 × 10 9/l: 95% of
healthy people will have platelet counts in this range. Some will have statistically abnormal platelet
counts while having no abnormality, although the likelihood increases if the platelet count is either
very low or very high.
Generally, low platelet counts increase the risk of bleeding (although there are exceptions, such as
immune heparin-induced thrombocytopenia). Low platelet counts are generally not corrected by
transfusion unless the patient is bleeding or the count has fallen below 5 × 109/l; it is contraindicated
in thrombotic thrombocytopenic purpura (TTP) as it fuels the coagulopathy. In patients having
surgery, a level below 50 × 10 9/l) is associated with abnormal surgical bleeding, and regional
anaesthetic procedures such as epidurals are avoided if levels are below 80–100 × 109/l.
Normal platelet counts are not a guarantee of adequate function. In some states the platelets, while
being adequate in number, are dysfunctional. For instance, aspirin irreversibly disrupts platelet
function by inhibiting cyclo-oxygenase-1 (COX1), and therefore also disrupts normal haemostasis;
normal platelet function might not return until the aspirin has been stopped and all the affected
platelets have been replaced by new ones, which can take over a week. Similarly, uraemia (resulting
from renal failure) leads to platelet dysfunction that can be ameliorated by the administration of
desmopressin.
6.3
Answer: C Scurvy
Scurvy is a deficiency disease that results from lack of vitamin C, which is required for normal
collagen synthesis in humans. Vitamin C is essential for wound healing and facilitates recovery from
burns. The scientific name of vitamin C, ascorbic acid, is derived from the Latin name of scurvy,
scorbutus. Scurvy leads to the formation of petechiae on the skin, spongy gums, and bleeding from all
mucous membranes. The spots are most abundant on the thighs and legs, and a person with the ailment
looks pale, feels depressed, and is partially immobilised. In advanced scurvy there are open,
suppurating wounds and loss of teeth. Scurvy was at one time common among sailors who spent
longer periods at sea than perishable fruits and vegetables could be stored for and by soldiers who
were similarly separated from these foods for extended periods. It was described by Hippocrates (c.
460 BC to c. 380 BC). Its cause and cure has been known in many native cultures since prehistory.
The bleeding is due to capillary fragility and not to some defect of coagulation. Normal collagen
synthesis depends on the hydroxylation of proline and lysine residues in the endoplasmic reticulum, to
form hydroxyproline and hydroxylysine respectively. Prolyl and lysyl hydroxylase, the enzymes that
catalyse the hydroxylation, require ascorbic acid to function correctly. With no ascorbic acid, the
enzymes cannot hydroxylate proline and lysine, and so normal collagen synthesis cannot be
performed.
6.4
Haemophilia A is a clotting disorder caused by a mutation of the factor VIII gene, leading to a
deficiency in factor VIII. It is the most common haemophilia. Inheritance is X-linked recessive, and
so males are affected while females are carriers or (very rarely) display a mild phenotype; 1 in
10,000 males are affected. Haemophilia leads to a severely increased risk of bleeding from common
injuries. The sites of bleeding are joints, muscles, digestive tract and brain. The muscle and joint
haemorrhages are quite typical of haemophilia.
Coagulation testing reveals a prolonged partial thromboplastin time in the context of a normal
prothrombin time and normal bleeding time. The diagnosis is made in the presence of very low levels
of factor VIII (<10 IU). A family history is frequently present, although not essential. Nowadays,
genetic testing might also be performed.
The most important differential diagnosis is that of haemophilia B (also known as ‘Christmas
disease’) or von Willebrand’s disease: the former is usually considered if factor VIII levels are
normal in a person with a haemophilia phenotype; the latter is excluded on routine testing for that
condition. A very small minority of patients have antibodies against factor VIII that impair its
functioning.
Most people with haemophilia require regular supplementation with intravenous recombinant factor
VIII. This is highly individuallydetermined. Apart from ‘routine’ supplementation, extra factor
concentrate is given around surgical procedures and after trauma. Some patients are managed on
desmopressin if the clotting factor is still partially active.
6.5
All cell lines can be increased with myeloproliferative disorders. Thrombocytosis is one
manifestation. The myeloproliferative diseases are a group of diseases of the bone marrow in which
excess cells are produced. They are related to, and can evolve into, myelodysplastic syndrome and
acute myeloid leukaemia, although the myeloproliferative diseases on the whole have a much better
prognosis than these conditions.
6.6
Answer: C Antithrombin III
Antithrombin III is a member of a larger family of antithrombins (numbered I–VI). They are all are
serpins. Only antithromin III (and possibly antithrombin I) is medically significant and this is often
referred to simply as ‘antithrombin’. Antithrombin III deficiency is a rare hereditary disorder that
generally only comes to light when a patient suffers recurrent venous thrombosis and pulmonary
embolism. The patients are treated with anticoagulants or, more rarely, with antithrombin concentrate.
In renal failure, and especially in nephrotic syndrome, antithrombin is lost in the urine, leading to a
higher activity of factor II and factor X and an increased tendency to thrombosis.
6.7
This patient has disseminated intravascular coagulation (DIC). One way for this to happen is through
the release of endotoxins from Gram-negative bacteria with sepsis. The result is widespread
activation of the coagulation system leading to DIC (see also Answer to 6.1).
6.8
Answer: D Haemarthrosis
Haemarthrosis is more of a complication with haemophilia. von Willebrand’s disease (vWD) is the
most common hereditary coagulation abnormality described in humans, although it can also be
acquired as a result of other medical conditions. It arises from a qualitative or quantitative deficiency
of von Willebrand factor (vWF), a multimeric protein that is required for platelet adhesion. There are
three types of hereditary vWD, but other factors such as ABO blood group may also play a part in the
cause of the condition.
When suspected, blood plasma of a patient needs to be investigated for quantitative and qualitative
deficiencies of vWF, by measuring the amount of vWF in a vWF antigen assay and the functionallity
of vWF with a glycoprotein (GP)Ib binding assay, a collagen binding assay, or a ristocetin cofactor
activity (RiCof) or ristocetin-induced platelet agglutination (RIPA) assay. Factor VIII levels are also
measured because factor VIII is bound to vWF, which protects the factor VIII from rapid breakdown
within the blood. Deficiency of vWF can therefore lead to a reduction in factor VIII levels. Normal
levels do not exclude all forms of vWD, particularly type II vWD, which might only be revealed by
investigating platelet interaction with subendothelium under flow, a highly specialised coagulation
study not routinely performed in most medical laboratories. A platelet aggregation assay will show an
abnormal response to ristocetin with normal responses to the other agonists used. A platelet function
assay will give an abnormal collagen/adrenaline closure time but a normal collagen/ADP time.
6.9
6.10
D-Dimer is a blood test performed in the medical laboratory to diagnose thrombosis. Since its
introduction in the 1990s, it has become an important test performed in patients suspected of
thrombotic disorders. While a negative result practically rules out thrombosis, a positive result can
indicate thrombosis but does not rule out other potential underlying conditions. Its main use, therefore,
is to exclude thromboembolic disease where the probability is low.
D-Dimer testing is of clinical use when there is a suspicion of deep venous thrombosis or pulmonary
embolism. In patients suspected of disseminated intravascular coagulation, D-dimers can aid in the
diagnosis. Most sampling kits have a normal range of 0–300 ng/ml; values exceeding 250 ng/ml, 300
ng/ml or 500 ng/ml (different for various kits) are considered positive.
6.11
• Fluctuating neurological symptoms, such as bizarre behaviour, altered mental status, stroke or
headaches (65%)
• Fever (33%)
6.12
Joint haemorrhages (haemarthroses) are typical of haemophilia. The diagnosis might be suspected as
coagulation testing reveals an increased partial thromboplastin time in the context of a normal
prothrombin time and a normal bleeding time. The diagnosis is made in the presence of very low
levels of factor VIII (<10 IU). A family history is frequently present, although not essential (see also
Answer to 6.4).
6.13
Mutations of the factor IX gene leads to an increased propensity for haemorrhage. This occurs in
response to mild trauma or can even occur spontaneously, for example into joints (haemarthrosis) or
muscles. Deficiency of factor IX causes Christmas disease (haemophilia B). Over 100 mutations of
factor IX have been described: some cause no symptoms, but many lead to a significant bleeding
disorder.
6.14
Researchers have identified many variations of the disease, but most fall into the following
classifications:
• Type I vWD. This is the most common and mildest form of von Willebrand’s disease. Levels of
vWF are lower than normal, and levels of factor VIII can also be reduced.
• Type II vWD. In these people, the vWF factor itself has an abnormality. Depending on the
abnormality, they may be classified as having type IIa or type IIb vWD. In type IIa vWD, the level
of vWF is reduced, as is the ability of platelets to clump together. In type IIb vWD, although the
factor itself is defective, the ability of platelets to clump together is actually increased.
• Type III vWD. This is severe von Willebrand’s disease. These people can have a total absence of
vWF, and factor VIII levels are often less than 10% normal.
• Pseudo- (or platelet-type) vWD. This disorder resembles type IIb vWD, but the defects appear to
be in the platelets, rather than in vWF.
vWD is a genetic disease that can be inherited from either parent. It affects males and females
equally. A man or woman with vWD has a 50% chance of passing the gene on to his or her child.
There are no racial or ethnic associations with the disorder. A family history of a bleeding disorder
is the primary risk factor (see also Answer to 6.8).
6.15
Aspirin, or acetylsalicylic acid, is a drug in the family of salicylates, often used as an analgesic and
antipyretic, and as an anti-inflammatory agent. It also has an antiplatelet effect and is used chronically
in low doses to prevent myocardial infarction and thrombus formation in hypercoaguable states. Low-
dose, long-term aspirin use irreversibly blocks the formation of thromboxane A2 in platelets,
producing an inhibitory effect on platelet aggregation.
6.16
Tissue factor (TF), also called ‘thromboplastin’ or ‘factor III’, is a protein present in subendothelial
tissue, platelets and leukocytes that is necessary for the initiation of thrombin formation from the
zymogen prothrombin. Thrombin formation ultimately leads to the coagulation of blood. TF is the cell
surface receptor for the serine protease factor VIIa. The best-known function of TF is its role in blood
coagulation. The complex of TF with factor VIIa catalyses the conversion of the inactive protease
factor X into the active protease factor Xa.
Together with factor VII, TF forms the tissue factor or extrinsic pathway of coagulation. The intrinsic
(amplification) pathway involves both activated factor IX and factor VIII. Both pathways lead to the
activation of factor X (the common pathway), which combines with activated factor V in the presence
of calcium and phospholipid to produce thrombin (thromboplastin activity).
TF is expressed by cells which are normally not exposed to flowing blood, such as subendothelial
cells (eg smooth muscle cells) and cells surrounding blood vessels (eg fibroblasts). This can change
when the blood vessel is damaged, for example by physical injury or rupture of atherosclerotic
plaques. Exposure of TF-expressing cells during injury allows the complex formation of TF with
factor VIIa, accelerating activity of factor VIIa about a thousand-fold.
The inner surface of the blood vessel consists of endothelial cells. Endothelial cells do not express
TF except when they are exposed to inflammatory molecules such as tumour necrosis factor-alpha
(TNF-α). Another cell type that expresses TF on the cell surface in inflammatory conditions is the
monocyte, a white blood cell.
6.17
Answer: A D-Dimer
A schistocyte is a red blood cell undergoing fragmentation, or a fragmented part of a red blood cell.
Schistocytes can be seen in disseminated intravascular coagulation (DIC) and the clinical scenario in
this scenario fits with DIC. Measurements ofD-dimer are ordered, along with other tests, to help
diagnose DIC. DIC is a complex acute condition that can arise from a variety of situations, including
some surgical procedures, septic shock, poisonous snake bites and liver disease, and can occur
postpartum. With DIC, clotting factors are activated and then used up throughout the body. This
creates numerous minute blood clots and at the same time leaves the patient vulnerable to excessive
bleeding. Steps are taken to support the patient while the underlying condition resolves. D-Dimer
levels can be used to monitor the effectiveness of DIC treatment (see also Answer to 6.10).
6.18
The main role of factor VII (FVII) is to initiate the process of coagulation in conjunction with tissue
factor (TF). Tissue factor is found on the outside of blood vessels and is not normally exposed to the
bloodstream. When a vessel is injured, tissue factor is exposed to the blood and circulating factor
VII. Once bound to TF, FVII is activated to form FVIIa by different proteases, among which are
thrombin (factor IIa), activated factor X and the FVIIa-TF complex itself. The most important
substrates for FVIIa-TF are factor IX and factor X. Factor VII is vitamin K-dependent; it is produced
in the liver and its production can be affected by liver disease. Warfarin and similar anticoagulants
also impair its function.
6.19
Aspirin inhibits platelet function, but has no effect on a clot that is already formed. Once the clot has
formed, neither warfarin nor heparin are suitable as initial therapy for the condition. Fibrinolytic
drugs are given once a clot is formed. Fibrinolytic drugs are used, for example, to reperfuse the
myocardium by dissolving the thrombus blocking the cornary arteries and in massive pulmonary
embolism. This process is called ‘thrombolysis’. Thrombolysis requires the use of thrombolytic or
fibrinolytic drugs, which are either derived from Streptomyces species or (more recently) by
recombinant technology, where human activators of plasminogen (eg tissue plasminogen activator,
tPA) are manufactured by bacteria.
• Streptokinase
• Urokinase
• Reteplase
• Tenecteplase.
All thrombolytic agents work by activating the enzyme plasminogen, which clears the cross-linked
fibrin mesh (the backbone of a clot). This makes the clot soluble and subject to further proteolysis by
other enzymes, and restores blood flow in occluded blood vessels.
6.20
Warfarin is an anticoagulant medication that is administered orally or, very rarely, by injection. It is
used for the prophylaxis of thrombosis and embolism in many disorders. Its activity has to be
monitored by frequent blood testing for the international normalised ratio (INR). Warfarin is a
synthetic derivative of coumarin, a chemical found naturally in many plants, notably woodruff, and at
lower levels in liquorice, lavender and various other species. Warfarin decreases blood coagulation
by interfering with vitamin K metabolism and for this reason it is referred to as a vitamin K
antagonist.
Warfarin inhibits the synthesis of biologically active forms of the vitamin K-dependent clotting
factors II, VII, IX and X, as well as the regulatory factors, protein C, protein S and protein Z. Other
proteins not involved in blood clotting, such as osteocalcin, or matrix Gla protein, can also be
affected.
The precursors of these factors require carboxylation of their glutamic acid residues to allow the
coagulation factors to bind to phospholipid surfaces inside blood vessels, on the vascular
endothelium. The enzyme that carries out the carboxylation of glutamic acid is gamma-glutamyl
carboxylase. The carboxylation reaction will only proceed if the carboxylase enzyme is able to
convert a reduced form of vitamin K (vitamin K hydroquinone) to vitamin K epoxide at the same time.
The vitamin K epoxide is in turn recycled back to vitamin K and vitamin K hydroquinone by another
enzyme, the vitamin K epoxide reductase. Warfarin inhibits epoxide reductase, thereby diminishing
available vitamin K and vitamin K hydroquinone in the tissues, which inhibits the carboxylation
activity of the glutamyl carboxylase. When this occurs, the coagulation factors are no longer
carboxylated at certain glutamic acid residues, and are incapable of binding to the endothelial surface
of blood vessels, and are thus biologically inactive. As the body stores of previously produced active
factors degrade (over several days) and are replaced by inactive factors, the anticoagulation effect
becomes apparent. The coagulation factors are produced, but have decreased functionality due to
undercarboxylation; they are collectively referred to as PIVKAs (proteins induced [by] vitamin K
absence/antagonism). The effect of warfarin, therefore, is to diminish blood clotting in the patient.
Concomitant intake of aspirin can displace warfarin from plasma protein, leading to a warfarin
overdose. If aspirin is prescribed, then dose of warfarin must be decreased.
6.21
Answer: C Normal PT, increased PTT, low factor VIII AHF, normal vWFAg
The partial thromboplastin time will be increased with classic haemophilia and factor VIII
antihaemophilic factor (also called ‘factor VIIIc’) is decreased, but von Willebrand factor antigen is
usually normal or increased (see also Answer to 6.4).
6.22
The patient in this scenario has disseminated intravascular coagulation (DIC) due to Gram-negative
sepsis. Bacterial infection, in particular septicaemia, is commonly associated with DIC. No
difference exists in the incidence of DIC in patients with Gram-negative sepsis or Gram-positive
sepsis. In addition, systemic infections with other micro-organisms, such as viruses and parasites, can
also lead to DIC. Factors involved in the development of DIC in patients with infections can be
specific cell membrane components of the micro-organism (lipopolysaccharide or endotoxin) or
bacterial exotoxins (eg staphylococcal alpha toxin). These components cause a generalised
inflammatory response, characterised by the systemic dissemination of pro-inflammatory cytokines
(see also Answer to 6.1).
6.23
Antiphosphilipid syndrome can cause arterial or venous blood clots in any organ system or can lead
to pregnancy-related complications (especially miscarriage in the second or third trimester). The
most common venous event is deep venous thrombosis of the lower extremities and the most common
arterial event is stroke. Other common findings, although not part of the APS classification criteria,
are thrombocytopenia, heart valve disease and the skin condition, livedo reticularis. Some patients
report headaches and migraines. Antiphospholipid syndrome can rarely mimic multiple sclerosis and
it is estimated that 10% of patients are misdiagnosed. Very few patients with primary APS go on to
develop SLE.
6.24
Answer: E Thrombocytopenia
The platelets are responsible for dealing with small leaks in vessels. Thrombocytopenia is marked by
petechiae and purpura. Generally speaking, the normal range for the platelet count is 150–450 × 109/l.
Decreased platelet counts can be due to decreased production or to increased destruction of platelets;
medication-induced low counts can be caused by either.
• Dengue fever, due to direct infection of bone marrow megakaryocytes as well as immunologically
mediated shortened platelet survival
• Hereditary syndromes:
Fanconi’s anaemia
Bernard–Soulier syndrome (associated with large platelets)
Alport syndrome.
• Antiphospholipid syndrome
• Post-transfusion purpura
• Heparin
• Valproic acid
• Sulfonamides
• Clopidogrel
• Vancomycin.
If the cause of the low platelet count remains unclear, bone marrow biopsy is often undertaken in
order to assess whether the low platelet count is due to decreased production or peripheral
destruction.
6.25
Fresh frozen plasma (FFP) is an effective treatment for coagulopathies resulting from disseminated
intravascular coagulation, liver disease, and massive transfusion. FFP is defined as the fluid portion
of one unit of human blood that has been centrifuged, separated, and frozen solid at –18 °C (or
colder) within 6 hours of collection. FFP contains the labile as well as the stable components of the
coagulation, fibrinolytic and complement systems; the proteins that maintain oncotic pressure and
modulate immunity; and other proteins that have diverse activities. In addition, fats, carbohydrates
and minerals are present in concentrations similar to those in the circulation. Although well-defined
indications exist for the use of FFP in single or multiple coagulation deficiencies, indications for
many of its other uses is often empirical.
FFP is efficacious for the treatment of deficiencies of factors II, V, VII, IX, X and XI when specific
component therapy is either not available or not appropriate. Requirements for FFP vary with the
specific factor being replaced. For example, haemostatic levels of factor IX in a patient with severe
deficiency are difficult to achieve with FFP alone, whereas patients with severe factor X deficiency
require factor levels of about 10% to achieve haemostasis and are easily treated with FFP.
In this scenario the patient has coagulopathy due to possible deficiency of coagulation factors, as
suggested by prolonged partial thromboplastin time.
6.26
The prothrombin time (PT) and its derived measures of prothrombin ratio (PR) and international
normalised ratio (INR) are measures of the extrinsic pathway of coagulation. The reference range for
prothrombin time is usually around 12–15 seconds; the normal range for the INR is 0.8–1.2. PT
measures factors II, V, VII, and X and fibrinogen. It is used in conjunction with the activated partial
thromboplastin time (aPTT), which measures the intrinsic pathway. The speed of the extrinsic
pathway is greatly affected by levels of factor VII in the body. Factor VII has a short half-life and its
synthesis requires vitamin K. The prothrombin time can be prolonged as a result of deficiencies in
vitamin K, which can be caused by warfarin, malabsorption, or a lack of intestinal colonisation by
bacteria (eg in newborns). In addition, poor factor VII synthesis (due to liver disease) or increased
consumption (in disseminated intravascular coagulation) can prolong the PT.
6.27
Answer: C Hypofibrinogenaemia
The thrombin clotting time (TCT), also known as the ‘thrombin time’ (TT), is a coagulation assay that
is usually performed in order to assess for the therapeutic level of heparin. It is also sensitive in
detecting the presence of fibrinogen abnormalities. The reference interval of the TCT is generally less
than 21 seconds, depending on the method and the patient population. Results outside of the reference
interval indicate heparin therapy, fibrinogen abnormality or the presence of lupus anticoagulant.
6.28
Factor VII (formerly known as ‘proconvertin’) is one of the central proteins in the coagulation
cascade. It is an enzyme of the serine protease class. The main role of factor VII (FVII) is to initiate
the process of coagulation in conjunction with tissue factor (TF). TF is found on the outside of blood
vessels and so not normally exposed to the bloodstream. On vessel injury, TF is exposed to the blood
and circulating factor VII. Once bound to TF, FVII is activated to FVIIa by different proteases, among
which are thrombin (factor IIa), activated factor X and the FVIIa-TF complex itself. The most
important substrates for FVIIa-TF are factor X and factor IX.
The action of factor VII is impeded by tissue factor pathway inhibitor (TFPI), which is released
almost immediately after initiation of coagulation. Factor VII is vitamin K-dependent, and is
produced in the liver. The use of warfarin or similar anticoagulants impairs its function.
6.29
Answer: C Protein C
Warfarin necrosis is an acquired protein C deficiency due to treatment with the vitamin K-inhibitor
anticoagulant, warfarin. It is a feared (but rare) complication of warfarin treatment. This rare reaction
usually occurs between the third and tenth days of therapy with warfarin derivatives, usually in
women. Lesions are sharply demarcated, erythematous, indurated and purpuric, and can either resolve
or progress to form large, irregular, haemorrhagic bullae, with eventual necrosis and slow-healing
eschar formation. Development of the syndrome is unrelated to drug dose or underlying condition.
The most usual sites affected are breasts, thighs and buttocks. The course is not altered by
discontinuation of the drug once the lesions have appeared. In initial stages of action, inhibition of
protein C can be stronger than inhibition of the vitamin K-dependent coagulation factors (factors II,
VII, IX and X), leading to paradoxical activation of coagulation and necrosis of skin areas. It occurs
mainly in patients with a deficiency of protein C. Protein C is an innate anticoagulant, and as warfarin
further decreases protein C levels by inhibiting vitamin K, this can lead to massive thrombosis, with
necrosis and gangrene of the limbs.
6.30
Protein S is a vitamin K-dependent plasma glycoprotein that is synthesised in the liver. In the
circulation, protein S exists in two forms: a free form and a complex form bound to complement
protein C4b. The best characterised function of protein S is its role in the anticoagulation pathway,
where it functions as a cofactor to protein C in the inactivation of factors Va and VIIIa. Only the free
form has cofactor activity.
Protein S can bind to negatively charged phospholipids via the carboxylated GLA domain. This
property allows protein S to function in the removal of cells which are undergoing apoptosis.
Apoptosis is a form of cell death that is used by the body to remove unwanted or damaged cells from
tissues. Cells which are apoptotic no longer actively manage the distribution of phospholipids in their
outer membrane and so begin to display negatively charged phospholipids, such as phosphatidyl
serine on the cell surface. In healthy cells, an adenosine triphosphate- (ATP-) dependent enzyme
removes these from the outer leaflet of the cell membrane. These negatively charged phospholipids
are recognised by phagocytes such as macrophages. Protein S can bind to the negatively charged
phospholipids and function as a bridging molecule between the apoptotic cell and the phagocyte. The
bridging property of protein S enhances the phagocytosis of the apoptotic cell, allowing it to be
removed ‘cleanly’ without any symptoms of tissue damage (such as inflammation).
Protein S deficiency is a rare blood disorder which can lead to an increased risk of thrombosis.
6.31
Factor XIII (FXIII), or ‘fibrin stabilising factor’ is an enzyme of the coagulation cascade that cross-
links fibrin. When thrombin has converted fibrinogen to fibrin, the latter forms a proteinaceous
network in which every E unit is cross-linked to only one D unit. Factor XIII is activated by thrombin
into factor XIIIa; its activation into factor XIIIa requires calcium as a cofactor. FXIII is known also as
‘Laki–Lorand’ factor, after the scientists who first proposed its existence in 1948. Factor XIII
deficiency is a rare deficiency, and can cause a severe bleeding tendency. The incidence is 1 in 1
million to 1 in 5 million people.
6.32
Answer: B Factor V
The coagulation factors are generally serine proteases. There are some exceptions. For example,
factor VIII and factor V are glycoproteins and factor XIII is a transglutaminase. Factor V is
occasionally referred to as ‘pro-accelerin’ or ‘labile factor’. In contrast to most other coagulation
factors, it is not enzymatically active but functions as a cofactor. Deficiency leads to predisposition to
haemorrhage, while some mutations (most notably factor V Leiden) predispose to thrombosis.
Factor V circulates in plasma as a single-chain molecule with a plasma half-life of about 12 hours,
though this has been reported to be up to 36 hours. Factor V is able to bind to activated platelets and
is activated by thrombin. On activation, factor V is spliced into two chains (a heavy and a light chain,
with molecular masses of 110 kDa and 73 kDa respectively) which are non-convalently bound to
each other by calcium. Factor V is active as a cofactor of the thrombinase complex. The activated
factor X (FXa) enzyme requires calcium and activated factor V to convert prothrombin to thrombin on
the cell surface membrane. This is considered to be part of the common pathway in the coagulation
cascade.
6.33
Answer: E Thrombomodulin
Thrombomodulin is an integral membrane protein expressed on the surface of endothelial cells. The
protein has a molecular mass of 74 kDa, and consists of a single chain with five distinct domains. It
functions as a cofactor in the thrombin-induced activation of protein C in the anticoagulant pathway
by forming a 1 : 1 stochiometric complex with thrombin. This raises the speed of protein C activation
1000-fold. Thrombomodulin-bound thrombin has no pro-coagulant effect. The thrombomodulin-
thrombin complex also inhibits fibrinolysis by cleaving thrombin-activatable fibrinolysis inhibitor
(TAFI) into its active form.
6.34
Heparin is a naturally occurring anticoagulant produced by basophils and mast cells. A highly
sulphated glycosaminoglycan, heparin is widely used as an injectable anticoagulant and has the
highest negative charge density of any known biological molecule. Pharmaceutical-grade heparin is
commonly derived from mucosal tissues of slaughtered meat animals such as porcine intestine or
bovine lung.
6.35
Heparin binds to the enzyme inhibitor, antithrombin III (AT-III), causing a conformational change that
results in its active site being exposed. The activated AT-III then inactivates thrombin and other
proteases involved in blood clotting, most notably factor Xa. The rate of inactivation of these
proteases by AT-III increases 1000-fold due to the binding of heparin. AT-III binds to a specific
pentasaccharide sulphation sequence contained within the heparin polymer (GlcNAc/NS(6S)-GlcA-
GlcNS(3S,6S)-IdoA(2S)-GlcNS(6S)). The conformational change in AT-III on heparin binding
mediates its inhibition of factor Xa. For thrombin inhibition, however, thrombin must also bind to the
heparin polymer at a site proximal to the pentasaccharide. The highly negative charge density of
heparin contributes to its very strong electrostatic interaction with thrombin. The formation of a
ternary complex between AT-III, thrombin and heparin results in the inactivation of thrombin. For this
reason heparin‘s activity against thrombin is size-dependent, the ternary complex requiring at least 18
saccharide units for efficient formation. In contrast, anti-factor Xa activity only requires the
pentasaccharide binding site.
6.36
6.37
Answer: D Factor X
Inborn deficiency of factor X is very uncommon (1 in 500,000) and can present with epistaxis,
haemarthrosis and gastrointestinal blood loss. Apart from congenital deficiency, low factor X levels
can occasionally occur in a number of disease states. Deficiency of vitamin K or antagonism by
warfarin (or similar medications) leads to the production of an inactive factor X. (In warfarin
therapy, this is desirable to prevent thrombosis.)
6.38
Answer: D Factor XI
Deficiency of factor XI causes the rare haemophilia C. This mainly occurs in Ashkenazi Jews and is
believed to affect approximately 8% of that population (both sexes). One per cent of cases occur in
other population groups. It is an autosomal recessive disorder. There is little spontaneous bleeding,
but surgical procedures can cause excessive blood loss, and prophylaxis is required.
6.39
Answer: D Factor XI
Factor XI (or plasma thromboplastin antecedent) is one of the enzymes of the coagulation cascade.
Like many other coagulation factors, it is a serine protease. Factor XI (FXI) is produced by the liver
and circulates as a homo-dimer in its inactive form. The plasma half-life of FXI is approximately 52
hours. The zymogen factor is activated into factor XIa by factor XIIa (FXIIa), thrombin, and it is also
autocatalytic. FXI is a member of the ‘contact pathway’ due to activation by FXIIa (with includes
high-molecular-weight kininogen, prekallikrein, factor XII, factor XI and factor IX). Factor XIa
activates factor IX by selectively cleaving arg-ala and arg-val peptide bonds. Factor IXa, in turn,
activates factor X.
6.40
7.1
Shock, or cardiovascular collapse, is the final common pathway for a number of potentially lethal
clinical events, including severe haemorrhage, extensive trauma or burns, a large myocardial
infarction, massive pulmonary embolism and microbial sepsis. Regardless of the underlying
pathology, shock gives rise to systemic hypoperfusion caused by reduction either in cardiac output or
in the effective circulating blood volume. The end results are hypotension, followed by impaired
tissue perfusion and cellular hypoxia. Although the hypoxic and metabolic effects of hypoperfusion
initially cause only reversible cellular injury, persistence of shock eventually causes irreversible
tissue injury and can culminate in the death of the patient.
• Cardiogenic shock results from myocardial pump failure. This can be caused by intrinsic
myocardial damage (infarction), ventricular arrhythmias, extrinsic compression (cardiac
tamponade) or outflow obstruction (eg due to pulmonary embolism).
• Hypovolaemic shock results from loss of blood or plasma volume. This can be caused by
haemorrhage as in this case, fluid loss from severe burns or trauma.
• Septic shock is caused by systemic microbial infection. Most commonly, this occurs in the setting
of Gram-negative infections (endotoxic shock), but it can also occur with Gram-positive and fungal
infections.
The mechanisms underlying cardiogenic and hypovolaemic shock are fairly straightforward,
essentially involving low cardiac output. Septic shock, in comparison, is substantially more
complicated and results from a complex interplay of peripheral vasodilation and pooling of blood,
endothelial activation/injury, leukocyte-induced damage, disseminated intravascular coagulation, and
activation of cytokine cascades.
Less commonly, shock can occur in the setting of anaesthetic accident or spinal cord injury
(neurogenic shock), owing to loss of vascular tone and peripheral pooling of blood. Anaphylactic
shock, initiated by a generalised IgE-mediated hypersensitivity response, is associated with systemic
vasodilation and increased vascular permeability. In these instances, widespread vasodilatation
causes a sudden increase in the vascular bed capacitance, which is not adequately filled by the
normal circulating blood volume. Hypotension, tissue hypoperfusion and cellular anoxia result.
7.2
Infective endocarditis, one of the most serious of all infections, is characterised by colonisation or
invasion of the heart valves or the mural endocardium by a microbe, leading to the formation of bulky,
friable vegetations composed of thrombotic debris and organisms, often associated with destruction
of the underlying cardiac tissues. The aorta, aneurysmal sacs, other blood vessels and prosthetic
devices can also become infected. Although fungi, rickettsiae (Q fever), and chlamydiae have at one
time or another been responsible for these infections, most cases are bacterial. Prompt diagnosis and
effective treatment of infective endocarditis can significantly alter the outlook for the patient.
Infective endocarditis can develop on previously normal valves, but a variety of cardiac and vascular
abnormalities predispose to this form of infection. In years past, rheumatic heart disease was the
major antecedent disorder, but more common now are myxomatous mitral valve, degenerative
calcific valvular stenosis, bicuspid aortic valve (whether calcified or not) and artificial (prosthetic)
valves. Host factors such as neutropenia, immunodeficiency, malignancy, therapeutic
immunosuppression, diabetes mellitus, and alcohol or intravenous drug abuse are predisposing
influences. Sterile platelet-fibrin deposits that accumulate at sites of impingement of jet-streams
caused by pre-existing cardiac disease or indwelling vascular catheters can also be important in the
development of endocarditis.
The causative organisms differ somewhat in the major high-risk groups. Endocarditis of native but
previously damaged or otherwise abnormal valves is caused most commonly by Streptococcus
viridans (50%–60% of cases). Note that this is not the organism responsible for rheumatic disease. In
contrast, the more virulent Staphylococcus aureus organisms commonly found on the skin can attack
either healthy or deformed valves and are responsible for 10%–20% of cases overall; S. aureus is
the major offender in intravenous drug abusers. The roster of the remaining bacteria includes
enterococci and the so-called HACEK group (Haemophilus, Actinobacillus, Cardiobacterium,
Eikenella, and Kingella), all commensals in the oral cavity. Prosthetic valve endocarditis is caused
most commonly by coagulase-negative staphylococci (eg S. epidermidis). Other agents causing
endocarditis include Gram-negative bacilli and fungi. In about 10% of all cases of endocarditis, no
organism can be isolated from the blood (‘culture-negative’ endocarditis) because of prior antibiotic
therapy, difficulties in isolating the offending agent, or because deeply embedded organisms within
the enlarging vegetation are not released into the blood.
7.3
Giant-cell (temporal) arteritis, the most common form of systemic vasculitis in adults, is an acute or
chronic, often granulomatous, inflammation of arteries of large to small size. It affects principally the
arteries in the head, especially the temporal arteries, but also the vertebral and ophthalmic arteries
and the aorta, where it can cause thoracic aortic aneurysm. Ophthalmic arterial involvement can lead
to permanent blindness, and visual loss caused by giant-cell arterits is a medical emergency that
requires prompt recognition and treatment. Lesions can be found in other arteries throughout the body,
including the aorta (giant cell aortitis).
Temporal arteritis is most common in older individuals and rare before age 50 years. Symptoms are
either just vague and constitutional (fever, fatigue, weight loss) without localising signs or symptoms,
or comprise facial pain or headache, often most intense along the course of the superficial temporal
artery, which can be painful to palpation. More serious are the ocular symptoms (associated with
involvement of the ophthalmic artery), which appear quite abruptly in about half of patients and range
from diplopia to transient or complete vision loss. The diagnosis depends on biopsy and histological
confirmation, but because of the segmental nature of the involvement, adequate biopsy requires at
least a 2- to 3-cm length of artery, and negative or atypical findings on biopsy do not rule out the
condition. Treatment with anti-inflammatory agents is generally very effective.
7.4
Atrial fibrillation (AF) is one of the most common arrhythmias. The incidence is higher in men than in
women and it is also more common in white people than it is in black people. The prevalence
increases with age, and almost 10% of people over the age of 80 years are affected. It tends to occur
in patients with a heart disorder, sometimes precipitating heart failure because cardiac output
decreases in the absence of atrial contraction. The absent atrial contractions also predispose to
thrombus formation, and the annual risk of cerebrovascular embolic events is about 7%. The risk of
stroke is higher in patients with a rheumatic valvular disorder, hyperthyroidism, hypertension,
diabetes, left ventricular systolic dysfunction or previous thromboembolic events. Systemic emboli
can also cause malfunction or necrosis of other organs (eg heart, kidneys, gastrointestinal tract, eye)
or of a limb.
AF is often asymptomatic, but many patients have palpitations, vague chest discomfort, or symptoms
of heart failure (eg weakness, light-headedness, dyspnoea), particularly when the ventricular rate is
very rapid (it is often 140–160/minute). Patients can also present with symptoms and signs of acute
stroke or of other organ damage due to systemic emboli. The pulse is irregularly irregular with loss of
a waves in the jugular venous pulse. A pulse deficit (the apical ventricular rate is faster than the rate
palpated at the wrist) might be present because the left ventricular stroke volume is not always
sufficient to produce a peripheral pressure wave at fast ventricular rates.
The diagnosis is made by electrocardiography (ECG). Findings include the absence of P waves, f
waves (‘fibrillatory’ waves) between QRS complexes (irregular in timing, irregular in morphology;
baseline undulations at rates >300/minute but not always apparent in all leads), and irregularly
irregular R–R intervals. Other irregular rhythms can resemble AF on ECG but can be distinguished by
the presence of discrete P waves or flutter waves, which can sometimes be made more visible with
vagal manoeuvres.
7.5
Answer: C ST-segment elevation
Electrocardiography (ECG) is the most important test and should be done within 10 minutes of
presentation in a patient suspected of having a myocardial infarction (MI). It is the centre of the
decision-making pathway because fibrinolytics benefit patients with ST-segment-elevation MI
(STEMI) but can increase the risk for those with non-ST-segment-elevation MI (NSTEMI). Fo
STEMI, the initial ECG is usually diagnostic, showing ST-segment elevation of 1 mm or more in two
or more contiguous leads subtending the damaged area. Pathological Q waves are not necessary for
the diagnosis. The ECG must be read carefully because ST-segment elevation can be subtle,
particularly in the inferior leads (II, III, aVF); sometimes the reader’s attention is mistakenly focused
on leads with ST-segment depression. If symptoms are characteristic, ST-segment elevation on ECG
has a specificity of 90% and a sensitivity of 45% for diagnosing MI. Serial tracings (obtained q 8 h
for 1 day, then daily) showing a gradual evolution towards a stable, more normal pattern or the
development of abnormal Q waves over a few days tends to confirm the diagnosis.
If right ventricular infarction is suspected, a 15-lead ECG is usually recorded; additional leads are
placed at V4R and, to detect posterior infarction, at V8 and V9. ECG diagnosis of MI is more difficult
when a left bundle branch block configuration is present because it resembles STEMI changes. ST-
segment elevation concordant with the QRS complex strongly suggests MI, as does an ST-segment
elevation of more than 5 mm in at least two precordial leads. But generally, any patient with suspect
symptoms and new-onset (or not known to be old) left bundle branch block is treated as for STEMI.
7.6
Pulmonary embolism (PE) nearly always arises from thrombus in the lower extremity or pelvic veins
(deep venous thrombosis, or DVT). Thrombi in either system can be occult. Thromboemboli can also
originate in upper extremity veins or in the right cardiac chambers. Risk factors for DVT and PE are
similar in children and adults and include conditions that impair venous return or that cause
endothelial injury or dysfunction, particularly in patients with an underlying baseline hypercoagulable
state. Bedrest and confinement without walking, even for a few hours, are common precipitators as a
result of stasis of blood (the most likely mechanism in this case).
Once a DVT develops, the clot can dislodge and travel through the venous system and right heart to
lodge in the pulmonary arteries, where it partially or completely occludes one or more vessels. The
consequences depend on the size and number of emboli, the pulmonary reaction, and the ability of the
body’s intrinsic thrombolytic system to dissolve the clot. Small emboli might have no acute
physiological effects; many begin to lyse immediately and resolve within hours or days. Larger
emboli can cause a reflex increase in ventilation (tachypnoea); hypoxaemia from ventilation/perfusion
mismatch and shunting; atelectasis from alveolar hypocapnia and abnormalities in surfactant; and an
increase in pulmonary vascular resistance caused by mechanical obstruction and vasoconstriction.
Endogenous lysis reduces most emboli, even those of moderate size, without treatment, and
physiological alterations decrease over hours or days. Some emboli resist lysis and can organise and
persist. Occasionally, chronic residual obstruction leads to pulmonary hypertension (chronic
thromboembolic pulmonary hypertension) that can develop over years and result in chronic right heart
failure. When large emboli occlude major arteries, or when several small emboli occlude over 50%
of the distal arterial system, the right ventricular pressure increases, causing acute right ventricular
failure, failure with shock (massive PE), or sudden death in severe cases. The risk of death depends
on the degree and rate of rise of right-sided pressures and on the patient’s underlying
cardiopulmonary status; higher pressures occur more commonly in patients with preexisting
cardiopulmonary disease. Healthy patients may survive a PE that occludes more than 50% of the
pulmonary vascular bed.
Pulmonary infarction occurs in fewer than 10% of patients diagnosed with PE. This low rate has been
attributed to the dual blood supply to the lung (ie bronchial and pulmonary). Infarction is typically
characterised by a radiographic infiltrate, chest pain, fever and, occasionally, haemoptysis.
7.7
Fat embolism is caused by the introduction of fat or bone marrow particles into the systemic venous
system and then to the pulmonary arteries. Causes include long-bone fractures (as in this case),
orthopaedic procedures, microvascular occlusion or necrosis of bone marrow in patients with sickle
cell crisis and, rarely, toxic modification of native or parenteral serum lipids. Fat embolism causes a
pulmonary syndrome similar to the acute respiratory distress syndrome, with severe hypoxaemia of
rapid onset, often accompanied by neurological changes and a petechial rash.
7.8
7.10
Phaeochromocytomas in the adrenal medulla occur equally in both sexes, are bilateral in 10% of
cases (in 20% in children), and are malignant in fewer than 10% of cases. Of extra-adrenal tumours,
30% are malignant. Although phaeochromocytomas occur at any age, the peak incidence is between
the twenties and forties. Phaeochromocytomas vary in size but are on average 5–6 cm in diameter.
They weigh 50–200 g, but tumours weighing several kilograms have been reported. Rarely, they are
large enough to be palpated or cause symptoms due to pressure or obstruction. Regardless of the
histological appearance, the tumour is considered to be benign if it has not invaded the capsule and no
metastases are found, although exceptions occur.
Phaeochromocytomas are sometimes part of the syndrome of familial multiple endocrine neoplasia
(MEN), types IIA or IIB, in which other endocrine tumours (parathyroid or medullary carcinoma of
the thyroid) coexist or develop subsequently. Phaeochromocytoma develops in 1% of patients with
neurofibromatosis (von Recklinghausen’s disease) and can occur with haemangiomas and renal cell
carcinoma, as in von Hippel–Lindau disease. Familial phaeochromocytomas and carotid body
tumours may be due to mutations of the enzyme, succinate dehydrogenase.
7.11
Pulmonary embolism (PE) is the occlusion of one or more pulmonary arteries by thrombi that
originate elsewhere, typically in the large veins of the lower extremities or pelvis. Risk factors are
conditions that impair venous return and that cause endothelial injury or dysfunction, especially in
patients with an underlying hypercoagulable state.
Most PEs are small, physiologically insignificant, and asymptomatic. Even when present, symptoms
are nonspecific and vary in frequency and intensity, depending on the extent of pulmonary vascular
occlusion and pre-existing cardiopulmonary function. Larger emboli cause acute dyspnoea and
pleuritic chest pain and, less commonly, cough and/or haemoptysis. Massive PE presents with
hypotension, tachycardia, syncope or cardiac arrest.
The most common signs of PE are tachycardia and tachypnoea. Less commonly, patients have
hypotension, a loud second heart sound (S2) due to a loud pulmonary component (P2) and/or crackles
or wheezing. In the presence of right ventricular failure, distended internal jugular veins and a right
ventricular heave may be evident, and right ventricular gallop (third and fourth heart sounds [S3 and
S4]), with or without tricuspid regurgitation, may be audible. Fever can occur, and in fact deep
venous thrombosis and PE are often overlooked causes of fever. Chronic thromboembolic pulmonary
hypertension causes symptoms and signs of right heart failure, including exertional dyspnoea, easy
fatigue, and peripheral oedema that develops over a period of months to years.
The diagnosis of PE is challenging, because symptoms and signs are non-specific and diagnostic tests
are either imperfect or invasive. Diagnosis starts by including PE in the differential diagnosis of a
large number of conditions with similar symptoms, including cardiac ischaemia, heart failure, chronic
obstructive pulmonary disease (COPD) exacerbation, pneumothorax, pneumonia, sepsis, acute chest
syndrome (in sickle cell patients) and acute anxiety with hyperventilation. Initial evaluation should
include pulse oximetry, electrocardiography (ECG) and chest X-ray. The chest X-ray usually is non-
specific but might show atelectasis, focal infiltrates, an elevated hemidiaphragm and/or a pleural
effusion. The classic findings of focal loss of vascular markings (Westermark’s sign), a peripheral
wedge-shaped density (Hampton’s hump) or enlargement of the right descending pulmonary artery
(Palla’s sign) are suggestive but very insensitive.
Pulse oximetry provides a quick way to assess oxygenation; hypoxaemia is one sign of PE, and other
significant disorders must be investigated. ECG most often shows tachycardia and various STT-wave
abnormalities, which are not specific for PE. An S1Q3T3 or a new right bundle branch block might
indicate the effect of an abrupt rise in right ventricular pressure on right ventricular conduction. These
are specific but insensitive signs, occurring in only about 5% of patients. Right axis deviation (R > S
in V1) and P-pulmonale can be present. T-wave inversion in leads V1 to V4 also occurs.
7.12
Acute aortic dissection is the surging of blood through a tear in the aortic intima, with separation of
the intima and media and creation of a false lumen. The intimal tear can be a primary event or can be
secondary to haemorrhage within the media. The dissection can occur anywhere along the aorta and
extend proximally or distally into other arteries. Hypertension is an important contributor. Symptoms
and signs include abrupt onset of tearing chest or back pain, and dissection can result in aortic
regurgitation and compromised circulation in branch arteries. Diagnosis is by imaging tests (eg
transoesophageal echocardiography, computed tomographic angiography, magnetic resonance
imaging, contrast aortography).
Treatment always involves aggressive blood pressure control and serial imaging to monitor
progression of the dissection; surgical repair of the aorta and placement of a synthetic graft is needed
for ascending aortic dissections and for certain descending aortic dissections. One fifth of patients die
before reaching the hospital, and up to a third die of operative or perioperative complications.
7.13
Answer: C Echocardiography
In this clinical scenario the clinical features are suggestive of a valvular lesion which can best be
assessed by echocardiography. Echocardiography uses ultrasound waves to produce an image of the
heart and great vessels. It helps assess heart wall thickness (eg in hypertrophy or atrophy) and motion,
and provides information about ischaemia and infarction and valvular function and structure. It can be
used to assess diastolic filling patterns of the left ventricle, which can help in the diagnosis of left
ventricular hypertrophy, hypertrophic or restrictive cardiomyopathy, severe heart failure, constrictive
pericarditis, and severe aortic regurgitation.
7.14
The clinical features in this case are suggestive of deep venous thrombosis. Duplex ultrasonography
is a safe, non-invasive, portable technique for detecting lower extremity (primarily femoral vein)
thrombi. A clot can be detected in up to three ways: by visualising the lining of the vein, by
demonstrating lack of compressibility of the vein, and by demonstrating reduced flow by Doppler.
The test has a sensitivity of over 90% and a specificity of over 95% for thrombus. It cannot reliably
detect a clot in calf or iliac veins. The absence of thrombi in the femoral veins does not exclude the
possibility of thrombus from other sources, but patients with negative duplex test results have an
event-free survival rate of over 95% because thrombi from other sources are so much less common.
Ultrasonography has been incorporated into many diagnostic algorithms because an ultrasound
positive for femoral vein thrombosis indicates the need for anticoagulation, which may make further
testing for pulmonary embolism or other thrombi unnecessary.
7.15
Acute pericarditis can result from infection, autoimmune and inflammatory disorders, uraemia,
trauma, myocardial infarction (MI) or certain drugs. Infectious pericarditis is most often viral.
Purulent bacterial pericarditis is uncommon but can follow infective endocarditis, pneumonia,
septicaemia, penetrating trauma or cardiac surgery. Often, the cause cannot be identified (non-
specific or idiopathic pericarditis) but many of these cases are probably viral. Overall, the most
common causes are viral and idiopathic. Acute MI causes 10%–15% of cases of acute pericarditis.
Post-MI syndrome (Dressler syndrome) is a less common cause now, occurring mainly when
reperfusion with percutaneous transluminal coronary angioplasty (PTCA) or thrombolytic drugs are
ineffective in patients with transmural infarction. Pericarditis occurs after pericardiotomy (post-
pericardiotomy syndrome) in 5%–30% of cardiac operations.
Pericarditis causes chest pain and a pericardial rub, sometimes with dyspnoea. The first evidence can
be tamponade, with hypotension, shock or pulmonary oedema. Because the innervation of the
pericardium and myocardium is the same, the chest pain of pericarditis is sometimes similar to that of
myocardial inflammation or ischaemia: Dull or sharp precordial or substernal pain can radiate to the
neck, trapezius ridge (especially on the left) or shoulders. Pain ranges from mild to severe. Unlike
ischaemic chest pain, pain due to pericarditis is usually aggravated by thoracic movement, cough,
breathing or swallowing food; it can be relieved by sitting up and leaning forwards. There can be
tachypnoea and non-productive cough, and fever, chills and weakness are common. In 15%–25% of
patients with idiopathic pericarditis, symptoms recur intermittently for months or years.
The most important physical finding is a triphasic or a systolic and diastolic precordial friction rub.
However, the rub is often intermittent and evanescent; it might be present only during systole or, less
frequently, only during diastole. If no rub is heard with the patient seated and leaning forwards,
auscultation could be attempted with the patient ‘on all fours’ and listening with the diaphragm of the
stethoscope. Sometimes, a pleural component to the rub is noted during breathing, which is due to
inflammation of the pleura adjacent to the pericardium. Considerable amounts of pericardial fluid can
muffle the heart sounds, increase the area of cardiac dullness and change the size and shape of the
cardiac silhouette.
The diagnosis is based on the presence of typical clinical findings and ECG abnormalities. Serial
ECGs may be needed to show abnormalities. The ECG in acute pericarditis might show
abnormalities confined to the ST segments and T waves, usually in most of the leads. The ST
segments in two or three of the standard leads become elevated but subsequently return to baseline.
Unlike MI, acute pericarditis does not cause reciprocal depression in the ST segments (except in
leads aVR and V1), and there are no pathological Q waves. The PR segment can be depressed. After
several days or longer, the T waves can become flattened and then inverted throughout the ECG,
except in lead aVR; T-wave inversion occurs after the ST segment has returned to baseline, in
contrast to the pattern seen in acute ischaemia or MI.
7.16
Cardiac markers or cardiac enzymes are proteins from cardiac tissue found in the blood. These
proteins are released into the bloodstream when damage to the heart occurs, as in myocardial
infarction. Until the 1980s, the enzymes serum aspartate aminotransferase (AST) and lactate
dehydrogenase (LDH) were used to assess cardiac injury. It was then found that a disproportional
elevation of the MB subtype of the enzyme creatine kinase (CK) was very specific for myocardial
injury. Current guidelines are generally in favour of troponin subunits I or T, which are very specific
for the heart muscle and are thought to rise before permanent injury develops. The CK enzyme
consists of two subunits, which can be either ‘B’ (brain-type) or ‘M’ (muscle-type). There are,
therefore, three different isoenzymes: CKMM, CK-BB and CK-MB. The genes for these subunits are
located on different chromosomes: B on 14q32 and M on 19q13. In addition to those, there are two
mitochondrial creatine kinases, the ubiquitous and sarcomeric forms. Isoenzyme patterns differ in
tissues. CK-BB occurs mainly in tissues, and its levels rarely have any significance in the
bloodstream. Skeletal muscle expresses CK-MM (98%) and CK-MB at low levels (1%). The
myocardium, in contrast, expresses CK-MM at 70% and CK-MB at 30%.
7.17
Persistent truncus arteriosus (or truncus arteriosus) is a rare form of congenital heart disease that
presents at birth. It derives its name from the embryological structure it is derived from, also known
as the ‘truncus arteriosus’. In this condition, the vessel never properly divides into the pulmonary
artery and aorta
Usually, this defect occurs spontaneously. Genetic disorders and teratogens (viruses, metabolic
imbalance and industrial or pharmacological agents) have been associated as possible causes. Up to
50% of cases are associated with chromosome 22q11 deletions. The neural crest, specifically a
population of cells known as the ‘cardiac neural crest’, directly contributes to the aorticopulmonary
septum. Microablation of the cardiac neural crest in developing chick embryos and genetic anomalies
affecting this population of cells in rodents results in persistent truncus arteriosus. Numerous
abnormalities affecting the cardiac neural crest have been associated with persistent truncus
arteriosus, including abnormalities of growth factors (fibroblast growth factor 8 and bone
morphogenetic protein), transcription factors (T-box, Pax, Nkx2-5, GATA-6 and Forkhead) and gap
junction proteins (connexin). The cardiac neural crest also contributes the smooth muscle of the great
arteries.
7.19
A mycotic aneurysm is a focal dilatation of an artery that occurs as a result of infection, necrosis and
weakening of the arterial wall. Because it is usually associated with bacterial endocarditis, the term
is inappropriate as the causative agents are usually bacteria and not a fungus. In this case it should be
called ‘bacterial endocarditis’ and not ‘mycotic endocarditis’. The bacteria most commonly involved
a r e Staphylococcus aureus, Pneumococcus and group A streptococci. Bacterial aneurysms are
believed to start when an infected embolus lodges in the vessel wall; infection, inflammation and
necrosis lead to weakening of the wall and focal aneurysmal dilatation that follows pulsations.
7.20
Eisenmenger syndrome (or Eisenmenger’s reaction) is defined as the process in which a left-to-right
shunt in the heart causes increased flow through the pulmonary vasculature, causing pulmonary
hypertension, which in turn causes increased pressures in the right side of the heart and reversal of the
shunt into a right-to-left shunt. Defining conditions that are required for the diagnosis of Eisenmenger
syndrome are:
• An underlying heart defect that allows blood to pass between the left and right sides of the heart
• Pulmonary hypertension, or elevated blood pressure in the lungs
The left side of the heart supplies blood to the whole body and as a result has higher pressures than
the right side, which supplies only deoxygenated blood to the lungs. If a large anatomical defect exists
between the sides of the heart, blood will flow from the left side to the right side. This results in high
blood flow and pressure travelling through the lungs. The increased pressure causes damage to
delicate capillaries, which are then replaced with scar tissue. Scar tissue does not contribute to
oxygen transfer, leading to a decrease in the useful volume of the pulmonary vasculature. The scar
tissue is also less flexible than normal lung tissue, and this causes further increases in blood pressure,
so that the heart must pump harder to continue supplying the lungs, leading to further capillary
damage.
The reduction in oxygen transfer reduces oxygen saturation in the blood, leading to increased
production of red blood cells in an attempt to bring the oxygen saturation up. The excess of red blood
cells is called ‘polycythaemia’. Desperate for enough circulating oxygen, the body begins to dump
immature red cells into the bloodstream. Immature red cells are not as efficient at carrying oxygen as
mature red cells and they are less flexible and less able to squeeze easily through tiny capillaries in
the lungs, and so contribute to the death of pulmonary capillary beds. The increase in red blood cells
also causes hyperviscosity syndrome.
A person with Eisenmenger syndrome is paradoxically subject to the possibility of both uncontrolled
bleeding due to damaged capillaries and high pressure and the development of random clots due to
hyperviscosity and stasis of blood. The rough places in the heart lining at the site of the septal
defects/shunts tend to gather platelets and keep them out of circulation, and can become the source of
random clots. Eventually, due to increased resistance, pulmonary pressures can increase sufficiently
to cause a reversal of blood flow, so blood begins to travel from the right side of the heart to the left
side, and the body is supplied with deoxygenated blood, leading to cyanosis and organ damage.
SECTION 8:
PULMONARY PATHOLOGY—ANSWERS
8.1
Answer: B Bronchiectasis
Cystic fibrosis is one of the most common life-shortening, childhood-onset inherited diseases. In the
United States the incidence is 1 in 1000. In Victoria, Australia the incidence is 1 in 3600. In northern
Italy the incidence is 1 in 4300. It is most common among Europeans and Ashkenazi Jews; 1 in 22
people of European descent carry one gene for cystic fibrosis, making it the most common genetic
disease among this population.
Individuals with cystic fibrosis can be diagnosed prior to birth by genetic testing or in early
childhood by a sweat test. Newborn screening tests are increasingly common and effective. There is
no cure for cystic fibrosis, and most people with cystic fibrosis will die young (many in their twenties
and thirties) from respiratory failure, although with many new treatments being introduced, the life
expectancy is increasing. Ultimately, lung transplantation is often necessary as the cystic fibrosis
worsens.
CF is caused by a mutation in a gene called the ‘cystic fibrosis transmembrane conductance regulator’
(CFTR). The product of this gene helps to make sweat, digestive juices and mucus. Although most
people without cystic fibrosis have two working copies of CFTR, only one is needed to prevent
cystic fibrosis. Cystic fibrosis is an autososmal recessive disease and only develops when neither
gene works normally. The name ‘cystic fibrosis’ refers to the characteristic ‘fibrosis’ and cyst
formation within the pancreas, first recognized in the 1930s.
Cystic fibrosis affects the entire body and affects growth, breathing, digestion and reproduction. The
newborn period can be marked by poor weight gain and intestinal blockage caused by thick faeces.
Other symptoms of cystic fibrosis appear during the remainder of childhood and early adulthood.
These include continued problems with growth, the onset of lung disease, and increasing difficulties
with poor absorption of vitamins and nutrients by the gastrointestinal tract. In addition, difficulties
with fertility will manifest.
Lung disease in cystic fibrosis results from clogging of airways due to inflammation. Inflammation
and infection cause injury to the lungs and structural changes that lead to a variety of symptoms. In the
early stages, incessant coughing, copious phlegm production and reduced ability to exercise are
common. Many of these symptoms occur when bacteria that normally inhabit the thick mucus grow out
of control and cause pneumonia. In the later stages of cystic fibrosis, changes in the architecture of the
lung further exacerbate chronic difficulties in breathing.
8.2
Lobar pneumonia is the result of alveolar wall injury with severe haemorrhagic oedema and is caused
by inhaled infectious organisms that reach the subpleural zone of the lung. This injury is followed by
a rapid multiplication of organisms and invasion of the infected oedematous fluid by polynuclear
leukocytes. The process spreads rapidly through the pores of Kohn, leading to consolidation of an
entire lobe or segment.
The typical radiological pattern is one of air-space consolidation involving an entire lobe containing
air bronchograms. Because of the use of antibiotics, the pneumonia is limited to one or more segments
within a lobe. Necrosis and cavitation are potential complications of lobar pneumonia. Pulmonary
gangrene can occur (rarely).
8.3
This is a description of the typical ‘Ghon complex’ of an initial, or primary, tuberculosis (TB)
infection. Air-borne droplet nuclei lodge in subpleural terminal air spaces, predominantly in the
lower lung, usually in only one site. Tubercle bacilli replicate inside macrophages, ultimately killing
them; inflammatory cells are attracted to the area, causing a tubercle and sometimes pneumonitis. In
the early weeks of infection some infected macrophages are borne to regional lymph nodes (eg hilar,
mediastinal). Haematogenous spread to any part of the body, particularly the apical-posterior portion
of the lungs, epiphyses of the long bones, kidneys, vertebral bodies or meninges can occur. In 95% of
cases, after about 3 weeks of uninhibited growth, the immune system suppresses bacillary replication
before symptoms or signs develop. Foci of infection in the lung or other sites resolve into epithelioid-
cell granulomas, which can have caseous and necrotic centres; tubercle bacilli can survive in this
material for years, the host’s resistance determining whether the infection ultimately resolves without
treatment, remains dormant or becomes active. Foci can leave nodular scars in the apices of one or
both lungs (‘Simon foci’), calcified scars from the primary infection (the Ghon foci) or calcified hilar
lymph nodes. The tuberculin skin test is positive.
Rarely, the primary focus immediately progresses, causing acute illness with pneumonia (sometimes
cavitary), pleural effusion and marked mediastinal or hilar lymph node enlargement (which can
compress bronchi in children). Small pleural effusions are predominantly lymphocytic, typically
contain few organisms, and clear within a few weeks. Primary extrapulmonary TB at any site can
sometimes present without any evidence of lung involvement. TB lymphadenopathy is the most
common extrapulmonary presentation; meningitis is the most feared complication because of its high
mortality in the very young and very old.
8.4
Clinical features of pulmonary embolism (PE) are sudden-onset dyspnoea, tachypnoea, chest pain of
‘pleuritic’ nature (worsened by breathing), cough, haemoptysis, and, in severe cases, cyanosis,
tachycardia, hypotension, shock, loss of consciousness and death. Although in most cases there is no
clinical evidence of deep venous thrombosis (DVT) in the legs, findings that indicate DVT may aid in
the diagnosis.
The most common sources of embolism are proximal leg deep venous thrombosis or pelvic vein
thromboses. Any risk factor for DVT also increases the risk that the venous clot will dislodge and
migrate to the lung circulation, which happens in up to 15% of all DVTs. The conditions are generally
regarded as a continuum called ‘venous thromboembolism’.
• Alterations in blood flow: immobilisation (after surgery, injury or long-distance air travel),
pregnancy (also pro-coagulant), obesity (also pro-coagulant)
• Factors in the vessel wall (of limited direct relevance in venous thromboembolism)
8.5
8.6
Most lung abscesses develop after aspiration of oral secretions by patients with gingivitis or poor
oral hygiene who are unconscious or obtunded from alcohol, illicit drugs, anaesthesia, sedatives or
opioids. Older patients and those unable to handle their oral secretions, often because of neurological
disease, are also at risk. Lung abscess less commonly complicates necrotising pneumonia that may
have developed from haematogenous seeding of the lungs due to septic embolism from intravenous
drug use or suppurative thromboembolism. In contrast to aspiration, these conditions typically cause
multiple rather than isolated lung abscesses.
The most common pathogens are anaerobic bacteria, but about a half of all cases involve both
anaerobic and aerobic organisms. The most common aerobic pathogens are streptococci.
Immunocompromised patients with lung abscess are more likely to have infection with Nocardia,
mycobacteria or fungi. People from developing countries are at risk of abscess due to
Mycobacterium tuberculosis, amoebic infection (Entamoeba histolytica), paragonimiasis or
Burkholderia pseudomallei.
Lung abscess is suspected on the basis of the history, physical examination and chest X-ray. In
anaerobic infection due to aspiration, the chest X-ray classically shows consolidation, with a single
cavity containing an air–fluid level in portions of the lung that are dependent when the patient is
recumbent (eg the posterior segment of the upper lobe or the superior segment of the lower lobe).
This pattern helps to distinguish anaerobic abscess from other causes of cavitary pulmonary disease,
such as diffuse or embolic pulmonary disease, which may cause multiple cavitations, or tuberculosis,
which involves the apices. Computed tomography is not required routinely, but can be useful when
the X-ray suggests a cavitating lesion or when an underlying pulmonary mass obstructing the drainage
of a lung segment is suspected.
8.7
Answer: A Asbestosis
Asbestosis, a form of interstitial pulmonary fibrosis, is much more common than malignant disease.
Shipbuilders, textile and construction workers, workers who work with asbestos removal, and miners
exposed to asbestos fibres are among the many categories of workers at risk of the disease. Second-
hand exposure can occur among family members of exposed workers and among those who live close
to mines. The pathophysiology is similar to that of other pneumoconioses – alveolar macrophages
attempting to engulf inhaled fibres release cytokines and growth factors that stimulate inflammation,
collagen deposition and ultimately fibrosis – in addition to the fact that asbestos fibres themselves
can also be directly toxic to lung tissue. The risk of disease is generally related to duration and
intensity of exposure and the type, length and thickness of the inhaled fibres.
Asbestosis is initially asymptomatic but can cause progressive dyspnoea, non-productive cough and
fatigue. The disease progresses in over 10% of patients after exposure ceases. Advanced asbestosis
can cause clubbing, dry bibasilar crackles and, in severe cases, symptoms and signs of right
ventricular failure (cor pulmonale). The diagnosis is based on the history of exposure and chest X-ray
or computed tomography (CT). Chest X-rays show linear reticular or nodular opacities suggestive of
fibrosis, usually in the peripheral lower lobes, often accompanied by pleural changes. Honeycombing
signifies more advanced disease, which can involve the mid-lung fields. As with silicosis, the
severity is graded on the International Labor Organization scale on the basis of size, shape, location
and profusion of opacities. In contrast to silicosis, however, asbestosis produces reticular opacities
with a lower lobe predominance. Hilar and mediastinal adenopathy are uncharacteristic and suggest a
different diagnosis. Chest X-ray is insensitive; high-resolution chest CT (HRCT) is useful when
asbestosis is a likely diagnosis. HRCT is also superior to the chest X-ray in identifying the pleural
abnormalities. Pulmonary function tests, which might show reduced lung volumes, and the diffusing
capacity of the lung for carbon monoxide (DLCO) are non-specific but help to characterise changes in
lung function over time once the diagnosis has been made. Bronchoalveolar lavage or lung biopsy is
indicated only when non-invasive measures fail to provide a conclusive diagnosis; the demonstration
of asbestos fibres indicates asbestosis in people with pulmonary fibrosis, although such fibres can
occasionally be found in the lungs of exposed people without disease.
8.8
Squamous cell carcinoma accounts for 25%–30% of all lung cancers. The classic manifestation is a
cavitary lesion in a proximal bronchus. Histologically, this tumour is characterised by the presence of
keratinisation and/or intercellular bridges. Keratinisation can take the form of squamous pearls or
individual cells with markedly eosinophilic (pink), dense cytoplasm. These features are prominent in
the well-differentiated tumours.
8.9
• Acute onset
• Bilateral infiltrates
• Pulmonary artery wedge pressure less than 19 mmHg (or no clinical signs of congestive heart
failure)
• p(O2)/Fi(O2) ratio less than 200 (ARDS) or less than 300 (acute lung injury, a milder clinical
expression of the injury of ARDS that may or may not progress to ARDS).
Several conditions have been found to precipitate ARDS, though in some cases a predisposing
condition cannot be identified. The following is a list of the most common predisposing conditions:
• Infection: pneumonia of any cause (especially viral) and systemic sepsis (especially Gram-
negative)
ARDS is thought to develop when pulmonary or systemic inflammation leads to systemic release of
cytokines and other pro-inflammatory molecules. The cytokines activate alveolar macrophages and
recruit neutrophils to the lungs, which in turn release leukotrienes, oxidants, platelet-activating factor
and proteases. These substances damage capillary endothelium and alveolar epithelium, disrupting
the barriers between capillaries and air spaces. Oedema fluid, protein, and cellular debris flood the
air spaces and interstitium, causing disruption of surfactant, air-space collapse, ventilation–perfusion
mismatch, shunting, stiffening of the lungs with decreased compliance and pulmonary hypertension.
The injury is distributed heterogeneously but mainly affects dependent lung zones.
Histopathologically, diffuse alveolar damage results, with intra-alveolar neutrophils, red blood cells
and cellular debris, and denuded epithelial basement membranes with formation of hyaline
membranes.
8.10
8.11
Answer: C Sarcoidosis
Sarcoidosis is characterised by non-caseating granulomas in one or more organs and tissues. The
aetiology of sarcoidosis is unknown. Sarcoidosis primarily affects people between the ages of 20
years and 40 years but occasionally affects children and older adults. The lungs and lymphatic system
are most often affected, but sarcoidosis can affect any organ. Pulmonary symptoms range from none
(limited disease) to exertional dyspnoea and, rarely, respiratory or other organ failure (advanced
disease). Diagnosis usually is first suspected because of pulmonary involvement and is confirmed by
chest X-ray, biopsy and exclusion of other causes of granulomatous inflammation.
8.12
Small cell lung cancers (SCLCs) are considered to be distinct from the other lung cancers, the non-
small-cell lung cancers (NSCLCs), because of their clinical and biological characteristics. SCLCs
exhibit aggressive behaviour, with rapid growth, early spread to distant sites, exquisite sensitivity to
chemotherapy and radiation and frequent association with distinct paraneoplastic syndromes (see
table below). Surgery usually plays no role in its management, except in rare situations (<5% of
patients) in which it presents at a very early stage as a solitary pulmonary nodule. Even then, adjuvant
chemotherapy after surgical resection is recommended, because SCLC should always be considered
to be a systemic disease.
In this scenario the patient has features of Cushing syndrome secondary to ectopic ACTH production.
8.13
The lung is the most common site for metastatic neoplasms. Pulmonary metastases occur most
frequently with tumours that have rich systemic venous drainage. Examples of such metastases include
renal cancers, bone sarcomas, choriocarcinomas, melanomas, testicular teratomas and thyroid
carcinomas. Most pulmonary metastases arise from common tumours, such as breast, colorectal,
prostate, bronchial, head and neck, and renal cancers.
8.14
Transudates are ultrafiltrates of plasma in the pleura caused by a small, defined group of underlying
conditions:
• Hypoalbuminaemia
• Nephrotic syndrome
• Peritoneal dialysis
• Myxoedema
• Constrictive pericarditis.
Laboratory testing helps to distinguish pleural fluid transudates from exudates. However, certain
types of exudative pleural effusions might be suspected simply by observing the quality of the fluid
obtained during thoracentesis:
• A milky, opalescent fluid suggests a chylothorax, resulting most often from lymphatic obstruction by
malignancy or thoracic duct injury by trauma or surgical procedures.
• Grossly bloody fluid indicates the need for a spun haematocrit test of the sample. A pleural fluid
haematocrit level of more than 50% of the peripheral hematocrit level defines a haemothorax,
which often requires tube thoracostomy.
The initial diagnostic consideration is distinguishing transudate from exudate. The fluid is considered
an exudate if any of the following apply:
• Pleural fluid to serum lactate dehydrogenase (LDH) ratio more than 0.6
• Pleural fluid LDH more than two-thirds of the upper limits of normal serum value.
8.15
‘Chylothorax’ refers to the presence of lymphatic fluid in the pleural space, secondary to leakage
from the thoracic duct or one of its main tributaries. A chylothorax can be non-traumatic or traumatic,
or can be a pseudochylothorax.
Non-traumatic chylothorax:
• Malignant causes account for more than 50% of chylothorax diagnoses and are separated into
lymphomatous and non-lymphomatous. Lymphoma is the most common cause, representing about
60% of all cases, with non-Hodgkin’s lymphoma more likely than Hodgkin’s lymphoma to cause a
chylothorax. In contrast, non-lymphomatous causes are rare.
• Non-malignant causes are separated into idiopathic, congenital and miscellaneous. Clinicians must
rule out all possible malignant causes before designating the chylothorax as idiopathic.
Traumatic chylothorax:
• Trauma is the second leading cause of chylothorax (25%). Iatrogenic injury to the thoracic duct has
been reported with most thoracic procedures. In particular, cardiothoracic surgery has been
associated with 69%– 85% of cases of chylothorax in children.
• Non-surgical traumatic injury is a rare cause, and usually secondary to penetrating trauma.
Pseudochylothorax:
8.16
Sputum cytology consists of the microscopic examination of a sample of sputum in order to determine
whether abnormal cells are present. Sputum has some normal lung cells in it. Sputum cytology can be
done as the first diagnostic test to help detect lung cancer and certain non-cancerous lung conditions.
A sputum sample can be collected by coughing up mucus, by breathing in a saline mist and then
coughing, or during bronchoscopy.
8.17
Several Nocardia species (family Actinomycetaceae) cause human disease. The most common human
pathogen is N. asteroides, which usually causes pulmonary and disseminated infection. N.
brasiliensis most commonly causes skin infection, particularly in tropical climates. Infection is via
inhalation or by direct inoculation of the skin. Nocardiosis occurs worldwide in all age groups, but
the incidence is greater among older adults, especially men. Person-to-person spread is rare.
Lymphoreticular malignancies, organ transplantation, high-dose corticosteroid or other
immunosuppressive therapy and underlying pulmonary disease are predisposing factors, but about
50% of patients have no pre-existing disease. Nocardiosis is also an opportunistic infection in
patients with advanced human immunodeficiency virus (HIV) infection.
8.18
Answer: E Pneumothorax
The risk of pneumothorax is 20%–40%, but it is rarely significant enough to require insertion of a
chest tube. Although it is impossible to predict which patients will experience this complication,
pneumothorax is more frequent and more serious in patients with severe emphysema and in patients in
whom the biopsy was difficult to perform.
8.19
Bronchial carcinoids are rare, slow-growing neuroendocrine tumours arising from bronchial mucosa
that affect patients in their forties to sixties. Half the patients with these tumours are asymptomatic,
and half present with symptoms of airway obstruction, including dyspnoea, wheezing and cough,
which often leads to a misdiagnosis of asthma. Recurrent pneumonia, haemoptysis and chest pain are
also common. Paraneoplastic syndromes, including Cushing’s syndrome due to ectopic
adrenocorticotrophic hormone (ACTH), acromegaly due to ectopic growth hormone-releasing factor
and Zollinger–Ellison syndrome due to ectopic gastrin production, are more common than carcinoid
syndrome, which occurs in fewer than 3% of patients with the tumour. A left-sided heart murmur
(mitral stenosis or regurgitation) occurs rarely, due to serotonin-induced valvular damage (in contrast
to the right-sided valvular lesions associated with gastrointestinal carcinoids). Diagnosis is based on
bronchoscopic biopsy, but evaluation often initially involves computed tomography of the chest,
which reveals tumour calcifications in up to a third of patients. Indium111-labelled octreotide scans
are useful for determining regional and metastatic spread. Increased urinary serotonin and 5-
hydroxyindoleacetic acid levels support the diagnosis but are not commonly present.
8.20
The size of the lesion and the fact that the patient is healthy, asymptomatic and a non-smoker all point
towards a benign lung lesion. Hamartomas are the third most common cause of a solitary pulmonary
nodule and the most common benign tumour of the lung, accounting for 75% of all benign lung
tumours. They are composed of tissues that are normally present in the lung, including fat, epithelial
tissue, fibrous tissue and cartilage, but they exhibit disorganised growth. Although most hamartomas
are asymptomatic and although they have no malignant potential, bronchogenic carcinoma is an
important differential diagnosis, and an accurate imaging interpretation and diagnosis is important.
Peripheral tumours are usually simply observed after the definitive diagnosis, while central tumours
might be excised. The prognosis is excellent.
SECTION 9:
RENAL PATHOLOGY — ANSWERS
9.1
The lack of findings in the bladder but the presence of atypical cells suggests that the lesion is located
higher up, possibly in the renal pelvis or ureter. Transitional cell carcinoma (TCC) of the renal pelvis
accounts for about 7% of all kidney tumours; TCC of the ureters accounts for about 4% of upper tract
tumours. Risk factors are the same as those for bladder cancer. Also, inhabitants of the Balkans with
endemic familial nephropathy are inexplicably predisposed to develop upper tract TCC.
Most patients present with haematuria; dysuria and frequency can occur if the bladder is also
involved. Colicky pain can accompany obstruction. Uncommonly, hydronephrosis results from a renal
pelvic tumour. Evaluation typically includes ultrasound or computed tomography with contrast.
Diagnosis must be confirmed by cytological or histological analysis. Ureteroscopy, nephroscopy or
both are done when biopsy of the upper tract is needed or when urine cytology is positive but no
source of the malignant cells is obvious. Staging for obviously superficial tumours is probably
unnecessary. For other tumours, abdominal and pelvic computed tomography and chest X-ray are
done to determine tumour extent and to check for metastases. The standard TNM staging system is
used.
9.2
The patient has Bence Jones protein. The dipstick is most sensitive for albumin, not globulins such as
seen in patients with multiple myeloma. Multiple myeloma is a malignancy of plasma cells that
produce monoclonal immunoglobulin and invade and destroy adjacent bone tissue. Common
manifestations include bone pain, renalinsufficiency, hypercalcaemia, anaemia and recurrent
infections. Diagnosis requires demonstration of M-protein (sometimes present in urine and not serum)
and either lytic bone lesions, light-chain proteinuria or excessive marrow plasma cells. A bone
marrow biopsy is usually needed.
The incidence of multiple myeloma is 2–4 per 100,000 population, the male to female ratio is 1.6 to
1, and most patients are aged over 40 years. The prevalence in blacks is twice that in whites. The
aetiology is unknown, although chromosomal and genetic factors, radiation, and chemicals have been
suggested.
Plasma cell tumours (plasmacytomas) produce IgG in about 55% of myeloma patients and IgA in
about 20%; of these patients, 40% also have Bence Jones proteinuria, which is free monoclonal
kappa or lamda light chains in the urine. In 15%–20% of patients, plasma cells secrete only Bence
Jones protein. These patients tend to have a higher incidence of lytic bone lesions, hypercalcaemia,
renal failure and amyloidosis than do other myeloma patients. IgD myeloma accounts for about 1% of
cases.
9.3
Squamous epithelium is normally not seen above the outer urethra. All the other structures are
present.
9.4
Nephrotic syndrome is defined by the urinary excretion of over 3 g of protein per day due to
glomerular disease. Nephrotic syndrome occurs at any age but is more prevalent in children, mostly
between the ages of 18 months and 4 years. At younger ages, boys are affected more often than girls,
but both are affected equally at older ages. Causes differ by age. The most common primary causes
are minimal-change disease, focal segmental glomerulosclerosis and membranous nephropathy.
Secondary causes account for fewer than 10% of childhood cases but over 50% of adult cases, most
commonly diabetic nephropathy and pre-eclampsia. Amyloidosis is an under-recognised cause in 4%
of cases.
Proteinuria occurs because of changes to capillary endothelial cells, the glomerular basement
membrane or podocytes, which normally filter serum protein selectively by size and charge. The
mechanism of damage to these structures is unknown in primary glomerular disease, but evidence
suggests that T cells up-regulate a circulating permeability factor or down-regulate an inhibitor of
permeability factor in response to unidentified immunogens and cytokines. The result is urinary loss
of macromolecular proteins, primarily albumin but also opsonins, immunoglobulins, erythropoietin,
transferrin, hormone-binding proteins and antithrombin III in conditions that cause non-selective
proteinuria. As a result, patients with nephrotic syndrome develop peripheral oedema, ascites and
effusions, and are at increased risk of infection (especially cellulitis and, in 2%– 6%, spontaneous
bacterial peritonitis), anaemia, abnormal thyroid function and thromboembolism (especially renal
vein thrombosis and pulmonary embolism in up to 5% of children and 40% of adults).
Thromboembolism can develop not only because of urinary loss of antithrombin III but also because
of increased hepatic synthesis of clotting factors, platelet abnormalities and hyperviscosity from
hypovolaemia.
Renal failure is rarely a presenting finding but can occur after a prolonged illness. However, patients
with nephrotic syndrome due to a secondary cause frequently have renal insufficiency at onset or soon
thereafter.
9.5
Answer: B Concentration
Renal concentrating ability is reflected by the specific gravity. In humans, normal specific gravity
values range from 1.002 g/ml to 1.028 g/ml. Increased specific gravity (ie increased concentration of
solutes in the urine) is associated with dehydration, diarrhoea, vomiting, excessive sweating,
glucosuria and the syndrome of inappropriate antiduretic hormone secretion (SIADH). Decreased
specific gravity (ie decreased concentration of solutes in urine) is associated with renal failure,
pyelonephritis, diabetes insipidus, acute tubular necrosis, interstitial nephritis and excessive fluid
intake.
9.6
The diffuse form of scleroderma can be associated with hyperplastic arteriolosclerosis and malignant
hypertension. Post-streptococcal glomerulonephritis produces glomerular hypercellularity. End-stage
kidneys all look alike: thickened arteries, globally sclerotic glomeruli and interstitial scarring with
chronic inflammation. The child is most likely to have minimal-change disease and the kidney will be
grossly normal, will be normal on light microscopy, and will show only fusion of foot processes on
electron microscopy. An acute myocardial infarction could cause decreased cardiac output with
decreased renal perfusion and ischaemia, leading to acute tubular necrosis.
9.7
Wilms’ tumour, an embryonal cancer of the kidney composed of blastemal, stromal and epithelial
elements, usually presents in children under the age of 5 years but occasionally presents in older
children and rarely in adults. Wilms’ tumour accounts for about 6% of cancers in children aged under
15 years. Bilateral synchronous tumours occur in about 4% of cases, with bilateral disease more
common in very young children, especially in girls. A chromosomal deletion of WT1, the Wilms’
tumour suppressor gene, on chromosome 11 has been identified in some cases. Other associated
genetic abnormalities include deletion of WT2 (another Wilms’ tumour suppressor gene), deletion of
chromosome 16 and duplication of chromosome 12.
About 10% of cases manifest other congenital abnormalities, especially genitourinary abnormalities,
but also commonly hemihypertrophy (asymmetry of the body). ‘WAGR syndrome’ is the combination
o f Wilms’ tumour (with the WT1 deletion), aniridia, genitourinary malformations (eg renal
hypoplasia, cystic disease, hypospadias, cryptorchidism) and mental retardation.
The most common presentation is a painless, palpable abdominal mass. Less common findings
include abdominal pain, haematuria, fever, anorexia, nausea and vomiting. Haematuria (occurring in
15%–20%) indicates invasion of the collecting system. Hypertension can occur if compression of the
renal pedicle or renal parenchyma causes ischaemia.
9.8
Minimal-change disease (MCD) is the most common cause of nephrotic syndrome in children aged 4–
8 years (80%–90% of cases of childhood nephrotic syndrome), but it also occurs in adults (20% of
cases of adult nephrotic syndrome). The cause is almost always unknown, although rarely it occurs
secondary to drug use (especially non-steroidal anti-inflammatory drugs) and haematological
malignancies (especially Hodgkin’s lymphoma).
MCD causes nephrotic syndrome without hypertension or azotaemia; microscopic haematuria occurs
in about 20% of patients. Azotaemia can occur in non-idiopathic cases and in patients aged over 60
years. Albumin is lost in the urine of patients with MCD to a greater extent than larger serum proteins,
probably because MCD causes changes in the charge barrier rather than the size barrier in the
glomerular capillary wall.
Diagnosis in children is most often clinical, but biopsy is required in atypical cases and in adults.
Electron microscopy demonstrates oedema with diffuse swelling (effacement) of foot processes of the
epithelial podocytes. Although effacement is not observed in the absence of proteinuria, heavy
proteinuria can occur with normal foot processes.
9.9
The ethylene glycol causes toxic injury to renal tubules and this is called ‘acute tubular necrosis’
(ATN). ATN is characterised by acute tubular cell injury and dysfunction and causes renal
insufficiency or failure. The most common causes of ATN are hypotension and nephrotoxins.
Common agents include aminoglycoside antibiotics, amphotericin, cisplatin and radiocontrast
(particularly at volume greater than 100 ml). Major surgery and advanced hepatobiliary disease, poor
perfusion states and advanced age all increase the risk of aminoglycoside toxicity. Less common
causes include haem pigments (myoglobin and haemoglobin), poisons (ethylene glycol) and herbal
and folk remedies (eg ingestion of fish gallbladder in Southeast Asia). Certain drug combinations (eg
aminoglycosides with amphotericin B) can be especially nephrotoxic. Toxic exposures cause patchy,
segmental, tubular luminal occlusion with casts and cellular debris or segmental tubular necrosis.
ATN is more likely to develop in those with a baseline creatinine clearance <47 ml/minute, diabetes
mellitus and pre-existing hypovolaemia or poor renal perfusion.
9.10
Therapy may depend on determination of the severity and nature of the renal disease with systemic
lupus erythematosus (SLE) and so a percutaneous needle biopsy is appropriate and helpful in this
situation. You should treat the acute infection. A urinalysis with microscopic examination and urine
culture is indicated if you suspect a urinary tract infection. Clinical examination with palpation of the
prostate can help confirm a diagnosis of prostatic hyperplasia, and an ultrasound of the prostate
would also be useful. Hydronephrosis can be assessed with an intravenous pyelogram.
Ultrasonography can help determine whether the kidneys are cystic. A family history also helps.
Renal cysts should not be biopsied. They are incidental findings
9.11
Transitional cell carcinomas can occur anywhere in the urothelium and multicentricity and recurrence
are common. There is only one prostate gland. Adenocarcinomas in the prostate can spread within the
prostate, and most arise in the posterior lobe. Renal cell carcinomas are sometimes bilateral or
multicentric, but not often. Generally, penile carcinomas are solitary (but infiltrative) mass lesions.
Wilms’ tumour usually presents as a solitary mass, though about 10% are bilateral or multicentric at
the time of diagnosis (see also Answer to 9.1).
9.12
The presentation is usually insidious, with weakness, fatigue, fever, nausea and vomiting, anorexia,
arthralgia and abdominal pain. About 50% of patients have oedema and a history of an acute
influenza-like illness within 4 weeks of the onset of renal failure, usually followed by severe
oliguria. Nephrotic syndrome is present in 10%–30%. Hypertension is uncommon and rarely severe.
Patients with anti-glomerular basement membrane antibody disease can have pulmonary haemorrhage,
which can present with haemoptysis or might be detectable only by the presence of diffuse alveolar
infiltrates on chest X-ray.
9.13
• Inorganic salts
• Malignant tumours (particularly carcinoma of the lung and colon, and melanoma).
MGN is caused by circulating immune complex. Current research indicates that the majority of the
immune complexes are formed via binding of antibodies to antigens in situ to the glomerular basement
membrane. The antigens can be endogenous to the basement membrane, or ‘planted’ from systemic
circulation.
The immune complex serves as an activator that triggers a response from the C5b–C9 complements,
which form a membrane attack complex (MAC) on the glomerular epithelial cells. This in turn
stimulates the release of proteases and oxidants by the mesangial and epithelial cells, damaging the
capillary walls and causing them to become ‘leaky’. In addition, the epithelial cells also seem to
secrete an unknown mediator that reduces nephrin synthesis and distribution.
9.14
Ischaemia, typically in hypotensive hospitalised patients, is the most frequent antecedent to acute
tubular necrosis (see also Answer to 9.9).
9.15
Answer: E White blood cell casts
The white blood cell casts are most characteristic of an acute tubulointerstitial nephritis (acute
pyelonephritis). Tubulointerstitial nephritis is a primary injury to renal tubules and interstitium that
results in decreased renal function. The acute form is most often due to allergic drug reactions or to
infections.
9.16
The features in this case are suggestive of a ureteric calculus. During passage through the ureter a
calculus irritates the ureter and can become lodged, obstructing urine flow and causing hydroureter
and sometimes hydronephrosis. Common sites for stones to become stuck include the ureteropelvic
junction, the distal ureter (at the level of the iliac vessels) and the ureterovesical junction. Typically,
a calculus must have a diameter of >5 mm to become lodged; calculi with a diameter of ≤5 mm are
likely to pass spontaneously.
Even partial obstruction causes decreased glomerular filtration, which can persist briefly after the
calculus has passed. With hydronephrosis and elevated glomerular pressure, renal blood flow
declines, which further worsens renal function. Generally, however, permanent renal dysfunction
occurs only after about 28 days of complete obstruction. Secondary infection can occur with
longstanding obstruction.
9.17
In bladder exstrophy the bladder is open suprapubically, and urine drips from the opening rather than
through the urethra. The bladder mucosa is continuous with the abdominal skin and the pubic bones
are separated. Despite the seriousness of the deformity, normal renal function is usually maintained.
The bladder can usually be reconstructed and returned to the pelvis, although vesicoureteral reflux
invariably occurs. Continent urinary diversion can be used to treat a bladder reservoir that fails to
expand sufficiently or has sphincter insufficiency. Reconstruction of the genitals is required.
9.18
Answer: A Chlamydia
Sexually transmitted urethritis, cervicitis, proctitis and pharyngitis infections that are not due to
gonorrhoea are caused predominantly by Chlamydia and infrequently by Mycoplasma or Ureaplasma
organisms. Because the organisms that cause most cases of non-gonococcal, sexually transmitted
cervicitis in women and most cases of urethritis, proctitis and pharyngitis in both sexes have been
identified, the previously used terms ‘non-specific urethritis’ and ‘non-gonococcal urethritis’ are
imprecise. Causal agents include Chlamydia trachomatis (responsible for about 50% of such cases
of urethritis and most such cases of mucopurulent cervicitis), Mycoplasma genitalium, Ureaplasma
urealyticum, and Trichomonas vaginalis.
9.19
Oval fat bodies appear with pronounced proteinuria and lipiduria, as seen in nephrotic syndrome.
9.20
Answer: A Myoglobinuria
Myoglobinuria is the presence of myoglobin in the urine, usually associated with rhabdomyolysis or
muscle destruction. Trauma, including electrical injuries and burns, vascular problems, venoms and
certain drugs can destroy or damage the muscle, releasing myoglobin into the circulation and thus to
the kidneys. Under ideal situations myoglobin will be filtered and excreted with the urine, but if too
much myoglobin is released into the circulation or where there are renal problems, it can occlude the
renal filtration system, leading to acute tubular necrosis and acute renal insufficiency.
SECTION 10:
GASTROINTESTINAL AND HEPATOBILIARY PATHOLOGY
— ANSWERS
10.1
Hepatitis A virus is a single-stranded RNA picornavirus. It is the most common cause of acute viral
hepatitis and is particularly common in children and young adults. In some countries more than 75%
of adults have been exposed. Hepatitis A virus spreads primarily by faecal–oral contact and so can
occur in areas of poor hygiene. Water-borne and food-borne epidemics occur, especially in
underdeveloped countries. Eating contaminated raw shellfish is sometimes responsible. Sporadic
cases are also common, usually as a result of person-to-person contact. Faecal shedding of the virus
occurs before symptoms develop and usually ceases a few days after symptoms begin. This means
that infectivity often has already ceased when the hepatitis becomes clinically evident. Hepatitis A
virus has no known chronic carrier state and does not lead to chronic hepatitis or cirrhosis. If a liver
biopsy is performed after the patient has completely recovered from this infection it will therefore
show normal architecture.
10.2
Mallory bodies (also known as ‘alcoholic hyaline’ or ‘Mallory’s hyaline’) are pathological
inclusions found in the cytoplasm of liver cells. They are most commonly found in the livers of
people suffering from alcoholic hepatitis. They are highly eosinophilic and appear pink on
haematoxylin and eosin staining. The bodies themselves are made up of intermediate keratin filament
proteins that have been ubiquinated, or bound by other proteins such as heat shock proteins or p62.
Mallory bodies are also seen in Wilson’s disease.
10.3
Crohn’s disease is a chronic transmural inflammatory disease that usually affects the distal ileum and
colon but which can occur in any part of the gastrointestinal tract. The most common initial
presentation is chronic diarrhoea with abdominal pain, fever, anorexia and weight loss. The abdomen
is tender, and a mass or fullness might be palpable. Gross rectal bleeding is unusual except in
isolated colonic disease, which can manifest similarly to ulcerative colitis. Some patients present
with an acute abdomen that simulates acute appendicitis or intestinal obstruction. About 33% of
patients have perianal disease (especially fissures and fistulas), which is sometimes the most
prominent or even the initial complaint. In children, extraintestinal manifestations frequently
predominate over gastrointestinal symptoms: arthritis, pyrexia of unknown origin, anaemia or growth
retardation can be the presenting symptoms, whereas abdominal pain or diarrhoea might be absent.
Crohn’s disease begins with crypt inflammation and abscesses, which progress to tiny focal aphthoid
ulcers. These mucosal lesions can develop into deep longitudinal and transverse ulcers with
intervening mucosal oedema, creating a characteristic cobblestoned appearance in the bowel.
Transmural spread of inflammation leads to lymphoedema and thickening of the bowel wall and
mesentery. Mesenteric fat typically extends onto the serosal surface of the bowel. Mesenteric lymph
nodes often enlarge. Extensive inflammation can result in hypertrophy of the muscularis mucosae,
fibrosis and stricture formation, which can lead to bowel obstruction. Abscesses are common, and
fistulas often penetrate into adjoining structures, including other loops of bowel, the bladder or psoas
muscle. Fistulas can even extend to the skin of the anterior abdomen or flanks. Independently of intra-
abdominal disease activity, perianal fistulas and abscesses occur in 25%–33% of cases; these
complications are frequently the most troublesome aspects of Crohn’s disease.
Non-caseating granulomas can occur in lymph nodes, the peritoneum, the liver and in all layers of the
bowel wall. Although pathognomonic when present, granulomas are not detected in about half of
patients with Crohn’s disease. The presence of granulomas does not seem to be related to the clinical
course.
Segments of diseased bowel are sharply demarcated from adjacent normal bowel (‘skip areas’). In
about 35% of patients with Crohn’s disease only the ileum is involved (ileitis); in around 45% the
ileum and colon are involved (ileocolitis), with a predilection for the right side of the colon; and in
about 20% only the colon is involved (granulomatous colitis), in most cases sparing the rectum
(unlike ulcerative colitis). Occasionally, the entire small bowel is involved (jejunoileitis). The
stomach, duodenum and oesophagus are rarely affected clinically, although microscopic evidence of
disease is often detectable in the gastric antrum, especially in younger patients. In the absence of
surgical intervention, the disease almost never extends into areas of small bowel that are not involved
at first diagnosis.
There is an increased risk of cancer in affected small-bowel segments. Patients with colonic
involvement have a long-term risk of colorectal cancer equal to that of patients with ulcerative
colitis, given the same extent and duration of disease.
10.4
Answer: A AFP
Alpha-fetoprotein (AFP), a glycoprotein normally synthesised by the yolk sac in the embryo and then
the fetal liver, is elevated in the newborn and therefore also in the pregnant woman. AFP decreases
rapidly during the first year of life, reaching adult values (normally <20 ng/ml) by the age of 1 year.
Marked elevations (>500 ng/ml) in a high-risk patient (for example a patient with a liver mass
detected on ultrasound) is diagnostic of primary hepatocellular carcinoma, although not all
hepatocellular carcinomas produce AFP. Because small tumours can have low levels of AFP, rising
values suggest the presence of hepatocellular carcinoma. The degree of AFP elevation, however, is
not prognostic. In populations in which chronic hepatitis B infection and hepatocellular carcinoma are
common (eg sub-Saharan Africans, ethnic Chinese), AFP can reach levels as high as 100,000 ng/ml,
whereas regions with lower frequencies of the tumour have more modest levels (about 3000 ng/ml).
A few other conditions (eg embryonic teratocarcinomas, hepatoblastomas, some hepatic metastases
from gastrointestinal tract cancers, some cholangiocarcinomas) cause levels to rise to 500 ng/ml or
above. In fulminant hepatitis, AFP can occasionally rise to 500 ng/ml; lesser elevations occur in acute
and chronic hepatitis. These levels probably reflect liver regeneration. The sensitivity and specificity
of AFP therefore vary according to population, but values of ≥20 ng/mL have a sensitivity range of
39%–64% and a specificity range of 76%–91%. Because values of ≤500 ng/ml are non-specific, 500
ng/ml has been suggested as the diagnostic cut-off level.
10.5
Answer: D Leucocytosis
The clinical features in this case are typical of acute appendicitis. Appendicitis is thought to result
from obstruction of the appendiceal lumen, typically by lymphoid hyperplasia, but occasionally by a
faecolith, foreign body or even worms. The obstruction leads to distension, bacterial overgrowth,
ischaemia and inflammation. If untreated, necrosis, gangrene and perforation occur. If the perforation
is contained by the omentum, an appendiceal abscess results.
Over 5% of the population develop appendicitis at some point. It is most common in the teens and
twenties but can occur at any age. The classic symptoms of acute appendicitis are epigastric or
periumbilical pain, followed by brief nausea, vomiting and anorexia; after a few hours the pain shifts
to the right lower quadrant. Pain increases with cough and movement. The classic signs are right
lower quadrant direct and rebound tenderness located at McBurney’s point (the junction of the middle
and outer thirds of the line joining the umbilicus to the anterior superior spine). Additional signs are
pain felt in the right lower quadrant with palpation of the left lower quadrant (Rovsing’s sign), an
increase in pain on passive extension of the right hip joint (this stretches the iliopsoas muscle and the
sign is known as the ‘psoas sign’) and pain on passive internal rotation of the flexed thigh (obturator
sign). Low-grade fever (rectal temperature 37.7 °C to 38.3 °C) is common.
Unfortunately, these classic findings appear in fewer than 50% of patients. Many variations in
symptoms and signs occur. Pain may not be localised, particularly in infants and children. Tenderness
can be diffuse or, in rare instances, absent. Bowel movements are usually less frequent or absent; if
diarrhoea is a sign, a retrocaecal appendix should be suspected. Red blood cells or white blood cells
might be found in the urine. Atypical symptoms are common in elderly patients and in pregnant
women; in particular, pain is less severe and local tenderness is less marked.
When classic signs and symptoms are present, the diagnosis is clinical. In such patients, delaying
laparotomy in order to perform imaging tests only increases the likelihood of perforation and
subsequent complications. In patients with atypical or equivocal findings, imaging studies should be
performed without delay. Contrast-enhanced computed tomography is reasonably accurate for
diagnosing appendicitis and can also reveal other causes of an acute abdomen. Graded compression
ultrasound can usually be obtained more quickly than computed tomography but is occasionally
limited by the presence of bowel gas and is less useful for recognising non-appendiceal causes of
pain. The use of these studies has reduced the rate of negative laparotomy.
Laparoscopy can be used for diagnosis; it can be especially helpful in women with lower abdominal
pain of unclear aetiology. Laboratory studies typically show leucocytosis (12–15 × 109/l), but this
finding is highly variable; a normal white blood cell count should not be used to exclude appendicitis.
10.6
10.7
Patients most commonly present early in life: 15% in the first month, 60% by the age of 1 year, and
85% by the age of 4 years. The infant presents with obstipation, abdominal distension and finally
vomiting, as in other forms of distal bowel obstruction. Occasionally, infants with ultra-short segment
aganglionosis have only mild or intermittent constipation, often with intervening bouts of mild
diarrhoea, resulting in a delay in making the diagnosis. In older infants, symptoms and signs can
include anorexia, lack of a physiological urge to defecate and, on examination, an empty rectum with
stool palpable higher up in the colon. The child might also fail to thrive. A diagnosis should be made
as soon as possible. The longer the disease goes untreated, the greater the chance of developing
Hirschsprung’s enterocolitis (toxic megacolon), which can be fulminant and fatal. Most cases can be
diagnosed in early infancy.
The initial approach is typically with a barium enema or sometimes a rectal suction biopsy. Barium
enema can show a transition in diameter between the dilated, normally innervated colon proximal to
the narrowed distal segment (which lacks normal innervation). Barium enema should be done without
prior preparation, which can dilate the abnormal segment, rendering the test non-diagnostic. Because
characteristic findings might not be present in the neonatal period, a 24-hour post-evacuation film
should be obtained: if the colon is still filled with barium, Hirschsprung’s disease is likely. A rectal
suction biopsy can reveal the absence of ganglion cells. Acetylcholinesterase staining can be
performed to highlight the enlarged nerve trunks. Some centres also have the facility to perform rectal
manometrics, which can reveal abnormal innervation. Definitive diagnosis requires a full-thickness
biopsy of the rectum.
10.8
Answer: B Haemolysis
10.9
Hepatocellular carcinoma (hepatoma) usually occurs in patients with cirrhosis and is common in
areas where infection with hepatitis B and hepatitis C viruses is prevalent. Symptoms and signs are
usually non-specific. The diagnosis is based on alpha-fetoprotein (AFP) levels, imaging tests, and
sometimes liver biopsy. Screening with periodic AFP measurement and ultrasound is sometimes
recommended for high-risk patients. The prognosis is grim, but small localised tumours can
sometimes be cured by surgical resection or liver transplantation (see also Answer to 10.4).
10.10
Answer: D Pseudopolyps
The presence of sclerosing cholangitis (an extraintestinal manifestation) along with the bloody
diarrhoea in this scenario is indicative of ulcerative colitis, which affects the mucosa and submucosa
of the rectum and colon, with a sharp border between normal and affected tissue. Only in severe
disease is the muscularis layer involved. In early cases, the mucous membrane is erythematous, finely
granular and friable, with loss of the normal vascular pattern and often with scattered haemorrhagic
areas. Large mucosal ulcers with copious purulent exudate are characteristic of severe disease.
Islands of relatively normal or hyperplastic inflammatory mucosa (pseudopolyps) project above
areas of ulcerated mucosa. Fistulas and abscesses do not occur.
10.11
10.12
Answer: A Aflatoxin
Aflatoxins are naturally occurring mycotoxins that are produced by many species of the Aspergillus
fungus, most notably A. flavus and A. parasiticus. Aflatoxins are toxic and carcinogenic. After
entering the body, aflatoxins are metabolised by the liver to a reactive intermediate, aflatoxin M1’ an
epoxide. Aflatoxin-producing Aspergillus organisms are common and widespread in nature. They can
colonise and contaminate grain before harvest or during storage. Host crops are particularly
susceptible to infection by Aspergillus after prolonged exposure to a high-humidity environment or
after damage resulting from stressful conditions such as drought. The native habitat of Aspergillus is
soil, decaying vegetation, hay and grains undergoing microbiological deterioration, and it invades all
types of organic substrates, whenever conditions are favourable for its growth. Favourable conditions
include high moisture content (at least 7%) and high temperature.
High-level aflatoxin exposure causes acute necrosis, cirrhosis and carcinoma of the liver. There can
be haemorrhage, acute liver damage, oedema, and alteration in digestion and in absorption and/or
metabolism of nutrients. No animal species is immune to the acute toxic effects of aflatoxins,
including humans. However, humans have an extraordinarily high tolerance for aflatoxin exposure and
rarely succumb to acute aflatoxicosis. Chronic, subclinical exposure does not lead to symptoms that
are as dramatic as the symptoms of acute aflatoxicosis. Children, however, are particularly affected
by aflatoxin exposure, which leads to stunted growth and delayed development. Chronic exposure
also carries a high risk of developing liver cancer, as the metabolite aflatoxin M1 can intercalate into
DNA and alkylate the bases through its epoxide moiety.
10.13
Answer: E Pancreatic carcinoma
In this scenario the presence of jaundice accompanied by pale stools, weight loss and a raised
alkaline phosphatase suggests obstructive jaundice, most probably due to pancreatic carcinoma.
10.14
Primary biliary cirrhosis is an autoimmune liver disease that is characterised by the progressive
destruction of intrahepatic bile ducts, leading to cholestasis, cirrhosis and liver failure. Patients are
usually asymptomatic at presentation but can experience fatigue, symptoms of cholestasis (eg pruritus,
steatorrhoea), or symptoms of cirrhosis (eg portal hypertension, ascites). Laboratory tests reveal
cholestasis, increased IgM and, characteristically, antimitochondrial antibodies in the serum. Liver
biopsy is usually required for diagnosis and staging.
10.15
10.16
Duodenal ulcers tend to be associated with a fairly consistent pattern of pain. Pain is absent when the
patient awakens but appears mid-morning, is relieved by food, but recurs 2–3 hours after a meal. Pain
that awakens a patient at night is common and is highly suggestive of duodenal ulcer.
10.17
These findings point to scleroderma, probably CREST syndrome, in which the ‘e’ represents
(o)esophageal dysmotility. Scleroderma varies in severity and progression, ranging from generalised
skin thickening with rapidly progressive and often fatal visceral involvement to isolated skin
involvement (often just the fingers and face) and slow progression (often several decades) before the
development of serious visceral disease. The latter form is known as ‘limited cutaneous scleroderma’
or ‘CREST syndrome’ (calcinosis cutis, Raynaud’s phenomenon, (o)esophageal dysmotility,
sclerodactyly, and telangiectasia).
10.18
Cirrhosis is fibrosis that progresses to produce diffuse disorganisation of normal hepatic structure,
characterised by regenerative nodules surrounded by dense fibrotic tissue. Symptoms might not
develop for years and are often non-specific, for example anorexia, fatigue and weight loss. Late
manifestations include portal hypertension, ascites and liver failure.
10.19
Answer: E Ulceration of mucosa by ectopic gastric tissue
A Meckel’s diverticulum is present at birth, unlike colonic diverticula, which are acquired in
adulthood. There can be ectopic tissue in the Meckel’s diverticulum, most likely pancreatic tissue, but
sometimes gastric mucosa, which can cause ulceration and bleeding, as is the case in this scenario. In
early fetal life the vitelline duct that runs from the terminal ileum to the umbilicus and yolk sac is
normally obliterated by the seventh week. If the portion connecting to the ileum fails to atrophy, a
Meckel’s diverticulum results. This congenital diverticulum arises from the antimesenteric margin of
the intestine and contains all layers of the normal bowel. About 50% of diverticula also contain
heterotopic tissue of the stomach (and so contain parietal cells that secrete hydrochloric acid),
pancreas or both.
Only about 2% of people with Meckel’s diverticulum develop complications. Although diverticula
are equally common in males and females, males are two to three times more likely to have
complications. Complications include bleeding, obstruction, diverticulitis and tumours. Bleeding is
more common in young children (<5 years) and occurs when acid secreted from ectopic gastric
mucosa in the diverticulum ulcerates the adjacent ileum. Obstruction can occur at any age but is more
common in older children and adults. In children, intussusception of the diverticulum is the most
likely cause. Obstruction can also result from adhesions, volvulus, retained foreign bodies, tumours
or incarceration in a hernia (Littre’s hernia). Acute Meckel’s diverticulitis can occur at any age, but
its incidence peaks in older children. Tumours, including carcinoids, are rare and occur mainly in
adults.
10.20
The findings point to an acute bowel obstruction. The abdominal scar indicates prior surgery and
suggests the development of peritoneal adhesions that have predisposed to obstruction. Hepatitis can
cause vague abdominal pain, and there can be distension from ascites with liver dysfunction, but not
dilated loops of bowel. Amoebiasis can cause an inflammatory bowel disease that mainly affects the
colon, with diarrhoea that can be positive for occult blood. Primary adenocarcinoma of the ileum is
rare. A Meckel’s diverticulum is a small focal lesion that can cause obstruction, but the clinical
features and radiological findings are more consistent with obstruction due to adhesions from
previous surgery.
SECTION 11:
HAEMATOPATHOLOGY — ANSWERS
11.1
This is the appearance of a benign, reactive lymph node. Lymphadenopathy is common in children.
11.2
Acute myeloid leukaemia (AML), also known as ‘acute myelogenous leukaemia’, is a cancer of the
myeloid line of white blood cells, characterised by the rapid proliferation of abnormal cells which
accumulate in the bone marrow and interfere with the production of normal blood cells. AML is the
most common acute leukaemia in adults, and its incidence increases with age. While AML is a
relatively rare disease overall, its incidence is expected to increase as the population ages.
The symptoms of AML are caused by replacement of normal bone marrow with leukaemic cells,
resulting in a drop in red blood cells, platelets and normal white blood cells. These symptoms
include fatigue, shortness of breath, easy bruising and bleeding and increased risk of infection. While
a number of risk factors for AML have been elucidated, the specific cause of AML remains unclear.
AML progresses rapidly and is typically fatal in weeks to months if left untreated.
Auer rods can be seen in the leukaemic blasts of acute myeloid leukaemia. Auer rods are clumps of
azurophilic granular material that form elongated needles seen in the cytoplasm of leukaemic blasts.
They are composed of fused lysosomes and contain peroxidase, lysosomal enzymes and large
crystalline inclusions. Auer rods are classically seen in myeloid blasts of M1, M2, M3 and M4 acute
leukaemias. They are also used to distinguish the preleukaemia myelodysplastic syndromes.
11.3
The most common type of leukaemia in the Western world, chronic lymphocytic leukaemia (CLL)
involves mature-appearing but defective neoplastic lymphocytes with an abnormally long life span.
The peripheral blood, bone marrow, spleen and lymph nodes undergo leukaemic infiltration. The
incidence of CLL increases with age: 75% of cases are diagnosed in patients aged over 60 years.
CLL is twice as common in men. Although the cause is unknown, some cases are familial. CLL is rare
in Japan and China and does not seem to increase among Japanese expatriates in the United States,
suggesting a genetic factor. CLL is more common among Jews of Eastern European descent.
In about 98% of cases, CD5+ B cells undergo malignant transformation, with lymphocytes initially
accumulating in the bone marrow and then spreading to lymph nodes and other lymphoid tissues,
eventually inducing splenomegaly and hepatomegaly. As the disease progresses, abnormal
haematopoiesis results in anaemia, neutropenia, thrombocytopenia and decreased immunoglobulin
production. Many patients develop hypogammaglobulinaemia and impaired antibody response,
perhaps related to increased T-suppressor cell activity. Patients have increased susceptibility to
autoimmune disease, characterized by immunohaemolytic anaemias (usually Coombs’ test-positive)
or thrombocytopenia, and a modest increase in the risk of developing other cancers.
In 2%–3% of cases, the clonal expansion is T-cell in type, and even this group has a subtype (eg large
granular lymphocytes with cytopenias). In addition, other chronic leukaemic patterns have been
categorised under CLL: prolymphocytic leukaemia, the leukaemic phase of cutaneous T-cell
lymphoma (Sézary syndrome), hairy cell leukaemia and lymphoma/leukaemia (ie the leukaemic
changes seen in the advanced stages of malignant lymphoma). Differentiation of these subtypes from
typical CLL is usually straightforward.
The onset is usually insidious and CLL is often diagnosed incidentally during routine blood tests or
on investigation of asymptomatic lymphadenopathy. The symptomatic patient usually has non-specific
complaints of fatigue, anorexia, weight loss, dyspnoea on exertion, or a sense of abdominal fullness
(secondary to an enlarged spleen). Initial findings include generalised lymphadenopathy and minimal-
to-moderate hepatomegaly and splenomegaly. With progressive disease, there might be pallor due to
anaemia. Skin infiltration, either maculopapular or diffuse, can be a feature of T-cell CLL.
Hypogammaglobulinaemia and granulocytopenia in late CLL can predispose to bacterial, viral and
fungal infection, especially pneumonia. Herpes zoster is common and usually dermatomic.
CLL is confirmed by examining the peripheral blood smear and bone marrow; the hallmark is a
sustained, absolute peripheral lymphocytosis (>5 ×109/l) and an increased proportion of lymphocytes
in the bone marrow (>30%). The differential diagnosis is simplified by immunophenotyping. Other
findings at diagnosis can include hypogammaglobulinaemia (<15% of cases) and, rarely, an elevated
lactate dehydrogenase (LDH). Only 10% of patients present with moderate anaemia (sometimes
immunohaemolytic) and/or thrombocytopenia. A monoclonal serum immunoglobulin spike of the same
type might be found on the leukaemic cell surface in 2%–4% of cases.
11.4
The markedly elevated mean corpuscular volume (MCV) suggests a megaloblastic anaemia, and the
hypersegmented neutrophils are consistent with this. Either a B12 or a folate deficiency is the likely
cause. Megaloblastic states result from defective DNA synthesis. RNA synthesis continues, resulting
in a large cell with a large nucleus. All cell lines have dyspoiesis, in which cytoplasmic maturity is
greater than nuclear maturity. This produces megaloblasts in the marrow before they appear in the
peripheral blood. Dyspoiesis results in intramedullary cell death, making erythropoiesis ineffective
and causing indirect hyperbilirubinaemia and hyperuricaemia. Because dyspoiesis affects all cell
lines, reticulocytopenia and, during later stages, leukopenia and thrombocytopenia develop. Large,
oval red blood cells (macro-ovalocytes) enter the circulation. Hypersegmentation of
polymorphonuclear neutrophils is common; the mechanism of their production is unknown.
11.5
Epstein–Barr virus infection, causing infectious mononucleosis often leads to the appearance of
atypical lymphocytes in the peripheral blood. The symptoms of infectious mononucleosis develop
most often in older children and adults. The incubation period is about 30–50 days. Usually, fatigue
develops initially, lasting several days to a week or longer, followed by fever, pharyngitis and
adenopathy. However, not all of these symptoms will appear. Fatigue can last months, but is usually
maximal in the first 2–3 weeks. Fever usually peaks in the afternoon or early evening, with a
temperature around 39.5 °C, although it can reach 40.5 °C. When fatigue and fever predominate (the
so-called ‘typhoidal form’), the onset and resolution can be much slower. The pharyngitis can be
severe, painful and exudative, and can resemble streptococcal pharyngitis. Lymphadenopathy is
usually symmetric and can involve any group of nodes, particularly the anterior and posterior cervical
chains. Adenopathy is sometimes the only manifestation.
Splenomegaly, which occurs in about 50% of cases, is maximal during the second and third weeks
and is usually barely palpable. Mild hepatomegaly and hepatic percussion tenderness can occur. Less
frequent findings include maculopapular eruptions, jaundice, periorbital oedema and palatal
enanthema.
Laboratory diagnosis usually involves a full blood count and a heterophil antibody test. Lymphocytes
that are morphologically atypical account for up to 80% of the white blood cells. Although individual
lymphocytes can resemble leukaemic lymphocytes, the lymphocytes are heterogeneous, which is
unlikely with leukaemia.
11.6
This patient has a leucoerythroblastic picture in the peripheral blood. At this age, prostatic
adenocarcinoma or lung cancer are the likely primaries.
11.7
‘Autosplenectomy’ is the rule with sickle cell anaemia by late childhood or teenage years. Vaso-
occlusive crises are caused by sickle-shaped red blood cells that obstruct capillaries and restrict
blood flow to an organ, resulting in ischaemia, pain and organ damage. Because of its narrow vessels
and its function in clearing defective red blood cells, the spleen is frequently affected. It is usually
infarcted before the end of childhood in individuals suffering from sickle cell anaemia. This
autosplenectomy increases the risk of infection with encapsulated organisms, and so prophylactic
antibiotics and vaccinations are recommended for people with this type of asplenia.
11.8
There is a markedly increased haematocrit. It is possible that this is secondary, but the
thrombocytosis and the leucocytosis suggest a myeloproliferative disorder. Polycythaemia vera is the
most common of the myeloproliferative disorders, occurring in about 5 in 1 million people, more
often in males (about 1.4 to 1). The mean age at diagnosis is 60 years (range 15–90 years, rare in
childhood); 5% of patients are aged under 40 years at onset.
Polycythaemia vera involves increased production of all cell lines, including red blood cells, white
blood cells and platelets. Increased production confined to the red blood cell line is known as
‘primary erythrocytosis’. In people with polycythaemia vera red blood cell production proceeds
independently of erythropoietin. Extramedullary haematopoiesis occurs in the spleen, liver and other
sites with the potential for blood cell formation. Peripheral blood cell turnover increases. Eventually,
about 25% of patients have reduced red blood cell survival and inadequate erythropoiesis. Anaemia,
thrombocytopenia and myelofibrosis can develop, and red blood cell and white blood cell precursors
are released into the circulation. Depending on treatment received, the incidence of transformation to
acute leukaemia varies between 1.5% and 10%. In polycythaemia vera the blood volume expands and
hyperviscosity develops, predisposing to thrombosis. Platelets function abnormally, predisposing to
increased bleeding. Patients can become hypermetabolic, and increased cell turnover leads to
hyperuricaemia.
Polycythaemia vera itself is often asymptomatic. Occasionally, increased blood volume and viscosity
cause weakness, headache, light-headedness, visual disturbances, fatigue and dyspnoea. Pruritus is
common, particularly after a hot bath. The face may be red and the retinal veins engorged. The lower
extremities can be red, warm and painful, sometimes with digital ischaemia (erythromelalgia).
Hepatomegaly is common and more than 75% of patients have splenomegaly, which may be massive.
Thrombosis can occur in most vessels, resulting in stroke, transient ischaemic attacks, deep venous
thrombosis, myocardial infarction, retinal artery or vein occlusion, splenic infarction (often with a
friction rub) or Budd–Chiari syndrome.
11.9
Answer: C Hodgkin’s lymphoma
11.10
Leukaemoid reactions are generally benign and are not inherently dangerous, although they are often a
response to a significant disease state. However, leukaemoid reactions can resemble more serious
conditions such as chronic myelogenous leukaemia (CML), which can present with identical findings
on peripheral blood smear. Historically, various clues, including the LAP score and the presence of
basophilia were used to distinguish CML from a leukaemoid reaction. However, at present the test of
choice in adults to distinguish CML from a leukaemoid reaction is an assay for the presence of the
Philadelphia chromosome, either using cytogenetics and fluorescent in-situ hybridisation or using a
polymerase chain reaction for the Bcr/abl fusion protein. The LAP score is high in reactive states but
is low in CML.
• Haemorrhage
• Drugs:
glucorticoids
11.11
The serum iron, iron-binding capacity and saturation are all low, but there is plenty of storage iron
that is not being utilised. This is typical of anaemia of chronic disease. In haemolytic anaemia the
haemolysed cells release haemoglobin that is usually not lost from the body and is recycled, so the
iron is not usually low. Chronic blood loss can lead to iron deficiency because of the loss of iron,
with a low serum iron, high total iron-binding capacity (TIBC) and low ferritin. Malabsorption,
particularly in disease processes involving the duodenum, can occasionally explain iron deficiency
anaemia, which has a high TIBC and low ferritin. Vitamin B12 and folate deficiencies lead to a
megaloblastic anaemia that is not associated with abnormalities in iron metabolism.
11.12
11.13
Answer: C Myelofibrosis
• Lymphomas
• Polycythaemia vera
• Gaucher’s disease
• Malaria.
11.14
The hyaline thrombi are typical of thrombotic thrmobocytopenic purpura (TTP). Platelet transfusion
is contraindicated. TTP is caused by non-immunological platelet destruction. Loose strands of fibrin
are deposited in multiple small vessels, which damage passing platelets and red blood cells. Platelets
are also destroyed within the multiple small thrombi. Multiple organs develop bland platelet–fibrin
thrombi (without the granulocytic infiltration of the vessel wall that is characteristic of vasculitis) that
are localised primarily to arteriocapillary junctions, described as ‘thrombotic microangiopathy’.
Treatment is by plasma exchange.
11.15
Answer: A β-Thalassaemia
Thalassaemias are suspected in patients with a family history, suggestive symptoms or signs or
microcytic haemolytic anaemia. If thalassaemias are suspected, laboratory tests for microcytic and
haemolytic anaemias and quantitative haemoglobin studies are performed. Serum bilirubin, iron and
ferritin levels are increased.
In β-thalassaemia major the anaemia is severe, often with haemoglobin levels of 6 g/dl or less. The
red blood cell count is elevated relative to the haemoglobin because the cells are very microcytic.
The blood smear is virtually diagnostic, with many nucleated erythroblasts, target cells, small pale
red blood cells, and punctate and diffuse basophilia.
11.16
11.17
The features are suggestive of multiple myeloma. Multiple myeloma is a malignancy of plasma cells
that produce monoclonal immunoglobulin and invade and destroy adjacent bone tissue. Common
manifestations include bone pain, renal insufficiency, hypercalcaemia, anaemia and recurrent
infections. Diagnosis requires the demonstration of M-protein (sometimes present in urine and not
serum) and either lytic bone lesions, light-chain proteinuria or excessive marrow plasma cells.
11.18
Idiopathic thrombocytopenic purpura (ITP) usually results from the development of an antibody
directed against a structural platelet antigen (ie an autoantibody). In childhood ITP the autoantibody
can be triggered by binding of viral antigen to megakaryocytes. The symptoms and signs are petechiae
and mucosal bleeding. Although in ITP there are circulating antibodies to platelets that lead to
platelet destruction, the spleen itself is usually not enlarged unless it is enlarged as a result of a
coexistent childhood viral infection.
ITP is suspected in patients with unexplained thrombocytopenia. The peripheral blood is normal
except for reduced platelet numbers. Bone marrow is examined if blood counts or a blood smear
reveal abnormalities in addition to the thrombocytopenia. Bone marrow examination reveals normal
or possibly increased numbers of megakaryocytes in an otherwise normal marrow. Because
diagnostic findings are non-specific, diagnosis of this condition requires exclusion of other
thrombocytopenic disorders suggested by clinical or laboratory test data.
11.19
11.20
Mycosis fungoides is an uncommon, chronic T-cell lymphoma primarily affecting the skin and
occasionally the internal organs. It is rarer than Hodgkin’s lymphoma and non-Hodgkin’s lymphoma.
Unlike most other lymphomas, it is insidious in onset, sometimes appearing as a chronic, pruritic rash
that is difficult to diagnose. It begins focally but can spread to involve most of the skin. Lesions are
plaque-like but can become nodular or ulcerated. Eventually, systemic involvement of lymph nodes,
liver, spleen and lungs occurs, resulting in the advent of symptoms, which include fever, night sweats
and unintentional weight loss. Diagnosis is based on skin biopsy, but the histology can be equivocal
early in the course of the disease because of insufficient quantities of lymphoma cells. The malignant
cells are mature T cells (T4+, T11+, T12+). Characteristic Pautrier’s microabscesses are present in
the epidermis. In some cases, a leukaemic phase known as ‘Sézary syndrome’ occurs, characterised
by the appearance of malignant T cells with serpentine nuclei in the peripheral blood.
Once mycosis fungoides has been confirmed, the stage is determined by computed tomography of the
torso and bone marrow biopsy for blood or lymph node involvement. Positron emission tomography
(PET) can also be used for investigation of suspected visceral involvement. Most patients are aged
over 50 years at diagnosis, and the average life expectancy is 7–10 years after diagnosis, even
without treatment. Survival rates depend on the stage at diagnosis.
SECTION 12:
ENDOCRINE PATHOLOGY — ANSWERS
12.1
Answer: D Calcitonin
Patients typically present with an asymptomatic thyroid nodule, although in many cases the tumour is
now diagnosed during routine screening of affected relatives with MEN IIA or MEN IIB before a
palpable tumour develops. Medullary carcinoma can have a dramatic biochemical presentation when
associated with ectopic production of other hormones or peptides (eg, adrenocorticotrophic hormone,
vasoactive intestinal polypeptide, prostaglandins, kallikreins, serotonin). Metastases spread via the
lymphatic system to cervical and mediastinal nodes and sometimes to liver, lungs and bone. The best
test is the serum calcitonin, which is greatly elevated. A challenge with calcium (15 mg/kg
intravenously over 4 hours) provokes excessive secretion of calcitonin. X-rays may show a dense,
homogenous, conglomerate calcification.
12.2
Papillary carcinoma accounts for 70%–80% of all thyroid cancers. The female to male ratio is 3 : 1,
and is familial in up to 5% of patients. Most patients present between the ages of 30 years and 60
years. The tumour is often more aggressive in elderly patients. It is also the predominant cancer type
in children with thyroid cancer, and in patients with thyroid cancer who have had previous radiation
to the head and neck (in this group the cancer tends to be multifocal, with early lymphatic spread and
a relatively poor prognosis). Thyroglobulin can be used as a tumour marker for well-differentiated
papillary thyroid cancer. Many papillary carcinomas contain follicular elements. The tumour spreads
via lymphatics to regional lymph nodes in a third of patients and can metastasise to the lungs. Patients
aged under 45 years with small tumours that are confined to the thyroid have an excellent prognosis.
• Characteristic ‘Orphan Annie’ eye nuclear inclusions and psammoma bodies on light microscopy.
• Multifocality is common.
• The so-called ‘lateral aberrant thyroid’ is actually a lymph node metastasis from papillary thyroid
carcinoma.
12.3
Answer: D Phaeochromocytoma
• Approximately 10% of tumours are located in chromaffin tissue outside the adrenal gland.
Up to 25% of phaeochromocytomas are be familial. Mutations of the genes VHL, RET, NF1, SDHB
and SDHD are all known to cause familial phaeochromocytoma/extra-adrenal paraganglioma. The
clinical features of a phaeochromocytoma are those of sympathetic nervous system hyperactivity,
which can generally be described as those of ‘impending doom’, including:
• Tachycardia
• Palpitations
• Anxiety, often resembling that of a panic attack
• Excessive sweating
• Headaches
• Pallor
• A phaeochromocytoma can also cause resistant arterial hypertension, and can be fatal if it causes
malignant hypertension.
12.4
A glucagonoma is a rare tumour of the alpha cells of the pancreas that results in up to a 1000-fold
overproduction of the hormone glucagon. Alpha cell tumours are commonly associated with
glucagonoma syndrome, though similar symptoms are present in cases of pseudoglucagonoma
syndrome in the absence of a glucagon-secreting tumour. The primary physiological effect of
glucagonoma is an overproduction of the peptide hormone glucagon, which enhances blood glucose
levels through the activation of gluconeogenesis and lipolysis. Gluconeogenesis produces glucose
from protein and amino acids. The net result is hyperglucagonaemia, decreased blood levels of amino
acids (hypoaminoacidaemia), anaemia, diarrhoea, and weight loss of 5–15 kg.
Necrolytic migratory erythema (NME) is a classic symptom observed in patients with glucagonoma
and is present in 80% of cases. Associated NME is characterised by the spread of erythematous
blisters and swelling across areas subject to greater friction and pressure, including the lower
abdomen, buttocks, perineum and groin. Diabetes mellitus also frequently results from the insulin and
glucagon imbalance that occurs in glucagonoma. Diabetes mellitus is present in 80%–90% of cases of
glucagonoma, and is exacerbated by pre-existing insulin resistance.
A plasma glucagon concentration of 1000 pg/ml or greater is indicative of glucagonoma (the normal
range is 50–200 pg/ml). Blood tests can also reveal abnormally low concentrations of amino acids,
zinc and essential fatty acids, which are thought to play a role in the development of NME. Skin
biopsies can also be taken to confirm the presence of NME. A full blood count can uncover anaemia.
The tumour itself can be located by any number of radiographic modalities, including angiography,
computed tomography, magnetic resonanace imaging, positron emission tomography and endoscopic
ultrasound. Laparotomy/laparoscopy is useful for obtaining histological samples for analysis and
confirmation of the glucagonoma.
Heightened glucagon secretion can be treated with octreotide, a somatostatin analogue, which inhibits
the release of glucagon. Doxorubicin and streptozotocin have also been used successfully to
selectively damage alpha cells of the pancreatic islets. These do not destroy the tumour, but help to
minimise progression of symptoms. The only curative therapy for glucagonoma, however, is surgical
resection, which has been known to reverse symptoms in some patients.
Fewer than 250 cases of glucagonoma have been described in the literature since their first
description by Becker in 1942. Because of its rarity (fewer than one in 20 million worldwide), long-
term survival rates are as yet unknown.
12.5
Answer: B Hyperthyroidism
Hyperthyroidism is a clinical syndrome caused by an excess of circulating free thyroxine (T4) or free
triiodothyronine (T3) or both. Major causes in humans are:
• Graves’ disease (the most common cause, responsible for 70%–80% cases)
Excess thyroid hormone intake can also cause hyperthyroidism. Amiodarone, an antiarrhythmic
medication, can sometimes cause hyperthyroidism. Other causes of hyperthyroxinaemia (high blood
levels of thyroid hormones) are not to be confused with true hyperthyroidism and include subacute
and other forms of thyroiditis (inflammation). Thyrotoxicosis (the symptoms caused by
hyperthyroxinemia) can occur in both hyperthyroidism and thyroiditis. When the metabolic rate is
acutely raised this is sometimes known as a ‘thyroid storm’.
The major clinical features include weight loss (often accompanied by a ravenous appetite),
intolerance to heat, fatigue, weakness, hyperactivity, irritability, apathy, depression, polyuria and
sweating. Patients can present with a variety of symptoms, including palpitations and arrhythmias
(notably atrial fibrillation), shortness of breath, loss of libido, nausea, vomiting and diarrhoea. In the
elderly these classic symptoms are sometimes absent and they can present with fatigue and weight
loss alone, leading to ‘apathetic hyperthyroidism’. Neurological manifestations include tremor,
chorea, myopathy and periodic paralysis. Stroke of cardioembolic origin due to coexisting atrial
fibrillation is one of the most serious complications of hyperthyroidism.
12.6
Answer: A Hyperprolactinaemia
Hyperprolactinaemia is the presence of abnormally high levels of prolactin in the blood. Normal
levels are less than 580 mIU/l for women and less than 450 mIU/l for men. The hormone prolactin is
down-regulated by dopamine and is up-regulated by oestrogen. A falsely high measurement can occur
due to the presence of the biologically inactive macroprolactin in the serum. This can show up as a
high prolactin in some types of tests, but is asymptomatic.
Hyperprolactinaemia can be caused either by disinhibition (eg compression of the pituitary stalk or
reduced dopamine levels) or excess production from a prolactinoma (a pituitary gland adenoma). A
prolactin level of 1000–5000 mIU/l could occur with either mechanism, but a level of over 5000
mIU/l is likely to be due to an adenoma, with macroadenomas (large tumours over 10 mm diameter)
being associated with levels of up to 100,000 mIU/l. Hyperprolactinaemia inhibits gonadotrophin-
releasing hormone (GnRH) by increasing the release of dopamine from the arcuate nucleus of the
hypothalamus (dopamine inhibits GnRH secretion), thus inhibiting gonadal steroidogenesis, which is
the cause of many of the symptoms associated with hyperprolactinaemia.
In women, a high blood level of prolactin often causes hypooestrogenism, with anovulatory infertility
and a decrease in menstruation. In some women, menstruation can disappear altogether
(amenorrhoea). In others, menstruation can become irregular or the menstrual flow can change.
Women who are not pregnant or nursing might begin to produce breast milk. Some women experience
loss of libido, and intercourse can become painful because of vaginal dryness. In addition, the hypo-
oestrogenism associated with the hyperprolactinaemia can lead to osteoporosis.
In men, the most common symptoms of prolactinoma are impotence, decreased libido, erectile
dysfunction and infertility. Because men have no reliable indicator such as menstruation to signal a
problem, many men with hyperprolactinoma caused by an adenoma delay going to the doctor until
they have headaches or eye problems caused by the enlarged pituitary pressing against the optic
chiasma. They may not recognise a gradual loss of sexual function or libido.
12.7
Adrenal adenomas are common (one in 10 people have them), benign and asymptomatic. They are
often found as an incidental finding on computed tomography of the abdomen (an ‘incidentaloma’).
Only about one in 10,000 are malignant (adrenal adenocarcinoma). A biopsy is rarely called for,
especially if the lesion is homogeneous and smaller than 3 cm in diameter. Follow-up images taken
after 3–6 months can confirm their stability in terms of size. While some adrenal adenomas do not
secrete hormones at all (ie non-functional adenomas, often incidentalomas), some secrete cortisol
(causing Cushing syndrome), aldosterone (causing Conn syndrome) or androgens (causing
hyperandrogenism).
This adrenal gland is an unlikely site for a traumatic lesion such as a haematoma. Granulomatous
involvement in Histoplasma capsulatum infection is typically bilateral and the granulomas are
multiple. Metastases are often multiple and/or larger than 1 cm and the patient will have a primary
lesion in the lung. Simple cysts can occur anywhere, but this would be an unusual location.
12.8
Measuring aldosterone alone is not considered adequate to make a diagnosis of Conn syndrome.
Rather, both renin and aldosterone are measured, and it is the ratio of these that is diagnostic. Usually,
renin levels are suppressed, leading to a very low renin to aldosterone ratio (<0.05). This test is
confounded by antihypertensive drugs, which have to be stopped for up to 6 weeks. If there is
biochemical proof of hyperaldosteronism, computed tomographic scanning can confirm the presence
of an adrenal adenoma.
Congenital adrenal hyperplasia presents in the paediatric age group and the most common variety, 21-
OH deficiency, is associated with salt wasting. Cushing’s syndrome does not explain the
hypokalaemia and normal blood glucose. Nelson’s syndrome is increased adrenocorticotrophic
hormone (ACTH) secretion from a pituitary adenoma, which does not fit with the results of the
laboratory investigations. A phaeochromocytoma could be part of a MEN IIA or MEN IIB syndrome
and asymptomatic, but many episodically secrete catecholamines and can produce hypertension.
However, hypokalemia will not be a feature of a phaeochromocytoma.
12.9
12.10
Answer: C Craniopharyngioma
Craniopharyngiomas comprise 9% of all paediatric brain tumours and usually occur in children
between the ages of 5 years and 10 years. They are very slow-growing tumours. They arise from the
cells along the pituitary stalk. Craniopharyngioma is a rare, usually suprasellar neoplasm, which can
be cystic. It develops from the nests of epithelium derived from Rathke’s pouch. The histological
pattern consists of nests of squamous epithelium bordered by radially arranged cells. It is often
accompanied by calcium deposition and shows a papillary type of architecture. These tumours are
also known as ‘Rathke pouch tumours’, ‘hypophyseal duct tumours’ or ‘adamantinomas’.
ACTH-secreting adenomas of the anterior pituitary are rare and are often microadenomas.
Astrocytomas in the paediatric age range are usually in the posterior fossa. Gliomas are unlikely to
erode bone and have calcifications. Null cell adenomas of the anterior pituitary can produce mass
effects, with headache and visual field defects but they are not suprasellar. Prolactinomas arise in the
anterior pituitary in the sella and are often small and slow-growing, though they are commonly
associated with headaches and visual field disturbances.
12.11
Answer: C Meningococcaemia
12.12
The multiple endocrine neoplasia (MEN) syndromes comprise three genetically distinct familial
diseases in which adenomatous hyperplasia and malignant tumours are found in several endocrine
glands. The clinical features depend on the glandular elements present. Each syndrome is inherited as
an autosomal dominant trait with a high degree of penetrance and variable expressivity, with the
production of seemingly unrelated effects by a single mutant gene. The specific genetic abnormalities
are not always known.
In this scenario the patient has multiple endocrine neoplasia, type IIA (MEN IIA) which is
characterised by medullary carcinoma of the thyroid, phaeochromocytoma and hyperparathyroidism.
MEN IIA should be suspected in patients with bilateral phaeochromocytoma, a familial history of
MEN, or at least two of its characteristic endocrine manifestations. The diagnosis is confirmed by
genetic testing. Many cases come to medical attention after bilateral phaeochromocytomas are
diagnosed in a relative.
Medullary carcinoma of the thyroid is diagnosed by measuring plasma calcitonin after provocative
infusion of pentagastrin and calcium. In most patients with palpable thyroid lesions, basal calcitonin
levels are elevated; in early disease, the basal levels may be normal, and the medullary carcinoma
can be diagnosed only by an exaggerated response to calcium and pentagastrin. Early diagnosis of
medullary carcinoma of the thyroid is important so that the tumour can be removed while still
localised.
Because phaeochromocytoma can be asymptomatic, its exclusion can be difficult. The most sensitive
tests are plasma free metanephrines and fractionated urinary catecholamines, particularly adrenaline
(epinephrine). Computed tomography or magnetic resonance imaging are useful for locating the
phaeochromocytoma or establishing the presence of bilateral lesions. Hyperparathyroidism is
diagnosed by the presence of hypercalcaemia, hypophosphataemia and an increased parathyroid
hormone level.
Genetic testing, used to confirm the diagnosis, is highly accurate. First-degree relatives and any
symptomatic relatives of the index patient should also undergo genetic testing. Annual screening for
hyperparathyroidism and phaeochromocytoma should begin in early childhood and continue
indefinitely. Screening for hyperparathyroidism is performed by measuring the serum calcium.
Screening for phaeochromocytoma includes a symptom history, measurement of the blood pressure
and laboratory testing.
12.13
Adrenocortical carcinoma is a rare malignancy and has a poor prognosis. The reported incidence of
adrenal carcinoma is two per million. When identified, tumours are often large, measuring 4–10 cm in
diameter. Adrenal carcinomas arise from the adrenal cortex and are bilateral in up to 10% of patients.
Approximately 50%–80% are functional tumours, with most causing Cushing syndrome (as is the case
in this scenario). Overall, there is an approximately equal sex distribution, but functional tumours are
slightly more common in women, while non-functional tumours are slightly more common in men.
Men with adrenocortical carcinoma often tend to be older than women who have this tumour and
appear to have a worse prognosis.
12.14
Papillary carcinoma of the thyroid has the best prognosis. It accounts for 70%–80% of all thyroid
cancers. The female to male ratio is 3 : 1, and the tumour is familial in up to 5% of patients. Most
patients present between the ages of 30 years and 60 years. The tumour is often more aggressive in
elderly patients. Many papillary carcinomas contain follicular elements. The tumour spreads via
lymphatics to regional lymph nodes in a third of patients and can metastasise to the lungs. Patients
aged under 45 years with small tumours confined to the thyroid have an excellent prognosis. About
10%–20% of patients, mainly the elderly, have recurrent or persistent disease.
12.15
The elevated serum calcium and hypophosphataemia point to parathyroid disease, and adenomas are
more common than hyperplasia. Metastatic carcinoma would be unusual at this age. Primary
hyperparathyroidism is caused by parathyroid adenoma in 80%–85% of patients, by multiple
parathyroid adenomas in 2%–3%, by parathyroid hyperplasia in 10%–15% and by parathyroid
carcinoma in 2%–3%. Common clinical presentations include nephrolithiasis (stones), bone pain
(bones), arthralgias, muscular aches, peptic ulcer disease, pancreatitis (groans), fatigue, depression
(moans), anxiety and other mental disturbances.
12.16
12.17
Answer: D Prolactinoma
This finding is typical of a pituitary adenoma that is enlarging and compressing the optic chiasma.
Prolactinoma is the most common adenoma. The symptoms experienced in women and men are as for
hyperprolactinaemia from any cause. In addition, macroprolactinomas, by their very size, can press
on surrounding structures, causing headaches or loss of vision from pressure on the optic chiasma.
Unlike women, who can observe a disruption of menstruation, men have no reliable indicator to
signal a problem and so often delay going to the doctor until they have headaches or eye problems. A
craniopharyngioma is an uncommon lesion that usually appears in children and young adults.
12.18
These findings point to Cushing syndrome arising from glucocorticoid excess. Ectopic
adrenocorticotrophic hormone from an oat cell carcinoma is one possible cause. The presence of
tenderness in the lower back could be due to metastatic disease because oat cell carcinoma is the
most widely metastatic of all tumours.
12.19
The total thyroidectomy might also have damaged or removed the parathyroids. The serum calcium
level is requested postoperatively to prevent potential problems associated with hypocalcaemia due
to hypoparathyroidism.
12.20
A thyroglossal duct cyst is a vestigial remnant of thyroid development that can become manifest at any
age. It is the most common clinically significant congenital thyroid anomaly. The most common
locations for a thyroglossal cyst are midline or slightly off-midline, between the isthmus of the thyroid
and the hyoid bone or just above the hyoid bone. A thyroglossal cyst can develop anywhere along the
thyroglossal duct, though cysts within the tongue or in the floor of the mouth are rare.
Women are affected more commonly than men. A thyroglossal cyst will move upwards with
protrusion of the tongue. Thyroglossal cysts are associated with an increased incidence of ectopic
thyroid tissue. Occasionally, a lingual thyroid can be seen as a flattened, strawberry-like lump at the
base of the tongue. Very rarely, the persistent duct can become cancerous (thyroglossal duct
carcinoma), where the cancerous cells are ectopic thyroid tissue that has been deposited along the
thyroglossal duct. This usually follows exposure to radiation.
SECTION 13:
BREAST AND FEMALE REPRODUCIVE PATHOLOGY —
ANSWERS
13.1
Lobular carcinoma of the breast can be divided into in-situ and invasive forms, and they both arise
from the acini or terminal ductules of the lobule. It accounts for about 10% of breast carcinomas. It
tends to be multifocal within the same breast, and is bilateral in 20% of cases. Metastases often seed
the peritoneum. In-situ disease progresses to invasive carcinoma at a rate of 15% over a 20-year
period.
13.2
Virginal breast hypertrophy is also known as ‘juvenile macromastia’ and ‘juvenile gigantomastia’. It
causes excessive growth of the breasts during puberty and is much more common than the rare cases
of breast hypertrophy in pregnancy. Virginal breast hypertrophy normally starts when puberty starts,
soon after the girl’s first menstrual period. The breast growth is sometimes not constant and comes in
‘growth spurts’. At times, women can have minimal or no breast growth and then experience a growth
spurt where the breasts grow very rapidly in a short space of time. These growth spurts cause great
physical discomfort, the main symptoms being red, itchy skin and sometimes a general ache in the
breasts. At puberty the breasts can also grow continuously at an even pace over several years. This
process can overdevelop a completely normal and healthy breast, sometimes to gigantic proportions.
Enlargement of the nipples usually also occurs, and the nipples can grow to an enormous size.
13.3
Answer: B Fibroadenoma
Fibroadenoma of the breast is a benign tumour that is characterised by proliferation of both glandular
and stromal elements. Most often it appears before the age of 30 years, and results from oestrogenic
hormonal excess. The tumour is usually solitary and multiple tumours are rare. The tumour is mobile
and not adherent to adjacent structures.
Macroscopically, the tumour is round, elastic, nodular and encapsulated (ie well circumscribed), its
cut surface being grey-white in colour. Microscopically, the epithelial proliferation describes duct-
like spaces surrounded by a fibroblastic stroma. Depending on the amount and the relationship
between these two components, there are two main histological types: intracanalicular and
pericanalicular. Both types are commonly found in the same tumour. In intracanalicular fibroadenoma
the stromal proliferation predominates and compresses the ducts, which are irregular and reduced to
slits. In pericanalicular fibroadenoma the fibrous stroma proliferates around the ductal spaces, so that
they remain round or oval on cross-section. The basement membrane is intact.
13.4
Acute mastitis is a bacterial infection of the breast, and results in the classic signs of infection – pain
(dolor), redness (rubor), swelling (turgor) and warmth (calor). Most often it occurs 2–3 weeks after
delivery but it can occur at any time. Typical causative organisms include Staphylococcus aureus,
Streptococcus species, and Escherichia coli. Prompt treatment can prevent complications such as
abscess formation. Antibiotics, continued breastfeeding, and plenty of rest are the treatments of
choice. In severe cases intravenous antibiotics can be required.
13.5
Breast cysts are very common and are rarely associated with cancer. With a cyst, the lump feels
smooth and firm, moves easily, and might be tender. If several cysts are grouped together, any
swelling may feel irregular. Cysts often occur in both breasts and can develop anywhere within them.
About 10% of women develop recurrent cysts. The fluid within the cyst comes from fluid in the breast
which occurs as part of a woman’s normal menstrual cycle. Towards the end of the menstrual cycle
the cells in the breasts enlarge and swell. After a period the cells shrink and the fluid that is released
disappears. But in some cases this fluid remains in the breast and forms a cyst. Cysts are most
common in women in their thirties and forties and tend to disappear after the menopause. They are
linked to oral contraceptive intake. Women who take hormone replacement therapy can also develop
breast cysts because their breasts are similar to those of a younger woman.
13.6
Phyllodes tumours are typically large, fast-growing masses that arise from the periductal stroma of
the breast. They account for less than 1% of all breast neoplasms. Phyllodes tumours are
fibroepithelial tumours that are composed of an epithelial component and a cellular stromal
component. They can be benign, borderline or malignant, depending on the histological features
(stromal cellularity, infiltration at the tumour’s edge, mitotic activity). This is predominantly a tumour
of adult women, with very few cases reported in adolescents. Most occur in women between the ages
of 40 years and 50 years, prior to the menopause. This is about 15 years older than the typical age of
patients with fibroadenoma, a condition with which phyllodes tumours can be confused.
13.7
A breast implant is a prosthesis used to enlarge the size of a woman’s breasts for cosmetic reasons, to
reconstruct the breast (eg after a mastectomy or to correct genetic deformities), or as an aspect of
male-to-female sex reassignment surgery. There are two primary types of breast implants, saline-
filled and silicone gel-filled implants. Saline implants have a silicone elastomer shell filled with
sterile saline liquid. Silicone gel implants have a silicone shell filled with a viscous silicone gel. A
variety of other types of breast implant have been developed, such as polypropylene string or soy oil,
but these are not seen commonly.
Local complications that can occur with breast implants include postoperative bleeding (haematoma),
fluid collections (seroma), surgical site infection, breast pain, alterations in nipple sensation,
interference with breastfeeding, visible wrinkling, asymmetric appearance, wound dehiscence (with
potential implant exposure), thinning of the breast tissue and synmastia (disruption of the natural plane
between breasts).
13.8
Invasion of the lymphatics is a poor prognostic sign, because it suggests that the carcinoma is
spreading. A family history makes breast cancer more likely, but does not predict the prognosis.
Aneuploidy, diagnosed with flow cytometry, suggests a worse prognosis. An oestrogen receptor-
postitive tumour has a better prognosis because it will respond to certain therapies. In-situ lesions are
the lowest stage possible for a carcinoma and carry an excellent prognosis if excised.
13.9
A breast abscess due to staphylococcal infection is the most likely diagnosis. These are most common
in the postpartum period when the nipple becomes cracked or fissured with nursing. Fat necrosis is
usually caused by trauma and leads to the formation of an ill-defined mass. Ductal carcinomas are ill-
defined masses that are not usually tender. They are extremely rare before age 25 years. Sclerosing
adenosis is a part of fibrocystic disease, which can lead to a cystic mass that can be tender, but
without nipple fissuring. A ‘plasma cell mastitis’ is the result of inspissated duct secretions and is
seen in women in their forties and fifties.
13.10
Answer: E Paget’s disease of the breast
Paget’s disease of the breast, also known as ‘Paget’s disease of the nipple’, is a condition that
outwardly can resemble eczema, with skin changes involving the nipple of the breast. Because of its
seemingly innocuous appearance, it often presents late, but it is a condition that can be fatal. There is
typically an underlying carcinoma. It results when malignant cells from an underlying carcinoma that
originated in the ducts of the mammary glands spread to the epithelium. It usually affects only one
nipple and presents with redness, oozing and crusting, and a sore that does not heal.
13.11
Intraductal papilloma is a small benign tumour that grows within a milk duct of the breast, and is most
common in women aged 35–55 years. The causes and risk factors are unknown. Intraductal papilloma
is the most common cause of spontaneous nipple discharge from a single duct. A small lump beneath
the nipple might be felt by the examiner, but it is not always palpable. A mammogram often does not
show papillomas, but ultrasound and ductography can be helpful. Cytologicical examination of
discharge might identify potentially malignant cells, but a breast biopsy is necessary to make a
definitive diagnosis and rule out cancer.
13.12
Medullary carcinoma of breast is a special type of infiltrating breast cancer that has a rather well-
defined, distinct boundary between tumour tissue and normal tissue. It also has some other special
features, including the large size of the cancer cells and the presence of lymphoid cells at the edges of
the tumour. They can be difficult to distinguish from invasive ductal carcinoma. Medullary carcinoma
accounts for about 5% of breast cancers. The prognosis for medullary carcinoma is better than for
other types of invasive breast cancer.
13.13
Gynaecomastia is hypertrophy of breast tissue in males. During puberty, enlargement of the male
breast is normal. It is usually transient, bilateral, smooth, firm and symmetrically distributed under the
areola; the breasts can be tender. Similar changes can occur during old age but are more often
unilateral. Most of the enlargement is due to proliferation of stroma, not of breast ducts.
Gynaecomastia can be caused by various disorders (especially hepatic and renal failure; less
commonly, endocrine disorders), drugs (eg anabolic steroid abuse, antineoplastic drugs, calcium-
channel blockers, cimetidine, digitalis, oestrogens, isoniazid, ketoconazole, methadone,
metronidazole, reserpine, spironolactone, theophylline) and marijuana. Gynaecomastia is rarely
confused with cancer, which is asymmetric, hard, and often fixed to dermis or fascia. Usually, the
only imaging or other diagnostic test needed is mammography. In most cases, no specific treatment is
needed because gynaecomastia usually remits spontaneously or disappears after any causative drug
(except perhaps anabolic steroids) is stopped or underlying disorder is treated.
13.14
13.15
Answer: B Fibrothecoma
A fibrothecoma is a benign ovarian tumour. The thecoma component of the neoplasm gives the tumour
a yellowish appearance because of the lipid content and can also produce oestrogen, which is
responsible for the endometrial hyperplasia. These are tumours that arise from the ovarian stroma.
They are bilateral in only about 10% of cases. A right-sided hydrothorax in association with this
tumour is known as ‘Meigs’ syndrome’.
13.16
The most widely accepted pathophysiological hypothesis is that endometrial cells are transported
from the uterine cavity and subsequently become implanted at ectopic sites. Retrograde flow of
menstrual tissue through the Fallopian tubes could transport endometrial cells intra-abdominally; the
lymphatic or circulatory system could transport endometrial cells to distant sites (such as the pleural
cavity). Another hypothesis is coelomic metaplasia, where coelomic epithelium is transformed into
endometrium-like glands. Microscopically, endometriotic implants consist of glands and stroma
identical to intrauterine endometrium. These tissues contain oestrogen and progesterone receptors and
so usually grow, differentiate, and bleed in response to changes in hormone levels during the
menstrual cycle.
The symptoms depend on the location of the implants and can include dysmenorrhoea, dyspareunia,
infertility, dysuria and pain during defecation. The incidence of endometriosis is increased in first-
degree relatives of women with endometriosis, suggesting that heredity is a factor. The incidence is
also increased in women who delay childbearing, who have shortened menstrual cycles (<27 days)
with menses that are abnormally long (>8 days), or who have Müllerian duct anomalies. The reported
incidence varies but is probably about 10%–15% in actively menstruating women aged 25–44 years.
The average age at diagnosis is 27 years, but endometriosis also occurs in adolescents. About 25%–
50% of infertile women have endometriosis. In patients with severe endometriosis and distorted
pelvic anatomy, the incidence of infertility is high because mechanisms of ovum pick-up and tubal
transport are impaired. Some patients with minimal endometriosis and normal pelvic anatomy can
also be infertile: in these patients, fertility might be decreased because the incidence of luteal phase
dysfunction or luteinised unruptured ovarian follicle syndrome (trapped oocyte) is increased, because
peritoneal prostaglandin production or peritoneal macrophage activity is increased (resulting in
phagocytosis), or because the endometrium is non-receptive. Potential protective factors seem to be
multiple pregnancies, the use of low-dose oral contraceptives (continuous or cyclic), and regular
exercise (especially if begun before the age of 15 years or if undertaken for more than 7 hours per
week, or both).
13.17
Ovarian carcinomas are often associated with ascites because they seed throughout the peritoneal
cavity. Psammoma body formation is common. Ovarian cancer affects mainly perimenopausal and
postmenopausal women. Nulliparity, delayed childbearing and delayed menopause increase the risk;
oral contraceptive use decreases the risk. A personal or family history of endometrial, breast or colon
cancer also increases the risk. Probably 5%–10% of ovarian cancers are related to mutations in the
autosomal dominant BRCA gene. XY gonadal dysgenesis predisposes to ovarian germ cell cancer.
Ovarian cancers are histologically diverse. At least 80% originate in the epithelium; 75% of these
cancers are serous cystadenocarcinomas, and the rest include mucinous, endometrioid, transitional
cell, clear cell and unclassified carcinomas, and Brenner tumours. The remaining 20% of ovarian
cancers originate in primary ovarian germ cells or in sex cord and stromal cells, or are metastases to
the ovary (most commonly from the breast or gastrointestinal tract). Germ cell cancers usually occur
in women under the age of 30 years and include dysgerminomas, immature teratomas, endodermal
sinus tumours, embryonal carcinomas, choriocarcinomas and polyembryomas. Stromal (sex cord–
stromal) cancers include granulosa-theca cell tumours and Sertoli–Leydig cell tumours.
Ovarian cancer spreads by direct extension, exfoliation of cells into the peritoneal cavity (peritoneal
seeding), lymphatic dissemination to the pelvis and around the aorta or, less often, haematogenously
to the liver or lungs.
13.18
Cervical cancer results from cervical intraepithelial neoplasia (CIN), which appears to be caused by
infection with human papillomavirus (HPV) types 16, 18, 31, 33, 35 or 39. Risk factors for cervical
cancer include younger age at the time of first intercourse, a high lifetime number of sex partners and
intercourse with men whose previous partners had cervical cancer. Other factors, including cigarette
smoking and immunodeficiency also appear to contribute to the risk. CIN is graded as grade 1 (mild
cervical dysplasia), grade 2 (moderate dysplasia) or grade 3 (severe dysplasia and carcinoma in
situ). CIN 3 is unlikely to regress spontaneously; if untreated, it can, over months or years, penetrate
the basement membrane, becoming invasive carcinoma. About 80%–85% of all cervical cancers are
squamous cell carcinoma; most of the rest are adenocarcinomas. Sarcomas and small-cell
neuroendocrine tumours are rare. Invasive cervical cancer usually spreads by direct extension into
surrounding tissues or via the lymphatics to the pelvic and para-aortic lymph nodes. Haematogenous
spread also occurs.
13.19
Gestational trophoblastic disease is a tumour originating from the trophoblast, which surrounds the
blastocyst and develops into the chorion and amnion. This disease can occur during or after an
intrauterine or ectopic pregnancy. If the disease occurs during a pregnancy, spontaneous abortion,
eclampsia or fetal death typically occur; the fetus rarely survives. Some forms are malignant; others
are benign but behave aggressively.
If gestational trophoblastic disease is suspected, testing includes measurement of serum beta subunit
of human chorionic gonadotrophin (β-hCG) and pelvic ultrasonography. A very high β-hCG level
might suggest the diagnosis, but a biopsy is required to confirm it. After the tumour has been removed,
gestational trophoblastic disease is classified clinically to determine whether additional treatment is
needed. (The clinical classification system does not correspond to the morphological classification
system.) A chest X-ray is taken and the serum β-hCG is measured. If the β-hCG level does not
normalise within 10 weeks, the disease is classified as persistent. Persistent disease requires
investigation by computed tomography of the brain, chest, abdomen and pelvis, the results of these
dictating whether disease is classified as non-metastatic or metastatic. Persistent disease is usually
treated with chemotherapy. Treatment is considered successful if at least three consecutive serum β-
hCG measurements at 1-week intervals are normal. Beta-hCG is used for follow-up.
13.20
Answer: D Imperforate hymen
Haematocolpos (accumulation of menstrual blood in the vagina), which can cause the vagina to bulge,
and haematometra (accumulation of blood in the uterus), which can cause uterine distension or a
mass, are most likely to be associated with an imperforate hymen. In about 1 in 2000 females, the
hymen fails to develop any opening at all: this is called an imperforate hymen and if it does not
spontaneously resolve itself before puberty a physician will need to make a hole in the hymen to
allow menstrual discharge to escape.
SECTION 14:
MALE REPRODUCTIVE PATHOLOGY — ANSWERS
14.1
Balanitis is inflammation of the glans penis. When the foreskin (or prepuce) is also affected, it is
called ‘balanoposthitis’. Lack of aeration and irritation because of smegma and discharge surrounding
the glans penis causes inflammation and oedema. Staphylococcal and streptococcal infections are
most likely to be present in this situation. While any man can develop balanitis, the condition is most
likely to occur in men who have a tight foreskin that is difficult to pull back, or who have poor
hygiene. Diabetes can make balanitis more likely, especially if the blood sugar is poorly controlled.
14.2
Answer: E Phimosis
Phimosis is a medical condition in which the foreskin of the penis of an uncircumcised male cannot
be fully retracted. In most (but not all) infants phimosis is physiological rather than pathological,
whereas in older children and adults phimosis is more often pathological than physiological. It has
been suggested that physiological infantile phimosis should be referred to as ‘developmental
nonretractility of the foreskin’ in order to distinguish this normal stage of development more clearly
from pathological forms of phimosis.
When phimosis develops in an uncircumcised adult who was previously able to retract his foreskin, it
is nearly always due to a pathological cause, and is far more likely to cause problems for the man. An
important cause of acquired, pathological phimosis is chronic balanitis xerotica obliterans, a skin
condition of unknown origin that causes a whitish ring of indurated tissue (a cicatrix) to form near the
tip of the prepuce. This inelastic tissue prevents retraction.
Some evidence suggests that balanitis xerotica obliterans could be the same disease as lichen
sclerosus et atrophicus of the vulva in females. Infectious, inflammatory and hormonal factors have
all been implicated or proposed as contributing factors. Circumcision is usually recommended,
though alternatives have been advocated. Phimosis can also occur after other types of chronic
inflammation (eg balanoposthitis), repeated catheterisation, or forceful foreskin retraction.
14.3
Answer: D Hypospadias
Hypospadias is a birth defect of the urethra in the male that involves an abnormally placed urethral
meatus. Instead of opening at the tip of the glans penis, a hypospadic urethra opens anywhere along a
line (the urethral groove) running from the tip, along the underside or ventral aspect of the shaft, to the
junction of the penis and scrotum or perineum. A distal hypospadias might be suspected, even in an
uncircumcised boy, if there is an abnormally formed foreskin and downward tilt of the glans.
The urethral meatus opens on the glans penis in about 50%–75% of cases; these are categorised as
first-degree hypospadias. Second-degree hypospadias (when the urethra opens on the shaft) and third-
degree hypospadias (when the urethra opens on the perineum) occur in up to 20% and 30% of cases,
respectively. The more severe degrees are more likely to be associated with chordee, in which the
phallus is incompletely separated from the perineum or is still tethered downwards by connective
tissue, or with undescended testes (cryptorchidism).
Hypospadias is one of the most common birth defects of the male genitalia (second to
cryptorchidism), but widely varying incidences have been reported from different countries, from as
low as 1 in 4000 to as high as 1 in 125 boys. Most cases of hypospadias are sporadic, without any
inheritance pattern or family recurrence. For most cases, no cause can be identified, though a number
of hypotheses related to inadequate androgen effect, or environmental agents interfering with
androgen effect have been offered.
14.4
Primary syphilis is typically acquired via direct sexual contact with the infectious lesions of a person
with syphilis. A skin lesion might be seen on the genitalia approximately 10–90 days after the initial
exposure (average 21 days). This lesion, called a ‘chancre’, is a firm, painless skin ulceration
located at the point of initial exposure to the spirochaete, often on the penis, vagina or rectum. Rarely,
there can be multiple lesions present, although typically only one lesion is seen. The lesion can
persist for 4–6 weeks and usually heals spontaneously. Local lymph node swelling can occur. During
the initial incubation period individuals are otherwise asymptomatic. As a result, many patients do
not seek medical care immediately.
Microscopy of fluid from the primary or secondary lesion using dark-field illumination can be used to
diagnose treponemal disease with high accuracy. As there are other treponemes that may be confused
with Treponema pallidum, care must be taken in evaluating with microscopy to correlate symptoms
with the correct disease. Present-day syphilis screening tests, such as the rapid plasma reagin (RPR)
and Venereal Diseases Research Laboratory (VDRL) tests are cheap and fast but not completely
specific, as many other conditions can cause a positive result. False positives can be seen in viral
infections (Epstein–Barr virus, hepatitis, varicella, measles), lymphoma, tuberculosis, malaria,
endocarditis, connective tissue disorders, pregnancy, intravenous drug abuse or with contamination.
As a result, these two screening tests should always be followed up by a more specific treponemal
test. Tests based on monoclonal antibodies and immunofluorescence, including the Treponema
pallidum haemagglutination assay (TPHA) and the fluorescent treponemal antibody absorption (FTA-
ABS) test are more specific and more expensive. Unfortunately, false positives can still occur in
related treponomal infections such as yaws and pinta. Tests based on enzyme-linked immunoassays
are also used to confirm the results of simpler screening tests for syphilis.
14.5
Embryonal carcinoma is one of the four germ cell tumours, the other three being seminoma, teratoma
and choriocarcinoma. Like any cancer, embryonal carcinoma grows because its cells are dividing
rapidly and indefinitely. The embryonal carcinoma can spread up the epididymis to the vas deferens
and can also spread to the rest of the body, including the lymph nodes that run along the aorta.
Unlike the other types of testicular cancers, an embryonal carcinoma can contain several elements of
a maldeveloped fetus, including cartilage. The main tumour is, on average, 2.5 cm in length and can
also stem out approximately 9 cm up the testicular cord. Embryonal carcinomas are usually seen in
males aged 25–35 years, but have also been found in males in their late teens. The chances of an
embryonal carcinoma spreading from one testicle to the other are less than 1%. Embryonal
carcinomas account for approximately 25% of testicular tumours. Rarely, embryonal carcinomas are
seen in females, in the ovaries.
14.6
Answer: D Spermatocoele
Spermatocoele (spermatic cyst) usually occurs at the upper pole of the testis, adjacent to the
epididymis, and appears as a cystic scrotal mass. A large spermatocoele might be difficult to
differentiate from a hydrocoele, which also is cystic and painless and transilluminates.
Ultrasonography can be helpful in reaching a diagnosis. Surgical excision is indicated if the
spermatocoele becomes large and bothersome.
14.7
Answer: E Varicocoele
A varicocoele is an abnormal enlargement of the veins in the scrotum draining the testis. The
testicular blood vessels originate in the abdomen and course down through the inguinal canal as part
of the spermatic cord on their way to the testis. Upward flow of blood in the veins is ensured by
small one-way valves. Defective valves, or compression of the vein by a nearby structure, can cause
dilatation of the veins near the testis, leading to the formation of a varicocoele.
The term ‘varicocoele’ specifically refers to dilatation and tortuosity of the pampiniform plexus,
which is the network of veins that drain the testicle. This plexus travels along the posterior portion of
the testicle with the epididymis and vas deferens, and then into the spermatic cord. This network of
veins coalesces into the gonadal (or testicular) vein. The right gonadal vein drains into the inferior
vena cava, while the left gonadal vein drains into the left renal vein, which then drains into the
inferior vena cava. The small vessels of the pampiniform plexus normally range from 0.5–1.5 mm in
diameter. A varicocoele is dilatation of these vessels to greater than 2 mm.
Idiopathic varicocoele occurs when the valves within the veins along the spermatic cord do not work
properly. This is essentially the same process as varicose veins, which are common in the legs. This
results in backflow of blood into the pampiniform plexus and causes increased pressures, ultimately
leading to damage to the testicular tissue. Varicocoeles usually develop slowly and might not cause
any symptoms. They are most commonly diagnosed when a patient is 15–25 years of age, and rarely
develop after the age of 40 years. They occur in 15%–20% of all males, and in 40% of infertile
males. Approximately 98% of idiopathic varicocoeles occur on the left side, apparently because of
the way the left testicular vein runs vertically up to the renal vein rather than directly into the vena
cava. In 70% of cases the varicocoele occurs bilaterally. Isolated right-sided varicocoeles are rare,
and should prompt evaluation for an abdominal or pelvic mass.
Secondary varicocoele is caused by compression of the venous drainage of the testicle. A pelvic or
abdominal malignancy is a definite concern when a varicocoele is newly diagnosed in a patient over
the age of 40 years. One non-malignant cause of a secondary varicocoele is the so-called ‘nutcracker
superior mesenteric artery’, a condition in which the superior mesenteric artery compresses the left
renal vein, causing increased pressures there to be transmitted in retrograde fashion into the left
pampiniform plexus.
• Infertility
Clinically, palpation of the scrotum will reveal a non-tender, twisted mass along the spermatic cord
(classically likened to a bag of worms). The mass might not be obvious, especially when the patient
is lying down. The testis on the side of the varicocoele might or might not be smaller compared with
the testis on the other side. Varicocoele can be reliably diagnosed with ultrasound, which will show
dilatation of the vessels of the pampiniform plexus to more than 2 mm. The patient should undergo a
provocative manoeuvre, such as a Valsalva manoeuvre, or stand up during the examination, both of
which are designed to increase intra-abdominal venous pressure and increase the dilatation of the
veins.
14.8
Answer: A Adenocarcinoma
Adenocarcinoma of the prostate is the most common non-dermatological cancer in men aged over 50
years. The incidence increases with each decade of life; autopsy studies show prostate cancer in
15%–60% of men aged 60–90 years, with the incidence increasing with age. The median age at
diagnosis is 72 years, and over 75% of prostate cancers are diagnosed in men over the age of 65
years. The risk is highest for black men. Sarcoma of the prostate is rare, occurring primarily in
children. Undifferentiated prostate cancer, squamous cell carcinoma, and ductal transitional
carcinoma also occur. Hormonal influences contribute to the pathogenesis of adenocarcinoma but
almost certainly not to that of other types of prostate cancer. Prostatic intraepithelial neoplasia is a
precancerous histological change, and can be either low-grade or high-grade, which is considered to
be a precursor of invasive cancer.
14.9
Peyronie’s disease is a connective tissue disorder involving the growth of fibrous plaques in the soft
tissue of the penis and affects as many as 1% of men. A French surgeon, François de la Peyronie, first
described the disease in 1743. The disease can cause pain, hardened, cord-like lesions (scar tissue
known as ‘plaques’) or abnormal curvature of the penis when erect. In addition, narrowing and/or
shortening of the penis can occur. The pain felt in the early stages of the disease often resolves after
12–18 months. Erectile dysfunction, in varying degrees, often accompanies these symptoms in the
later stages of the disease process. The condition can also make sexual intercourse painful and/or
difficult, though many men report satisfactory intercourse in spite of the disease.
Although it can affect men of any race and age, it is most commonly seen in white men over the age of
40 years. The disease only affects men and is confined to the penis, although about 30% of men with
Peyronie’s disease develop fibrosis in other elastic tissues of the body, such as on the hand or foot,
including Dupuytren’s contracture of the hand. An increased incidence in male relatives suggests a
genetic component.
14.10
Answer: C Seminoma
Germ cell tumours of the testis occur as the following subtypes and with the following frequencies:
seminoma (40%), embryonal (25%), teratocarcinoma (25%), teratoma (5%) and choriocarcinoma
(pure) (1%).
14.11
Answer: C N2
Testicular seminoma is staged according to the American Joint Committee on Cancer (AJCC) 2002
staging guidelines. This is a TNM (tumour-node-metastasis) staging system comprising separate
categories for the primary tumour, regional lymph nodes, distant metastases, and serum tumour
markers; these four categories are used to determine the stage of the patient’s disease. Modern
treatment decisions are based in part on the subdivisions of this staging system.
14.12
Answer: C T2
According to the AJCC guidelines (see Answer to 14.11), the primary tumour staging for testicular
seminoma is as follows:
14.13
Answer: B S1
According to AJCC guidelines (see Answer to 14.11), the serum tumour marker staging is as follows:
14.14
According to AJCC guidelines (see Answer to 14.11), the clinical staging for testicular seminoma is
as follows:
Stage IA: T1 N0 M0 S0
Stage IB: T2/3/4 N0 M0 S0
Stage IC: any T N0 M0 S1/2/3
Stage IIA: any T N1 M0 S0/1
Stage IIB: any T N2 M0 S0/1
Stage IIC: any T N3 M0 S0/1
Stage IIIA: any T any N M1a S0/1
Stage IIIB: any T any N M0/1a S2
Stage IIIC: any T any N M1a/1b S3
In this scenario, therefore, the patient has a T2 N3 M0 S1 seminoma, which fits into stage IIC disease.
14.15
Prostate-specific antigen (PSA) is an enzyme produced by the prostate. Specifically, PSA is a serine
protease similar to kallikrein. Its normal function is to liquify gelatinous semen after ejaculation,
allowing spermatozoa to ‘swim’ more easily through the uterine cervix. PSA levels under 4 ng/ml are
generally considered normal, while levels over 4 ng/ml are considered abnormal (although in men
over the age of 65 years levels up to 6.5 ng/ml might be acceptable, depending upon each
laboratory’s reference ranges). PSA levels between 4 ng/ml and 10 ng/ml indicate a risk of prostate
cancer that is higher than normal, but the risk does not seem to rise within this six-point range. When
the PSA level is above 10 ng/ml, the association with cancer becomes stronger. However, PSA is not
a perfect test. Some men with prostate cancer do not have an elevated PSA, and most men with an
elevated PSA do not have prostate cancer.
PSA levels can change for many reasons other than cancer. Two common causes of high PSA levels
are benign prostatic hyperplasia and prostatitis. It can also be raised for 24 hours after ejaculation
and for several days after catheterisation. PSA levels are lowered in men who use medications used
to treat benign prostatic hyperplasia or baldness. These medications, finasteride and dutasteride, can
decrease the PSA levels by 50% or more.
14.16
Answer: C T1c
Prostatic cancer is staged according to the American Joint Committee on Cancer (AJCC) 2002
staging guidelines. This is a TNM staging system comprising separate categories for the primary
tumour, regional lymph nodes and distant metastases. It also uses histological grade information in
conjunction with TNM status to group cases into four overall stages. According to AJCC guidelines,
the primary tumour staging for prostatic cancer is as follows:
14.17
Answer: C Stage II
According to the AJCC guidelines (see Answer to 14.16), the overall staging for prostatic cancer is
summarised as in the following table:
In this scenario, therefore, the patient has a T2 N0 M0 G2 tumour, which makes it a stage II cancer.
14.18
Finasteride is an anti-androgen which acts by inhibiting type 2 5-alpha reductase, the enzyme that
converts testosterone to dihydrotestosterone. It is used as a treatment in benign prostatic hyperplasia
in low doses, and in prostate cancer in higher doses. It is also indicated for use in combination with
doxazosin therapy to reduce the risk of symptomatic progression of benign prostatic hyperplasia.
Additionally, it is registered in many countries for male-pattern baldness.
14.19
Answer: B T1
The TNM (tumour-node-metastasis) classification of the primary tumour of the penis is given below:
14.20
Answer: E Teratoma
Testicular teratomas occur in both children and adults, but their incidence and natural history are
quite different. Pure teratomas are fairly common in children, comprising nearly half of all germ cell
tumours. They are relatively rare after puberty and comprise only 2%–3% of germ cell tumours in this
age group. In children, they behave as a benign tumour, whereas in adults and adolescents they are
invariably malignant.
Childhood testicular teratomas are uniformly benign, with no documented cases of retroperitoneal or
lung metastasis in differentiated lesions. Most morbidity results from surgical or postoperative
complications, such as haemorrhage or infection. The mortality is less than one per million. During
and after puberty, all teratomas are regarded as malignant because even mature teratomas (composed
of entirely mature histological elements) can metastasise to retroperitoneal lymph nodes or to other
systems. Morbidity is associated with the growth of the tumour, which can invade or obstruct local
structures and become unresectable.
Malignant transformation is significantly more common in testicular teratomas than in their ovarian
counterparts, and the risk of recurrence is around 20% in both mature and immature testicular
teratomas.
Testicular teratomas usually present as a painless scrotal mass, but sometimes present as testicular
torsion. The masses are firm or hard in 83% of cases. Most are not tender and do not transilluminate.
Testicular pain and scrotal swelling occasionally are reported with teratomas, but these are non-
specific symptoms and simply indicate torsion until proved otherwise. Hydrocoele is often associated
with teratoma in childhood. On examination, the testis is diffusely enlarged, rather than nodular,
although a discrete nodule in the upper or lower pole can sometimes be palpated.
SECTION 15:
SECTION 15: BONE AND JOINT PATHOLOGY — ANSWERS
15.1
Answer: A Gout
Gout is precipitation of monosodium urate crystals into tissue, usually in and around joints. This
usually causes recurrent acute or chronic arthritis. Acute arthritis is initially monoarticular and often
involves the first metatarsophalangeal joint. Symptoms include acute pain, tenderness, warmth,
redness and swelling. Diagnosis requires identification of crystals in synovial fluid.
Patients suspected of acute gouty arthritis should have arthrocentesis and synovial fluid analysis on
initial presentation. Synovial fluid analysis can confirm the diagnosis by identifying needle-shaped,
strongly negatively birefringent urate crystals that are either free in the fluid or engulfed by
phagocytes. During attacks, synovial fluid shows inflammatory characteristics. The table below
summarises the findings of microscopic examination of crystals in joints.
*Crystals that have negative elongation are yellow parallel to the axis of slow vibration marked on
the compensator; positive elongation appears blue in the same direction.
†Occur primarily in patients with renal failure.
15.2
Paget’s disease of bone is a chronic disorder of the adult skeleton in which bone turnover is
accelerated in localised areas. Normal matrix is replaced with softened and enlarged bone. The
disease can be asymptomatic or can cause gradual onset of bone pain or deformity. The cause of this
disorder is unknown. The appearance of involved bone on electron microscopy suggests a viral
infection, but a viral cause has not been established. Paget’s disease is sometimes familial, but a
specific genetic pattern has been suggested. About 1% of adults in the US aged over 40 years have
Paget’s disease, with a 3 : 2 male predominance. The disease is most common in Europe (except
Scandinavia), Australia, and New Zealand.
Any bone can be involved. The bones most commonly affected are, in decreasing order: the pelvis,
femur, skull, tibia, vertebrae, clavicle and humerus. Bone turnover is accelerated at involved sites.
Pagetic lesions are metabolically active and highly vascular. Excessively active osteoclasts are often
large and contain many nuclei. Osteoblastic repair is also hyperactive, producing coarsely woven,
thickened lamellae and trabeculae. This abnormal structure weakens the bone, despite bony
enlargement and heavy calcification. Overgrown bone can compress nerves and other structures
passing through small foramina. Osteoarthritis can develop in joints adjacent to involved bone.
Paget’s disease should be suspected in patients with unexplained bone pain or deformity. It should
also be suspected in patients who have suggestive findings on X-ray, in patients who have an
unexplained elevation of serum alkaline phosphatase on laboratory tests performed for other reasons,
or in older patients who develop hypercalcaemia during bedrest.
If Paget’s disease is suspected, plain X-rays and serum alkaline phosphatase, calcium, and phosphate
levels should be obtained. Characteristic X-ray findings include increased bone density, abnormal
architecture with coarse cortical trabeculation or cortical thickening, bowing and bony enlargement;
there might also be stress microfractures of the tibia or femur. Characteristic laboratory findings
include elevated serum alkaline phosphatase (due to the increased anabolic activity of the bone) but
usually normal serum phosphate levels. The serum calcium is usually normal but can increase due to
immobilisation or hyperparathyroidism. If the alkaline phosphatase is not elevated or if it is unclear
whether the increased serum alkaline phosphatase is of bony origin, a bone-specific fraction can be
measured.
15.3
Osteoid osteoma, which tends to affect young adults, can occur in any bone but is most common in
long bones. It can cause pain (usually worse at night) that is typically relieved by mild analgesics,
particularly aspirin. Physical examination can show atrophy of regional muscles. The characteristic
appearance on X-ray is a small radiolucent zone surrounded by a larger sclerotic zone. If a tumour is
suspected, a technetium-99m bone scan should be performed; an osteoid osteoma appears as an area
of increased uptake.
15.4
Polymyalgia rheumatica (PMR) is a syndrome closely associated with temporal arteritis. It affects
older adults. The onset can be acute or subacute. PMR is characterised by severe pain and stiffness of
the neck and pectoral and pelvic girdles; the stiffness is particularly severe in the morning or after a
period of inactivity. Pain is most often localised to the proximal muscles rather than the joints, and
symptoms are usually bilateral. Systemic symptoms, such as weight loss, malaise, fever and
depression are common. PMR does not cause muscle weakness, although pain might limit muscular
effort.
PMR should be suspected in older adults with the typical symptoms. Establishing the diagnosis
requires the presence of characteristic symptoms and signs and the exclusion of alternative diagnoses.
The erythrocyte sedimentation rate (ESR), full blood count, thyroid-stimulating hormone levels and
creatine kinase are usually obtained. In most people, the ESR is elevated, often to over 100 mm/hour
and usually to over 50 mm/hour (Westergren method). Normochromic normocytic anaemia can be
present. Electromyography, biopsy and other tests (eg rheumatoid factor) are normal in PMR but are
sometimes performed in order to rule out alternative diagnoses.
15.5
These tumours, which most commonly affect people in their twenties and thirties, occur in the
epiphyses and can erode the rest of the bone and extend into the soft tissues. Giant-cell tumours are
notorious for their tendency to recur. Rarely, a giant-cell tumour can metastasise, even though it
remains histologically benign. Benign giant-cell tumours appear lytic on X-ray.
15.6
Answer: C Osteoarthritis
Osteoarthritis is a chronic arthropathy of an entire joint, with disruption and potential loss of joint
cartilage, along with other joint changes, including bone hypertrophy (osteophyte formation). This
most common joint disorder often becomes symptomatic in the forties and fifties and is nearly
universal by the age of 80 years. Only half of those with the pathological changes of osteoarthritis
have symptoms. Below the age of 40 years, most osteoarthritis occurs in men and results from trauma.
Women predominate between the ages of 40 years and 70 years, after which men and women are
affected equally.
Symptoms include gradually developing pain that is aggravated or triggered by activity, with stiffness
relieved less than 30 minutes after activity, and occasional joint swelling. The joints most often
affected in generalised osteoarthritis include the distal interphalangeal and proximal interphalangeal
joints (producing Heberden’s and Bouchard’s nodes), the thumb carpometacarpal joint, the
intervertebral discs and zygapophyseal joints in the cervical and lumbar vertebrae, the first
metatarsophalangeal joint, the hips and the knees.
Osteoarthritis should be suspected in patients with gradual onset of symptoms and signs, particularly
in older adults. If osteoarthritis is suspected, plain X-rays should be obtained of the most
symptomatic joints. X-rays generally reveal marginal osteophytes, narrowing of the joint space,
increased density of the subchondral bone, subchondral cyst formation, bony remodelling and joint
effusions. Standing X-rays of the knees are more sensitive to joint-space narrowing.
Laboratory studies are normal in osteoarthritis but might be required to rule out other disorders (eg
rheumatoid arthritis) or to diagnose an underlying disorder causing secondary osteoarthritis. If
osteoarthritis causes joint effusions, synovial fluid analysis can sometimes be used to differentiate it
from inflammatory arthritides; in osteoarthritis, synovial fluid is usually clear, viscous, and contains
2000 white blood cells per microlitre or less.
Osteoarthritic involvement outside the usual joints suggests secondary OA; further evaluation might
be required to determine the underlying primary disorder (endocrine, metabolic, neoplastic or
biomechanical disorders).
15.7
The features in this scenario are characteristic of chondrosarcoma. Chondrosarcomas are malignant
tumours of cartilage. They differ from osteosarcomas clinically, therapeutically and prognostically.
Around 90% of chondrosarcomas are primary tumours. Rarely, chondrosarcomas arise in other, pre-
existing conditions, particularly multiple osteochondromas. Chondrosarcomas tend to occur in older
adults. They often develop in flat bones (eg pelvis, scapula) but can develop in any portion of any
bone and can implant in surrounding soft tissues.
Plain X-rays often reveal punctate calcifications. Primary chondrosarcomas also often exhibit
cortical bone destruction and loss of normal bone trabeculae. The appearance of punctate
calcifications or an increase in size of an osteochondroma are findings that are suggestive of
secondary chondrosarcoma. Technetium-99m bone scintigraphy is an accurate method for screening.
Biopsy is required for diagnosis and can also determine the grade of the tumour (ie the probability of
metastasis).
15.8
The features in this case are suggestive of multiple myeloma. Multiple myeloma is the most common
primary malignant bone tumour and is of haematopoietic derivation. It occurs mostly in older adults.
The neoplastic process is usually multicentric and often involves the bone marrow so diffusely that
bone marrow aspiration is diagnostic. Aspiration reveals sheets or clusters of plasma cells that are
diagnostic of myeloma. X-rays usually show sharply circumscribed lytic lesions or diffuse
demineralisation.
15.9
15.10
A prostatic adenocarcinoma should be the first guess (particularly in a man!) with osteoblastic (bone-
forming) tumour metastases. Extensive metastases can act in a myelophthisic manner that leads to
peripheral blood leucoerythroblastosis.
15.11
An aneurysmal bone cyst is idiopathic and usually develops before the age of 20 years. This cystic
lesion usually occurs in the metaphyseal region of the long bones, but almost any bone can be
affected. It tends to grow slowly. Periosteal new bone formation tends to limit the periphery of the
mass. Pain and swelling are common. The lesion can be present for anything between a few weeks
and a few years before diagnosis. The appearance on X-ray is often characteristic: the rarefied area
is usually well circumscribed and eccentric; its periosteum bulges, extending into the soft tissues, and
might be surrounded by new bone formation.
15.12
Answer: C Osteoporosis
15.13
Answer: D Osteochondroma
Osteochondromas (osteocartilaginous exostoses), the most common benign bone tumours, can arise
from any bone but tend to occur near the ends of long bones. They occur most often in people aged
10–20 years and can be single or multiple. Multiple osteochondromas tend to run in families.
Secondary malignant chondrosarcoma develops in about 10% of patients with multiple
osteochondromas but in fewer than 1% of those with single lesions.
15.14
Answer: B HA-B27
Ankylosing spondylitis is three times more frequent in men than in women and usually begins between
the ages of 20 years and 40 years. It is 10 to 20 times more common in first-degree relatives of
patients with ankylosing spondylitis than in the general population. The risk of ankylosing spondylitis
in first-degree relatives with the HLA-B27 allele is about 20%. The increased prevalence of
HLAB27 in white people or HLA-B7 in black people supports a genetic predisposition. However,
the concordance rate in identical twins is only about 50%, suggesting that environmental factors must
also contribute. The pathophysiology probably involves immune-mediated inflammation.
15.15
Answer: A Bursitis
A bursa is a sac-like cavity or potential cavity that contains fluid, and is found where friction occurs
(eg where tendons or muscles pass over bony prominences). Bursas minimise friction between
moving parts and facilitate movement. They can communicate with joints. Bursitis can occur in the
shoulder (subacromial or subdeltoid bursitis), secondary to rotator cuff tendinitis, which is usually
the primary lesion in the shoulder. Other commonly affected bursas include the olecranon bursa
(miner’s elbow), the prepatellar bursa (housemaid’s knee), the suprapatellar bursa, the retrocalcaneal
(Achilles) bursa, the iliopectineal (iliopsoas) bursa, the ischial bursa (tailor’s or weaver’s bottom),
the greater trochanteric bursa, the anserine bursa and the first metatarsal head bursa (bunion).
Bursitis can be caused by injury, chronic overuse, inflammatory arthritis (eg due to gout or
rheumatoid arthritis) or acute or chronic infection (for example with pyogenic organisms, particularly
Staphylococcus aureus). Infection is the most common cause in the olecranon and prepatellar bursas.
Acute bursitis might occur following unusual exercise or strain and usually causes bursal effusion.
Chronic bursitis can follow previous attacks of bursitis or repeated trauma. The bursal wall is
thickened, with proliferation of its synovial lining; bursal adhesions, villi, tags and chalky deposits
can form. Bursitis occasionally causes inflammation in a communicating joint.
Acute bursitis causes pain, particularly on movement, and localised tenderness. Swelling, sometimes
with other signs of inflammation, is common if the bursa is superficial (eg prepatellar bursa,
olecranon bursa). Crystal-induced or bacterial-induced bursitis is usually erythematous as well as
painful and warm. Bursitis should be suspected in patients with swelling or signs of inflammation
over bursas. It can generally be diagnosed clinically. If the swelling is particularly painful, red or
warm, or if bursitis involves the olecranon bursa or the prepatellar bursa, then infection and crystal-
induced disease should be excluded by bursal puncture. Using local anaesthesia and sterile technique,
fluid is withdrawn from the bursa and analysed for cell count, Gram staining and culture, and
microscopy for the detection of crystals. Gram staining is not sensitive, however, and white blood
cell counts in infection can be lower than those found in septic joints. Urate crystals are easily seen
with polarised light, but the apatite crystals typical of calcific tendinitis appear only as shiny, non-
birefringent chunks. X-rays should be obtained if the bursitis is persistent or if infection or
calcification is suspected.
15.16
This cricketer has a herniated disc which is impinging on spinal nerve root L5 to produce sensory
disturbances. Pain radiates along the course of the sciatic nerve, most often down the buttocks and the
posterior aspect of the leg to below the knee. The pain is typically burning, lancinating or stabbing. It
can occur with or without low back pain. Performing a Valsalva manoeuvre can exacerbate the pain.
Nerve root compression can produce sensory, motor or (the most objective finding) reflex deficits.
L5–S1 disc herniation can affect the ankle jerk reflex, whereas L3–L4 herniation can affect the knee
jerk. Straight-leg raising can produce pain that radiates down the leg when the leg is raised above 60°
(and sometimes when the leg is raised even less than this). This is sensitive for sciatica; pain
radiating down the involved leg with lifting of the contralateral leg (crossed straight-leg raising) is
more specific for sciatica.
15.17
Answer: B Fibromatosis
The description is suggestive of Dupuytren’s contracture due to palmar fibromatosis. This is one of
the more common hand deformities; the incidence is higher in men and increases after the age of 45
years. This autosomal dominant condition with variable penetrance occurs more commonly in patients
with diabetes, alcoholism or epilepsy. However, the specific factor that causes the palmar fascia to
thicken and contract is unknown. The earliest manifestation is usually a tender nodule in the palm,
most often near the middle or ring finger; it gradually becomes painless. Next, a superficial cord
forms and contracts the metacarpophalangeal joints and interphalangeal joints of the fingers. The hand
eventually becomes arched. The disease is occasionally associated with fibrous thickening of the
dorsum of the proximal interphalangeal joints (Garrod’s pads), Peyronie’s disease (penile
fibromatosis) in about 7%–10% of patients and, rarely, with nodules on the plantar surface of the feet
(plantar fibromatosis). Other types of flexion deformities of the fingers can also occur in diabetes, in
systemic sclerosis, and in chronic reflex sympathetic dystrophy.
15.18
Answer: A Carpal tunnel syndrome
Carpal tunnel syndrome is very common and usually occurs in women aged 30–50 years. Risk factors
include rheumatoid arthritis or other types of wrist arthritis (sometimes this is the presenting
manifestation), diabetes mellitus, hypothyroidism, acromegaly, amyloidosis and pregnancy-induced
oedema in the carpal tunnel. Activities or jobs that require repetitive flexion and extension of the
wrist can also be contributory factors. Most cases are idiopathic.
Symptoms include pain in the hand and wrist, associated with tingling and numbness, classically
distributed along the median nerve (the palmar side of the thumb, the index and middle fingers, and
the radial half of the ring finger) but sometimes involving the whole hand. Typically, the patient
wakes at night with burning or aching pain and with numbness and tingling, and shakes the hand to
obtain relief and restore sensation. Thenar atrophy and weakness of thumb opposition and abduction
can also develop later on.
The diagnosis is strongly suggested by Tinel’s sign, in which median nerve paraesthesiae are
reproduced by tapping at the volar surface of the wrist over the site of the median nerve in the carpal
tunnel. Reproduction of tingling with wrist flexion (Phalen’s sign) is also suggestive. However,
clinical differentiation from other types of peripheral neuropathy can sometimes be difficult. If
symptoms are severe or the diagnosis is uncertain, conduction testing of the median nerve should be
performed.
15.19
This is a rheumatoid nodule, a peculiar form of granulomatous inflammation that can be seen in the
soft tissues of people with rheumatoid arthritis. Rheumatoid nodules develop in about 30% of patients
with rheumatoid arthritis. They consist of a central necrotic area surrounded by palisaded histiocytic
macrophages, all enveloped by lymphocytes, plasma cells and fibroblasts. Nodules and vasculitis can
also develop in many of the visceral organs.
15.20
Answer: D Pseudogout
CPPD deposition disease should be suspected in older patients with arthritis, particularly
inflammatory arthritis. The diagnosis is established by identifying rhomboid or rod-shaped, weakly
positively birefringent crystals on polarised light microscopy of synovial fluid. Coincident infectious
arthritis must be ruled out by Gram staining and culture. X-rays are indicated if synovial fluid cannot
be obtained for analysis: findings of multiple linear or punctate calcification in articular cartilages,
especially fibrocartilages, support the diagnosis.
SECTION 16:
CENTRAL NERVOUS SYSTEM PATHOLOGY — ANSWERS
16.1
Answer: C Thromboembolism
The appearance of the infarction in a major blood flow distribution and the previous history of heart
disease suggests embolic disease. This patient probably had a large left atrium filled with mural
thrombus.
16.2
Answer: C Meningioma
Meningiomas, particularly those less than 2 cm in diameter, are among the most common intracranial
tumours. Meningioma is the only brain tumour that is more common in women. They tend to occur
between the ages of 40 years and 60 years but can occur during childhood. These benign tumours can
develop wherever there is dura, most commonly over the convexities near the venous sinuses, along
the base of the skull, in the posterior fossa and, rarely, within the ventricles. Multiple meningiomas
can develop. Meningiomas compress but do not invade brain parenchyma. They can invade and
distort adjacent bone. There are many histological types, but they all follow a similar clinical course,
and some become malignant.
Symptoms depend on which part of the brain is compressed and so on the tumour’s location. Midline
tumours in the elderly can cause dementia with few other focal neurological findings. Diagnosis is
similar to that of other brain tumours, usually by magnetic resonance imaging with a paramagnetic
contrast agent. Bony abnormalities (eg hyperostosis around the cerebral convexities, changes in the
tuberculum sellae) might be seen incidentally on computed tomography or on plain X-rays.
16.3
Glioblastoma multiforme, also known as ‘grade 4 astrocytoma’, is the most common and aggressive
type of primary brain tumour, accounting for 52% of all primary brain tumours and 20% of all
intracranial tumours. Despite being the most prevalent form of primary brain tumour, there are only
two to three cases of glioblastoma multiforme per 100,000 people in Europe and North America.
Glioblastoma multiforme tumours are characterised by the presence of small areas of necrotising
tissue that are surrounded by highly anaplastic cells. This characteristic differentiates the tumour from
grade 3 astrocytomas, which do not have regions of necrotic tissue. Although glioblastoma multiforme
can be derived from lower-grade astrocytomas, autopsies have revealed that most do not arise from
precursor lesions in the brain.
Unlike oligodendrogliomas, glioblastoma multiforme can form in either the grey matter or the white
matter of the brain, but most arise from the deep white matter and quickly infiltrate the brain, often
becoming very large before causing any symptoms. The tumour can extend to the meningeal or
ventricular wall, leading to a characteristically high protein content of the cerebrospinal fluid (>100
mg/dl), as well as an occasional pleocytosis of 10 to 100 cells, mostly lymphocytes. Malignant cells
carried in the cerebrospinal fluid can spread to the spinal cord or cause meningeal gliomatosis.
However, metastasis of glioblastoma multiforme beyond the central nervous system is extremely rare.
About 50% of these tumours occupy more than one lobe of a hemisphere or are bilateral. Tumours of
this type usually arise from the cerebrum, and can exhibit the classic infiltration across the corpus
callosum, producing a butterfly (bilateral) glioma.
The tumour can express a variety of appearances, depending on the amount of haemorrhage or
necrosis, or on its age. A computed tomographic scan usually shows a non-homogeneous mass with a
hypointense centre and a variable ring of enhancement surrounded by oedema. Part of a lateral
ventricle is usually deformed and both lateral and third ventricles can be displaced.
Although common symptoms of the disease can include seizures, nausea and vomiting, headache and
hemiparesis, the most common symptom is progressive memory, personality or neurological deficit
due to temporal and frontal lobe involvement. The kind of symptoms produced depend very much on
the location of the tumour, more so than on its pathological properties. These tumours can start
producing symptoms quickly, but are occasionally asymptomatic until they reach an enormous size.
16.4
The brain has a high lipid content and typically undergoes liquefaction with ischaemic injury.
16.5
Maternal serum screening during the second trimester is a non-invasive way of identifying women at
increased risk of having children with a neural tube defect (usually open spina bifida or
anencephaly), Down’s syndrome or trisomy 18. Screening should be offered to all pregnant women.
When amniocentesis is recommended to test for fetal abnormalities, some women request serum
screening before they agree to the procedure, so that risk of such abnormalities can be more precisely
defined. Results are most accurate when the initial sample is obtained between 16 weeks’ and 18
weeks’ gestation, although screening can be done between 15 weeks and 20 weeks; normal values
vary with gestational age. Corrections for maternal weight, diabetes mellitus, race and other factors
are also necessary.
Maternal alpha-fetoprotein levels are measured first: elevated levels suggest open spina bifida,
anencephaly, increased risk of pregnancy complications (eg intrauterine growth restriction, abruptio
placentae) or, occasionally, twins or other multiple pregnancy. Closed spina bifida is not usually
detected by this test. Designating a cut-off value to determine whether further testing is warranted
involves weighing the risk of missed abnormalities against the risk of complications from
unnecessary testing. Usually, a cut-off value in the 95th to 98th percentile or 2.0 to 2.5 times the
normal pregnancy median (multiples of the median, or MOM) is used. This value is about 80%
sensitive for open spina bifida and 90% sensitive for anencephaly. When this value is used,
amniocentesis is eventually required in 1% to 2% of women originally screened. Lower cut-off
values increase sensitivity but decrease specificity, resulting in more amniocenteses being performed.
16.6
A brain abscess can result from direct extension of cranial infections (eg osteomyelitis, mastoiditis,
sinusitis, subdural empyema), penetrating head wounds (including neurosurgical procedures),
haematogenous spread (bacterial endocarditis, congenital heart disease with right-to-left shunt,
intravenous drug abuse) or from unknown causes. The bacteria involved are usually anaerobic and
sometimes mixed, often including anaerobic streptococci or Bacteroides species. Staphylococci are
common after cranial trauma, neurosurgery, or endocarditis. Enterobacteria are common with an ear
source. Fungi (eg Aspergillus) and protozoa (eg Toxoplasma gondii, particularly in HIV-infected
patients) can cause cerebral abscesses.
An abscess forms when an area of cerebral inflammation becomes necrotic and encapsulated by glial
cells and fibroblasts. Oedema around the abscess can increase the intracranial pressure. Symptoms
result from increased intracranial pressure and mass effects. Headache, nausea, vomiting, lethargy,
seizures, personality changes, papilloedema and focal neurological deficits develop over days to
weeks. Fever, chills and leucocytosis can develop before the infection is encapsulated, and then
subside.
16.7
Vestibular neuronitis causes a self-limited episode of vertigo, presumably due to inflammation of the
vestibular division of cranial nerve VIII; some vestibular dysfunction can persist. Although the cause
is unclear, a viral cause is suspected. Symptoms include a single attack of severe vertigo, with nausea
and vomiting and persistent nystagmus towards the affected side, which lasts 7–10 days. The
nystagmus is unidirectional, horizontal and spontaneous, with fast-beat oscillations in the direction of
the unaffected ear. The absence of concomitant tinnitus or hearing loss is a hallmark of vestibular
neuronitis. The condition slowly subsides after this initial episode. Some patients have residual
disequilibrium, especially with rapid head movements, probably due to permanent vestibular injury.
16.8
Answer: D Schwannoma
Schwannomas are histologically benign neoplasms that arise from nerve sheaths, most commonly
from sensory nerve roots. Schwannomas are clearly delineated from the nerve root and so surgical
resection of schwannomas can be accomplished without sacrifice of the associated nerve root.
Because they are histologically benign they have an excellent prognosis.
Intracranially, schwannomas most commonly arise from the vestibular nerve. Although the most
accurate term for these neoplasms is ‘vestibular schwannoma’, the term ‘acoustic neuroma’ is firmly
entrenched in the neurosurgical literature. The most common presentation of the acoustic neuroma is
progressive, unilateral hearing loss due to compression of the adjacent cochlear nerve. Tinnitus is
another common early symptom. As it grows the tumour can compress the adjacent trigeminal nerve
and brainstem. Nerve-sheath tumours also occur on other cranial nerves, including the trigeminal and
glossopharyngeal nerves.
16.9
Epidural haematomas (a collection of blood between the skull and the dura mater) are usually caused
by arterial bleeding, classically due to damage to the middle meningeal artery by a temporal bone
fracture. Without intervention, patients with large or arterial epidural haematomas can deteriorate
rapidly and die. Small, venous epidural haematomas are rarely lethal.
Symptoms of epidural haematoma usually develop within minutes to several hours after the injury and
consist of increasing headache, decreased level of consciousness, hemiparesis and pupillary dilation
with loss of light reactivity. Some patients lose consciousness, followed by a transient lucid interval,
then subsequent neurological deterioration.
16.10
Answer: E White matter
Brain function can be impaired immediately by direct damage of brain tissue (eg by crush or
laceration injuries). Further damage can occur shortly thereafter as a result of the cascade of events
triggered by the initial injury. Traumatic brain injury of any sort can cause oedema in the damaged
tissues. The greatest amount of salt and water increase with cerebral oedema occurs within the white
matter. The cranial vault is fixed in size (constrained by the skull) and almost completely filled by
non-compressible cerebrospinal fluid and minimally compressible brain tissue. Consequently, any
swelling from oedema, haemorrhage or haematoma has nowhere to expand into and so increases the
intracranial pressure (ICP). Cerebral blood flow is proportional to the cerebral perfusion pressure,
which is the difference between mean arterial pressure and the mean ICP. Therefore, as the ICP
increases (or the mean arterial pressure decreases), the cerebral perfusion pressue decreases, and
when it is below about 50 mmHg, brain tissue can become ischaemic. This mechanism can lead to
ischaemia at a local level when compression from focal oedema or haematoma compromises blood
flow in the region of the lesion. Ischaemia and oedema then trigger release of excitatory
neurotransmitters and free radicals, causing further oedema, which further increases the ICP.
Systemic complications from trauma (eg hypotension, hypoxia) can also contribute to cerebral
ischaemia and are therefore often called ‘secondary brain insults’.
Excessive ICP initially causes global cerebral dysfunction. If excessive ICP is unrelieved, it can push
brain tissue across the tentorium or through the foramen magnum, causing herniation, which
significantly increases morbidity and mortality risks. Also, if the ICP increases to equal the mean
arterial pressure, the cerebral perfusion pressure becomes zero, resulting in complete brain
ischaemia, which rapidly leads to brain death; absent cranial blood flow can be used as one criterion
for brain death.
16.11
Subarachnoid haemorrhage is bleeding between the arachnoid mater and the pia mater. In general,
head trauma is the most common cause, but traumatic subarachnoid haemorrhage is usually
considered as a separate disorder. Spontaneous (primary) subarachnoid haemorrhage usually results
from ruptured aneurysms. A congenital intracranial saccular or berry aneurysm is the cause in about
85% of patients. Aneurysmal haemorrhage can occur at any age but is most common between the ages
of 40 years and 65 years. Less common causes are mycotic aneurysms, arteriovenous malformations
and bleeding disorders.
Blood in the subarachnoid space causes a chemical meningitis that commonly raises the intracranial
pressure for several days or even for a few weeks. Secondary vasospasm can cause focal brain
ischaemia; about 25% of patients develop signs of a transient ischaemic attack or ischaemic stroke.
Brain oedema is maximal and the risk of vasospasm and subsequent infarction (called ‘angry brain’)
is highest between 72 hours and 10 days after the bleed. Secondary acute hydrocephalus is also
common. A second rupture (rebleeding) sometimes occurs, most often within about 7 days of the first
bleed.
16.12
A subdural haematoma is blood between the dura mater and the pia-arachnoid mater. Acute subdural
haematomas, which are often caused by laceration of brain or cortical veins or avulsion of bridging
veins between the cortex and dural sinuses, often occur after falls or motor vehicle crashes. Oedema
can occur as the haematoma compresses brain tissue, resulting in signs of increased intracranial
pressure. The morbidity and mortality can be significant.
A chronic subdural haematoma can form and cause symptoms gradually, over a period of several
weeks after trauma. These haematomas are more common in elderly patients (especially those taking
antiplatelet drugs or anticoagulants), who might have thought that the head injury was relatively trivial
or might even have forgotten about it. In contrast to acute subdural haematoma, oedema and increased
intracranial pressure are unusual.
16.13
A brain abscess can result from direct extension of cranial infections (eg osteomyelitis, mastoiditis,
sinusitis, subdural empyema), penetrating head wounds (including neurosurgical procedures),
haematogenous spread (bacterial endocarditis, congenital heart disease with right-to-left shunt,
intravenous drug abuse) or from unknown causes. The bacteria involved are usually anaerobic and
sometimes mixed, often including anaerobic streptococci or Bacteroides species. Staphylococci are
common after cranial trauma, neurosurgery, or endocarditis. Enterobacteria are common with an ear
source. Fungi (eg Aspergillus) and protozoa (eg Toxoplasma gondii, particularly in HIV-infected
patients) can cause cerebral abscesses.
An abscess forms when an area of cerebral inflammation becomes necrotic and encapsulated by glial
cells and fibroblasts. Oedema around the abscess can increase the intracranial pressure. Symptoms
result from increased intracranial pressure and mass effects. Headache, nausea, vomiting, lethargy,
seizures, personality changes, papilloedema and focal neurological deficits develop over days to
weeks. Fever, chills and leucocytosis can develop before the infection is encapsulated, and then
subside.
Answer: B Astrocytoma
Astrocytomas are primary intracranial tumours derived from astrocyte cells of the brain. They can
arise in the cerebral hemispheres, in the posterior fossa, in the optic nerve and, rarely, in the spinal
cord. Well-differentiated astrocytomas comprise 25%–30% of cerebral gliomas. Although
astrocytomas have many different histological characteristics, the most common type is the well-
differentiated fibrillary astrocytoma. These tumours express glial fibrillary acidic protein (GFAP),
which possibly functions as a tumour suppressor, and is a useful diagnostic marker in a tissue biopsy.
In almost half of cases, the first symptom of an astrocytoma is the onset of a focal or generalised
seizure. Between 60% and 75% of patients will have recurrent seizures during the course of their
illness. Headache and signs of increased intracranial pressure (headache, vomiting) usually present
late in the disease course. In children, the tumour is usually located in the cerebellum and will present
with some combination of gait instability, unilateral ataxia and signs of increased intracranial
pressure. Children with astrocytoma usually have decreased memory, attention and motor abilities,
but unaffected intelligence, language and academic skills. When the tumour metastasises, it can spread
via the lymphatic system and cause death even when the primary tumour is well controlled.
Computed tomography (CT) or magnetic resonance imaging is necessary to characterise the anatomy
of this tumour (size, location, consistency). CT will usually show distortion of the third and lateral
ventricles, with displacement of the anterior and middle cerebral arteries. Histological diagnosis
with tissue biopsy will normally reveal an infiltrative character suggestive of the slow growing
nature of the tumour. The tumour can be cavitating, pseudocyst-forming or non-cavitating, and is
usually white-grey, firm, and almost indistinguishable from normal white matter.
16.15
Haemorrhagic lesions of the temporal lobe are typical of herpes simplex virus (HSV) infection. HSV
encephalitis occurs at any time of the year, has a bimodal age distribution, tending to affect patients
aged under 20 years or over 40 years, and is often fatal if left untreated. In acute encephalitis,
cerebral oedema and petechial haemorrhages occur throughout the hemispheres, brainstem,
cerebellum and, occasionally, the spinal cord. Direct viral invasion of the brain usually damages
neurones, sometimes with visible inclusion bodies. Severe infection, particularly if untreated, can
lead to cerebral haemorrhagic necrosis. Magnetic resonance imaging is sensitive for early HSV
encephalitis, showing oedema in the orbitofrontal and temporal areas, which HSV typically infects
preferentially.
16.16
Overall, risk factors for a chronic subdural haematoma include chronic alcoholism, epilepsy,
coagulopathy, arachnoid cysts, anticoagulant therapy (including aspirin), cardiovascular disease
(hypertension, arteriosclerosis), thrombocytopenia and diabetes. In younger patients, alcoholism,
thrombocytopenia, coagulation disorders and oral anticoagulant therapy have been found to be more
prevalent. Arachnoid cysts are associated with chronic subdural haematoma more commonly in
patients under the age of 40 years. In older patients, cardiovascular disease and arterial hypertension
are found to be more prevalent. Severe dehydration is a less commonly associated condition and is
found concurrently in only 2% of patients. Clinically, the presentation is often insidious, with
symptoms of decreased level of consciousness, balance problems, cognitive dysfunction and memory
loss, motor deficit (eg hemiparesis), headache or aphasia. Some patients present acutely, with a
seizure for example.
Neurological examination can reveal hemiparesis, papilloedema, hemianopia, or third cranial nerve
dysfunction, such as an unreactive dilated pupil or a laterally deviated eye with limited movement. In
patients aged 60 years or older, hemiparesis and reflex asymmetry are common presenting signs. In
patients younger than 60 years, headache is a common presenting symptom. Chronic subdural
haematomas are bilateral in 8.7%–32% of cases.
16.17
The features described in this scenario are suggestive of Guillain–Barré syndrome, which is the most
common acquired inflammatory neuropathy. Although the cause is not fully understood, it is thought to
be autoimmune. There are several variants. In some, demyelination predominates; others affect the
axons. In about two-thirds of patients, the syndrome begins 5 days to 3 weeks after an infectious
disease, surgery or vaccination. Infection is the trigger in over 50% of patients; common pathogens
include Campylobacter jejuni, enteric viruses, herpesviruses (including cytomegalovirus and those
causing infectious mononucleosis) and Mycoplasma species.
Flaccid weakness predominates in most patients, which is always more prominent than sensory
abnormalities and can be most prominent proximally. Relatively symmetric weakness with
paraesthesiae usually begins in the legs and progresses to the arms, but it occasionally begins in the
arms or head. In 90% of patients weakness is maximal at 3 weeks. Deep tendon reflexes are lost, but
sphincters are usually spared. Facial and oropharyngeal muscles are weak in over 50% of patients
with severe disease. Respiratory paralysis severe enough to require endotracheal intubation and
mechanical ventilation occurs in 5%–10%.
A few patients (possibly with a variant form) have significant, life-threatening autonomic dysfunction
causing fluctuations in the blood pressure, inappropriate antidiuretic hormone secretion, cardiac
arrhythmias, gastrointestinal stasis, urinary retention and pupillary changes. An unusual variant (the
Fisher variant) causes only ophthalmoparesis, ataxia and areflexia.
16.18
Diabetic neuropathy is probably the most common form of peripheral neuropathy in the Western
world. This man also has a ‘diabetic foot’ as a result of severe peripheral vascular atherosclerosis,
and the history of myocardial infaction is consistent with severe occlusive coronary atherosclerosis.
16.19
The location of the mass at the grey–white junction is typical of a metastasis. Secondary or metastatic
brain tumours originate from malignant tumours located primarily in other organs. Their incidence is
higher than that of primary brain tumours. The most frequent types of metastatic brain tumours
originate in the lung, skin, kidney, breast and colon. These tumour cells reach the brain via the
bloodstream. In this scenario the patient is most likely to have a lung cancer because she is a chronic
smoker.
16.20
The location of the haemorrhages suggests Duret haemorrhages. Duret haemorrhages are small areas
of bleeding in the ventral and paramedian parts of the upper brainstem (midbrain and pons). They
occur secondary to raised intracranial pressure with formation of a transtentorial pressure cone in
which the cerebellar tonsils are impacted in the foramen magnum by the high pressure. Kernohan’s
notch is a groove in the cerebral peduncle caused by this displacement of the brainstem against the
incisura of the tentorium in some cases. The resulting ipsilateral hemiparesis is a false localising
sign, known as the ‘Kernohan–Woltman syndrome’. This can either succeed or accompany temporal
lobe (uncal) herniation and subfalcial herniation secondary to a supratentorial mass.
The common causes are an acute haematoma, oedema following trauma, abscess or tumour. Duret
haemorrhages are demonstrated by computed tomography or magnetic resonance imaging, and they
usually indicate a fatal outcome. The mechanism is uncertain but is probably caused by the
displacement of the brainstem stretching and lacerating pontine perforating branches of the basilar
artery; venous infarction might play a role.
SECTION 17:
PHARMACOLOGY — ANSWERS
17.1
Cimetidine is a histamine H2-receptor anatgonist that inhibits the production of acid in the stomach. It
is largely used in the treatment of heartburn and peptic ulcers. Cimetidine is a known inhibitor of
many isozymes of the cytochrome P450 enzyme system (specifically CYP1A2, CYP2C9, CYP2C19
CYP2D6, CYP2E1 and CYP3A4). This inhibition forms the basis of the numerous drug interactions
that occur between cimetidine and other drugs. For example, cimetidine can decrease the metabolism
of some drugs, such as oral contraceptives. Cimetidine interferes with oestrogen metabolism,
enhancing oestrogen activity. This can lead to gynaecomastia. Adverse drug reactions are also found
to be relatively common with cimetidine. The development of longer-acting H2-receptor antagonists
with fewer adverse effects, such as ranitidine, has proved to be the downfall of cimetidine and, while
it is still used, it is not among the more widely used H2-receptor antagonists.
17.2
Ondansetron is a serotonin 5-HT3-receptor antagonist used mainly to treat nausea and vomiting
caused by chemotherapy. Its effects are thought to be on both peripheral and central nerves: one
mechanism is reduction of the activity of the vagus nerve, which is the nerve that activates the
vomiting centre in the medulla oblongata; another mechanism is blockage of serotonin receptors in the
chemoreceptor trigger zone. It does not have much effect on vomiting due to motion sickness. This
drug does not have any effect on dopamine receptors or on muscarinic receptors.
17.3
Diphenoxylate is an opiate agonist used for the treatment of diarrhoea. It acts by slowing down
intestinal contractions. It is a congener to the narcotic meperidine. This medication is therefore
potentially habit-forming, particularly in high doses or when long-term. Because of this,
diphenoxylate is manufactured and marketed as a combination drug with atropine (Lomotil®). This
pharmaceutical strategy is designed to discourage abuse, because the anticholinergic effect of
atropine will cause severe weakness and nausea if the standard dosage is exceeded.
17.4
17.5
Psyllium is a laxative belonging to the subclass of bulk-forming laxatives. It holds water in the stool.
Its efficacy is not well documented but it is believed to provide some benefit in irritable bowel
syndrome.
17.6
17.7
Metoclopramide is a potent dopamine-receptor antagonist used for its anti-emetic and prokinetic
properties. It is therefore primarily used to treat nausea and vomiting, and to facilitate gastric
emptying in patients with gastric stasis. It appears to bind to dopamine D2 receptors, where it is a
receptor antagonist, and it is also a mixed 5-HT3-receptor antagonist/5-HT4-receptor agonist. The
anti-emetic action of metoclopramide is due to its antagonist activity at D2 receptors in the
chemoreceptor trigger zone (CTZ) in the central nervous system. This action prevents nausea and
vomiting triggered by most stimuli. At higher doses, 5-HT3-antagonist activity can also contribute to
the anti-emetic effect. The prokinetic activity of metoclopramide is mediated by muscarinic activity,
D2 receptor-antagonist activity and 5-HT4-receptor agonist activity. The prokinetic effect itself might
also contribute to the anti-emetic effect.
17.8
Answer: D Lowers the surface tension of the stool, facilitating penetration of water and fats
Docusate sodium (dioctyl sodium sulphosuccinate) surfactant is used as a laxative and stool softener.
It is given to make stools softer and easier to pass. It is used to treat constipation due to hard stools,
in painful anorectal conditions such as haemorrhoids, and for people who should avoid straining
during bowel movements. Of note is that the effect of docusate sodium might not necessarily be
exclusively due to its surfactant properties. Perfusion studies have suggested that docusate sodium
inhibits fluid absorption or stimulates secretion in the jejunum. While the use of docusate sodium is
widespread, data to support its efficacy in treating chronic constipation is lacking. Also, although
more research is needed, long-term use of docusate sodium seems to decrease levels of magnesium
and potassium in the blood.
17.9
Loperamide is an opioid-receptor agonist and acts on the mu opioid receptors in the myenteric plexus
of large intestine; it does not affect the central nervous system, unlike other opioids. It works by
decreasing the activity of the myenteric plexus, which decreases the motility of the circular and
longitudinal smooth muscles of the intestinal wall. This increases the amount of time substances stay
in the intestine, allowing for more water to be absorbed out of the faecal matter. Loperamide also
decreases colonic mass movements and suppresses the gastrocolic reflex. Loperamide does not cross
the blood–brain barrier and has no analgesic properties or addictive potential. Tolerance in response
to long-term use has not been reported.
17.10
Sufentanil is a synthetic opioid analgesic drug that is approximately 5–10 times more potent than
fentanyl. The main use of this medication is in operating suites and critical care units, where pain
relief is required for a short period of time. It also has sedative properties and this makes it a good
analgesic component of anaesthetic regimes during surgery.
17.11
17.12
Atorvastatin is a member of the drug class of statins, used for lowering cholesterol (ie a
hypolipidaemic agent) in people with hypercholesterolaemia and so for preventing cardiovascular
disease. The mode of action of statins is inhibition of 3-hydroxy-3-methylglutaryl-coenzyme A
(HMG-CoA) reductase. This enzyme is needed by the body to make cholesterol. Atorvastatin causes
cholesterol to be lost from low-density lipoprotein (LDL), but also reduces the concentration of
circulating LDL particles. Apolipoprotein B concentration falls substantially during treatment with
atorvastatin. Atorvastatin’s ability to lower LDL is thought to be due to a reduction in very-low-
density lipoprotein (VLDL), which is a precursor of LDL. Also, atorvastatin can increase the number
of LDL receptors on the surface of cell membranes, and so increase the breakdown of LDL.
Atorvastatin can also produce slight to moderate increases in high-density lipoprotein (HDL), and
slight to moderate decreases in triglycerides. Both of these effects are benefical in a patient with a
poor lipid profile.
17.13
Because streptokinase is a bacterial product the body will build up an immunity to it. It is
recommended that this medication should not be used again after 4 days from the first administration
because it might not be as effective and can also cause an allergic reaction. For this reason, it is
usually only given for a person’s first heart attack.
17.14
17.15
Answer: A Antifungal
Griseofulvin is an antifungal drug. It is used in both animals and humans to treat ringworm infections
of the skin and nails. It is derived from the mould Penicillium griseofulvum. It is administered orally.
It binds to keratin in keratin precursor cells and makes them resistant to fungal infections. It is only
when hair or skin is replaced by the keratin–griseofulvin complex that the drug reaches its site of
action. Griseofulvin will then enter the dermatophyte through energy-dependant transport processes
and bind to fungal microtubules. This alters the processing for mitosis and also the underlying
information for deposition of fungal cell walls. Known side-effects of griseofulvin include hives, skin
rashes, confusion, dizziness, diarrhoea, fatigue, headache, impairment of performance of routine
activities, inability to fall or stay asleep, nausea, oral thrush, upper abdominal pain, vomiting,
swelling, itching, tingling in the hands or feet and loss of taste sensation.
17.16
Mexiletine belongs to the class IB anti-arrhythmic group of medicines. It is used to treat ventricular
arrhythmias. It slows nerve impulses in the heart and makes the heart tissue less sensitive. Dizziness,
heartburn, nausea, nervousness, trembling and unsteadiness are common side-effects. It is available in
injection and capsule form. Class IB anti-arrhythmics decrease the duration of the action potential by
shortening the repolarisation phase. This is achieved by blocking sodium channels.
17.17
Nadolol is a non-selective beta-blocker used in the treatment of high blood pressure and angina.
Nadolol is non-polar and hydrophobic, with low lipid solubility. It is a beta-specific sympatholytic
which non-selectively blocks beta-1 adrenergic receptors, which are mainly located in the heart,
inhibiting the effects of the catecholamines adrenaline (epinephrine) and noradrenaline
(norepinephrine) and decreasing the heart rate and blood pressure. It also blocks beta-2 adrenergic
receptors, which are located in bronchial smooth muscle, causing bronchoconstriction. By binding
beta-2 receptors in the juxtaglomerular apparatus, nadolol inhibits the production of renin, thereby
inhibiting angiotensin II and aldosterone production. Nadolol therefore inhibits the vasoconstriction
and water retention due to angiotensin II and aldosterone, respectively. It also impairs
atrioventricular node conduction and decreases the sinus rate. Nadolol can also increase plasma
triglycerides and decrease high-density lipoprotein- (HDL-) cholesterol levels.
17.18
Answer: C It induces the release of stored factor VIII and von Willebrand factor
Desmopressin is a synthetic drug that mimics the action of antidiuretic hormone. It can be taken
nasally, intravenously or orally as a recently developed pill. Desmopressin is used to reduce urine
production in patients with central diabetes insipidus and to promote the release of von Willebrand
factor and factor VIII in patients with coagulation disorders such as type I von Willebrand’s disease,
mild haemophilia A and thrombocytopenia (which occurs after prolonged surgery on
cardiopulmonary bypass). Desmopressin is not effective in the treatment of haemophilia B or severe
haemophilia A.
17.19
Answer: C Anxiolytic
17.20
Answer: B Penicillin
17.21
Answer: E Vasodilators
Sodium nitroprusside is a potent peripheral vasodilator that affects both arterioles and venules. It is
often administered intravenously to patients who are experiencing a hypertensive emergency.
‘Nitroprusside’ in fact is an anion that is usually available as the dihydrate of its disodium salt. It
reduces both total peripheral resistance as well as venous return, so decreasing both preload and
afterload. For this reason it can be used in severe cardiogenic heart failure where this combination of
effects can act to increase cardiac output. In situations where cardiac output is normal, the effect is to
reduce the blood pressure.
Nitroprusside is light-sensitive and breaks down in sunlight, producing cyanide. Despite its toxicity,
however, nitroprusside is still used because it remains an effective drug in certain clinical
circumstances such as malignant hypertension or for rapid control of blood pressure during vascular
surgery and neurosurgery. Its mechanism of action appears to be liberation of nitric oxide as it is
metabolised in the erythrocyte, converting haemoglobin to cyanomethaemaglobin. Nitroprusside also
releases cyanide ions, which are converted in the liver to thiocyanate by the enzyme rhodanase, a
reaction which requires a sulphur donor such as thiosulphate. Thiocyanate is then excreted by the
kidney. In the absence of sufficient thiosulphate, cyanide ions can quickly reach toxic levels. The half-
life of nitroprusside is less than 10 minutes, although thiocyanate has an excretion half-life of several
days. The duration of treatment should not exceed 72 hours and plasma concentrations of thiocyanate
should be monitored.
17.22
Spironolactone also has anti-androgen activity via binding to the androgen receptor and thus
preventing it from interacting with dihydrotestosterone, and so it can also be used to treat hirsutism
and is a common component in hormone therapy for male-to-female transgender people. It is also
used for treating hair loss and for acne in women.
17.23
Abciximab (previously known as ‘c7E3 Fab’) is a platelet aggregation inhibitor that is mainly used
during and after coronary artery procedures such as angioplasty to prevent platelets from sticking
together and causing thrombus formation in the coronary artery. Its mechanism of action is inhibition
of glycoprotein IIb/IIIa. Although abciximab has a short plasma half-life due to its strong affinity for
its receptor on the platelets, it can occupy some receptors for weeks. In practice, platelet aggregation
gradually returns to normal about 24–48 hours after discontinuation of the drug. Abciximab is made
from the Fab fragments of an immunoglobulin that targets the glycoprotein IIb/IIIa receptor on the
platelet membrane. It is indicated for use in people who are undergoing percutaneous coronary
intervention (angioplasty, with or without stent placement). The use of abciximab in this setting is
associated with a decreased incidence of ischaemic procedure-related complications and a decreased
need for repeated coronary artery revascularisation in the first month after the procedure. Many of the
side-effects of abciximab are due to its antiplatelet effects. This includes an increased risk of
bleeding. The most common type of bleeding due to abciximab is gastrointestinal haemorrhage.
Thrombocytopenia is a rare but known serious risk. Abciximab-induced thrombocytopenia can
typically be treated with transfusion of platelets. Abciximab has a plasma half-life of about 10
minutes, with a second-phase half-life of about 30 minutes. However, its effects on platelet function
can be seen for up to 48 hours after the infusion has finished, and low levels of glycoprotein IIb/IIIa
receptor blockade are present for up to 15 days after the infusion is stopped.
17.24
Fluconazole is a triazole antifungal drug used in the treatment and prevention of superficial and
systemic fungal infections. Like other imidazole- and triazole-class antifungals, fluconazole inhibits
the fungal cytochrome P450 enzyme, 14-demethylase. Mammalian demethylase activity is much less
sensitive to fluconazole than fungal demethylase. This inhibition prevents the conversion of lanosterol
to ergosterol, an essential component of the fungal cell wall, and subsequent accumulation of 14-
methyl sterols. Fluconazole is primarily fungistatic, but can be fungicidal against certain organisms in
a dose-dependent manner.
17.25
17.26
17.27
17.28
Trimethoprim is a bacteriostatic antibiotic that is mainly used in the prophylaxis and treatment of
urinary tract infections. Trimethoprim acts by interfering with the action of bacterial dihydrofolate
reductase, inhibiting synthesis of tetrahydrofolic acid. Tetrahydrofolic acid is an essential precursor
in the de novo synthesis of the DNA nucleosides thymidine and uridine. Bacteria are unable to take up
folic acid from the environment (ie the infected host) and are thus dependent on their own de novo
synthesis. Inhibition of the enzyme starves the bacteria of two bases necessary for DNA replication
and transcription.
17.29
Tazobactam is a compound which inhibits the action of bacterial beta-lactamases. It is added to the
extended-spectrum beta-lactam antibiotic piperacillin to produce Tazocin®. It broadens the spectrum
of piperacillin by making it effective against organisms that express beta-lactamase and would
normally degrade piperacillin. Tazobactam sodium is a derivative of the penicillin nucleus and is a
penicillanic acid sulphone.
17.30
17.31
Methimazole is an antithyroid drug similar in action to propylthiouracil, and is part of the thioamide
group. Methimazole is a drug used to treat hyperthyroidism. It is also taken before thyroid surgery or
radioactive iodine therapy, to lower thyroid hormone levels and minimise the effects of thyroid
manipulation. Thioamides inhibit several steps in the synthesis of thyroid hormones, including the
addition of iodide to thyroglobulin by the enzyme thyroperoxidase, a necessary step in the synthesis of
thyroxine, and by inhibiting the enzyme 5’-deiodinase which converts T4 to T3. Notably, they do not
inhibit the action of the sodium-dependent iodide transporter located on the basolateral membranes of
follicular cells. Inhibition of this step requires competitive inhibitors such as perchlorate and
thiocyanate.
17.32
Cyclophosphamide is a nitrogen mustard alkylating agent, used to treat various types of cancer and
some autoimmune disorders. It is a ‘prodrug’, being converted in the liver to active forms that have
chemotherapeutic activity. The main use of cyclophosphamide is in combination with other
chemotherapy agents in the treatment of lymphomas, some forms of leukaemia and some solid
tumours. It is a chemotherapy drug that works by slowing or stopping cell growth. It also works by
decreasing the immune system’s response to various diseases.
The main effect of cyclophosphamide is due to its metabolite, phosphoramide mustard. This
metabolite is only formed in cells which have low levels of aldehyde dehydrogenase (ALDH).
Phosphoramide mustard forms DNA cross-links between (inter-strand) and within (intra-strand)
DNA strands at guanine N-7 positions. This leads to cell death.
Answer: C Piperacillin
17.34
Answer: A Azlocillin
Azlocillin is an acylampicillin antibiotic with an extended spectrum of activity and greater in vitro
potency than the carboxypenicillins. Azlocillin is similar to mezlocillin and piperacillin. It
demonstrates antibacterial activity against a broad spectrum of bacteria, including Pseudomonas
aeruginosa and, in contrast to most cephalosporins, also exhibits activity against enterococci.
17.35
Three antibiotics are effective against Clostridium difficile. Metronidazole 500 mg orally three times
daily is the drug of choice, because of superior tolerability, lower price and comparable efficacy.
Oral vancomycin 125 mg four times daily is second-line therapy, but is avoided due to theoretical
concerns of converting intestinal flora into vancomycin-resistant organisms. However, it is used in
the following cases: when there has been no response to oral metronidazole; when the organism is
resistant to metronidazole; when the patient is allergic to metronidazole; when the patient is either
pregnant or younger than 10 years of age; and when the patient is critically ill because of C. difficile
diarrhoea (the duration of diarrhoea is reduced to 3 days, compared with 4.6 days with
metronidazole). Vancomycin must be administered orally because intravenous administration does not
achieve minimum therapeutic concentrations in the gut lumen. Thirdly, the use of linezolid might also
be considered.
17.36
Linezolid is a synthetic antibiotic, the first of the oxazolidinone class, used for the treatment of
infections caused by multiresistant bacteria, including streptococci and methicillin-resistant
Staphylococcus aureus (MRSA). It was the first commercially available oxazolidinone antibiotic
and is usually reserved for the treatment of serious bacterial infections where older antibiotics have
failed due to antibiotic resistance. Conditions such as skin infections or nosocomial pneumonia where
methicillin or penicillin resistance is found are indications for linezolid use. Compared with the older
antibiotics it is quite expensive.
The drug works by inhibiting the initiation of bacterial protein synthesis. Linezolid is effective against
Gram-positive pathogens, notably Enterococcus faecium, S. aureus, Streptococcus agalactiae,
Streptococcus pneumoniae and Streptococcus pyogenes. It has almost no effect on Gram-negative
bacteria and is only bacteriostatic against most enterococci. Linezolid has also been used to treat
tuberculosis.
17.37
Linezolid works on the initiation of protein synthesis. It does this by stopping the 30S and 50S
subunits from binding together. Linezolid binds onto the 50S subunit, close to the peptidyl transferase
and chloramphenicol binding sites. This then stops the interaction with the 30S subunit.
17.38
Answer: D Erythromycin
17.39
Erythromycin prevents bacteria from growing by interfering with their protein synthesis.
Erythromycin binds to the 23S rRNA molecule in the 50S subunit of the bacterial ribosome, blocking
the exit of the growing peptide chain, thus inhibiting the translocation of peptides.
17.40
Quinolones are bactericidal drugs, actively killing bacteria. Quinolones inhibit the bacterial DNA
gyrase or the topoisomerase IV enzyme, thereby inhibiting DNA replication and transcription.
Quinolones can enter cells easily and therefore are often used to treat intracellular pathogens such as
Legionella pneumophila and Mycoplasma pneumoniae. DNA gyrase is the target for many Gram-
negative bacteria; topoisomerase IV is the target for many Gram-positive bacteria.
17.41
Rifampicin is a bactericidal drug of the rifamycin group. It is a semisynthetic compound derived from
Amycolatopsis mediterranei (formerly known as Streptomyces mediterranei). Rifampicin is
typically used to treat mycobacterial infections, including tuberculosis and leprosy; and also has a
role in the treatment of methicillin-resistant Staphylococcus aureus (MRSA) infections, in
combination with fusidic acid. It is used in prophylactic therapy against Neisseria meningitidis (ie
meningococcal) infection.
Rifampicin inhibits DNA-dependent RNA polymerase in bacterial cells by binding its beta subunit,
thus preventing transcription of messenger RNA (mRNA) and subsequent translation to proteins. Its
lipophilic nature makes it a good drug for treating the meningitis form of tuberculosis, which requires
distribution to the central nervous system and penetration through the blood–brain barrier.
17.42
Fusidic acid is an antibiotic that is used particularly for eye and skin infections. It works by
interfering with bacterial protein synthesis, specifically by preventing the translocation of elongation
factor G (EF-G) from the ribosome, although it works only on Gram-positive bacteria such as
Staphylococcus aureus, streptococci and Corynebacterium minutissimum. Because it primarily
inhibits their reproduction (as opposed to killing them directly) it is bacteriostatic.
17.43
17.44
Isoniazid reaches therapeutic concentrations in serum, cerebrospinal fluid, and within caseous
granulomas. Isoniazid is metabolised in the liver by acetylation. There are two forms of the enzyme
responsible for acetylation, so that some patients metabolise the drug more quickly than others. The
metabolites are excreted in the urine. Doses do not usually have to be adjusted in patients who are in
renal failure. Isoniazid is bactericidal to rapidly-dividing mycobacteria, but is bacteriostatic if the
mycobacterium is slow-growing.
17.45
Folinic acid or leucovorin, generally administered as calcium folinate (or leucovorin calcium), is an
adjuvant drug used in cancer chemotherapy involving the drug methotrexate. It is also used in
synergistic combination with the chemotherapeutic agent, 5-fluorouracil. (Note that folinic acid is not
the same as folic acid.) Folinic acid is a 5-formyl derivative of tetrahydrofolic acid. It is readily
converted to other reduced folic acid derivatives (eg tetrahydrofolate). Because it does not require
the action of dihydrofolate reductase for its conversion, it will be unaffected by inhibition of this
enzyme by drugs such as methotrexate. This therefore allows for purine/pyrimidine synthesis to occur;
so normal DNA replication and RNA transcription processes can proceed.
Folinic acid is administered at the appropriate time following methotrexate as part of a total
chemotherapeutic plan, where it can ‘rescue’ bone marrow and gastrointestinal mucosal cells from
methotrexate. There is no apparent effect on pre-existing methotrexate-induced nephrotoxicity,
however. While not specifically an antidote for methotrexate, folinic acid can also be useful in the
treatment of acute methotrexate overdose.
Folinic acid is used in combination with 5-fluorouracil for the treatment of colon cancer. In this case,
folinic acid is not used for ‘rescue’ purposes; rather, it enhances the effect of 5-fluorouracil on the
inhibition of thymidylate synthase. Folinic acid is also sometimes used to prevent the toxic effects of
high doses of antimicrobial dihydrofolate reductase inhibitors such as trimethoprim and
pyrimethamine.
17.46
Answer: E FOLFOX
FOLFOX is a chemotherapy regimen used for the treatment of colorectal cancer, made up of the
following drugs:
Adjuvant treatment in patients with stage III colon cancer is recommended for 12 cycles, every 2
weeks. The recommended dose schedule (given every 2 weeks) is as follows:
Day 2: Leucovorin 200 mg/m intravenous infusion over 120 minutes, followed by 5-FU 400 mg/m
intravenous bolus given over 2–4 minutes, followed by 5-FU 600 mg/m intravenous infusion in 500
ml 5% dextrose (recommended) as a 22-hour continuous infusion.
17.47
Answer: A ABVD
ABVD is a chemotherapy regimen used in the first-line treatment of Hodgkin’s lymphoma. It consists
of concurrent treatment with the chemotherapy drugs, adriamycin, bleomycin, vinblastine and
dacarbazine. One cycle of ABVD chemotherapy is typically given over 4 weeks, with two doses in
each cycle (on day 1 and day 15). All four of the chemotherapy drugs are given intravenously. ABVD
chemotherapy is usually given in an outpatient setting.
The total number of cycles given depends upon the stage of the disease and on how well the patient
tolerates chemotherapy. Doses might be delayed because of neutropenia, thrombocytopenia or other
side-effects.
17.48
Answer: C CHOP
CHOP is the acronym for a chemotherapy regimen used in the treatment of non-Hodgkin’s lymphoma,
comprising cyclophosphamide, hydroxyrubicin (adriamycin), Oncovin® (vincristine) and prednisone.
This regimen can also be combined with the monoclonal antibody rituximab if the lymphoma is of B-
cell origin (R-CHOP or CHOPR). Typically, courses are administered at an interval of 3 weeks. A
staging computed tomographic scan is generally performed after three cycles to assess whether the
disease is responding to treatment. In patients with a history of cardiovascular disease, the
doxorubicin (which is cardiotoxic) is often considered to be too great a risk and is omitted from the
regimen. The combination is then referred to as ‘COP’ or ‘CVP’.
17.49
Methotrexate competitively and reversibly inhibits dihydrofolate reductase (DHFR), an enzyme that
is part of the folate synthesis metabolic pathway. The affinity of methotrexate for DHFR is about one
thousand times that of folate for DHFR. Dihydrofolate reductase catalyses the conversion of
dihydrofolate to the active tetrahydrofolate. Folic acid is needed for the de novo synthesis of the
nucleoside thymidine, required for DNA synthesis. Methotrexate, therefore, inhibits the synthesis of
DNA, RNA, thymidylate and proteins.
Methotrexate acts specifically during DNA and RNA synthesis, and it is therefore cytotoxic during the
S-phase of the cell cycle. Logically, it therefore has a greater toxic effect on rapidly dividing cells
(such as malignant and myeloid cells, but also gastrointestinal and oral mucosal cells), which
replicate their DNA more frequently, and thus inhibits the growth and proliferation of both cancerous
and non-cancerous cells, leading to the therapeutic and toxic effects respectively.
Lower doses of methotrexate have been shown to be very effective for the management of rheumatoid
arthritis, Crohn’s disease and psoriasis. In these cases inhibition of DHFR is not thought to be the
main mechanism, which is thought to be the inhibition of enzymes involved in purine metabolism,
leading to accumulation of adenosine, or the inhibition of T-cell activation and suppression of
intercellular adhesion molecule expression by T cells.
17.50
(The number in bold indicates section and the number in italics indicates question number)
A
Abciximab 17.23
ABVD 17.47
Acetylsalicylic acid ingestion and bleeding 6.15
Actinomycosis 4.20
Acute aortic dissection 7.12
Acute appendicitis 10.5
Acute mastitis 13.4
Acute myelogenous leukaemia 11.2
Acute pericarditis 7.15
Acute rejection 2.37
Acute small-bowel obstruction due to adhesions 10.20
Acute tubular necrosis 9.9, 9.14
Adrenal cortical carcinoma 12.13
Adult respiratory distress syndrome 8.9
Aetiology of acute adrenal insufficiency 12.11
Aetiology of adenocarcinoma of oesophagus 3.43
Aetiology of bladder cancer 3.48
Aetiology of chylothorax 8.15
Aflatoxin 10.12
Alpha-fetoprotein 10.4
Alpha-1-antitrypsin deficiency 8.5
Amikacin 4.30
Ammonia synthesis in hypokalaemia 5.28
Anaemia in parasitic infestation 4.24
Anaemia of chronic disease 11.11
Aneurysmal bone cyst 15.11
Ankylosing spondylitis 15.14
Antibody-mediated (type II) hypersensitivity reaction 2.3
Antifungal agents 4.40
Anti-inflammatory action of aspirin 2.19
Antinuclear antibody test 2.22
Antiphospholipid syndrome 6.23
Antithrombin III deficiency 6.6
Apoptosis 1.6
Arachidonic acid 2.47
Arrhenoblastoma 3.11
Asbestosis 8.7
Association of malignant potential and tumour size 3.39
Astrocytoma 16.14
Atorvastatin 17.12
ATP depletion 1.24
Atrial fibrillation 7.4
Atrophy 1.3, 1.8
Autophagic vacuoles 1.32
Autosplenectomy 11.7
Azlocillin 17.34
B
Bacteroides fragilis 4.7
Balanitis 14.1
Benign giant-cell tumour 15.5
Beta-thalassaemia 11.15
Bladder cancer staging 3.47
Bladder exstrophy 9.17
Blastomycosis 4.14
Brain abscess 16.13
BRCA1 and BRCA2 genes 3.13, 3.23
Breast abscess 13.9
Breast cancer staging 3.17, 3.20
Breast cyst 13.5
Breast implants 13.7
Bronchial carcinoid 8.19
Bronchoalveolar carcinoma 8.10
Brucellosis 4.19
Bursitis 15.15
Buspirone 17.19
C
CA-125 3.25
Calcitonin (Tumour marker for medullary carcinoma thyroid gland) 12.1
Calcium metabolism 5.20
Cancer staging 3.16
Carbachol 17.14
Carbon tetrachloride poisoning 1.23
Carcinoembryonic antigen 3.24
Carcinoma of oral cavity 3.29
Carpal tunnel syndrome 15.18
Caseous necrosis 1.2
Causative organism for infective endocarditis in intravenous drug abusers 4.16
Causative organism for infective endocarditis of native damaged or abnormal valves 4.23
Causes of decreased anion gap 5.21
Causes of euvolaemic hyponatraemia 5.16
Causes of hyperkalaemia 5.7
Causes of hyperphosphataemia 5.15
Causes of increased anion gap 5.22
Causes of non-gonococcal urethritis 9.18
Ceftriaxone 17.25
Cell types in chronic inflammation 2.31
Cells of mononuclear phagocyte system 2.34
Cerebral abscess 16.6
Cervical intraepithelial neoplasia 13.18
Chemotaxis 2.33
Chemotherapy-induced immunosuppression 4.37
Cholera toxin 4.8
Cholesterol crystals in haemophiliac joints 1.11
Chondrosarcoma 15.7
CHOP 17.48
Chronic lymphocytic leukaemia 11.3
Chronic subdural haematoma 16.16
Cimetidine 17.1
Ciprofloxacin 4.29
Cisplatin 17.50
Clinical features of hypomagnesaemia 5.26
Clinical staging of testicular seminoma 14.14
Clostridium botulinum food poisoning 4.9
Coagulase 4.39
Coagulative necrosis 1.35
Codman’s triangle 3.33
Common cancers in organ transplant recipients 3.38
Common primary site for tumour in adults 3.37
Complement C3b 2.20
Complement C5a 2.18
Complications of cystic fibrosis 8.1
Complications of fine-needle aspiration of the lung 8.18
Complications of wound healing 1.26
Concomitant hyperplasia and hypertrophy 1.34
Conn syndrome 12.8
Coronary atherosclerosis after cardiac transplantation 7.17
Corticosteroid-induced osteoporosis 15.12
Craniopharyngioma 12.10
Creatine kinase-MB 7.16
CREST syndrome 10.17
Crohn’s disease 10.3
Cryptosporidium parvum 4.22
Cyclophosphamide 17.32
Ciclosporin 2.29
Cytokeratins 1.16
D
D-Dimer 6.17
D-Dimer test 6.10
Dehydration 5.4
Dermoid cyst 3.32
Desmopressin 17.18
Diabetic neuropathy 16.18
Diagnosis of primary syphilis 14.4
Diapedesis 1.39
Dicloxacillin 17.20
Diphenoxylate 17.3
Diphtheria 4.38
Disseminated intravascular coagulation 6.1, 6.7, 6.22
Docusate sodium 17.8
Doxycycline 4.27
Drug of choice for pulmonary anthrax 4.12
Drugs causing hyperkalaemia 5.13
Duodenal ulcer 10.16
Duplex scan 7.14
Dupuytren’s contracture 15.17
Duret haemorrhages 16.20
E
ECG changes in acute myocardial infarction 7.5
ECG findings in hypokalaemia 5.11
ECG manifestations of hyperkalaemia 5.9
Echocardiography 7.13
Ectopic ACTH production 12.18
Effects of severe metabolic alkalosis 5.24
Eisenmenger’s syndrome 7.20
Embryonal carcinoma 14.5
Endocervical polyp 13.14
Endometriosis 13.16
Endotoxin 4.1
Eosinophils 2.43
Epidural haematoma 16.9
Epithelioid cells 1.36
Epstein–Barr virus 4.25
Erythromycin 17.38
Escherichia coli 4.2
Ethambutol 17.43
Etoposide 17.26
Events of inflammation 2.4
Ewing’s sarcoma 15.9
Extracellular fluid volume expansion 5.18
F
Factor IX deficiency 6.13
Factor V 6.32
Factor VII 6.28
Factor VII deficiency 6.18
Factor X 6.37
Factor XI 6.39
Factor XII deficiency 6.9
Factor XIII 6.31
Factors influencing wound healing 1.27, 1.28
Fat embolism 7.7
Fat necrosis 1.7
Features of acute inflammation 2.16
Features of hypermagnesaemia 5.14
Features of malignant neoplasms 3.6
Features of neoplasm 3.3
Fibrinolytic agents 6.19
Fibroadenoma 13.3
Fibrothecoma 13.15
Finasteride 14.18
Fluconazole 17.24
Fluid resuscitation 5.3
FOLFOX 17.46
Folinic acid 17.45
Folliculitis 4.10
Food allergy 2.21
Free radical formation 1.10
Free radical injury 1.22
Fresh frozen plasma 6.25
Fusidic acid 17.42
G
Gamma interferon 2.11
Generation of metabolic alkalosis 5.23
Gestational trophoblastic disease 13.19
Giant cells 2.13
Glioblastoma multiforme 16.3
Glucagonoma 12.4
Gout 15.1
Granulation tissue 2.32
Grey platelet syndrome 6.40
Griseofulvin 17.15
Guillain–Barré syndrome 16.17
Gynaecomastia 13.13
H
Haemolytic disease of the newborn 2.39
Haemophilia 6.12
Haemophilia A 6.4, 6.21
Haemophilia C 6.38
Hageman factor 1.30
Hamartoma 1.37
Healing after myocardial infarction 1.40
Heparin 6.34
Hepatic angiosarcoma 3.12
Hepatitis A 10.1
Hepatocellular carcinoma 10.9
Herpes simplex encephalitis 16.15
Hirschsprung’s disease 10.7
Histological subtypes of testicular germ cell tumours 14.10
Histopathology of osteosarcoma 3.4
HLA-DR antigens and tissue typing 2.24
Hodgkin’s lymphoma 11.9
Hyaline degeneration 1.31
Hydrogen peroxide 2.14
Hyperacute rejection 2.30
Hypercalcaemia as part of a paraneoplastic syndrome 3.14
Hyperprolactinaemia 12.6
Hypersegmented neutrophils 11.19
Hyperthyroidism 12.5
Hypertrophy 1.5, 1.17
Hypervolaemic hypernatraemia 5.30
Hyponatraemia 5.10
Hypoparathyroidism as a cause of hypocalcaemia 5.19
Hypospadias 14.3
Hypovolaemic shock 7.1
I
ICAM-1 & VCAM-1 2.5
Idiopathic thrombocytopenic purpura 11.18
Immune response to parasitic infestation 2.25
Imperforate hymen 13.20
Infectious mononucleosis 11.5
Infectious mononucleosis (Epstein–Barr virus) 4.3
Interleukin-1 2.28
Interleukin-10 2.45
Interleukin-5 2.46
Interleukin-6 2.44
Intraductal papilloma 13.11
Isoniazid 17.44
K
Karyorrhexis and cell fragmentation 1.20
k-ras mutations 3.2
Kupffer cells 2.49
L
Lactobacillus 4.4
Lambert–Eaton myasthenic syndrome 3.46
Langerhans giant cells 2.35
Lansoprazole 17.6
Leukaemoid reaction 11.10
Leukotrienes 2.48
Liddle syndrome 5.12
Linezolid 17.36
Lipochrome 1.15
Lipofuscin 1.25
Liposarcoma 3.19
Liquefactive necrosis 1.1, 1.13, 16.4
Lobar pneumonia 8.2
Lobular carcinoma of breast 13.1
Lobular hyperplasia in the breast 1.14
Loperamide 17.9
Low-molecular-weight heparins 6.36
Lung abscess 8.6
M
Macrophages in granulomatous inflammation 2.17
Mallory bodies 10.2
Marantic endocarditis 7.8
Massive splenomegaly 11.13
Mechanism for pathogenesis in Pseudomonas infection 4.15
Mechanism of action of erythromycin 17.39
Mechanism of action of heparin 6.35
Mechanism of action of linezolid 17.37
Mechanism of action of quinolones 17.40
Meckel’s diverticulum 10.19
Medullary carcinoma of breast 13.12
Medulloblastoma 3.31
Megaloblastic anaemia 11.4
Membranous glomerulonephritis 9.13
Meningioma 16.2
Meropenem 4.26
Mesothelioma and asbestos exposure 3.26
Metabolic alkalosis 5.1, 5.8
Metabolic response to trauma 1.9
Metaplasia 1.12
Metastatic brain tumour 16.19
Metastatic carcinoma and leucoerythroblastosis 11.6
Metastatic prostatic carcinoma 15.10
Methicillin 4.33
Methimazole 17.31
Methotrexate 17.49
Methylcellulose 17.4
Metoclopramide 17.7
Mexilitene 17.16
Micronodular cirrhosis 10.18
Minimal-change disease 9.8
Mismatched blood transfusion 2.38
Mithramycin 17.30
Monocytes 2.41
Multiple endocrine neoplasia type IIA 12.12
Multiple myeloma 15.8
Multiple myeloma 9.2
Mycosis fungoides 11.20
Mycotic aneurysm 7.19
Myeloproliferative disorder 6.5
Myoglobinuria 9.20
N
Nadolol 17.17
Nephroblastoma 3.22
Nephrotic syndrome 9.4
Neural tube defect 16.5
Neutrophils 2.2, 2.42
Nocardia asteroides 8.17
Nodal staging in NSCLC 3.42
Nodal staging of testicular seminoma 14.11
Non-functioning adrenal adenoma 12.7
Non-neoplastic colonic polyps 10.6
O
Oliguria 5.6
Oncogenes 3.34
Ondansetron 17.2
Oedema in nephrotic syndrome 5.2
Organisation of haematoma 1.38
Organisms responsible for infective endocarditis 7.2
Osteoarthritis 15.6
Osteochondroma 15.13, 3.30
Osteoid osteoma 15.3
Oval fat bodies 9.19
Ovarian serous adenocarcinoma 13.17
Overall staging for prostate cancer 14.17
P
Paget’s disease of bone 15.2, 3.27
Paget’s disease of breast 13.10
Pain in acute inflammation 2.12
Pap smear 3.21
Papillary carcinoma (thyroid gland) 12.2
Papillary carcinoma of thyroid 3.28
Paraneoplastic syndromes 3.35
Parathyroid adenoma 12.15
Pathophysiology of pulmonary embolism 7.6
Pathophysiology of traumatic brain injury 16.10
Percutaneous needle biopsy of kidney 9.10
Perforin 2.50
Pernicious anaemia and atrophic gastritis 11.16
Persistent truncus arteriosus 7.18
Peyronie’s disease 14.9
Phaeochromocytoma 12.3, 7.10
Phimosis 14.2
Phosphate metabolism 5.25
Phyllodes tumour 13.6
Piperacillin 17.33
Plasma cells 11.17
Platelet abnormalities 6.2
Platelet-derived growth factor 2.7
Pleomorphic adenoma 3.8
Pneumocystis carinii pneumonia 4.32
Polycythaemia vera 11.8
Polymyalgia rheumatica 15.4
Poorly differentiated lymphocytic lymphoma 11.12
Primary biliary cirrhosis 10.14
Primary penile tumour staging 14.19
Primary prostatic tumour staging 14.16
Primary testicular tumour staging 14.12
Primary tuberculosis 8.3
Prognosis in colonic adenocarcinoma 10.11
Prognosis of papillary carcinoma of thyroid 12.14
Prognostic indicators of breast carcinoma 13.8
Prolactinoma 12.17
Prophylaxis for haemolytic disease of the newborn 2.40
Prostate cancer 14.8
Prostate specific antigen (PSA) 3.9, 14.15
Protein S 6.30
Prothrombin time 6.26
Pseudogout 15.20
Pseudomonas aeruginosa 4.21
Psyllium 17.5
Pulmonary embolism 7.11
Pulmonary embolus 8.4
Pulmonary hamartoma 8.20
Pulmonary metastases 8.13
R
Rabies 4.34
Rapidly progressive glomerulonephritis 9.12
Reactive lymhadenitis 11.1
Regenerative ability of cells 1.33
Release of growth factors by macrophages 2.15
Renal cortex and medulla 9.3
Renal involvement in scleroderma 9.6
Rheumatoid nodule 15.19
Rifampicin 17.41
S
Sacrococcygeal teratoma 3.40
Sarcoidosis 8.11
Schwannoma 16.8
Sciatica 15.16
Sclerosing cholangitis 10.15
Scurvy 6.3
Secondary hyperparathyroidism 12.16
Seminoma 3.10
Sequelae of hyperphosphataemia 5.27
Serum calcium after total thyroidectomy 12.19
Serum tumour marker staging in testicular seminoma 14.13
Significance of raised alkaline phosphatase 10.13
Sinus histiocytosis 2.1
Skin tanning 1.18
Skin testing in allergies 2.6
Small-cell anaplastic carcinoma 8.12
Small-cell lung cancer 3.45
Small-intestinal infarction 1.21
Sodium nitroprusside 17.21
Solar keratosis 3.18
Spermatocoele 14.6
Spironolactone 17.22
Sputum culture 4.11
Sputum cytology 8.16
Squamous cell carcinoma 8.8
Squamous metaplasia 1.4
Staging and prognosis of breast cancer 3.1
Staging for head and neck cancers 3.50
Staging of breast carcinoma 3.5
Staging of renal cell carcinoma 3.49
Staging of tumours 3.7
Stevens–Johnson syndrome 4.28
Streptokinase 17.13
Structure of Gram-negative bacteria 4.5
Subacute granulomatous thyroiditis 12.9
Subarachnoid haemorrhage 16.11
Subdural haematoma 16.12
Sufentanil 17.10
Survival rate for colon cancer (5-year) 3.44
T
Tazobactam 17.29
Temporal arteritis 7.3
Testicular teratoma 14.20
Thrombin clotting time 6.27
Thromboangiitis obliterans 7.9
Thrombocytopenia 6.24
Thromboembolic stroke 16.1
Thrombomodulin 6.33
Thrombotic thrombocytopenic purpura 6.11, 11.14
Thyroglossal duct cyst 12.20
Tissue factor 6.16
Tramadol 17.11
Transitional cell carcinoma 9.11
Transitional cell carcinoma of renal pelvis 9.1
Transudate pleural effusion 8.14
Treatment of Clostridium difficile infection 17.35
Treatment of tuberculosis 4.6
Trimethoprim 17.28
Tuberculosis prophylaxis 4.35
Tuberculous granuloma 4.17
Tumour necrosis factor 2.27
Tumour staging in NSCLC 3.41
Tumour–chromosome associations 3.36
Type I hypersensitivity 2.9
Type II hypersensitivity 2.10
Type III hypersensitivity 2.8
Type IV hypersensitivity 2.26
Types of graft 2.36
Types of lactic acidosis 5.29
Tyrosine kinase 1.29
U
Ulcerative colitis 10.10
Ultraviolet light 4.36
Unconjugated hyperbilirubinaemia 10.8
Ureteric calculus 9.16
Urinary tract infection 4.31
Urine specific gravity 9.5
V
Varicocoele 14.7
Vestibular neuronitis 16.7
Vincristine 17.27
Virginal breast hypertrophy 13.2
Virulence of bacteria 4.18
Vitamin A deficiency 1.19
von Willebrand’s disease 6.8, 6.14
W
Warfarin necrosis 6.29
Warfarin therapy 6.20
Warthin’s tumour 3.15
White blood cell casts 9.15
Widal test 4.13
Wide anion gap metabolic acidosis 5.5
Wilms’ tumour 9.7
X
X-linked agammaglobulinaemia 2.23
What did you think of this book?
Click here to let us know!
pastest.co.uk
twitter.com/PasTest
joinourgroup